Download as pdf or txt
Download as pdf or txt
You are on page 1of 135

TOPNOTCH MEDICAL BOARD PREP SURGERY MAIN DIGITAL HANDOUT BY LOUBOMIR ANTONIO, MD

For inquiries visit www.topnotchboardprep.com.ph or https://www.facebook.com/topnotchmedicalboardprep/


This handout is only valid for the September 2021 PLE batch. This will be rendered obsolete for the next batch since we update our handouts regularly.
13. Esophagus 45
Important Legal Information 14. Stomach 51
The handouts, videos and other review materials, provided by Topnotch Medical Board
Preparation Incorporated are duly protected by RA 8293 otherwise known as the 15. Liver and Gallbladder 60
Intellectual Property Code of the Philippines, and shall only be for the sole use of the person: 16. Pancreas 74
a) whose name appear on the handout or review material, b) person subscribed to Topnotch 17. Small Intestines 80
Medical Board Preparation Incorporated Program or c) is the recipient of this electronic
communication. No part of the handout, video or other review material may be reproduced, 18. Appendix 87
shared, sold and distributed through any printed form, audio or video recording, electronic 19. Colon, Rectum, Anus 90
medium or machine-readable form, in whole or in part without the written consent of
Topnotch Medical Board Preparation Incorporated. Any violation and or infringement, This handout is only valid for the September 2021 PLE batch.
whether intended or otherwise shall be subject to legal action and prosecution to the full This will be rendered obsolete for the next batch
extent guaranteed by law. since we update our handouts regularly.

DISCLOSURE
The handouts/review materials must be treated with utmost confidentiality. It shall be the
BASIC CONSIDERATIONS
responsibility of the person, whose name appears therein, that the handouts/review RESPONSE TO INJURY, FLUIDS
materials are not photocopied or in any way reproduced, shared or lent to any person or
disposed in any manner. Any handout/review material found in the possession of another AND NUTRITION
person whose name does not appear therein shall be prima facie evidence of violation of RA
8293. Topnotch review materials are updated every six (6) months based on the current
FLUIDS, ELECTROLYTES, AND ACID-BASE
trends and feedback. Please buy all recommended review books and other materials listed BODY FLUIDS AND COMPARTMENT
below.
THIS HANDOUT IS NOT FOR SALE! • Water constitutes ~50-60% of total body weight
• Total body weight and total body water (TBW) is relatively
INSTRUCTIONS constant for a person and is primarily a reflection of body fat
To scan QR codes on iPhone and iPad o Lean tissues (e.g. muscle and solid organs) have higher water
1. Launch the Camera app on your IOS device content than fat and bone
2. Point it at the QR code you want to scan
3. Look for the notification banner at the top Parameter Total Body Water
of the screen and tap Adult male 60%
To scan QR codes on Android Adult female 50% - due to females having higher
1. Install QR code reader from Play Store
2. Launch QR code app on your device amount of fat relative to men
3. Point it at the QR code you want to scan Newborns 80%
4. Tap browse website • TBW is divided into 3 functional fluid compartments:
Approach to Topnotch Surgery 1. Plasma (extracellular)
• Surgery can be overwhelming. But believe it or not, you will now find it 2. Interstitial fluid (extracellular)
easier to integrate concepts and practices for you have already seen 3. Intracellular fluid
these surgical concepts and scenarios in action during your surgical
rotations. We encourage you to correlate the concepts being discussed
in this handout with actual cases that you had encountered – what was
the diagnosis? What surgery was done?
• Supplementary notes are written in special boxes like this!

SUPPLEMENT: APPROACH TO TOPNOTCH SURGERY


NORMAL FLUID
Pay close attention to those in bold, italicized, underlined as these are very
BALANCE
important facts to remember for the subject. Master topics written in our (i.e.,
quick review, subject cross overs and end of review question boxes because in a 70 kg man)
they provide high yield information, not just for surgery but for the rest of
the other subjects as well!
• Please find the time to finish the whole material “skin-to-skin.” Like any
surgery, do not take shortcuts, and please try to stay awake and alert
throughout the duration.
• We suggest that you buy the following:
o Schwartz Principles of Surgery, 11th ed.
o Schwartz Principles of Surgery, Absite and Board Review
10th ed. Extracellular Fluid Intracellular Fluid
o Surgery Platinum Na+
Cations K+, Mg2+
(principal cation of ECF)
You will also find several annotations of our faculty in your handout to Cl-, HCO3-
highlight boards-relevant concepts. Anions PO4-, proteins
Topnotch (principal anions of ECF)
• Gibbs-Donnan Effect
o Slightly higher protein content (anions) in plasma results in a
SURGERY higher plasma cation composition relative to ISF
Remember OPPOSITES ATTRACT: ionic interactions occur between
By Loubomir E. Antonio, MD, FSOSP, FPCS, FPSGS substances with 2 different charges (i.e. cations and anions).
Contributors: Dr. Rubio

Kurt Roland A. Asperas, MD


Angeli Andrea S. Cocos-Alcantara, MD
Teddy Carpio, MD-MBA
Julianne Cristy Lopez, MD-MBA
Frinz Moey C. Rubio, MD

OUTLINE PAGE
BASIC CONSIDERATIONS
1. Response to Injury, Fluids, and Nutrition 1
2. Surgical Metabolism 4
3. Hemostasis and Transfusion 7
4. Wound Healing 9
5. Sepsis and Septic Shock 10
6. Surgical Site Infections 10
7. Trauma and Burns 11
SPECIFIC CONSIDERATIONS The figure above shows the values we calculate/account for when we do
9. Head, Neck, and Thyroid 20 our I/O (input vs output).
10. Breast 30 • In surgery, we also account for losses like bleeding and/or outputs from
the surgical drains – JP Drains, Penrose, NGT, etc.
11. Skin 37
• Make sure you account for insensible losses when you make your calculations
12. Abdominal Wall Hernia 40 Dr. Asperas

TOPNOTCH MEDICAL BOARD PREP SURGERY MAIN DIGITAL HANDOUT BY LOUBOMIR ANTONIO, MD Page 1 of 101
For inquiries visit www.topnotchboardprep.com.ph or https://www.facebook.com/topnotchmedicalboardprep/
This handout is only valid for the September 2021 PLE batch. This will be rendered obsolete for the next batch since we update our handouts regularly.
TOPNOTCH MEDICAL BOARD PREP SURGERY MAIN DIGITAL HANDOUT BY LOUBOMIR ANTONIO, MD
For inquiries visit www.topnotchboardprep.com.ph or https://www.facebook.com/topnotchmedicalboardprep/
This handout is only valid for the September 2021 PLE batch. This will be rendered obsolete for the next batch since we update our handouts regularly.

SUPPLEMENT:
BASAL REQUIREMENTS FOR WATER STAGES OF DEHYDRATION
AND ELECTROLYTES MILD MODERATE SEVERE
DAILY REQUIREMENTS (2 to 5% (6 to 10% (11 to 15%
1,500 to 2,500 ml, or BW lost as water) BW lost as water) BW lost as water)
Fluid (70kg male)
about 25–30 ml/kg body weight • Thirst • Severe thirst • Stupor
Sodium 1–2 mEq/kg
• Low urine • Nausea • Hypotension
Potassium 0.5–1.0 mEq/kg volume • Dry axilla and groin • Severe oliguria
Electrolytes Calcium 1000 mg/day • Reduced • Tachycardia and • Thready pulse
Phosphate 700 mg sweating Orthostatic • Shock
Magnesium 300-400mg/day hypotension • Coma
Chloride 1–2 mEq/kg • Low central venous • Death
SUPPLEMENT: EXTRACELLULAR VOLUME DEFICIT VS. EXCESS pressure
EXTRACELLULAR VOLUME DEFICIT • Poor skin turgor
• Most common fluid disorder in surgical patients • Apathy
o Acute volume deficit is associated with cardiovascular and central • Oliguria
nervous system signs • Hemoconcentration
o Chronic volume deficit displays tissue signs such as decrease in skin
turgor and sunken eyes, in addition to acute signs ✔ GUIDE QUESTION
• Most common cause of volume deficit in surgical patients is a loss A 55-year-old man with Crohn’s disease had undergone resection of
of GI fluids from nasogastric suction, vomiting, diarrhea, or small bowel and anastomosis. Ten days later, he is found to have bilious
enterocutaneous fistula drainage of 1 L/d from the drains. He is started on total parenteral
nutrition (TPN). Four days later, his arterial blood gases (ABGs) are pH,
• Third-space or nonfunctional ECF losses that occur with
7.25; PO2, 98 mm Hg; and PCO2, 40 mmHg. His anion gap is 10. The
sequestration secondary to soft tissue injuries/infections, burns, and
most likely cause is which of the following?
intraabdominal processes such as peritonitis, obstruction, or
prolonged surgery can also lead to massive volume deficits (A) Diabetic ketoacidosis
(B) Renal failure
EXTRACELLULAR VOLUME EXCESS (C) Hypovolemic shock
• May be iatrogenic or secondary to renal dysfunction, congestive heart (D) Small-bowel fistula
failure, or cirrhosis DKA, renal failure and hypovolemic shock all results to high anion
• Both plasma and interstitial volumes are increased gap metabolic acidosis.
• Symptoms are primarily pulmonary and cardiovascular
• In healthy patients, edema and hyperdynamic circulation are SUPPLEMENT: ACID BASE BALANCE
common and well tolerated. However, the elderly and patients with ANION GAP
cardiac disease may quickly develop congestive heart failure and • Anion gap is an index of unmeasured anions
pulmonary edema • AG = 𝑁𝑎 − (𝐶𝑙 + 𝐻𝐶𝑂!) N= <12mmol/L
ELECTROLYTE SOLUTIONS FOR PARENTERAL HIGH ANION GAP METABOLIC ACIDOSIS (HAGMA)
ADMINISTRATION Exogenous acid ingestion Mnemonic: “MUDPILES”
Solution Na+ Cl- K+ HCO3- Ca2+ Mg2+ mOsm • Ethylene glycol • Methanol
280- • Salicylate • Uremia (Renal failure)
ECF 142 103 4 27 5 3
310 • Methanol • Diabetic ketoacidosis
Lactated • Propylene glycol, Paraldehyde
130 109 4 28 3 273 Endogenous acid production
Ringer’s • Infection, Iron, Isoniazid
0.9% NSS 154 154 308 • Ketoacidosis
• Lactic acidosis
D5 0.45% • Lactic acidosis
77 77 407 • Ethylene glycol
NSS • Renal insufficiency
• Salicylates
D5W 253
3% NSS 513 513 1026 NORMAL ANION GAP METABOLIC ACIDOSIS (NAGMA)
Plasma- Acid administration (HCl) Mnemonic: “HARD UP”
140 98 5 27 3 295
Lyte Loss of bicarbonate • Hyperalimentation
From Schwartz’s Principles of Surgery, 11th edition
GI losses (diarrhea, fistulas) • Acetazolamide
Presented above are the commonly used IVF solutions. They are listed Ureterosigmoidoscopy (Carbonic anhydrase inhibitor)
from lowest to highest osmolarity. The composition of the ECF is also Renal tubular acidosis • Renal tubular acidosis
included for comparison. Knowledge on the components and properties Carbonic anhydrase inhibitor • Diarrhea
of each fluid will guide you in choosing the appropriate IVF. • Ureteroenteric fistula
Example: If you are intending to expand the ECF compartment, choosing • Pancreaticoduodenal fistula
0.9% PNSS or LR is acceptable.
Dr. Asperas
✔ GUIDE QUESTION
HOLLIDAY-SEGAR METHOD In metabolic alkalosis, there is which of the following?
BODY FLUID VOLUME FLUID VOLUME (A) Gain in fixed acid
(B) Loss of base
WEIGHT (kg) (ml/kg/hr) (ml/kg/day)
(C) Hyperkalemia
First 10 kg 4 100 A (D) Rise in base excess
Simply put, metabolic alkalosis is loss of fixed acid, rise in base
Next
2 50 B excess (HCO3-) and hypokalemia, while metabolic acidosis is gain in
10-20 kg fixed acid, loss of base and associated with hyperkalemia.
20
For elderly patients or SUPPLEMENT: BICARBONATE GENERATION
Each kg
1 patients with cardiac disease, C INCREASED BICARBONATE GENERATION
>20 kg this amount is Chloride losing (urinary chloride > 20 mEq/l)
reduced to 15 mL/kg/d
• Mineralocorticoid excess
Maintenance Fluid Requirement = A + B + C • Profound potassium depletion
10 kgs: 10kg x 100ml/kg/day = 1L/day! Chloride sparing (urinary chloride < 20 mEq/l)
20 kgs: 10kg x 50 ml/kg/day = 1L + 500ml = 1.5L/day!
• Loss from gastric secretions (emesis or nasogastric suction)
50 kgs: 30kg x 20ml/kg/day = 1L + 500ml + 600ml = 2.1L/day!
• Diuretics
Example: Average 70kg man Excess administration of alkali
10kg + 10kg + 50kg x 20ml/kg/day = 1L + 500ml + 1L = 2.5L/day • Acetate in parenteral nutrition
*2500ml / 24 hours = ~104cc/hr. • Citrate in blood transfusions
*Chart order: 1L IVF to run at 100cc/hour • Antacids
(it means that the patient will be given 100% of his Maintenance • Bicarbonate
Fluid via IVF; TFR is equal to MF given that the patient has no fluid • Milk-alkali syndrome
loss and is clinically well hydrated - TFR = MF + OL + SOH)
Dr. Asperas
IMPAIRED BICARBONATE EXCRETION
Decreased glomerular filtration (as seen in CKD)
Increased bicarbonate reabsorption (hypercarbia or potassium
depletion)

TOPNOTCH MEDICAL BOARD PREP SURGERY MAIN DIGITAL HANDOUT BY LOUBOMIR ANTONIO, MD Page 2 of 101
For inquiries visit www.topnotchboardprep.com.ph or https://www.facebook.com/topnotchmedicalboardprep/
This handout is only valid for the September 2021 PLE batch. This will be rendered obsolete for the next batch since we update our handouts regularly.
TOPNOTCH MEDICAL BOARD PREP SURGERY MAIN DIGITAL HANDOUT BY LOUBOMIR ANTONIO, MD
For inquiries visit www.topnotchboardprep.com.ph or https://www.facebook.com/topnotchmedicalboardprep/
This handout is only valid for the September 2021 PLE batch. This will be rendered obsolete for the next batch since we update our handouts regularly.

COMMON ELECTROLYTE ABNORMALITIES • Clinical manifestations:


o Primarily related to failure of normal contractility of GI smooth
SODIUM muscle (ileus, constipation), skeletal muscle (decreased
1. HYPERNATREMIA reflexes, weakness, paralysis), and cardiac muscle (arrest)
• Results from either a loss of free water or a gain of sodium in • ECG changes:
excess of water o U waves, T-wave flattening,J ST-segment changes, and
o Associated with either an increased, normal, or decreased arrhythmias (with digitalis therapy)
extracellular volume • Treatment: Potassium repletion, the rate is determined by the
• Clinical manifestations: symptoms
o Symptoms are rare until serum sodium concentration exceeds o Mild, asymptomatic hypokalemia: oral repletion is adequate
160 mEq/l (KCl 40 mEq per enteral access x 1 dose)
o Mostly central nervous system in nature (restlessness, o Asymptomatic hypokalemia, not tolerating enteral nutrition:
irritability, seizures, coma) due to hyperosmolarity KCl 20 mEq IV q2h x 2 doses
o May lead to subarachnoid hemorrhage and death o If IV repletion is required, usually no more than 10 mEq/h is
• Treatment: Management of water deficit advisable in an unmonitored setting
o In hypovolemic patients, volume should be restored with o K+ supplementation can be increased to 40 mEq/h when
normal saline before concentration abnormality is addressed accompanied by continuous ECG monitoring, and even more in
o Once adequate volume is achieved, water deficit is replaced the case of imminent cardiac arrest from a malignant
using a hypotonic fluid arrhythmia associated hypokalemia
o Rate of fluid administration should be titrated to achieve a Caution should be done when oliguria or impaired renal function is
decrease in serum sodium concentration of no more than 1 coexistent!
mEq/l/h Dr. Asperas

o Overly rapid correction can lead to cerebral edema and 5. HYPERcalcemia


herniation • Serum calcium level above the normal range of 8.5-10.5
mEq/l or an increase in ionized calcium above 4.2-4.8 mg/dl
2. HYPONATREMIA
o Critical level for serum calcium is 15 mEq/l, when
• Occurs when there is an excess of extracellular water relative to symptoms noted earlier may rapidly progress to death
sodium • Caused by primary hyperparathyroidism in the outpatient
• Extracellular volume can be high, normal, or low setting and malignancy in hospitalized patients
• In most cases, sodium concentration is decreased as a • Clinical manifestations: Neurologic impairment,
consequence of either sodium depletion or dilution musculoskeletal weakness and pain, renal dysfunction, and GI
• Clinical manifestations: symptoms
o Symptomatic hyponatremia does not occur until serum • ECG changes: shortened QT interval, prolonged PR and QRS
sodium level is 120 mEq/l intervals, increased QRS voltage, T-wave flattening and
o Primarily central nervous system in origin (headache, widening, and atrioventricular block
confusion, seizures, coma) associated increases in intracranial • Treatment: Aimed at repleting the associated volume deficit and
pressure then inducing a brisk diuresis with normal saline
• Treatment: Water restriction and, if severe, the o Treatment is required when hypercalcemia is symptomatic,
administration of sodium which usually occurs when the serum level exceeds 12
o If symptomatic, 3% normal saline should be used to increase mEq/l
the sodium by no more than 1 mEq/l/h until the serum
sodium reaches 130 mEq/l or symptoms are improved 6. HYPOcalcemia
o If asymptomatic, correction should increase the sodium level • Serum calcium level below 8.5 mEq/l or a decrease in the
by no more than 0.5 mEq/l/hr. to a maximum increase of 12 ionized calcium level below 4.2 mg/dl
mEq/l/d • Causes include pancreatitis, malignancies associated with
increased osteoclastic activity (breast and prostate cancer),
POTASSIUM massive soft tissue infections such as necrotizing fasciitis, renal
failure, pancreatic and small bowel fistulas, hypoparathyroidism,
3. HYPERKALEMIA
toxic shock syndrome, and tumor lysis syndrome
• Serum K+ concentration above the normal range of 3.5-5 • Transient hypocalcemia also occurs after removal of a
mEq/l parathyroid adenoma due to atrophy of the remaining gland and
• Caused by excessive K+ intake, increased release of K+ from cells, avid bone remineralization
or impaired K+ excretion by the kidneys • Neuromuscular and cardiac symptoms do not occur until the
• Clinical manifestations: ionized fraction falls below 2.5 mg/dl
o Mostly GI (nausea/vomiting, diarrhea), neuromuscular • Clinical manifestations: Neuromuscular symptoms with
(weakness, paralysis), and cardiovascular (arrhythmia, arrest) decreased cardiac contractility
• ECG changes: • ECG changes: Prolonged QT interval, T-wave inversion, heart
o High peaked T waves J (early), widened QRS complex, block and ventricular fibrillation
flattened P wave, prolonged PR interval (first-degree block), • Treatment: Calcium supplementation and correction of other
sine wave formation and ventricular fibrillation metabolic derangements
• Treatment: o Asymptomatic hypocalcemia can be treated with oral or IV
o Reducing total body K+ (eg. furosemide), shifting K+ from calcium
extracellular to intracellular space (eg. glucose-insulin o Acute symptomatic hypocalcemia should be treated with IV
solution, salbutamol), and protecting cells from the effects of 10% calcium gluconate to achieve a serum concentration of
increased K+ (eg. Bicarbonate, calcium gluconate) 7-9 mg/dl
o K+ can be removed from the body using a cation-exchange o Associated deficits in magnesium, potassium, and pH must also
resin such as Kayexalate that binds K+ in exchange for Na+ be corrected
o When ECG changes are present, calcium chloride or calcium
Hypocalcemia will be refractory to treatment if coexisting
gluconate (5-10 ml of 10% solution) should be administered
hypomagnesemia is not corrected first! Also, check if serum albumin
immediately level is normal.
All measures are temporary, lasting from 1 to 4 hours. Dialysis should Dr. Asperas

be considered in severe hyperkalemia when conservative measures fail. Corrected Ca mg/dL= Measured Ca + 0.8 x (4.0-albumin g/dL)
Dr. Asperas Or
4. HYP0KALEMIA Corrected Ca mmol/L=Measured Ca 0.02 x (40-albumin g/L)
• More common than hyperkalemia in the surgical patient
• Caused by inadequate K+ intake, excessive renal K+ excretion, K+
loss in pathologic GI secretions, or intracellular shifts from
metabolic alkalosis or insulin therapy

TOPNOTCH MEDICAL BOARD PREP SURGERY MAIN DIGITAL HANDOUT BY LOUBOMIR ANTONIO, MD Page 3 of 101
For inquiries visit www.topnotchboardprep.com.ph or https://www.facebook.com/topnotchmedicalboardprep/
This handout is only valid for the September 2021 PLE batch. This will be rendered obsolete for the next batch since we update our handouts regularly.
TOPNOTCH MEDICAL BOARD PREP SURGERY MAIN DIGITAL HANDOUT BY LOUBOMIR ANTONIO, MD
For inquiries visit www.topnotchboardprep.com.ph or https://www.facebook.com/topnotchmedicalboardprep/
This handout is only valid for the September 2021 PLE batch. This will be rendered obsolete for the next batch since we update our handouts regularly.
7. HYPERMAGNESEMIA • The estimated protein maintenance requirements for non-
• Rare but can be seen with severe renal insufficiency and parallel stressed patients is 0.8-1 g/kg/day respectively.
changes in potassium excretion • Repletion requirements of the nutritionally compromised
• Clinical manifestations: Mainly GI with neuromuscular patient are adjusted depending on the activity and stress level
dysfunction and impaired cardiac conduction the body is subjected with.
• ECG changes: (similar to hyperkalemia) Increased PR interval, CORRECTION CORRECTION
widened QRS complex, elevated T waves ACTIVITY AND STRESS /
FACTOR FACTOR
INJURY LEVEL
• Treatment: Eliminate exogenous sources of magnesium, correct (CALORIE) (PROTEIN)
concurrent volume deficits and correct acidosis if present Confined to bed 1.2 1.0
o To manage acute symptoms, calcium chloride (5-10ml) Activity Ambulatory 1.3 1.2
should be administered to immediately antagonize the Fever (>27°C) 1.3 1.5
cardiovascular effects Mild stress
1.2 1.2
o If persistently elevated or with symptoms, dialysis may be (Minor operation)
necessary Moderate stress
Stress / 1.35 1.5
(Skeletal trauma)
Injury
8. HYPOMAGNESEMIA Severe stress
1.6 2.0
(Severe sepsis)
• Result from alterations of intake, renal excretion, and pathologic
Burns 2.0 2.5
losses
o Poor intake may occur in cases of starvation, alcoholism, ✔ GUIDE QUESTION
prolonged IV fluid therapy, and TPN with inadequate Surgical patients who are in a lot of stress like burn patients need how
supplementation of Magnesium many grams of protein per kg/day?
o Losses are seen in cases of increased renal excretion from (A) 1.2 (C) 2
alcohol abuse, diuretic use, administration of amphotericin B, (B) 1.5 (D) 2.5
and primary aldosteronism, as well as GI losses from diarrhea,
malabsorption, and acute pancreatitis
• Clinical manifestations: Neuromuscular and central nervous
system hyperactivity, similar to those of calcium deficiency Basal caloric
• ECG changes: Prolonged QT and PR intervals, ST-segment requirement of a
depression, flattening or inversion of P waves, torsade de normal health
pointes, and arrhythmias adult
• Treatment: Magnesium supplementation 25-30 kcal/Kg per
o Correction of magnesium depletion can be oral if day
asymptomatic and mild or IV if symptomatic and severe Figure 2-18. Schwartz’s
Principles of Surgery. 11th ed.
o For those with severe deficits (<1 mEq/L) or those who are 2019

symptomatic, 1 to 2 g of magnesium sulfate may be


administered IV over 15 minutes or 2 minutes if under ECG
monitoring to correct torsade
o To counteract the adverse side effects of a rapidly rising ✔ GUIDE QUESTION
magnesium level and correct hypocalcemia (frequently associated A 60-year-old man with carcinoma of the esophagus is admitted with
with hypomagnesemia), simultaneous administration of calcium severe malnutrition. Nutritional support is to be initiated.
gluconate is done What should be his daily caloric intake?
(A) 5 kcal/kg body weight/day
(B) 15 kcal/kg body weight/day
SURGICAL METABOLISM (C) 30 kcal/kg body weight/day
PREOPERATIVE NUTRITIONAL ASSESSMENT (D) 100 kcal/kg body weight/day
SEVERITY OF WEIGHT LOSS For chronically starved patient, calories are given initially HALF of
what is required to prevent Refeeding Syndrome.
SIGNIFICANT SEVERE Dr. Antonio
TIME
WEIGHT LOSS WEIGHT LOSS
1 week 1 to 2% >2% PHASES OF SURGICAL METABOLISM
1 month 5% >5% 1. EBB PHASE
3 months 7.5% >7.5% • within the first 24–48 hours after injury.
6 months 10% >10% • It is characterized by reconstruction of body’s normal tissue
perfusion and efforts to protect homeostasis.
• there is a decrease in total body energy and urinary nitrogen
MANAGING NUTRITION IN THE SURGICAL PATIENT excretion
ESTIMATING ENERGY AND PROTEIN REQUIREMENTS
A. ENERGY (CALORIC) REQUIREMENT 2. FLOW PHASE
• Basal Energy Expenditure (BEE) represents the minimum a. Catabolic phase (“after operation phase”)
caloric requirements of the resting, unstressed patient. § early period catabolism that provides compensating
response to the initial trauma and volume replacement
• The calculation is performed using the Harris-Benedict
b. Anabolic phase (“recovery from operation phase”)
equation:
o Female BEE
= 665.10 + (9.56 × 𝑊) + (1.85 × 𝐻) − (4.68 × 𝐴)
o Male BEE
= 66.47 + (13.75 × 𝑊) + (5 × 𝐻) − (6.76 × 𝐴)
§ W = actual weight in Kg
§ H = actual height in cm
§ A = age in years
• After the BEE has been determined from the Harris–Benedict
equation, the patient’s total daily maintenance energy
requirements are estimated by multiplying the BEE by an
activity factor and a stress/injury factor (Long modification).

B. PROTEIN REQUIREMENT
𝑔 𝑁𝑖𝑡𝑟𝑜𝑔𝑒𝑛 6.25𝑔 𝑃𝑟𝑜𝑡𝑒𝑖𝑛 Adapted from https://aneskey.com/how-does-critical-illness-alter-metabolism/
𝑃𝑟𝑜𝑡𝑒𝑖𝑛(") = 𝑇𝑜𝑡𝑎𝑙 𝑘𝑐𝑎𝑙 × ×
150 𝑘𝑐𝑎𝑙 𝑔 𝑁𝑖𝑡𝑟𝑜𝑔𝑒𝑛

TOPNOTCH MEDICAL BOARD PREP SURGERY MAIN DIGITAL HANDOUT BY LOUBOMIR ANTONIO, MD Page 4 of 101
For inquiries visit www.topnotchboardprep.com.ph or https://www.facebook.com/topnotchmedicalboardprep/
This handout is only valid for the September 2021 PLE batch. This will be rendered obsolete for the next batch since we update our handouts regularly.
TOPNOTCH MEDICAL BOARD PREP SURGERY MAIN DIGITAL HANDOUT BY LOUBOMIR ANTONIO, MD
For inquiries visit www.topnotchboardprep.com.ph or https://www.facebook.com/topnotchmedicalboardprep/
This handout is only valid for the September 2021 PLE batch. This will be rendered obsolete for the next batch since we update our handouts regularly.
FLOW PHASE
EBB PHASE
CATABOLIC ANABOLIC
Duration 24-48 hrs 1-2 Weeks Month/s
Repair – conserve
Mobilization of Replacement of
Role blood and energy
energy stores lost tissue
reserves
Decreased BMR, Increased BMR,
Positive nitrogen
Physiological Temperature, O2 Temperature, O2
balance
consumption consumption
Dr. Asperas

Metabolism After Injury


Adapted from Schwartz’s Principles of Surgery. 11th ed. 2019

• LIPIDS (TRIGLYCERIDE) become the primary source of energy


Summary: ENERGY SOURCE
Short-term Fasting (<5 days) LIPIDS
Prolonged Fasting KETONE BODIES
After Injury LIPIDS
Dr. Asperas

✔ GUIDE QUESTION
How many kilocalories does a Liter of D5LR provides?
(A) 170kcal
(B) 200kcal
(C) 400kcal
(D) 500kcal
(E) none, dextrose does not provide energy like glucose

CATEGORIES OF ENTERAL FORMULAS


TYPE COMMENTS
• Whole protein nitrogen source
Standard
• For patients with normal or near normal GI function
Intact
Homeostatic adjustments initiated after injury • Most products contain about 1.0 kcal/mL
The Absite Review. 5th ed. 2017 Nutrients
• Protein content varies
• Predigested nutrients
• Most have a low-fat content or a high percentage
Elemental of medium chain triglycerides
• For use in patients with severely impaired GI
absorption
Fluid • Intact nutrients
Restricted • Calorically dense (2.0 kcal/mL)
• Intact nutrients
Renal • Calorically dense (2.0 kcal/mL)
• Low phosphorus and low potassium
Other • Intact nutrients designed for feeding patients with
disease respiratory disease, diabetes, hepatic failure, and
specific immune compromise

Metabolism During Short-term Fasting (<5 days) OPTIONS FOR ENTERAL FEEDING ACCESS
Adapted from Schwartz’s Principles of Surgery. 11th ed. 2019
• Short-term use
• principal sources of fuel are derived from muscle protein and Nasogastric • Aspiration risks
lipids, with LIPIDS being the most abundant source of energy tube (NGT) • Nasopharyngeal trauma
• Frequent dislodgement
• Short-term use
Nasoduodenal
• Lower aspiration risks in jejunum
/ Nasojejunal
• Placement challenges (radiographic
tube
assistance often necessary)
• Endoscopy skills required
• May be used for gastric decompression or
Percutaneous
bolus feeds
Endoscopic
• Aspiration risks
Gastrostomy
• Can last 12-24 months
(PEG)
• Slightly higher complication rates with
placement and site leaks
• Requires general anesthesia and small
laparotomy
Surgical
• Procedure may allow placement of extended
Gastrostomy
duodenal/jejunal feeding ports
• Laparoscopic placement possible
Metabolism During Prolonged Fasting
Adapted from Schwartz’s Principles of Surgery. 11th ed. 2019 • Commonly carried out during laparotomy
• General anesthesia, laparoscopic placement
• Systemic proteolysis is reduced Surgical
usually requires assistant to thread catheter
• KETONE BODIES become an important fuel source (after 2 Jejunostomy
• Laparoscopy offers direct visualization of
days) catheter placement

TOPNOTCH MEDICAL BOARD PREP SURGERY MAIN DIGITAL HANDOUT BY LOUBOMIR ANTONIO, MD Page 5 of 101
For inquiries visit www.topnotchboardprep.com.ph or https://www.facebook.com/topnotchmedicalboardprep/
This handout is only valid for the September 2021 PLE batch. This will be rendered obsolete for the next batch since we update our handouts regularly.
TOPNOTCH MEDICAL BOARD PREP SURGERY MAIN DIGITAL HANDOUT BY LOUBOMIR ANTONIO, MD
For inquiries visit www.topnotchboardprep.com.ph or https://www.facebook.com/topnotchmedicalboardprep/
This handout is only valid for the September 2021 PLE batch. This will be rendered obsolete for the next batch since we update our handouts regularly.
• Jejunal placement with regular endoscope is • Zinc deficiency is the most common that
operator dependent manifests as diffuse eczematoid rash at
PEG-jejunal • Jejunal tube often dislodges retrograde intertriginous areas
tube • Two-stage procedure with PEG placement, • Copper deficiency is associated with
followed by fluoroscopic conversion with Microcytic anemia
jejunal feeding tube through PEG • Chromium deficiency is associated with
Glucose intolerance
TYPES OF PARENTERAL NUTRITION SUPPORT • May occur after initiation of parenteral
nutrition
TYPE COMMENT
• Manifests as glycosuria
Central • High osmolarity (1500-2800 mOsm/L) • If blood glucose levels remain elevated or
Parenteral • Requires administration into Relative glycosuria persists, dextrose concentration
Nutrition large veins with high blood flow (2-6 L/min) Glucose may be decreased, infusion rate slowed, or
(CPN or TPN) • Provides complete nutritional requirements Intolerance regular insulin added to each bottle
• Osmolarity limited to <900 mOsm/day • Rise in blood glucose may be temporary, as
Peripheral the normal pancreas increases its output of
• Lower concentrations of dextrose and amino
Parenteral insulin in response to the continuous
acid carbohydrate infusion
Nutrition
(PPN)
• Calories and protein provided are limited by • Lack of intestinal stimulation is associated
tolerated volume of IV fluid with intestinal mucosal atrophy, diminished
Intestinal villous height, bacterial overgrowth, reduced
Atrophy lymphoid tissue size, reduced
immunoglobulin A production, and impaired
gut immunity

As a general rule, if the GI tract is functioning, USE IT to feed your


patient!
http://www.rxkinetics.com/tpntutorial/1_4.html Dr. Asperas

SUPPLEMENT: PARENTERAL (IV) NUTRITION ✔ GUIDE QUESTION


Indications A 42-year-old man with small-bowel fistula has been receiving TPN
with standard hypertonic glucose-amino acid solution for 3 weeks. The
• Inability to absorb adequate nutrients via GI tract
patient is noticed to have scaly, hyperpigmented lesions over the acral
o Massive short bowel resection/short-bowel syndrome
surfaces of elbows and knees, similar to enterohepatic acrodermatitis.
o Radiation enteritis
What is the most likely cause of the condition?
o Severe diarrhea
(A) Copper deficiency
o Untreatable steatorrhea/malabsorption
(B) Essential fatty acid deficiency
• Complete bowel obstruction or intestinal pseudo-obstruction
(C) Excess glucose calories
• Severe catabolism ± malnutrition when GI tract not usable within
(D) Zinc deficiency
5-7 days
• Inability to obtain enteral access REFEEDING SYNDROME
• Potentially fatal shifts in fluids and electrolytes that may occur
SUPPLEMENT: PARENTERAL (IV) NUTRITION in malnourished patients receiving artificial refeeding
Contraindications • Symptoms: fatigue, lethargy, muscle weakness, fluid overload,
• Functional GI tract cardiac arrhythmias, and hemolysis
• Hemodynamically unstable • Management: Slow initiation in first week (50% of
• Hypovolemia, cardiogenic, or septic shock requirements based on dry weight)
• Severe pulmonary edema, fluid overload
• Anticipated treatment <5 days in the absence of severe
malnutrition
• Profound metabolic or electrolyte disturbances
• Unable to obtain venous access
• Prognosis doesn't warrant aggressive nutritional support
COMPLICATIONS OF PARENTERAL NUTRITION
• Rare occurrences if IV vitamin preparations
are used
Vitamin
• However, Vitamin K is not part of any
Deficiencies
commercially prepared vitamin solution so it
should be supplemented on a weekly basis
• Clinically apparent during prolonged
parenteral nutrition with fat-free solutions
Essential Fatty • Manifests as dry, scaly dermatitis and loss of
Acid (EFA) hair
Deficiency • Prevented by periodic infusion of a fat
emulsion at a rate equivalent to 10 to 15% of
total calories
Trace Mineral • Essential trace minerals may be required
Adapted from http://flipper.diff.org/app/items/info/4867
Deficiencies after prolonged TPN

TOPNOTCH MEDICAL BOARD PREP SURGERY MAIN DIGITAL HANDOUT BY LOUBOMIR ANTONIO, MD Page 6 of 101
For inquiries visit www.topnotchboardprep.com.ph or https://www.facebook.com/topnotchmedicalboardprep/
This handout is only valid for the September 2021 PLE batch. This will be rendered obsolete for the next batch since we update our handouts regularly.
TOPNOTCH MEDICAL BOARD PREP SURGERY MAIN DIGITAL HANDOUT BY LOUBOMIR ANTONIO, MD
For inquiries visit www.topnotchboardprep.com.ph or https://www.facebook.com/topnotchmedicalboardprep/
This handout is only valid for the September 2021 PLE batch. This will be rendered obsolete for the next batch since we update our handouts regularly.
OVERFEEDING SUPPLEMENT: COAGULATION CASCADE
• Provision of macronutrients in excess of metabolic demand Please watch the videos below as a review of coagulation.
• High doses of protein intake may lead to azotemia, hypertonic Dr. Asperas

dehydration, and metabolic acidosis


• High doses of glucose infusion may result in hyperglycemia, HEMOSTASIS – CLOTS,
hypertriglyceridemia, and hepatic steatosis THROMBI, AND
• Management: Prevented by insulin treatment targeting ANTI-PLATELETS
euglycemia https://qrs.ly/8ibi0hl

HEMOSTASIS AND TRANSFUSION


HEMOSTASIS
STAGES OF HEMOSTASIS
1. Vascular constriction
2. Platelet plug formation
3. Fibrin formation
4. Fibrinolysis

HEMOSTASIS –
COAGULATION CASCADE
https://qrs.ly/dnbi0nz

Adapted from Schwartz’s Principles of Surgery. 11th ed. 2019

PRIMARY HEMOSTASIS
• Refers to platelet plug formation and
aggregation

Adapted from The Absite Review. 5th ed. 2017

SECONDARY HEMOSTASIS
• Refers to the deposition of insoluble fibrin
generated by the proteolytic coagulation
cascade

APPROACH TO COAGULATION PROBLEMS Increased PT/INR


DERANGEMENT APPROACH Increased aPTT
• Assume DIC
• Consider platelet dysfunction Increased D-dimer
Normal PT/INR • Provide FFP or cryoprecipitate
• Provide platelets and initiate directed Decreased
Normal aPTT platelets
therapy
• Consider warfarin effect, cirrhosis, Increased PT/INR • Consider ESRD and multifactor
hepatic failure, malnutrition Increased aPTT deficiency
Increased PT/INR Normal D-dimer • Give FFP and initiate directed therapy
• Provide IV vitamin K, FFP and
Normal aPTT Dabigatran-
prothrombin complex concentrate
(PCC) as needed induced • Initiate reversal with idarucizumab
• Consider heparin/lepirudin effect, coagulopathy
acquired factor deficiency, and von SUPPLEMENT: DISORDERS OF HEMOSTASIS
Normal PT/INR Willebrand disease (vWD) • Most common congenital bleeding disorder
Increased aPTT • Provide (protamine) to reverse heparin Von
• Type I – partial deficiency
Willebrand
• Replace factors as needed • Type II – qualitative defect
Disease
• Initiate directed therapy to vWD • Type III – total deficiency
TOPNOTCH MEDICAL BOARD PREP SURGERY MAIN DIGITAL HANDOUT BY LOUBOMIR ANTONIO, MD Page 7 of 101
For inquiries visit www.topnotchboardprep.com.ph or https://www.facebook.com/topnotchmedicalboardprep/
This handout is only valid for the September 2021 PLE batch. This will be rendered obsolete for the next batch since we update our handouts regularly.
TOPNOTCH MEDICAL BOARD PREP SURGERY MAIN DIGITAL HANDOUT BY LOUBOMIR ANTONIO, MD
For inquiries visit www.topnotchboardprep.com.ph or https://www.facebook.com/topnotchmedicalboardprep/
This handout is only valid for the September 2021 PLE batch. This will be rendered obsolete for the next batch since we update our handouts regularly.
Factor VIII • Most common intrinsic platelet defect
Storage pool
deficiency • Most common coagulation factor deficiency • Dense granule deficiency – most prevalent
disease
(Hemophilia A) form of storage pool disease

TRAUMA-INDUCED
COAGULOPATHY (TIC)
• Features of TIC
o Independent of traditional causes
of posttraumatic coagulopathy
(acidosis, hypothermia, dilution
of coagulation factors – classical
trauma triad of death)
o Precipitated by tissue injury
and/or hemorrhagic shock
o Associated with higher mortality
risk
o Distinct process from DIC
o Widespread use of damage control
resuscitation has decreased its
frequency

Adapted from Schwartz’s Principles of Surgery. 11th ed. 2019

✔ GUIDE QUESTION MANAGEMENT OF IMMEDIATE TRANSFUSION REACTION


A 75-year-old man is found to have prolonged bleeding from • Stop the transfusion
intravenous puncture sites. Platelet aggregation is inhibited by which o Initiate diuresis → alkalinize urine for hemoglobinuria
of the following? • Differentiate vs. Delayed reactions
(A) Adenosine diphosphate (ADP) (D) Aspirin
(B) Calcium (E) Serotonin
o 2 – 10 days after transfusion
(C) Magnesium o Extravascular hemolysis
o No specific intervention needed
✔ GUIDE QUESTION See table below for a summary of transfusion reactions.
A 24-year-old woman is scheduled for an elective cholecystectomy. Dr. Asperas

The best method of identifying a potential bleeder is which of the ADULT TRANSFUSION CLINICAL PRACTICE
following? SUPPLEMENT:
GUIDELINES (Highlights)
(A) Platelet count A. INITIAL TRANSFUSION OF RED BLOOD CELLS (RBCS):
(B) A complete history and physical examination 1. A blood sample must be sent to blood bank for a type and cross.
(C) Bleeding time 2. If the blood type is not known and blood is needed immediately, O-
(D) Prothrombin time (PT) negative RBCs should be issued.
The rest of the choices are important diagnostic tests for bleeding o As soon as possible, switch to O negative for females and O positive
but a complete history and PE is paramount and will make you for males.
suspect a possible bleeding disorder firsthand. o Type-specific uncross-matched - available in approximately 5–10 min.
o Completely cross-matched - available in approximately 40 min.
BLOOD TRANSFUSION 3. RBCs will be transfused in the standard fashion.
4. Patients who are unstable or receive 1–2 RBCs and do not rapidly
GENERAL INDICATIONS FOR TRANSFUSION respond should be considered candidates for the massive transfusion
1. Improvement in Oxygen Carrying Capacity of the Patient (MT) guideline
2. Treatment of Anemia (American Association of Blood Banks) B. ADULT MASSIVE TRANSFUSION GUIDELINE:
o 7 g/dL – minimum threshold for hemodynamically stable 1. The Massive Transfusion Guideline (MTG) should be initiated as soon
patients as it is anticipated that a patient will require massive transfusion (≥10
o 8 g/dL – minimum threshold for patients undergoing cardiac U RBCs in 24 h).
o The Blood Bank should strive to deliver plasma, platelets, and RBCs
surgery, orthopedic surgery, and with preexisting CVD in a 1:1:1 ratio.
3. Volume Replacement o To be effective and minimize further dilutional coagulopathy, the
o Most common indication for blood transfusion in surgical 1:1:1 ratio must be initiated early, ideally with the first 2 units of
patients is the replenishment of the blood volume transfused RBCs.
o Blood loss of up to 20% of total blood volume: Replaced with o Crystalloid infusion should be minimized!
crystalloid solution 2. Once initiated, the MT will continue until stopped by the attending
o Blood loss >20% of total blood volume: Addition of packed RBC, and physician. MT should be terminated once the patient is no longer
in the case of massive transfusion, the addition of FFP actively bleeding.
3. Basic laboratory tests should be drawn immediately
Suggested laboratory values are: CBC, INR, fibrinogen, pH and/or base
deficit, TEG, where available.

REACTIONS COMPLICATION SSx FREQUENCY MECHANISM PREVENTION


Febrile, Preformed cytokines Use leukocyte-reduced
0.5%–1.5% of
nonhemolytic Fever Host Ab to donor blood
transfusions
transfusion reaction lymphocytes Store platelets <5 d
NHTR High fever, chills
<0.01% of blood
(Non- Bacterial Hemodynamic changes Infusion of
<0.05% of
Hemolytic) contamination DIC, Emesis, Diarrhea contaminated blood
platelets
Hemoglobinuria
0.1%–0.3% of Soluble transfusion Provide antihistamine
Allergic reactions Rash, hives, Itching
units constituents prophylaxis
Transfusion
1:200–1:10,00 of Large volume of blood Increase transfusion time
associated
TACO Pulmonary edema transfused transfused into an older Administer diuretics
circulatory
patients patient with CHF Minimize associated fluids
overload
Anti-HLA or anti-HNA
Acute (<6 h) hypoxemia
Transfusion-related Ab in transfused blood
TRALI Bilateral infiltrates ± Limit female donors
acute lung injury attacks circulatory and
Tachycardia, hypotension
pulmonary leukocytes

TOPNOTCH MEDICAL BOARD PREP SURGERY MAIN DIGITAL HANDOUT BY LOUBOMIR ANTONIO, MD Page 8 of 101
For inquiries visit www.topnotchboardprep.com.ph or https://www.facebook.com/topnotchmedicalboardprep/
This handout is only valid for the September 2021 PLE batch. This will be rendered obsolete for the next batch since we update our handouts regularly.
TOPNOTCH MEDICAL BOARD PREP SURGERY MAIN DIGITAL HANDOUT BY LOUBOMIR ANTONIO, MD
For inquiries visit www.topnotchboardprep.com.ph or https://www.facebook.com/topnotchmedicalboardprep/
This handout is only valid for the September 2021 PLE batch. This will be rendered obsolete for the next batch since we update our handouts regularly.
REACTIONS COMPLICATION SSx FREQUENCY MECHANISM PREVENTION
Fever
Transfusion of ABO
Hypotension Transfuse
Hemolytic reaction, 1:33,000– incompatible blood
DIC, Hemoglobinuria appropriately
acute 1:1,500,000 units Preformed IgM Ab to
Hemoglobinemia matched blood
ABO Ag
Renal insufficiency
HTR Anemia
Indirect
Hemolytic reaction, Identify patient’s
hyperbilirubinemia
delayed IgG-mediated Ag to prevent
↓Haptoglobin level
(2–10 days) recurrence
Positive result on direct
Coombs test

WOUND HEALING
CLASSIFICATION OF WOUNDS
ACUTE VS CHRONIC WOUNDS
• Acute wound: healing achieved in <4 weeks
• Chronic wound: a wound beyond 4 weeks

TETANUS PRONE VS NON-TETANUS PRONE WOUNDS


CLINICAL NON-TETANUS
TETANUS PRONE
FEATURES PRONE
Age of wound • More than 6 hours • 6 hours of less Figure 9-4. Schwartz’s Principles of Surgery. 11th ed. 2019

• Stellate wound,
Configuration • Linear • Hemostasis precedes inflammation
avulsion, abrasion
• PMNs (peaks at 24-48 hours post-injury)
Depth • More than 1 cm • 1 cm or less
– first cells to infiltrate wound
• Sharp surface Hemostasis →
Mechanism of • Missile, crush, • Macrophages (peaks at 48-96 hours
(e.g., knife or Inflammation
injury burn, frostbite post-injury)
glass) (Day 0-6)
• T cells (peaks 1 week post-injury) –
Signs of infection • Present • Absent
bridges transition from inflammation to
Devitalized proliferative phase
• Present • Absent
tissue
• For establishment of tissue continuity
Contaminants • Present • Absent
Surgery Platinum. 1st ed. 2018
• Involves the invasion of fibroblasts and
Proliferation endothelial cells
PHASES OF HEALING (Day 4-12) • PDGF – strongest chemotactic factor for
fibroblasts
• Deposition of type III collagen fibers
Maturation and • Shift of type III collagen fibers → type I
Remodeling collagen fibers
(Day 8 – months) • Mediated by matrix metalloproteinases
• Final step in establishing tissue integrity
• Characterized by proliferation and
Epithelialization migration of epithelial cells
• Once layering of the epithelium is re-
established, surface will keratinize
• Caused by myofibroblasts due to the
Wound
presence of actin filaments allowing
contraction
“contraction”
Please take note of the events, cells involved, and duration per phase.
Dr. Rubio

Type of
Primary closure Secondary healing Tertiary healing or Delayed primary closure
wound
(primary intention) (secondary intention) (tertiary intention)
healing
• Requires clean tissue to be • Wound left open to heal by • initially managed as secondary intention (i.e.,
approximated without granulation tissue formation and left open with dressing changes); wound is
tension by sutures contraction closed after a few days when wound is clean and
• Includes wound closure by • Spontaneous wound closure depends granulation tissue is abundant
direct approximation, on contraction and epithelialization • Usually for traumatic injuries, dog bites, or
Description pedicle flap, or skin grafts • Contraction results from centripetal lacerations from foreign bodies
force in wound margin probably
provided by myofibroblasts
• beneficial in heavily contaminated
wounds (e.g., abscess after incision or
drainage, perineal wounds, perineum)
Surgery Platinum. 1st ed. 2018 p.75

Figure 9-6. Schwartz’s Principles of Surgery. 11th ed. 2019

TOPNOTCH MEDICAL BOARD PREP SURGERY MAIN DIGITAL HANDOUT BY LOUBOMIR ANTONIO, MD Page 9 of 101
For inquiries visit www.topnotchboardprep.com.ph or https://www.facebook.com/topnotchmedicalboardprep/
This handout is only valid for the September 2021 PLE batch. This will be rendered obsolete for the next batch since we update our handouts regularly.
TOPNOTCH MEDICAL BOARD PREP SURGERY MAIN DIGITAL HANDOUT BY LOUBOMIR ANTONIO, MD
For inquiries visit www.topnotchboardprep.com.ph or https://www.facebook.com/topnotchmedicalboardprep/
This handout is only valid for the September 2021 PLE batch. This will be rendered obsolete for the next batch since we update our handouts regularly.
OPERATIVE INFECTION
DESCRIPTION EXAMPLE CASES
WOUND CLASS RATES
• Atraumatic, uninfected • Breast surgery (MRM)
• No entry of GU, GI or respiratory tract • Thyroid surgery
Class I-
1-2% • Primarily closed • Inguinal hernia
CLEAN
• If necessary, drained with closed drainage (e.g. JP • Operative incisional wounds that follow non-
drain) penetrating (blunt) trauma
2.1-9.5%
(Elective GI • Wound involving normal but colonized tissue
Class II- • Operations involving biliary tract, appendix, vagina,
Surgery) • Minor breaks in sterile technique
CLEAN- and oropharynx (provided no evidence of infection
4-14% • Entry of GU, GI, or respiratory tract without
CONTAMINATED or major break in technique is encountered)
(Colorectal significant change
Surgery)
• Operations with major breaks in sterile technique
• Wound containing foreign or infected material
(e.g. open cardiac massage)
• Traumatic wounds
Class III- • Incisions in which acute, non-purulent
3.4-13.2% • Open, fresh, accidental wounds
CONTAMINATED inflammation is encountered including necrotic
• Gross spillage from GI tract
tissue without evidence of purulent drainage (e.g.
• Entry into infected tissue, bone, urine, or bile
dry gangrene)
• Old traumatic wounds with retained devitalized
tissue and those that involve existing clinical • Drainage of abscess
Class IV-
3.1-12.8% infection or perforated viscera. • Debridement of soft tissue infection
DIRTY
• Organisms causing postoperative infection were • Resection of infarcted bowel
present in the operative field before operation
FACTORS IMPAIRING WOUND HEALING • > 50,000/mm3 – active bleeding
LOCAL FACTORS SYSTEMIC FACTORS Mechanical • Target TV = 6mL/Kg and Max plateau
• Infection • Malnutrition Ventilation pressure <30 cmH2O
• Foreign bodies • Cancer Glucose control • Target glucose: <18O mg/dL
• Ischemia/hypoxia • Diabetes mellitus • Intermittent HD (Stable) and Continuous
Renal replacement
HD (Unstable)
• Venous insufficiency • Uremia
• Prophylaxis for VTE
• Toxins (e.g. spider venom) • Jaundice
Prophylaxis • Stress ulcers
• Previous trauma • Old age
• VAP
• Radiation • Systemic corticosteroids
• Cigarette smoking • Chemotherapeutic agents ✔ GUIDE QUESTION
• Alcoholism Following an infection, a 68-year-old woman developed gram-negative
septicemia. Which statement is true for gram-negative bacterial
septicemia?
SEPSIS AND SEPTIC SHOCK (A) Pseudomonas is the most common organism isolated.
SIRS, SEPSIS, SEVERE SEPSIS, SEPTIC SHOCK (B) Adverse changes can be accounted for lipid A release.
(C) The cardiac index is low.
• SIRS criteria: (2 or more of the following) (D) Central venous pressure (CVP) is high.
o Temp ≥ 38°C or ≤ 36°C (E) It usually follows severe pulmonary infections
o HR ≥ 90 beats/min
o RR ≥ 20/min or PaCO2 ≤ 32mmHg or mechanical ventilation SUPPLEMENT: QUICK SOFA
o WBC ≥ 12,000/μL or ≤ 4,000/μL or ≥ 10% bands • Used to quickly assess patients without the use of laboratory
parameters
SUPPLEMENT: SEPSIS DEFINITION
• Score ≥2 warrants further investigation of organ dysfunction and/or
Sepsis escalate therapy
• Life-threatening “organ dysfunction” due to a dysregulated host o RR >22/min (1point)
response to infection o Altered mental status (or GCS <15) (1point)
• Definition of organ dysfunction: increase in Sequential (Sepsis- o SBP < 100mmHg (1point)
Related) Organ Failure Assessment (SOFA) score of ≥ 2 points Singer M, et al. 2016. The Third International Consensus Definitions for Sepsis and
Septic Shock (Sepsis-3). JAMA. 2016 February 23; 315(8): 801–810. doi:10.1001/jama.2016.0287.
Septic Shock
• Subset of sepsis where underlying circulatory and cellular/metabolic
abnormalities are profound enough to substantially increase SURGICAL SITE INFECTIONS
mortality
• are infections of the tissues, organs, or spaces exposed by
surgeons during performance of an invasive procedure
• Clinical criteria identifying such condition include:
o Need for vasopressors to obtain a MAP ≥ 65mmHg despite
adequate fluid resuscitation
o Increase in lactate concentration > 2 mmol/L in the absence of
hypovolemia
Singer M, et al. 2016. The Third International Consensus Definitions for Sepsis and
Septic Shock (Sepsis-3). JAMA. 2016 February 23; 315(8): 801–810. doi:10.1001/jama.2016.0287.

MANAGEMENT
• 30mL/Kg IV crystalloids in first 3 hours
Initial resuscitation
• Target MAP = > 65mmHg
diagnostics
• Cultures before antibiotics
• Empiric broad-spectrum IV Antibiotics
Antibiotics
then culture-guided antibiotics
Source control • Emergent surgery, drainage, evacuation
• Norepinephrine – 1st choice
• Vasopressin/Epinephrine
Vasopressors
• Dopamine or Dobutamine
• GOAL: MAP > 65mmHg
• Unresponsive septic shock
Steroids
• IV Hydrocortisone 200mg/day
• pRBC for Hgb < 7g/dL
• Platelet transfusion
Blood transfusion
• < 10,000/mm3 – even without bleeding Chopra et al., 2010. Adapted from CDC National Nosocomial Surveillance System SSI: Surgical site infection
• < 20,000/mm3 - high risk for bleeding
TOPNOTCH MEDICAL BOARD PREP SURGERY MAIN DIGITAL HANDOUT BY LOUBOMIR ANTONIO, MD Page 10 of 101
For inquiries visit www.topnotchboardprep.com.ph or https://www.facebook.com/topnotchmedicalboardprep/
This handout is only valid for the September 2021 PLE batch. This will be rendered obsolete for the next batch since we update our handouts regularly.
TOPNOTCH MEDICAL BOARD PREP SURGERY MAIN DIGITAL HANDOUT BY LOUBOMIR ANTONIO, MD
For inquiries visit www.topnotchboardprep.com.ph or https://www.facebook.com/topnotchmedicalboardprep/
This handout is only valid for the September 2021 PLE batch. This will be rendered obsolete for the next batch since we update our handouts regularly.
1. SUPERFICIAL INCISIONAL SSI
PRIMARY SURVEY: IMMEDIATE MANAGEMENT
• Infection occurs within 30 days after an operative procedure
OF LIFE-THREATENING INJURIES
• AND Involves only skin and subcutaneous tissue of the incision
AIRWAY BREATHING
• AND Patient has at least one of the following:
• Tension pneumothorax
o Purulent drainage from the superficial incision • Airway obstruction • Open pneumothorax
o Organisms isolated from an aseptically-obtained culture of • Airway injury • Flail chest with underlying pulmonary
fluid or tissue from the superficial incision contusion
o At least one of the following signs or symptoms: CIRCULATION
§ Pain or tenderness; • Massive hemothorax or hemoperitoneum
§ Localized swelling; redness; or heat. Hemorrhagic shock • Mechanically unstable pelvis fracture
§ Superficial incision that is deliberately opened by a surgeon • Extremity blood loss
unless incision culture is negative • T. pneumothorax
o Diagnosis of a superficial incisional SSI by the surgeon • Cardiac tamponade
• Do not report the ff. conditions as SSI Cardiogenic shock
• Blunt cardiac injury, MI
o Stitch abscess (minimal inflammation and discharge confined • Bronchovenous air embolism
to the points of suture penetration) Neurogenic shock
o Infection of an episiotomy or newborn circumcision site DISABILITY
Intracranial hemorrhage/mass
o Infected burn wound lesion/Cervical spine injury
o Incisional SSI that extends into the fascial and muscle layers completely undress the patient, usually by
EXPOSURE/
cutting off his or her garments to facilitate a
2. DEEP INCISIONAL SSI ENVIRONMENTAL
thorough examination and assessment.
• Infection occurs within 30 or 90 days after an operative
procedure if no implant is left in place or within 1 year if AIRWAY
implant is in place AIRWAY MAINTENANCE
• AND involves deep soft tissues of the incision (e.g., fascial and WITH CERVICAL SPINE IMMOBILIZATION
muscle layers • Securing the airway is the first priority in the primary survey
• AND Patient has at least one of the following: • Must ensure patency of airway and consider airway protection
o Purulent drainage from the deep incision but not from the o Visual inspection: airway cleared of any debris, blood and
organ/space component of the surgical site foreign bodies
o Deep incision that spontaneously dehisces or is deliberately o Administration of oxygen as needed
opened by a surgeon when the patient has at least one of the o Application of hard cervical collar or sandbags on both sides of
following signs or symptoms: the neck to immobilize neck (except with penetrating neck
§ Fever (>38°C); trauma)
§ Localized pain or tenderness • Manual airway maneuvers (to elevate the tongue out of the
§ Unless culture site is negative hypopharynx)
o An abscess or other evidence of infection involving the deep o Jaw thrust (Esmarch
incision that is found on direct examination, during maneuver)
reoperation, or by histopathologic or radiologic examination o Chin lift
o Diagnosis of a deep incisional SSI by a surgeon or attending o Head tilt is
physician. inappropriate with
3. ORGAN/SPACE SSI cervical spine
control
• Infection occurs within 30 days after the operation if no
implant is left in place or within 1 year if implant is in place INDICATIONS FOR
• AND Infection involves any part of the body (e.g. organs or EMERGENCY ENDOTRACHEAL INTUBATION
spaces) other than the incision, which was opened or 1. Acute airway obstruction
manipulated during an operation 2. Hypoventilation
• AND at least one of the following: 3. Severe hypoxemia despite supplemental oxygen
o purulent drainage from a drain that is placed through a stab 4. Altered mental status (e.g., GCS < 8)
wound into the organ/space 5. Cardiac arrest
o Organisms isolated from an aseptically-obtained culture of 6. Severe hemorrhagic shock
fluid or tissue in the organ/space
o an abscess or other evidence of infection involving the OPTIONS FOR ENDOTRACHEAL INTUBATION
organ/space that is found on direct examination, during OPTION ADVANTAGE DISADVANTAGE
reoperation, or by histopathologic or radiologic examination • Applicable in
o Diagnosis of an organ/space SSI by a surgeon or attending patients with
physician • Requires
apnea
A. Orotracheal neuromuscular
• Direct visualization
PARENTERAL ANTIBIOTICS RECOMMENDED FOR (preferred) blockade among
of the vocal cord
SURGICAL PROPHYLAXIS • Ability to use large-
conscious patients
• Preoperative dose timing for administration is: diameter tubes
o Within 60 minutes before incision (120 minutes for vancomycin) • Not applicable in
• Most cases can be treated with a single dose apnea, severe
• Applicability in
• Redosing: if duration of the surgery exceeds 2 half-lives of the pre-hospital setting
midface trauma
antimicrobial or there is excessive blood loss, intraoperative B. and or suspicion of
• May be inserted
redosing is needed to ensure adequate concentrations of the Nasotracheal basilar skull
without a
antimicrobial fracture (CSF
laryngoscope
rhinorrhea, racoon
TRAUMA AND BURNS eyes, Battle’s sign)
TRAUMA DEFINITION • Risk of subglottic
• First airway
stenosis if
• cellular disruption caused by an exchange with environmental C. Crico- control maneuver
performed among
energy that is beyond the body’s resilience which is thyroidotomy in cases of
children (less than
compounded by cell death due to ischemia / reperfusion craniofacial trauma
11 years)
Advanced Trauma Life Support (ATLS) by the American College • Last resort (best
of Surgeons provides a structured approach to trauma: performed in OR)
• Primary survey (ABCDE) / Concurrent resuscitation D. • Applicable in cases
• Requires more skill
• Secondary survey (AMPLE) / Diagnostic evaluation Tracheostomy of laryngeal fractures
• Definitive care • Preferred for those
• Tertiary survey under age 11 years
TOPNOTCH MEDICAL BOARD PREP SURGERY MAIN DIGITAL HANDOUT BY LOUBOMIR ANTONIO, MD Page 11 of 101
For inquiries visit www.topnotchboardprep.com.ph or https://www.facebook.com/topnotchmedicalboardprep/
This handout is only valid for the September 2021 PLE batch. This will be rendered obsolete for the next batch since we update our handouts regularly.
TOPNOTCH MEDICAL BOARD PREP SURGERY MAIN DIGITAL HANDOUT BY LOUBOMIR ANTONIO, MD
For inquiries visit www.topnotchboardprep.com.ph or https://www.facebook.com/topnotchmedicalboardprep/
This handout is only valid for the September 2021 PLE batch. This will be rendered obsolete for the next batch since we update our handouts regularly.
A B • Treatment: presumptive intubation and mechanical
ventilation (PEEP)

MASSIVE HEMOTHORAX
• Collection of blood in the space between the chest wall and the
lung (the pleural cavity)

MAJOR AIR LEAK DUE TO TRACHEOBRONCHIAL INJURY


• Type I – within 2 cm of carina
• Type II – almost always accompanied by pneumothorax
• Diagnosis and management: bronchoscopy

✔ GUIDE QUESTION
A 70-year-old man is brought into the emergency department following
his injury as a passenger in a car crash. He complains of right-side chest
pain. Physical examination reveals a respiratory rate of 42 breaths per
C D minute and multiple broken ribs of a segment of the chest wall that
moves paradoxically with respiration. What should the next step be?
BREATHING AND VENTILATION (A) Tube thoracostomy (D) Endotracheal intubation
• Check respiratory rate and oxygen saturation (B) Tracheostomy + PEEP (E) Intercostal nerve blocks
• Inspect for external signs of trauma and asymmetric chest (C) Needle Thoracostomy
movements This is a sample case of a flail chest injury. Prompt intubation is needed
to secure the airway.
• Palpate chest wall (e.g., crepitus, surgical emphysema, deviation
of trachea)
• Auscultate CIRCULATION AND CONTROL OF HEMORRHAGE
INADEQUATE VENTILATION • Look for evidence of bleeding: chest, abdomen, retroperitoneum,
TENSION PNEUMOTHORAX pelvis, long bones, and external bleeding
• Adequacy of circulation is assessed rapidly by evaluating the
• Respiratory distress + hypotension in combination with any of
pulse, color of skin, and capillary refill
the following physical signs in patients with chest trauma:
o Palpable carotid pulse: SBP > 60 mmHg
tracheal deviation away from the affected side, lack of or
o Palpable femoral pulse: SBP > 70 mmHg
decreased breath sounds on the affected side, & subcutaneous
o Palpable radial pulse: SBP > 80 mmHg
emphysema on the affected side
• Treatment: Immediate needle thoracostomy decompression
in the 4th or 5th ICS MAL (Adult) or 2nd ICS MCL (Child)
• Closed tube thoracostomy should be performed immediately
before a chest radiograph is obtained
SUPPLEMENT: THORACOSTOMY
4th or 5th ICS MAL

Figure 7-3. Schwartz’s Principles of Surgery. 11th ed. 2019

OPEN PNEUMOTHORAX (SUCKING CHEST WOUND)


APPROACH TO SHOCK
• Occurs with full thickness loss of the chest wall, permitting free • Hemorrhagic (ie. blood loss)
communication between the pleural space & the atmosphere • Cardiogenic (eg. blunt cardiac injury, ACS)
• Compromises ventilation due to equilibration of atmospheric Differentials
• Obstructive (e.g. tension pneumothorax, cardiac
and pleural pressures, which prevents lung inflation and for Shock
tamponade)
alveolar ventilation, and results in hypoxia and hypercarbia • Neurogenic
• Complete occlusion of the chest wall defect WITHOUT a tube • 20 mL/kg intravenous bolus isotonic crystalloid
thoracostomy may convert an open pneumothorax to a tension Initial Fluid (plain lactated ringer’s)
pneumothorax Resuscitation • Assessment of response to fluid resuscitation
• Initial treatment: temporary occlusion of the wound at its 3 • Facilitate blood transfusion as needed
sides (convert to a one-way valve) • Temporize with direct pressure, tourniquets, or
Hemorrhage
tying blood vessels
• Definitive treatment: closure of the chest wall defect and Control
• Surgical intervention
closed tube thoracostomy remote from the wound
SUPPLEMENT: CLASSIFICATION OF HEMORRHAGE
FLAIL CHEST WITH UNDERLYING PULMONARY CONTUSION (Schwartz 11th Edition)
• Occurs when 3 or more contiguous ribs are fractured in at CLASS I CLASS II CLASS III CLASS IV
least 2 locations Blood loss Up to 1500-
750-1500 >2000
(mL) 750 2000
• Paradoxical movement of this free floating segment of chest
Blood loss Up to
wall may be evident in patients with spontaneous ventilation, (% BV) 15%
15%-30% 30%-40% >40%
due to the negative intrapleural pressure of inspiration Pulse rate <100 >100 >120 >140
• Associated pulmonary contusion is typically the source of Blood
Normal Normal Decreased Decreased
postinjury pulmonary dysfunction (Decreased compliance and pressure
increased shunt fraction)
TOPNOTCH MEDICAL BOARD PREP SURGERY MAIN DIGITAL HANDOUT BY LOUBOMIR ANTONIO, MD Page 12 of 101
For inquiries visit www.topnotchboardprep.com.ph or https://www.facebook.com/topnotchmedicalboardprep/
This handout is only valid for the September 2021 PLE batch. This will be rendered obsolete for the next batch since we update our handouts regularly.
TOPNOTCH MEDICAL BOARD PREP SURGERY MAIN DIGITAL HANDOUT BY LOUBOMIR ANTONIO, MD
For inquiries visit www.topnotchboardprep.com.ph or https://www.facebook.com/topnotchmedicalboardprep/
This handout is only valid for the September 2021 PLE batch. This will be rendered obsolete for the next batch since we update our handouts regularly.
CLASS I CLASS II CLASS III CLASS IV
Pulse Normal
The preferred test in
pressure or Decreased Decreased Decreased
(mmHg) increased
the diagnosis of
Respiratory cardiac tamponade
14-20 >20-30 30-40 >35 • Echocardiography
rate
Urine
output >30 >20-30 5-15 Negligible
(mL/h) ✔ GUIDE QUESTION
Anxious Confused A patient presents with stable vital signs and respiratory distress after a
CNS/mental Slightly Mildly stab wound to the chest. Chest tubes are placed, and an air leak is noted.
and and
status anxious anxious The patient is electively intubated. The patient arrests after positive
confused lethargic
BV = blood volume; CNS = central nervous system pressure ventilation is started. What is the most likely diagnosis?
How I Remember it: (A) Unrecognized hemorrhage in the abdomen
• Average man has ~5L blood (B-L-O-O-D has 5 Letters!) (B) Tension pneumothorax
• CLASS II (C) Pericardial tamponade
o (Imagine 1.5L bottle)– blood loss >1.5L: AT LEAST CLASS II! (D) Air embolism
o With tachycardia (>100bpm)? Dyspnea (>20/min): AT LEAST SUPPLEMENT: AIR EMBOLI
CLASS II! • can occur after blunt or penetrating trauma
• CLASS III • air from an injured bronchus enters an adjacent injured pulmonary
o With hypotension? Decreased UO? AT LEAST CLASS III! vein (bronchovenous fistula) and returns air to the left heart
o “R-B-C” – three letters: Transfuse pRBC if at least CLASS III!
• air accumulation in the left ventricle impedes diastolic filling, and
*Two classifications in a single textbook. Almost the same idea, but
during systole air is pumped into the coronary arteries, disrupting
different cut off values. Personally, I prefer to memorize the
coronary perfusion
Classification in Trauma
Dr. Asperas • Management:
o Patient placed in Trendelenburg’s position to trap the air in the
✔ GUIDE QUESTION apex of the left ventricle.
A 30-year-old man is brought to the emergency department following a o Emergency thoracotomy is followed by cross clamping of the
high-speed car accident. He was the driver, and the windshield of the pulmonary hilum on the side of the injury to prevent further
car was broken. On examination, he is alert, awake, oriented, and in no introduction of air.
respiratory distress. He is unable to move any of his four extremities; o Air is aspirated from the apex of the left ventricle and the aortic root
however, his extremities are warm and pink. His vital signs on with an 18-g needle and 50-ml syringe.
admission are HR 54 bpm and BP70/40 mm Hg. What is the diagnosis? o Vigorous massage is used to force air bubble through the coronary
(A) Hemorrhagic shock (C) Neurogenic shock arteries. If unsuccessful, a tuberculin syringe may be used to
(B) Cardiogenic shock (D) Septic shock aspirate air from the right coronary artery.
Classic description of neurogenic shock: paralysis, warm and pink o Patient should be kept in Trendelenburg’s with the pulmonary hilum
extremities, bradycardia, hypotension. clamped until pulmonary venous injury is controlled operatively
INADEQUATE CIRCULATION DISABILITY AND NEUROLOGIC STATUS
• >1,500 ml of blood or, in the pediatrics, >25% • Assess the GCS
of the patient’s blood volume in the pleural • Assess pupillary size and responsiveness, motor & sensory
space
function, glucose level, etc.
• Blunt trauma: usually due to multiple rib
• Consider spinal injury in appropriate cases (e.g., priapism, loss
fractures with severed intercostal arteries, but
Massive of anal sphincter tone and bulbocavernosus reflex)
occasionally bleeding is from lacerated lung
Hemothorax
parenchyma 1. Glasgow Coma Scale (GCS)
• Penetrating trauma: a great vessel or EYE
MOTOR RESPONSE VERBAL OUTPUT
pulmonary hilar vessel injury OPENING
• Treatment: Operative intervention (after a 6 Obeys commands
CTT) 5 Localizes pain Oriented
• Occurs most commonly after penetrating 4 Withdraws from pain Confused Spontaneous
thoracic injuries, although occasionally blunt Abnormal flexion Inappropriate
3 To voice
rupture of the heart, particularly the atrial (decorticate) words
appendage, is seen Abnormal extension Incomprehensible
2 To pain
• <100 ml of pericardial blood may cause (decerebrate) sounds
pericardial tamponade 1 None None None
• Beck’s triad (dilated neck veins, muffled
2. Pediatric Glasgow Coma Scale
heart tones, and a decline in arterial
EYE
pressure) -rare MOTOR RESPONSE VERBAL OUTPUT
OPENING
• Diagnosis: 2decho
Spontaneous,
• Early in the course of tamponade, blood 6
purposeful
pressure and cardiac output will transiently
Alert, normal
improve with fluid administration 5 Localizes pain
vocalization
• Treatment: Pericardiocentesis- insertion of
Cries but
wide-bore needle to aspirate pericardial fluid; 4 Withdraws from pain Spontaneous
Cardiac consolable
successful in decompressing tamponade in ~
Tamponade 80% of cases; vs Subxiphoid pericardial Abnormal flexion Persistently
3 To voice
window- involves the excision of a portion of (decorticate) irritable
the pericardium, which allows the effusion to Abnormal extension Restless, agitated,
2 To pain
drain (decerebrate) moaning
• Removing as little as 15 to 20 ml of blood will 1 None None None
often temporarily stabilize the patient’s Formula to compute GCS if the patient is intubated or unable
hemodynamic status, prevent subendocardial to verbalize:
ischemia, and associated lethal arrhythmias, • GCS = E + DVS + M
and allow transport to the OR for sternotomy o E – Eye response, M – Motor response,
• Patients with an SBP <60 mmHg warrant DVS – Derived verbal response = M (0.5) + E (0.4)
emergency department thoracotomy (EDT) SUPPLEMENT: GCS-P SCORE
with opening of the pericardium to address the
• Developed to improve the ability to predict patient prognosis
injury
• Pupil Reactivity Score (number of non-reactive pupils)
• EDT is best accomplished using a left Mortality
anterolateral thoracotomy, with the incision o both pupils unreactive = 2 59%
started to the right of the sternum o only one pupil unreactive = 1 38%
o both pupils reacted = 0 16%
• GCS-P score = GCS – P (range = 1 to15)

TOPNOTCH MEDICAL BOARD PREP SURGERY MAIN DIGITAL HANDOUT BY LOUBOMIR ANTONIO, MD Page 13 of 101
For inquiries visit www.topnotchboardprep.com.ph or https://www.facebook.com/topnotchmedicalboardprep/
This handout is only valid for the September 2021 PLE batch. This will be rendered obsolete for the next batch since we update our handouts regularly.
TOPNOTCH MEDICAL BOARD PREP SURGERY MAIN DIGITAL HANDOUT BY LOUBOMIR ANTONIO, MD
For inquiries visit www.topnotchboardprep.com.ph or https://www.facebook.com/topnotchmedicalboardprep/
This handout is only valid for the September 2021 PLE batch. This will be rendered obsolete for the next batch since we update our handouts regularly.

MANAGEMENT OF DISABILITY measure for patients in impending herniation while awaiting


definitive surgical management.
• Airway maintenance
• If ICP with monitoring: ICP should be maintained <20 mmHg.
• Supportive: seizure control, treat hypoglycemia
CPP between 50-70 mmHg is desirable.
• Treat increased intracranial pressure (e.g., sedation, mannitol,
• In patients with severe head injury, prophylactic antiepileptic
hypertonic saline, surgical decompression)
drugs reduce the incidence of early seizures, defined as those
occurring during the first 7 days after head injury.
NEUROSURGICAL TRAUMA
• Even without neuroimaging, the following trauma patients
will need neurosurgical evaluation:
o Impaired sensorium
o Abnormal pupils
o Focal neurologic signs that suggest impending herniation
o Open skull fractures
o Suspected closed depressed skull fracture
o Suspected basal skull fracture
o Battle sign: ecchymosis behind the ear
o Raccoon eyes: periorbital ecchymosis
o Suspected CSF leak: Rhinorrhea, otorrhea, postnasal drip
• Halo sign: dried blood surrounded by a halo of dried CSF
• Gold standard: CSF is (+) for β2 transferrin

CORE PRINCIPLES OF HEAD TRAUMA MANAGEMENT


• Avoid hypotension (systolic BP <90 mm Hg) and hypoxemia
(pO2 <60 mm Hg and O2 sats <90%).
• Hyperventilation to lower pCO2 and decrease cerebral blood
flow (hence, lower ICP) is only recommended as a temporizing
PATHOPHYSIOLOGY MANIFESTATIONS AND DIAGNOSIS MANAGEMENT
Acute Epidural Hematoma (EDH)
• Lentiform and hyperdense hematoma in • Classic “lucid interval” followed by • EDH >30 mL in volume requires craniotomy
between the inner table of the skull and the dura sudden deterioration observed only in and evacuation, regardless of GCS
• Limited by suture lines 1 out of 5 patients
• Most commonly from ruptured middle • CT Scan: lenticular-shaped
meningeal artery hematoma
Acute Subdural Hematoma
• Crescentic and hyperdense hematoma in • CT Scan: crescent-shaped, curved • SDH >10 mm thick or with midline shift >5
between the dura and the arachnoid covering hematoma mm requires craniotomy and evacuation,
the brain regardless of GCS
• Not limited by suture lines, follows the • Underlying cerebral edema might warrant
convexity of the brain decompressive craniectomy
• Bleeding usually from torn bridging veins
Chronic Subdural Hematoma
• Crescentic and hypodense (liquefied) • Common in elderly (atrophic brain) • Requires burr hole and drainage of liquefied
hematoma and alcoholics (prone to head trauma blood clot (“motor oil” like fluid) with hydration
• Usually associated with significant mass effect and with hepatic coagulopathy) to allow brain expansion
Contusion and Parenchymal Hemorrhages
• Contusions represent focal regions of brain • These are evolving lesions that may • Surgical evacuation is recommended for focal
edema and are hypodense while acute progress and thus require serial lesions with significant mass effect while
parenchymal hematomas are hyperdense imaging and close clinical monitoring decompressive craniectomy is an option for
• May have an associated overlying acute SDH patients with medically refractory cerebral
(“burst lobe”) edema
Skull Fractures
• Differentiate from suture lines by knowledge • Best seen in bone window images of • Open skull fractures require debridement and
of anatomy and comparing with normal cranial CT, ideally with 3D antibiotics to prevent infection
contralateral side reconstruction • Inspect for CSF leak and subsequent meningitis
• Depressed skull fractures have to be elevated if
the depression is more than the thickness of the
calvarium, or if cosmetically indicated
Traumatic subarachnoid hemorrhage
• Hyperdensities in the subarachnoid space • Most common finding in fatal head
(cisterns and cortical sulci) trauma
• Massive subarachnoid hemorrhage most
commonly from rupture of the vertebral artery
Diffuse axonal injury
• Widespread shearing of axons due to forces • Suspected in patient with depressed • Medical management as per severe head injury
with severe angular acceleration- sensorium that is disproportionate to guidelines
deceleration findings in Cranial CT
• MRI may show punctate hemorrhages
in the grey-white matter junction,
corpus callosum, and brainstem
Surgery Platinum. 1st ed. 2018 p458-459
ACUTE SPINAL CORD INJURYAMERICAN SPINAL INJURY
ASSOCIATION (ASIA) IMPAIRMENT SCALE • Motor function is preserved below the
C
GRADE DESCRIPTION neurologic level; most key muscles below the
INCOMPLETE
A • No motor or sensory function is preserved in neurologic level have a muscle grade <3
COMPLETE sacral segments S4-5 • Motor function is preserved below the
D
neurologic level; most key muscles below the
• Sensory but no motor function is preserved INCOMPLETE
B neurologic level have a muscle grade ≥3
below the neurologic level and extends through
INCOMPLETE E
the sacral segment S4-5 • Motor and sensory function is intact
NORMAL
TOPNOTCH MEDICAL BOARD PREP SURGERY MAIN DIGITAL HANDOUT BY LOUBOMIR ANTONIO, MD Page 14 of 101
For inquiries visit www.topnotchboardprep.com.ph or https://www.facebook.com/topnotchmedicalboardprep/
This handout is only valid for the September 2021 PLE batch. This will be rendered obsolete for the next batch since we update our handouts regularly.
TOPNOTCH MEDICAL BOARD PREP SURGERY MAIN DIGITAL HANDOUT BY LOUBOMIR ANTONIO, MD
For inquiries visit www.topnotchboardprep.com.ph or https://www.facebook.com/topnotchmedicalboardprep/
This handout is only valid for the September 2021 PLE batch. This will be rendered obsolete for the next batch since we update our handouts regularly.
INCOMPLETE SPINAL CORD SYNDROMES SECONDARY SURVEY
• Incomplete spinal cord syndromes are caused by lesions of the
• Carried out after the primary survey
ascending or descending spinal tracts that result from trauma,
rapid, systematic, and head-to-toe examination approach to the
spinal compression, or occlusion of spinal arteries
injured patient to identify other injuries missed in the focused
• A spine MRI is the diagnostic modality of choice to determine the
primary survey
etiology, level, and extent of the lesion
• Aside from Hx and PE, it includes special procedures such as
• Spinal compression is a medical emergency and requires urgent
radiography, laboratory tests, scans, and peritoneal lavage
treatment with steroids and decompressive surgery
• Useful mnemonic – A-M-P-L-E
o A: Allergies
INCOMPLETE CORD SYNDROMES
o M: Medications
SYNDROME CLINICAL PRESENTATION
o P: Past medical history
• Motor weakness BELOW LESION
o L: Last meal
Anterior • Impaired pain and temperature sense BELOW
o E: Events and environment related to injury
cord LESION
syndrome • Results from hyperflexion
✔ GUIDE QUESTION
• Poorest prognosis
A 56-year-old man was stabbed in the right lower quadrant of his
Posterior • Impaired proprioception and vibration sense abdomen. He complains of pain at the wound site. His vital signs are
cord BELOW LESION normal and the findings from his abdominal examination are normal.
syndrome • Rare phenomenon Local wound exploration reveals penetration of the anterior fascia. and
• Weakness: UPPER EXTREMITIES > LOWER DPL reveals 7000 RBCs/mm3 and 750 WBC/mm3. Which of the
EXTREMITIES following is the most appropriate next step?
Central
• Cape-like pattern of numbness in the upper back A. Repeat the DPL in 4 hours
cord B. Obtain an abdominal CT scan
• Impaired bowel and bladder control
syndrome C. Perform a laparoscopy
• Results from hyperextension in the setting of
preexisting spinal canal stenosis D. Perform exploratory laparotomy
Indication for EL in this case is WBC >500 in DPL.
• IPSILATERAL motor weakness below the lesion
SUPPLEMENT: POSITIVE DPL
• IPSILATERAL impairment of proprioception and
vibration sense below the lesion Anterior
Brown- Anterior Thoracoabdominal
• CONTRALATERAL impaired pain and Abdominal
Séquard Abdominal Stab Wound
temperature sense below the lesion Stab Wound
syndrome
• Seen in penetrating spine injuries or epidural Red blood cell
>100,000/ml >10,000/ml
hematoma (RBC) count
• Best prognosis White blood cell
>500/ml
• Injuries below the L1 LEVEL (WBC) count
Cauda • Gradual and unilateral
equina • Results in a LOWER MOTOR NEURON LESION Amylase level >19 IU/l
syndrome • Results in flaccidity, areflexia, impairment of Alkaline
>2 IU/l
bowel & bladder function (late) phosphatase level
• Causes severe back pain Bilirubin level >0.01 mg/dl
Conus
• Sudden and bilateral (symmetric) • DPL RBC Between 1,000 and 10,000 – do laparoscopy/thoracoscopy
medullaris
• Results in impotence, hyperreflexia, impairment
syndrome
of bowel & bladder function (early)
PENETRATING ABDOMINAL ORGAN INJURIES

Adapted from Schwartz’s Principles of Surgery. 11th ed. 2019

TOPNOTCH MEDICAL BOARD PREP SURGERY MAIN DIGITAL HANDOUT BY LOUBOMIR ANTONIO, MD Page 15 of 101
For inquiries visit www.topnotchboardprep.com.ph or https://www.facebook.com/topnotchmedicalboardprep/
This handout is only valid for the September 2021 PLE batch. This will be rendered obsolete for the next batch since we update our handouts regularly.
TOPNOTCH MEDICAL BOARD PREP SURGERY MAIN DIGITAL HANDOUT BY LOUBOMIR ANTONIO, MD
For inquiries visit www.topnotchboardprep.com.ph or https://www.facebook.com/topnotchmedicalboardprep/
This handout is only valid for the September 2021 PLE batch. This will be rendered obsolete for the next batch since we update our handouts regularly.
BLUNT ABDOMINAL ORGAN INJURIES

Adapted from Schwartz’s Principles of Surgery. 11th ed. 2019

SPECIFIC ABDOMINAL ORGAN INJURIES


ORGAN MANAGEMENT OPTIONS
• Nonoperative: simple non-bleeding lacerations
• Direct suture: shallow slowly oozing injuries
LIVER • Hepatotomy or tractotomy: tracing and unroofing of the wound and bleeding sites directly ligated
(most commonly • Pringle maneuver: intermittent clamping of the hepatoduodenal ligament at the epiploic foramen in cycles (up to 10-15
injured intra- minutes)
abdominal • Selective hepatic artery ligation: either left or right hepatic artery may be ligated following a positive Pringle’s maneuver
organ in blunt (bleeding stops while the hepatoduodenal ligament is clamped and resumes when released)
abdominal • Resection or debridement: for extensive injury & devitalized tissues
trauma) • Atriocaval shunt: for significant hemorrhage likely due to major hepatic venous or retrohepatic vena caval injury (i.e. does
not stop with Pringle technique)
• Perihepatic packing: performed as damage control (unstable patients who are coagulopathic, acidotic and hypothermic)
SPLEEN • Splenorrhaphy: stable patients without associated multiple injuries
(second most • Splenectomy: hemodynamic instability, multiple associated injuries, hilar injuries, completely shattered parenchyma
frequently • Auto-transplantation in younger patients
injured organ in • Nonoperative: hemodynamically stable
blunt abdominal • Post-splenectomy measures: Beyond post-op Day 5 – WBC > 15,000 and Platelet/WBC ratio <20 – associated with sepsis
trauma) (consider sub-phrenic abscess)
• Simple repair and drainage: minor pancreatic lacerations or contusions
• Pyloric exclusion or diverticularization: repair of the duodenum, drainage, and closure of the pylorus with diversion of
gastric contents through a gastrojejunostomy
PANCREAS
• Pancreaticoduodenal resection: only for severe injuries to the head of the pancreas and the duodenum (rare)
• Distal pancreatectomy: distal pancreatic injuries with major ductal involvement
• Damage control surgery and postoperative ERCP: unstable patients
• Involves initial hemostasis and subsequent repair or resection
• Assume even number of wounds (each entry wound has an associated exit wound for GSW)
• Mesenteric border of the bowel, the retroperitoneal duodenum and colon, the cardia, posterior wall, and the greater and
lesser curvatures of the stomach are areas that might be missed
HOLLOW
• Stomach:
VISCUS INJURY
o Primary repair for small perforations
(stomach and
o Proximal or distal gastrectomy for extensive injuries
small bowel)
• Small bowel:
o Primary repair for perforations <50% bowel circumference
o Segmental resection & primary anastomosis for perforation >50% of the bowel circumference multiple injuries in close
proximity
• Primary suture repair without colostomy: for non-destructive injuries (perforations <50% of the bowel wall and without
devascularization)
• Segmental resection and primary anastomosis: for destructive injuries (loss >50% of bowel wall or with devascularization)
COLON • Alternatives for destructive injuries: debridement, primary repair and exteriorization, proximal colostomy with or without
repair
• Timing of colostomy closure after trauma: within 2 weeks after colostomy creation provided all wounds have completely
healed and nutritional state is optimal
• Intraperitoneal injuries: managed as colonic injuries
• Extraperitoneal injuries: 4-D’s of managing extraperitoneal injuries:
o Debridement of devitalized tissues
RECTUM
o Diversion: in selected cases where satisfactory repair cannot be performed because of anatomical location or extent of injury
o Drainage (presacral): not supported by current literature
o Distal rectal washout: no evidence that it can reduce morbidity

TERMS REMARKS • The difference between mean arterial


Abdominal
• The steady-state pressure concealed within pressure (MAP) and intraabdominal
perfusion
the abdominal cavity pressure (IAP)
pressure (APP)
• IAP is approximately 5-7 mmHg in critically • APP = MAP – IAP
Intraabdominal ill adults • Sustained IAP > 20 mmHg (with or
Abdominal
pressure (IAP) • The reference standard for intermittent without an APP <60mmHg) that is
compartment
IAP measurements is via the bladder associated with a new organ dysfunction or
syndrome (ACS)
with a maximal instillation volume of 25 failure
mL of sterile saline Surgery Platinum. 1st ed. 2018. p 157

Intraabdominal • Sustained or repeated pathological


hypertension elevation in IAP greater than or equal to
(IAH) 12 mmHg

TOPNOTCH MEDICAL BOARD PREP SURGERY MAIN DIGITAL HANDOUT BY LOUBOMIR ANTONIO, MD Page 16 of 101
For inquiries visit www.topnotchboardprep.com.ph or https://www.facebook.com/topnotchmedicalboardprep/
This handout is only valid for the September 2021 PLE batch. This will be rendered obsolete for the next batch since we update our handouts regularly.
TOPNOTCH MEDICAL BOARD PREP SURGERY MAIN DIGITAL HANDOUT BY LOUBOMIR ANTONIO, MD
For inquiries visit www.topnotchboardprep.com.ph or https://www.facebook.com/topnotchmedicalboardprep/
This handout is only valid for the September 2021 PLE batch. This will be rendered obsolete for the next batch since we update our handouts regularly.
Management of Intraabdominal Hypertension and Signs of Vascular Injury:
Abdominal Compartment Syndrome (ACS) HARD SIGNS SOFT SIGNS
CONDITION MANAGEMENT (Operation Mandatory) (Further Evaluation Indicated)
• Conservative management • Pulsatile hemorrhage • Proximity to vasculature
o Paracentesis • Absent pulses • Significant hematoma
o Nasogastric tubes with gastric suctioning • Acute ischemia • Associated nerve Injury
o Gastric prokinetics (metoclopramide, • A-A index <0.9 (SBP of injured side
erythromycin, etc.) compared with uninjured side)
o Rectal enemas and suctioning • Thrill or bruit
Intraabdominal
o Colonic prokinetics (prostigmine) Schwartz’s Principles of Surgery. 11th ed. 2019
hypertension
(>12 mmHg)
o Furosemide either alone or with human DIAGNOSTICS:
albumin 20% • Doppler assessment of pulsatile flow [compare injured side to
o Continuous venous hemofiltration or uninjured side] - (A-A index)
ultrafiltration o Less than 10% difference – assess significant injury to be
o Continuous negative abdominal pressure unlikely
o Sedation and muscle relaxation o More than 10% – needs CTA or angiography
o Upright (sitting) body positioning (pilot seat)
ACS
• Decompressive laparotomy MANAGEMENT:
(>20 mmHg)
Surgery Platinum. 1 ed. 2018. p 157
st • Completion of arteriograms to confirm adequacy of the repair
and distal flow
GENITOURINARY INJURIES • Primary repair – if vessels can be repaired without tension or
• Most present with hematuria narrowing
• The following indicate urethral disruption after lower Vessels for which repair should always be attempted:
abdominal trauma: ARTERIES VEINS
o Inability to void • Aorta • Hepatic • Superior vena cava
o Blood on the urethral meatus • Carotid proper • Suprarenal inferior
o Perineal or scrotal hematoma (Destot sign) • Brachiocephalic • Renal vena cava
o High riding or boggy prostate on digital rectal exam • Brachial • Iliac • Portal vein
• Blind urethral catheterization is contraindicated unless • Superior • Femoral
urethral injury is ruled out by a urethrogram mesenteric • Popliteal
ORGAN MANAGEMENT OPTIONS Surgery Platinum. 1st ed. 2018. p 158

• Non-operative management
• Embolization CURRENT INDICATIONS AND CONTRAINDICATIONS FOR
• Absolute indications for renal exploration for EMERGENCY DEPARTMENT THORACOTOMY
primary repair, or possible total or partial nephrectomies: INDICATIONS
Kidney 1. Expanding, pulsatile, uncontained retroperitoneal • Patients sustaining witnessed penetrating
hematomas trauma with <15 min of prehospital CPR
2. Renal pedicle avulsion Salvageable • Patients sustaining witnessed blunt
3. Persistent, life-threatening hemorrhage or shock postinjury trauma with <10 min of prehospital CPR
4. Ureteropelvic junction disruption cardiac arrest • Patients sustaining witnessed penetrating
• Proximal and mid injuries: trauma to the neck or extremities with <5
o ureteroureterostomy (primary anastomotic min of prehospital CPR
repair) over a double-J stent Persistent severe • Cardiac tamponade
• Distal injuries: postinjury • Hemorrhage (intrathoracic, intra-
Ureter
o Ureteroneocystostomy (Reimplantation of the hypotension abdominal, extremity, cervical)
ureter to adjacent bladder) (SBP £60 mmHg) • Air embolism
o Psoas hitch
o Boari flap: for bridging wide gaps in ureteral length CONTRAINDICATIONS
• Intraperitoneal bladder perforations: • Penetrating trauma: CPR >15 min and no signs of life
o primary “water tight” absorbable suture repair in (pupillary response, respiratory effort, motor activity)
2-3 layers with indwelling Foley
Urinary • Blunt trauma: CPR >10min and no signs of life or asystole
catheter/suprapubic cystostomy
bladder without associated tamponade
• Extraperitoneal bladder perforations:
o transurethral catheter drainage alone (ie. Foley
catheter) for 7-10 days
• Retrograde urethrogram: dx to rule out urinary
extravasation
Urethra
• Best managed by suprapubic cystostomy and delayed
urethral repair

EDT – Emergency Department Thoracotomy

PERIPHERAL VASCULAR INJURIES


• Initial physical examination of the limb should identify distal
pulses, capillary refill, color, motor and sensory function
TOPNOTCH MEDICAL BOARD PREP SURGERY MAIN DIGITAL HANDOUT BY LOUBOMIR ANTONIO, MD Page 17 of 101
For inquiries visit www.topnotchboardprep.com.ph or https://www.facebook.com/topnotchmedicalboardprep/
This handout is only valid for the September 2021 PLE batch. This will be rendered obsolete for the next batch since we update our handouts regularly.
TOPNOTCH MEDICAL BOARD PREP SURGERY MAIN DIGITAL HANDOUT BY LOUBOMIR ANTONIO, MD
For inquiries visit www.topnotchboardprep.com.ph or https://www.facebook.com/topnotchmedicalboardprep/
This handout is only valid for the September 2021 PLE batch. This will be rendered obsolete for the next batch since we update our handouts regularly.
SUPPLEMENT: DAMAGE CONTROL SURGERY MANDIBLE FRACTURE
• Purpose: break the vicious cycle by limiting operative time and start • Most common facial
physiologic restoration fracture (36% condyle –
• Indications: most common site
o refractory hypothermia (<35 C) affected)
o profound acidosis (pH <7.2 or base deficit > 15mmol/L)
• Imaging:
o refractory coagulopathy
• Goal: to control surgical bleeding and limit GI spillage o CT scan of the facial bones
• Return to the OR within 24-48 hours once the patient clinically and mandible
improves • Presenting signs and
symptoms: trismus,
✔ GUIDE QUESTION
malocclusion, numbness,
All of the ff. are considered manifestations of the bloody vicious cycle
and loose or missing teeth
and is an indication for damage control surgery except?
A. Coagulopathy C. Hypothermia • Treatment: maxillary-
B. Hypotension D. Metabolic acidosis mandibular fixation
(MMF)
EXTREMITY COMPARTMENT SYNDROME
• Manifestations: 6-Ps
o Pain as prominent symptom, aggravated by passive and active
motion
o Paresthesia-numbness between 1st and 2nd toes is the hallmark
of early leg compartment syndrome (deep peroneal nerve
involvement in anterior compartment)
o Pallor, Poikilothermia, Paralysis, Pulselessness NASAL FRACTURE
• Indications for Fasciotomy • Presenting signs and symptoms: epistaxis, nasal airway
o Gradient Pressure <30mmHg (diastolic P – compartment P) obstruction, alterations in smell, cosmetic deformity
o Absolute compartment P > 30mmHg
• Treatment
o Ischemic periods >6 hours
o Closed reduction – done within 10 days
o Combined arterial and venous injuries
o Open reduction with osteotomy – after 3 months
TRAUMATIC HEAD AND NECK INJURIES o Septal hematoma - should be drained as soon as possible to
prevent septal cartilage necrosis, septal perforation, and
• Evaluate each third of the face for neurovascular integrity, bony
potential saddle nose deformity
factures, and lacerations
• Cerebrospinal fluid (CSF) rhinorrhea
SUPPLEMENT: LE FORT FRACTURE o Clear nasal drainage in the patient with facial trauma with skull
LE FORT FRACTURE CLASSIFICATION base fracture
o Salty or metallic taste
• Le Fort Type 1: horizontal fracture o Screening test: Halo sign (differential capillary action of CSF
through the maxilla superior to the and blood on a white cloth or tissue)
maxillary dentition. o Confirmatory test: β2 Transferrin test
• Le Fort Type 2: pyramidal fracture
through the maxilla and orbit,
ORBITAL BLOWOUT FRACTURE
outlining the nose
• Le Fort Type 3: fracture of the facial • Isolated orbital floor fracture, presenting with a trapdoor
bones from the skull (complete pattern with associated extraocular muscle entrapment
craniofacial separation) • Presenting signs and symptoms: vision changes, forehead
Schwartz’s Principles of Surgery. 11th ed. 2019 and midface numbness, enophthalmos
• Imaging: CT scan of the facial bones and sinuses • Imaging: CT scan of the facial bones and sinuses
• Treatment • Treatment
o Goals of Le Fort fracture management o Goals of orbital fracture management
§ restoration of the continuity of the facial bones with the cranium § restoration of orbital structure
§ reduction of fractures with the goal of returning the patient to the
§ restoration of volume
preinjury occlusion
o Putting the patient into maxillomandibular fixation (MMF)
ensures satisfactory occlusion and provides a stable foundation for
the remainder of the repair

PENETRATING NECK INJURIES


• Zone I: inferior to the clavicles
encompassing the thoracic outlet
structures
• Zone II: between the thoracic outlet
and the angle of the mandible
• Zone III: above the angle of the
mandible
• Indications for IMMEDIATE
operative intervention for
penetrating cervical injury
1. Hemodynamic instability
2. Significant external hemorrhage
3. Evidence of aerodigestive injury

Adapted from Schwartz’s Principles of Surgery. 11th ed. 2019

Figure 7-20. Schwartz’s Principles of Surgery. 11th ed. 2019

TOPNOTCH MEDICAL BOARD PREP SURGERY MAIN DIGITAL HANDOUT BY LOUBOMIR ANTONIO, MD Page 18 of 101
For inquiries visit www.topnotchboardprep.com.ph or https://www.facebook.com/topnotchmedicalboardprep/
This handout is only valid for the September 2021 PLE batch. This will be rendered obsolete for the next batch since we update our handouts regularly.
TOPNOTCH MEDICAL BOARD PREP SURGERY MAIN DIGITAL HANDOUT BY LOUBOMIR ANTONIO, MD
For inquiries visit www.topnotchboardprep.com.ph or https://www.facebook.com/topnotchmedicalboardprep/
This handout is only valid for the September 2021 PLE batch. This will be rendered obsolete for the next batch since we update our handouts regularly.
BURN INJURIES 3. ESTIMATION OF BURN SIZE
MANAGEMENT o Most burn resuscitation formulas estimate fluid requirements
Resuscitative Period Definitive Period using the burn size as %Total Body Surface Area (TBSA).
(First 48 Hours) (>48 Hours) o Palmar surface – the surface area of a patient’s palm (including
• Assessment of burn injury • Excision and grafting fingers) is roughly 1% of total surface area.
• Classification of burn injury • Control of infection o “Rule of nines” is a crude but quick and effective method of
• Criteria for admission • Nutrition estimating burn size
• Initial (ER) management • Rehabilitation § The body is divided into areas of 9% and the total burn area
o Fluid resuscitation • Complication can be calculated.
o Wound dressing § Not accurate in children
o Monitoring o Lund and Browder chart – if used correctly, is the most
CLASSIFICATION OF BURN INJURY accurate method, considers the variation in body shape with age
1. FLAME BURN (favorable in the pediatric population)
o Most common cause for hospital admission o Thorough cleaning of soot and debris is mandatory to avoid
o Highest mortality (primary related to association with structural confusing areas of soiling with burns
fires and accompanying inhalational injury) o Superficial (first degree) burns SHOULD NOT be included when
o House fires, improper use of flammable liquids, kerosene lamps, calculating the %TBSA
careless smoking, vehicular accidents, clothing ignited from stove SUPPLEMENT: ESTIMATING BURN SIZE
2. FLASH BURN
Rule of Nines / Rule of Wallace
o Explosions of natural gas propane, gasoline and other flammable AREA %BSA
liquids causing intense heat for a very brief period of time (Adults)
3. ELECTRICAL BURN Head 9%
o Injury from electrical current classified as high voltage Chest 9%
o (> 1000 volts) or low voltage (<1000 volts) Abdomen 9%
o Concern with electrical burns: cardiac arrhythmias, Upper Back 9%
compartment syndrome with rhabdomyolysis Lower Back or 9%
4. SCALD BURN Buttocks
o Burns caused by hot liquids, eg. hot water, soups, and sauces Right Arm 9%
which are thicker in consistency, remain in contact with the skin Left Arm 9%
for a longer period of time Right thigh and 18%
Leg
5. CHEMICAL BURN
Left thigh and 18%
o Caused by strong alkali or acids Leg
o These cause progressive damage until chemical is deactivated Genitalia or 1%
with reaction with tissue or reaction with water Perineum
§ Acid burns: more self-limiting than alkaline burns; acids tend Surgery Platinum. 1st ed. 2018 p 161
to tan the skin, creating an impermeable barrier which limits Know at least one method – rule of 9!
further penetration Dr. Asperas

§ Alkali burns: combine with cutaneous lipids to create soap and 4. DIAGNOSIS OF CARBON MONOXIDE AND CYANIDE
thereby continue to dissolve the skin until they are neutralized POISONING
o Most important components of initial therapy: removal of toxic o Unexpected neurologic symptoms should raise the level of
substance and irrigation of affected area with water (~30 minutes) suspicion for CO poisoning
6. CONTACT BURN § Affinity of CO for hemoglobin is 200-250x more than that of O2
o Results from hot metals, plastic, glass or hot coils → decreases the levels of normal oxygenated hemoglobin and
o Usually limited in extent but very deep can quickly lead to anoxia and death
7. RADIATION § Treatment: Administration of 100% oxygen (gold standard)
o Exposure from laboratory accidents or damage from x-ray o Cyanide poisoning is seen in smoke inhalation injury
machines, meltdown of a nuclear power plant, and detonation of § May have lactic acidosis or ST elevation
a nuclear device § Cyanide inhibits cytochrome oxidase, which in turn inhibit
SUPPLEMENT: BURN CENTER REFERRAL cellular oxygenation
AMERICAN BURN ASSOCIATION: § Treatment: sodium thiosulfate, hydroxocobalamin, and 100% O2
Guidelines for Referral to a Burn Center
• Partial thickness burns >10% TBSA FLUID RESUSCITATION AND CALORIC NEEDS
• Burns involving face, hands, feet, genitalia, perineum, major joints 1. FLUID NEEDS FOR THE FIRST 24 HOURS POST-BURN
• Third-degree burns (any age group) Revised Parkland’s (Baxter) Formula = 2 ml plain
• Electrical, chemical burns LR/kg/%TBSA (American Burn Association; Schwartz’s 11th ed.)
• Inhalation injury • first ½ given in the first 8 hours
• Burn patients with co-morbidities • next ½ given for the next 16hours
• Burns with trauma
• No qualified personnel to handle burns (children) Parkland formula provides an estimate of the ADDITIONAL FLUID. To
• Need for special, social, emotional, rehabilitation reiterate, computed volume DOES NOT INCLUDE the maintenance fluid
requirements that can be computed using HOLLIDAY-SEGAR METHOD
INITIAL EVALUATION OF BURNS Superficial (first degree) burns SHOULD NOT be included when
1. AIRWAY MANAGEMENT calculating the %TBSA!
Dr. Asperas
o With direct thermal injury to the upper airway and/or smoke
inhalation (perioral burns, signed nasal hairs), rapid and severe Curreri Formula = 25kcal/kg/day + 40kcal/TBSA/day
airway edema is a potentially lethal threat • computation for caloric needs
o Anticipating the need for intubation and establishing an early Galveston Formula (for initial 24 hours) for children
airway is critical • 5,000 mL/m2 TBSA burned + 2,000 mL/m2 TBSA (D5% Lactated
o Signs of impending respiratory compromise: hoarse voice, Ringer’s)
wheezing, or stridor • Half given during the first 8 hours after burn
2. EVALUATION OF OTHER INJURIES • Remaining half over subsequent 16 hours
o Burn patients should be first considered trauma patients! • Add dextrose to the resuscitation fluid in children to prevent
(especially when details of the injury are unclear), as such, a hypoglycemia, because children have smaller glycogen stores
primary survey should be conducted than adults
o An early and comprehensive secondary survey must also be • The standard Parkland formula commonly underestimates fluid
performed in all burn patients requirements in a burned child
o Urgent radiology studies (i.e. CXR) should be performed in the 2. FLUID NEEDS FOR THE SECOND 24 HOURS POST-BURN
ER, but non urgent skeletal evaluation (i.e. extremity X-rays) can • Colloids (5% albumin in Ringer’s lactate or FFP 0.5mL/Kg/%
be done later to avoid hypothermia and delays in burn TBSA burned/day)
resuscitation • Colloid volume is subtracted from the calculated crystalloid rate
TOPNOTCH MEDICAL BOARD PREP SURGERY MAIN DIGITAL HANDOUT BY LOUBOMIR ANTONIO, MD Page 19 of 101
For inquiries visit www.topnotchboardprep.com.ph or https://www.facebook.com/topnotchmedicalboardprep/
This handout is only valid for the September 2021 PLE batch. This will be rendered obsolete for the next batch since we update our handouts regularly.
TOPNOTCH MEDICAL BOARD PREP SURGERY MAIN DIGITAL HANDOUT BY LOUBOMIR ANTONIO, MD
For inquiries visit www.topnotchboardprep.com.ph or https://www.facebook.com/topnotchmedicalboardprep/
This handout is only valid for the September 2021 PLE batch. This will be rendered obsolete for the next batch since we update our handouts regularly.
• Amount of crystalloid solution (D5 Water for adults, D5 ½ or ¼ SUPPLEMENT: JACKSON ZONES OF BURN INJURY
Normal Saline for children) depends on the maintenance ZONE DESCRIPTION
requirements plus the amount of transcutaneous evaporative • Most severely burned portion
losses from burn wounds (1 ml/kg/% TBSA/ day) • Center of the wound
• Amount is titrated to maintain normal urine output and mean Coagulation
• No capillary blood flow
arterial pressure (MAP) • Needs excision and grafting
• Optimal MAP (for adequate end-organ perfusion) = 60 mmHg • Surrounds the zone of coagulation
• Urine output goals: • Variable degrees of vasoconstriction, and
Stasis
o Adults: 0.5 mL/kg/hr resultant ischemia
o Children: 1-1.5 mL/kg/hour • Salvageable by appropriate resuscitation
• Outermost area surrounding of burn
DEFINITIVE MANAGEMENT • Related to vasodilation from inflammation
Components of Definitive Management Hyperemia
surrounding the burn wound
• Present trend: early (within 7 days post-burn) tangential excision • Heals with minimal or no scarring
of the burn wound, followed by skin grafting
• Attention to nutrition
• Pain control: meperidine, nalbuphine
• Rehabilitation
• Management of complications
ASSESSMENT OF BURN DEPTH
1ST DEGREE 2ND DEGREE BURN
3RD DEGREE BURN 4TH DEGREE 5TH DEGREE 6TH DEGREE
BURN (PARTIAL-
(FULL-THICKNESS) BURN BURN BURN
(SUPERFICIAL) THICKNESS)
Superficial (papillary):
• Extremely painful
• Painful • Blister formation • Painless • Affects
Clinical • Goes through • Charring bone
• No blisters • Blanching • Leathery underlying soft
Features muscle to bone is present
• Blanching Deep (reticular): • Non-blanching tissue
• Less painful
• White or pale
TOPICAL ANTIMICROBIAL AGENTS USED IN BURN CARE
AGENT REMARKS SPECIFIC CONSIDERATIONS
• Primarily as a prophylactic agent against burn HEAD, NECK, PARATHYROID, & THYROID
wound infections (rather than treatment of HEAD AND NECK
existing infections)
Silver • Can cause neutropenia (due to neutrophil ✔ GUIDE QUESTION
sulfadiazine migration to the burn site) A 33-year-old female noted a 4-day history of discharge from a sinus in
the overlying skin below the right angle of the mandible. She recalls
• Contraindicated on burns or donor sites in
previous episodes of fullness and mild pain in this region over the past
proximity to newly grafted areas – may retard
several years. What is the most likely cause?
epithelial migration (A) Thyroglossal duct cyst
• Effective in the presence of an eschar (B) Branchial cleft anomaly
• Can be used in both prophylaxis and (C) Teratoma
Mafenide
treatment of wound infections (D) Myeloma
acetate
• Can cause metabolic acidosis (due to (E) Trauma to the neck
carbonic anhydrase inhibition) Thyroglossal duct cysts are midline. Teratoma and myeloma could
• Has broad-spectrum microbial activity appear anywhere but usually doesn’t present with discharge and
Silver
• Can cause hyponatremia and episodes of fullness and pain. There was no mention of trauma and so
nitrate
methemoglobinemia, black stains is obviously wrong.
• Being used as an inexpensive topical A second branchial cleft sinus is suspected when clear fluid is noted
Dakin’s
antimicrobial draining from the external opening of the tract at the anterior border
solution
• Contains 0.5% sodium hypochlorite (bleach) of the lower third of the sternomastoid muscle.
COMMON COMPLICATIONS IN BURN PATIENTS It extends inward through the bifurcation of the carotid artery, and enters
• Most common cause of death in burns the posterolateral pharynx just below the tonsillar fossa. In contrast, a
Burn wound • The practice of early burn wound excision has third branchial cleft fistula passes posterior to the carotid bifurcation.
Dr. Cocos
sepsis significantly decreased the incidence of burn
HEAD AND NECK MASS – Evaluation
wound sepsis and improved survival
• Rule of 7: Provides a probable diagnosis of the neck mass
• Occurs in electrical or inhalational injury
based on the average duration of the patient's symptoms
ARDS • Presents as progressive hypoxemia
o 7 days – inflammatory
unresponsive to increasing FiO2
o 7 months – neoplastic
• Can be caused by massive fluid resuscitation
o 7 years – congenital
• Characterized by:
Abdominal
compartment
o ↑ airway pressures + hypoventilation CONGENITAL LESIONS
o ↓urine output THYROGLOSSAL DUCT CYST
syndrome
o Hemodynamic compromise
• Standard of care: decompressive laparotomy • Residual thyroid tissue left behind during its descent
Deep vein • Can cause fatal pulmonary embolus • Presents as midline neck mass in children
thrombosis • Heparin prophylaxis may be considered • 80% occurs at or just below the hyoid bone
• Stress ulcer prophylaxis is necessary in burn • Moves with deglutition or tongue protrusion
Stress ulcers • 5% contain functional thyroid tissue
patients
(Curling • Treatment: Sistrunk procedure
• Histamine receptor blockers, sucralfate, and
ulcers)
protein pump inhibitors BRANCHIAL CLEFT ANOMALIES
CRITERIA FOR DISCHARGE EXTERNAL OPENING INTERNAL OPENING
1. No existing complications of thermal injury such as 1st • Pre-auricular area • Middle ear
inhalational injury 2nd • Lateral neck at SCM • Tonsillar fossa
2. Fluid resuscitation completed 3rd • Lateral neck at SCM • Pyriform sinus
3. Adequate pain tolerance • Most common anomaly: 2nd branchial cleft anomaly
4. Adequate nutritional intake
• Usually presents on the 1st decade of life
5. No anticipated septic complications
• Treatment: Complete excision
TOPNOTCH MEDICAL BOARD PREP SURGERY MAIN DIGITAL HANDOUT BY LOUBOMIR ANTONIO, MD Page 20 of 101
For inquiries visit www.topnotchboardprep.com.ph or https://www.facebook.com/topnotchmedicalboardprep/
This handout is only valid for the September 2021 PLE batch. This will be rendered obsolete for the next batch since we update our handouts regularly.
TOPNOTCH MEDICAL BOARD PREP SURGERY MAIN DIGITAL HANDOUT BY LOUBOMIR ANTONIO, MD
For inquiries visit www.topnotchboardprep.com.ph or https://www.facebook.com/topnotchmedicalboardprep/
This handout is only valid for the September 2021 PLE batch. This will be rendered obsolete for the next batch since we update our handouts regularly.
• GIANT CELL REPARATIVE
GRANULOMA
o Peripheral
§ Soft tissue in origin
§ May be polypoid,
submucosal and fibrous
§ May ulcerate and bleed
§ Treatment: excision
o Central
§ Expansile endosteal lesion of the mandible of bony origin
§ Treatment: curettage
• GRANULAR CELL
MYOBLASTOMA
o Also called Abrikoss of tumor
Irace A, et al. 2017. Embryology of congenital neck masses. Operative Techniques in Otolaryngology-Head and Neck o Rare benign tumor of the tongue
Surgery Volume 28, Issue 3, September 2017, Pages 138-142. https://doi.org/10.1016/j.otot.2017.05.009
o derived from Schwann cells
CYSTIC HYGROMA o Treatment: wedge excision
• Result from maldevelopment & obstruction of lymphatic system
• Sequestrations of lymphatic tissue develops which does not ULCERATIVE LESIONS - ORAL CAVITY
communicate with the lymphatic system
• White sponge nevus, lichen planus, and oral hairy leukoplakia
• Features: soft, compressible, and non- tender masses, usually
o Usually resolves spontaneously; otherwise treated with
located in the lateral or posterior triangle of the neck
steroids
• Imaging: MRI
o Biopsy to rule out carcinoma
• Treatment: complete surgical resection (preserve vital neck
structures) BENIGN LESIONS – NOSE
POLYPS
NEOPLASMS OF THE HEAD AND NECK • SQUAMOUS PAPILLOMA
ANATOMIC AREA CLINICAL MANIFESTATION o May cause nasal obstruction
• Fleshy exophytic mass o Treatment: excision
• Heaped-up ulcers • JUVENILE NASOPHARYNGEAL ANGIOFIBROMA
Anterior oral cavity • Pain o Highly expansile and destructive fibrovascular neoplasms
• Tooth loosening o Typically occurs in adolescent males
• Impairment of tongue mobility o Presents with epistaxis which can be massive
Tonsillar fossa or • Trismus with infiltration of pterygoid o Treatment: angioembolization
retromolar trigone muscles
• Hoarseness, dysphagia, dyspnea, pain
Larynx
• Cervical lymphadenopathy
EPISTAXIS
• Nasal obstruction https://qrs.ly/lcckzwd
• Decreased hearing acuity
Nasopharynx
• Ocular motor palsy
• Cervical lymphadenopathy
• Cheek swelling ✔ GUIDE QUESTION
Maxillary sinus • Palatal bulging An 85-year-old hypertensive man is evaluated in the emergency
department for recent onset epistaxis. His blood pressure is 150/80 mm Hg,
• Orbit extension
and hematocrit is 39%. What is the most likely source of bleeding?
• Asymptomatic firm lump (A) Posterior nasal septum
• Skin or mucosal ulceration (B) Anterior nasal septum
Salivary glands
• Paresthesia or paralysis (C) Inferior turbinate
• Cervical lymphadenopathy (D) Middle turbinate
(E) Floor of nose
90% of epistaxis comes from the anterior nasal septum. Clues to
BENIGN LESIONS – LIP posterior nasal septum epistaxis: massive, bilateral, hemodynamic
• Lower lip is usually subject to chronic irritation, presenting as instability, aspiration etc.
scaling lesions Dr. Cocos

• Spectrum of Lesions: dysplasia, hyperkeratosis, parakeratosis, BENIGN LESIONS – LARYNX


dyskeratosis, carcinoma in situ, invasive carcinoma
PAPILLOMA
• Treatment: wide excision +/- reconstruction
• Usually arise from the true vocal cord, presenting with
hoarseness
BENIGN LESIONS - ORAL CAVITY
• Juvenile type usually recurs after treatment
• MUCOUS RETENTION CYST • Treatment: Excision or laser ablation
o Submucosal accumulation of mucus due to rupture of the
ductal system LARYNGOCELE
o (+) bluish hue cystic structure, less than 1 cm • Herniation of the laryngeal ventricles due to chronic increase
in intralaryngeal pressure
• RANULA – mucous retention cyst
• Treatment: Ligation of stalk and repair of the ventricles
involving the sublingual gland
o Treatment: excision or
marsupialization ODONTOGENIC TUMORS
• Arise from progenitors of tooth development
• EPULIS • Involves the maxilla and the mandible
o granulomatous lesions due to • Primary management: conservative
exaggerated inflammation AMELOBLASTOMA (ADAMANTINOMA)
o Congenital type: occurs in the
• Arises from the dental lamina
anterior maxilla
o Pregnancy type: occurs in 1% • Associated with impacted tooth
of pregnant women, resolves • Painless benign but locally aggressive mandibular mass that
spontaneously may erode the bone cortex
o Treatment: excision if • (+) soap-bubble appearance in radiographs
symptomatic and persistent • Treatment: Resection
TOPNOTCH MEDICAL BOARD PREP SURGERY MAIN DIGITAL HANDOUT BY LOUBOMIR ANTONIO, MD Page 21 of 101
For inquiries visit www.topnotchboardprep.com.ph or https://www.facebook.com/topnotchmedicalboardprep/
This handout is only valid for the September 2021 PLE batch. This will be rendered obsolete for the next batch since we update our handouts regularly.
TOPNOTCH MEDICAL BOARD PREP SURGERY MAIN DIGITAL HANDOUT BY LOUBOMIR ANTONIO, MD
For inquiries visit www.topnotchboardprep.com.ph or https://www.facebook.com/topnotchmedicalboardprep/
This handout is only valid for the September 2021 PLE batch. This will be rendered obsolete for the next batch since we update our handouts regularly.
• Mostly used to assess presence of neck
metastases
Ultrasound
• Can also be used to guide the needle in fine-
needle aspiration biopsy
• Tumors of the oral cavity can be biopsied by
punch or cup forceps biopsy
• Nasal, pharyngeal and laryngeal tumors are
Biopsy
biopsied by use of endoscopes
• Neck nodes can be biopsied using fine-needle
techniques
STAGING - TNM CLASSIFICATION
T Description
✔ GUIDE QUESTION
A24-year-old computer technician notes a progressive increase in the T1 2cm or less in diameter, confined to site of origin
size of his left jaw. After x-rays are taken and a biopsy is done, a T2 >2 to 4 cm in diameter
diagnosis of ameloblastoma is established. What should be the next step T3 >4 cm in diameter
in management? T4 Massive tumor with invasion of surrounding structures
(A) Radiotherapy N Description
(B) Laser beam therapy N0 No regional lymph node metastasis
(C) Curettage and bone graft N1 Metastasis in a single ipsilateral node <3cm
(D) Excision of lesions with 1–2 cm of normal mandible Metastasis in a single ipsilateral lymph node between
(E) Mandibulectomy with bilateral radical neck dissection N2a
3-6 cm
Excision with margins is sufficient since ameloblastoma is a benign
N2b Metastasis in multiple ipsilateral lymph node, node >6cm
lesion (but locally aggressive). There is no need for a neck dissection.
(E is wrong and mandibulectomy is overkill). Radiotherapy might be N2c Metastasis in bilateral or contralateral nodes <6cm
needed after surgery or if surgery isn’t an option. Curettage has high N3 Metastasis in a lymph node >6cms
recurrence rates. Laser beam therapy is not a therapeutic option. M Description
Dr. Cocos M0 No distant metastasis
M1 With distant metastasis
MALIGNANT HEAD AND NECK TUMORS
Note: Tip to memorize H&N staging: magic numbers for T are 2 and 4,
• Risk factors:
for N are 3 and 6. J
o Tobacco use Dr. Cocos
o Alcohol consumption CERVICAL LYMPH NODES
o Nutritional deficiencies Level Description
o Immunosuppression 1 • Submental / submandibular lymph nodes
o Viruses (HPV, EBV) 1a • Submental nodes (medial to anterior belly of digastric)
o Occupational exposures (wood dust, nickel, paint • Submandibular node (posterior to anterior belly of
1b
o fumes, metal works, textiles, radiation, asbestos) digastric)
o Older age 2 • Upper jugular lymph nodes
o Low socioeconomic status • Upper jugular chain
2a
• Clinical Presentation: otalgia, odynophagia, bleeding, (inferior to spinal accessory nerve)
hoarseness, trismus, dysphagia and loss of hearing • Submuscular recess
2b
(superior to spinal accessory nerve)
3 • Mid jugular lymph nodes
4 • Lower jugular lymph nodes
5 • Posterior triangle / suboccipital lymph nodes
5a • Superior to the spinal accessory nerve
5b • Inferior to the spinal accessory nerve
6 • Anterior / central lymph nodes
• Superior mediastinal / paratracheal / suprasternal
7
lymph nodes

HPV-positive
Feature HPV-negative HNSCC
HNSCC
Incidence Decreasing Increasing
Smoking, excessive
Etiology Oral sex
alcohol use
Under 60
Age Above 60 years
years
Field
Yes Unknown
cancerization
TP53 mutations Frequent Infrequent
Predilection site None Oropharynx
Prognosis Poor Favorable

DIAGNOSTIC REMARKS CERVICAL LN


• Necessary to rule out synchronous lesions https://qrs.ly/65cl0jt
Endoscopy
which can be present in 5-7% of cases
Dental • Help determine whether mandible involvement
panoramic is present NECK DISSECTIONS
radiograph • aka Panorex • Radical Neck Dissection (RND)
• Information on extent of local soft-tissue and o Removes levels I to V cervical nodes, spinal accessory nerve,
CT scan with internal jugular vein and sternocleidomastoid muscle
bony involvement of upper aerodigestive tract
Contrast
tumors, and to detect sites of metastasis • Modified Radical Neck Dissection (MRND) – functional neck
MRI • Excellent for evaluation of soft tissues dissection
Chest • Screen for lung and mediastinal metastases o Removes the same levels of cervical lymphatics as in RND but
radiograph (ideally, chest CT Scan with IV contrast) preserves the spinal accessory nerve, the IJV and the
sternocleidomastoid muscle

TOPNOTCH MEDICAL BOARD PREP SURGERY MAIN DIGITAL HANDOUT BY LOUBOMIR ANTONIO, MD Page 22 of 101
For inquiries visit www.topnotchboardprep.com.ph or https://www.facebook.com/topnotchmedicalboardprep/
This handout is only valid for the September 2021 PLE batch. This will be rendered obsolete for the next batch since we update our handouts regularly.
TOPNOTCH MEDICAL BOARD PREP SURGERY MAIN DIGITAL HANDOUT BY LOUBOMIR ANTONIO, MD
For inquiries visit www.topnotchboardprep.com.ph or https://www.facebook.com/topnotchmedicalboardprep/
This handout is only valid for the September 2021 PLE batch. This will be rendered obsolete for the next batch since we update our handouts regularly.
• Treatment:
o T1-T2: partial glossectomy
MRND • Presents with
plus supraomohyoid
https://qrs.ly/kfcl0lj ulcerations or
dissection if N0 or MRND if N+
Cancer exophytic masses
o If with mandibular
of the • Most commonly
involvement:
tongue located on the
• Selective Neck Dissection lateral and
§ composite resection
o preserves lymphatic structures normally removed in an RND § partial glossectomy + floor
ventral surface
or MRND of mouth and
o Supraomohyoid dissection (I, II, and III) mandibulectomy
§ for oral cavity malignancies • Usually at an • Treatment
o Lateral neck dissection (II, III, and IV) Cancer advanced stage o Early lesions: surgery or
§ for laryngeal, oropharyngeal, hypopharyngeal malignancies of the on diagnosis, and radiotherapy + plus bilateral
o Central neck dissection (VI) base of has a strong neck dissection
the propensity for o Advanced lesions: total
§ for thyroid malignancy
tongue lymph node resection of tongue base
o Posterolateral neck dissection (II,III,IV, and V)
metastasis with/without laryngectomy
§ for thyroid and posterior malignancies

CANCER OF THE LIP


https://qrs.ly/cccl0m3

✔ GUIDE QUESTION
A 62-year-old alcoholic presents with an indurated ulcer, 1.5 cm in
length, in the left lateral aspect of her tongue (not fixed to the alveolar
ridge). There are no clinically abnormal glands palpable in the neck,
and a biopsy of the tongue lesion reveals squamous cell carcinoma.
What should she undergo?
(A) Chemotherapy
(B) Wide excision of the ulcer
(C) Wide excision and left radical neck dissection
(D) Antibiotic therapy and should be encouraged to stop smoking
(E) Wide excision of ulcer and radiotherapy
Since this is a resectable mass then chemotherapy is a wrong choice.
There is no indication for antibiotic therapy because there were no
✔ GUIDE QUESTIONS signs pointing to an infection. There is a need for resection since this
The classic complete neck dissection for palpable adenopathy in the is a resectable mass.. Doing a radical neck dissection is not correct
posterior triangle of the neck includes removal of which of the since no neck nodes were palpable. Lesions >4mm depth should
following? undergo preferably a selective neck dissection (I,II,III) or adjuvant
(A) The transverse process, C2–C4 radiotherapy in addition to wide excision or resection.
Dr. Cocos
(B) The spinal accessory nerve
(C) Both thyroid lobes CANCER OF THE ALVEOLUS/GINGIVA
(D) The trapezius
(E) The vagus • Needs preoperative imaging evaluation of the involvement of
Classic or radical neck dissection includes the removal of the SCM, IJV bony structures
and the SAN (spinal accessory nerve). • Panorex/ CT scan: to demonstrate gross cortical invasion
Dr. Cocos
• MRI: to demonstrate invasion of the medullary cavity
A 70-year-old male complains of progressive weight loss and
hoarseness. Ear, nose, and throat (ENT) evaluation reveals right vocal • Treatment:
cord paralysis and several right neck masses, which fine needle o Resection of the tumor plus;
aspiration reveals to be squamous cell carcinoma. The patient o marginal mandibular resection or segmental mandibular
undergoes right hemilaryngectomy and right radical neck dissection. resection plus;
Postoperatively, right hemidiaphragm paralysis is noted. This is due to o ipsilateral supraomohyoid dissection (if N0) or MRND (if N+)
injury of which of the following:
(A) Vagus nerve
(B) Brachial plexus
(C) Cervical plexus
(D) Spinal accessory nerve
(E) Phrenic nerve
The phrenic nerve (C3-C5), which is the main innervation of the
diaphragm, arises in the posterior triangle, then descends to the
anterior surface of the anterior scalene muscle in the supraclavian
triangle. It may inadvertently be injured during a radical neck
dissection
Dr. Cocos

CANCERS OF THE LIP, TONGUE, AND BASE OF THE TONGUE


FEATURES MANAGEMENT
• Majority occurs • Treatment:
in the lower lip o Early lesions – surgery or
(basal cell radiotherapy
carcinoma o Presence of nodal
presents more involvement – neck
CANCER OF THE HYPOPHARYNGEAL AREA
commonly on dissection• Usually presents at an advanced stage
Cancer
the upper lip) • Presents with dysphagia starting with solid foods progressing
o Indications for adjuvant
of the
• Presentation: to liquids
radiotherapy:
lip
ulceration, • Majority have lymph node metastasis at the time of diagnosis
§ (+) margins at operated
paresthesia site • Treatment:
(indicating § (+) perineural invasion
o T1 – radiotherapy
involvement of § (+) nodal involvement
o T2 and T3 – chemoradiotherapy
the mental • Surgery:
nerve)
o Must not involve apex of pyriform sinus and vocal cord
o Adequate pulmonary reserve
TOPNOTCH MEDICAL BOARD PREP SURGERY MAIN DIGITAL HANDOUT BY LOUBOMIR ANTONIO, MD Page 23 of 101
For inquiries visit www.topnotchboardprep.com.ph or https://www.facebook.com/topnotchmedicalboardprep/
This handout is only valid for the September 2021 PLE batch. This will be rendered obsolete for the next batch since we update our handouts regularly.
TOPNOTCH MEDICAL BOARD PREP SURGERY MAIN DIGITAL HANDOUT BY LOUBOMIR ANTONIO, MD
For inquiries visit www.topnotchboardprep.com.ph or https://www.facebook.com/topnotchmedicalboardprep/
This handout is only valid for the September 2021 PLE batch. This will be rendered obsolete for the next batch since we update our handouts regularly.

CANCER OF THE LARYNX (A) Glandular hypertrophy, secondary to vitamin A deficiency


(B) Cystic dilation
• Presents with (C) Mikulicz disease
hoarseness, airway (D) Pleomorphic adenoma
(E) Papillary cystadenoma (Warthin tumor)
compromise, and
dysphagia Most common benign parotid gland lesion is pleomorphic adenoma.
• Treatment: The tumor presents as a slow-growing, painless, firm, and nontender
mass that is mobile when small but may become fixed as it enlarges.
o Supraglottic/
Second most common is papillary cystadenoma which is bilateral in
glottic 20% of cases.
§ Early lesions - Dr. Cocos

radiotherapy
§ Advanced
lesions -
surgery +
radiotherapy
o Subglottic: total
laryngectomy
plus neck
dissection
(regardless of N LARYNGEAL PRESERVING
status) PROCEDURE
Figure 18-33. Schwartz’s Principles of Surgery. 11th ed. 2019

UNKNOWN PRIMARY TUMORS


• Patients with cervical nodal metastases without clinical or
radiologic evidence of primary tumor Finding the Facial Nerve:
• Imaging: Pan-endoscopy • External auditory canal cartilage – a triangle that points to the
• Blind biopsies (on 4 areas): facial nerve, 1 cm deep and inferior to the “tragal pointer”
o Base of tongue • Styloid process – lateral lies the nerve
o Pyriform sinus • Posterior belly of digastric – superficial and parallel to the nerve
o Tonsillar fossa • Tympanomastoid suture- 6-8mm deep to this suture
o Nasopharynx • Groove between mastoid and bony EAC- bisected by facial nerve
• Treatment: ✔ GUIDE QUESTION
o Radical neck dissection A 43-year-old teacher underwent left parotidectomy. Upon awakening
o Empiric radiotherapy (nasopharynx to hypopharynx) from surgery, paralysis of the left lower lip was observed. This
complication was most likely due to injury to which of the following:
(A) Parotid duct
(B) Facial nerve - temporal branch
(C) Facial nerve – marginal mandibular branch
(D) Facial nerve - main trunk
(E) Platysma muscle – supplies motor function to the lower lip and
chin
The most commonly injured branch of the facial nerve is the marginal
mandibular branch followed by the temporal then buccal branches.
The temporal branch raises the eyebrows, the zygomatic branch closes
the eyes, the buccal branch compresses the cheek and moves the upper
SALIVARY GLAND TUMORS lip, the marginal mandibular branch moves the lower lip, and the
• Most salivary tumors are benign, slow-growing and well- cervical branch pulls lower lip and angle of mouth down.
Dr. Cocos
circumscribed
• Incidence of malignancy:
MALIGNANT SALIVARY GLAND TUMORS
o 25% - parotid gland FEATURES
o 50% - submandibular gland Muco-epidermoid • Most common malignant salivary
carcinoma gland tumor
o 75% - sublingual gland
• Second most common malignant
The smaller the salivary gland, the higher its propensity for malignant salivary gland tumor in ADULTS
transformation. Adenoid cystic
• Has propensity for neural spread
Dr. Rubio carcinoma
• Has indolent growth, but high
• Malignant features of salivary gland tumors: rapid growth, propensity for distant metastasis
painful, paresthesia, trismus • Second most common malignant
• Diagnostic imaging is the standard evaluating tool aided with Acinic cell carcinoma
salivary gland tumor in CHILDREN
FNAB (final histopathologic diagnosis by surgical excision) Carcinoma ex • Aggressive tumor arising from benign
• Salivary gland tumors can be classified as: epithelial, non- pleomorphic adenoma mixed tumor
epithelial, and metastatic • Treatment
o Parotid gland involvement
BENIGN EPITHELIAL AND NON-EPITHELIAL TUMORS § Lateral lobe: superficial parotidectomy with facial nerve
EXAMPLES MANAGEMENT preservation
• Pleomorphic adenoma • Treatment: surgical § Deep lobe: total parotidectomy with facial nerve
(most common) excision preservation
Epithelial § Facial Nerve sacrifice
• Monomorphic adenoma o Parotid
salivary
tumors
• Warthin tumor or involvement: - encased by tumor
papillary cystadenoma superficial - non-functional pre-operatively
• Oncocytoma parotidectomy
o Submandibular gland involvement:
Non- • Hemangioma with facial nerve
preservation
§ En bloc resection of the gland with submental and
epithelial • Neural sheath tumor
o Enucleation is NOT submandibular LN
salivary • Lipoma
tumors RECOMMENDED • Management of the Neck
o Indications for modified radial neck dissection (MRND)
✔ GUIDE QUESTION § N+
While shaving, a 45-year-old teacher notices a marble-sized mass § N0; if the risk of LN metastases exceeds 20% (high-grade
beneath his left ear. The mass is eventually excised, revealing which of
the following benign parotid gland lesions?
mucoepidermoid CA)
o Indications for radiotherapy
TOPNOTCH MEDICAL BOARD PREP SURGERY MAIN DIGITAL HANDOUT BY LOUBOMIR ANTONIO, MD Page 24 of 101
For inquiries visit www.topnotchboardprep.com.ph or https://www.facebook.com/topnotchmedicalboardprep/
This handout is only valid for the September 2021 PLE batch. This will be rendered obsolete for the next batch since we update our handouts regularly.
TOPNOTCH MEDICAL BOARD PREP SURGERY MAIN DIGITAL HANDOUT BY LOUBOMIR ANTONIO, MD
For inquiries visit www.topnotchboardprep.com.ph or https://www.facebook.com/topnotchmedicalboardprep/
This handout is only valid for the September 2021 PLE batch. This will be rendered obsolete for the next batch since we update our handouts regularly.
§ Presence of extraglandular disease • Nerves:
§ Perineural invasion REMARKS
§ Direct invasion of regional structures • From vagus nerve
§ Presence of regional metastases • Crosses arch of the aorta → loops around
Left recurrent
§ Presence of high-grade histology ligamentum arteriosum → ascends at
laryngeal nerve
tracheoesophageal groove
• Vulnerable to injury at the ligament of Berry
• From vagus nerve
• Crosses and loops around right subclavian
Right recurrent
artery
laryngeal nerve
• More oblique
• Vulnerable to injury at the ligament of Berry
Internal branch
• Provides sensation above the vocal folds
of the superior
• Injury: increases risk of aspiration
laryngeal nerve
Figure 18-27 a. Schwartz’s Principles of Surgery. 11th ed. 2019 External branch
• Innervates the cricothyroid muscle
of the superior
• Injury: causes inability to hit high notes
laryngeal nerve
RLN INJURY CLINICAL MANIFESTATION
• Vocal cord in paramedian position:
Unilateral RLN normal but weak voice
injury • Vocal cord in abducted position: hoarse
voice, weak cough
Bilateral RLN • Airway obstruction → necessitates
injury tracheostomy

EVALUATION OF PATIENTS WITH THYROID DISEASE


SUPPLEMENT: TEST OF THYROID FUNCTION
• normal: 0.5 – 5μU/mL
• only test necessary in most patients with
thyroid nodules that clinically appear
Serum TSH
euthyroid
THYROID • serum TSH levels reflect the ability of the anterior
pituitary to detect free T4 levels
THYROID ANATOMY • normal: T4: 55 – 150 nmol/L
• Lies posterior to the strap muscles, and weights 20 grams • Total T4 levels reflect the output from the thyroid
• Isthmus – connects the 2 lobes of the gland at the midline gland
• Pyramidal lobe – extra lobe present only in 50% of the • Not suitable as a general screening test
population • Increased levels seen in hyperthyroid patients,
elevated Tg levels secondary to pregnancy,
• Ligament of Berry – attaches the gland to the cricoid cartilage
Total T4 estrogen/progesterone use or congenital diseases
• Decreased levels seen in hypothyroid patients,
decreased Tg levels secondary to anabolic steroid
use and protein losing disorders (i.e. nephrotic
syndrome)
o These individuals maybe euthyroid if their free
T4 levels are normal
• Normal: 1.5 – 3.5 nmol/L
• Total T3 levels reflect peripheral thyroid hormone
metabolism
Total T3 • Not suitable as a general screening test
• Measurement of total T3 levels is important for
clinically hyperthyroid patients with normal T4
levels → think T3 thyrotoxicosis
• Normal: 12 – 28 pmol/L
• Half-life: about 7 days
• Measures the biologically active hormone
• Not performed as a routine screening test in
thyroid disease
Free T4 • Its utility is in detecting early hyperthyroidism in
which total T4 levels maybe normal but free T4
levels are raised
Surgery Platinum. 1st ed. 2018. p 233 o Refetoff syndrome: end-organ resistance to T4
ARTERIAL SUPPLY VENOUS DRAINAGE wherein free T4 are increased and TSH levels are
• Superior thyroid artery (from • Superior and middle normal
external carotid artery): thyroid veins → internal • normal: 3 – 9 pmol/L
supplies upper 1/3 of lobe and jugular vein • half-life: about one day
upper 1/2 of isthmus • Inferior thyroid veins → Free T3 • most useful in the diagnosis of early
• Inferior thyroid artery (from brachiocephalic vein hyperthyroidism in which levels of free T3 and T4
rise before total T3 and T4
thyrocervical trunk): supplies
Serum • used for the evaluation of pituitary TSH
lower 2/3 of lobe and lower
TRH secretory function
1/2 of isthmus
• include anti-Tg, antimicrosomal, or anti-TPO and
• Thyroidea ima artery (from
TSI
aorta): present in 1-4% of
Thyroid • anti-Tg & anti-TPO antibody levels: elevated if
patients antibodies with autoimmune thyroiditis
Surgery Platinum. 1st ed. 2018. p. 234

• Lymphatic Drainage • can be elevated in Hashimoto’s, Graves’,


multinodular goiter & thyroid neoplasms
o The first echelon are the nodes of the central compartment of
the neck (level 6), the nodes of the superior mediastinum (level
7), and the lateral cervical nodes (Level 3 and 4)

TOPNOTCH MEDICAL BOARD PREP SURGERY MAIN DIGITAL HANDOUT BY LOUBOMIR ANTONIO, MD Page 25 of 101
For inquiries visit www.topnotchboardprep.com.ph or https://www.facebook.com/topnotchmedicalboardprep/
This handout is only valid for the September 2021 PLE batch. This will be rendered obsolete for the next batch since we update our handouts regularly.
TOPNOTCH MEDICAL BOARD PREP SURGERY MAIN DIGITAL HANDOUT BY LOUBOMIR ANTONIO, MD
For inquiries visit www.topnotchboardprep.com.ph or https://www.facebook.com/topnotchmedicalboardprep/
This handout is only valid for the September 2021 PLE batch. This will be rendered obsolete for the next batch since we update our handouts regularly.
• increased in destructive processes of the • Antithyroid drugs side effects: reversible granulocytopenia,
thyroid gland (thyroiditis) or overactive states skin rashes; fever; peripheral neuritis, vasculitis, aplastic
(graves’ or toxic multinodular goiter) anemia, agranulocytosis
• most important use is for the monitoring of • Treatment of agranulocytosis
Serum
differentiated thyroid cancer recurrence, o Admission to the hospital
Thyroglobulin
after total thyroidectomy and RAI ablation
o Stopping the medication
o elevated anti-Tg antibodies can interfere with
the accuracy of Tg levels and should always be o Provision of broad-spectrum antibiotics
measured when interpreting Tg levels. o Delaying surgery
• normal: 0-4 pg/mL basal • Improved symptoms by 2 weeks, euthyroid in 6 weeks
Serum • secreted by C cells (parafollicular cells) • Associated with high relapse rate
Calcitonin • function: lower serum calcium • Indications
• sensitive marker for medullary thyroid cancer o small goiters (<40g)
o mildly elevated thyroid hormones
• Thyroid Imaging
o rapid decrease in gland size
o Radionuclide Imaging
RADIOACTIVE IODINE
§ Iodine 123: low dose radiation (half-life 12-14 hours) - SURGERY
THERAPY (I-131)
imaging thyroid tissues including lingual, ectopic, metastatic • Indications • Advantages
§ Iodine 131: higher radiation (half-life 8-10 days) - used to o Confirmed cancer or suspicious o Avoidance of surgery
screen and treat differentiated thyroid cancers (Papillary thyroid nodules o Reduced overall
and Follicular CA) o Young treatment cost
§ provides both anatomic and physiologic information o Pregnant or desire to conceive soon o Ease of treatment
§ Cold lesions: trap less iodine than surrounding gland; 20% after treatment § oral dose: 8-12 mCi
risk of malignancy o Severe reactions to antithyroid § euthyroid in 2 mos.
medications • Disadvantages
§ Hot/warm lesions: areas of increased activity
o Large goiters with compressive o Progression of
§ <5% risk of malignancy symptoms ophthalmopathy
Iodine 123 is used for diagnosis. o Reluctant to undergo RAI o Risk of nodular
Iodine 131 is used for treatment. o Moderate to severe ophthalmopathy goiter, thyroid
Dr. Cocos o Must be euthyroid prior to surgery cancer &
o Lugol iodine solution hyperparathyroidism
o KISS (Potassium Iodide Saturated o increased rate of
Solution) overall and
§ given 7 to 10 days pre-op to cardiovascular
decrease vascularity of thyroid and mortality
to lessen risk of thyroid storm • Indications
• Subtotal Thyroidectomy o Older patients with
o Bilateral Subtotal thyroidectomy small to moderate
o Hartley-Dunhill Procedure - size goiters
Removal of 1 entire lateral lobe with o Relapsed after
isthmus and partial/subtotal removal medical or surgical
o PET Scan – for screening for thyroid malignancies when other of opposite lateral lobe therapy
imaging studies are negative • Total or Near total Thyroidectomy o When surgery or
(TT/NTT) medications are
o Ultrasound – for the evaluation of thyroid nodules, cervical o co-existent thyroid CA contraindicated
lymphadenopathy and to guide FNAB o refuse RAI • Absolute
o CT/MRI – most useful for large, fixed, substernal goiters o ophthalmopathy Contraindications
o compressive symptoms o Pregnant women
CLINICAL EVALUATION OF THYROID SIZE o severe reactions to anti-thyroid o Breast feeding
WHO drugs
DESCRIPTION
GRADE
I Neither palpable nor visible goiter Subtotal
II Goiter palpable but not visible in normal neck position thyroidectomy.
III Goiter palpable and visible in normal neck position Done in
toxic/nontoxic
multinodular
BENIGN THYROID DISORDERS: HYPERTHYROIDISM goiter. Most of the
GRAVES DISEASE gland except
lower pole (4-8
• Autoimmune disorder with a strong familial predisposition gram) on both
and female preponderance sides is removed
• Presentation: thyrotoxicosis, diffuse goiter and extrathyroidal
manifestations (ophthalmopathy, dermopathy) Hartley Dunhill
procedure. One
• Etiology: unknown
entire lateral lobe,
o HLA- B8, HLA-DR3, HLA-DQA1 isthmus, and most
• Presentation: heat intolerance, weight loss, diarrhea, part of the opposite
palpitation, amenorrhea, atrial fibrillation, ophthalmopathy, lateral lobe except a
dermopathy small quantity of
• PE: diffusely enlarged thyroid gland tissue in the lower
• Diagnosis: suppressed TSH with or without elevated FT4/FT3 pole/tracheo-
esophageal groove
o (+) eye signs: no other tests needed

o (-) eye signs: iodine 123 uptake and scan (diffuse uptake) subtotal/partial/
one gram is
ANTI-THYROID DRUGS retained.
• Generally given prior to RAI ablation or surgery
• Propylthiouracil (PTU) or methimazole Total
o both inhibits organific binding of iodine and coupling of thyroidectomy is
iodotyrosines done for follicular
o PTU inhibits peripheral conversion of T4 to T3 carcinoma and
o both can cross the placenta and are excreted in breast milk but medullary
less for PTU carcinoma of
o Methimazole is associated with congenital aplasia cutis thyroid.

TOPNOTCH MEDICAL BOARD PREP SURGERY MAIN DIGITAL HANDOUT BY LOUBOMIR ANTONIO, MD Page 26 of 101
For inquiries visit www.topnotchboardprep.com.ph or https://www.facebook.com/topnotchmedicalboardprep/
This handout is only valid for the September 2021 PLE batch. This will be rendered obsolete for the next batch since we update our handouts regularly.
TOPNOTCH MEDICAL BOARD PREP SURGERY MAIN DIGITAL HANDOUT BY LOUBOMIR ANTONIO, MD
For inquiries visit www.topnotchboardprep.com.ph or https://www.facebook.com/topnotchmedicalboardprep/
This handout is only valid for the September 2021 PLE batch. This will be rendered obsolete for the next batch since we update our handouts regularly.
Near total SUBACUTE THYROIDITIS
thyroidectomy is PAINFUL TYPE PAINLESS TYPE
done in papillary
• Viral in origin or post viral • Autoimmune, that may
carcinoma of
inflammatory response occur sporadically or
thyroid. Most of
the gland except (mumps, influenza and other postpartum
lower small tissue respiratory viruses) • Treatment: short-term
of 1 g on one side • Occurs in 30-40 yrs. old thyroid replacement
is usually retained women, self-limited
to safeguard prognosis
recurrent • Has history of preceding
laryngeal nerve URTI
and parathyroid • Treatment: pain
gland. management, short-term
TOXIC MULTINODULAR GOITER (PLUMMER DISEASE) thyroid hormone
replacement, thyroidectomy
• Occurs in older individuals with prior history of nontoxic multi-
for failure of medical
nodular goiters
management
• Signs and symptoms are similar to Graves disease but extra
CHRONIC THYROIDITIS
thyroidal manifestations are absent
HASHIMOTO / LYMPHOCYTIC THYROIDITIS
• Diagnosis: suppressed TSH, elevated FT4 or FT3
o RAI uptake: multiple nodules with increased uptake • Most common inflammatory disorder of the thyroid gland
suppression of the remaining gland and leading cause of hypothyroidism in iodine-sufficient
regions
• Treatment: Near total/Total thyroidectomy
TOXIC ADENOMA (SOLITARY TOXIC NODULE) • Auto-immune disorder
o CD+8 T-cells and antibodies directed against antigens Tg, TPO
• Occurs in young patients with long standing nodule with and TSH-R, HLA B8, DR3 and DR5
symptoms of hyperthyroidism o More common in women (30-50 y)
• Dx: suppressed TSH, elevated FT4 or FT3 o Diffuse infiltration by small lymphocytes and plasma cells
o RAI uptake: solitary hot nodule and suppression of the • Presents with minimally or moderately painless diffusely
remaining gland enlarged thyroid gland
• Treatment: Unilateral lobectomy + Isthmusectomy • Diagnosis: elevated TSH, thyroid autoantibodies
o FNAB – for solitary nodules
BENIGN THYROID DISORDERS: THYROIDITIS • Treatment: thyroid hormone replacement, surgery only
ACUTE SUPPURATIVE THYROIDITIS when CA is suspected or for symptoms
• Common in children
• Often preceded by URTI RIEDEL THYROIDITIS
(Riedel’s struma or invasive fibrous thyroiditis)
• Severe neck pain, fever and chills
• Diagnosis: Leukocytosis , FNAB • Associated with autoimmune diseases and other focal
sclerosing syndromes (IgG4-related diseases)
• Treatment: parenteral antibiotics, drainage of abscess
• Presentation: Painless, hard anterior neck mass
• Recurrent acute thyroiditis - suspect persistent pyriform
• Physical examination: hard, woody thyroid gland
sinus fistula
• Diagnosis: open thyroid biopsy
• Treatment: Wedge excision of the thyroid isthmus

SOLITARY THYROID NODULE


Diagnostic Investigations:
• Fine Needle Aspiration
Biopsy (FNAB)
o Single most important test
in the evaluation of thyroid
nodules
o Optimum cytology: at least
6 follicles each containing at
least 10-15 cells from at
least 2 aspirates
o Disadvantage: Cannot
diagnose Follicular and
Hürthle cell carcinoma and
is less reliable in patients
with history of radiation or
family history of thyroid
cancer

THYROID NODULE
https://qrs.ly/8rcl0o9

Modified Figure 38-14. Schwartz’s Principles of Surgery. 11th ed. 2019

TOPNOTCH MEDICAL BOARD PREP SURGERY MAIN DIGITAL HANDOUT BY LOUBOMIR ANTONIO, MD Page 27 of 101
For inquiries visit www.topnotchboardprep.com.ph or https://www.facebook.com/topnotchmedicalboardprep/
This handout is only valid for the September 2021 PLE batch. This will be rendered obsolete for the next batch since we update our handouts regularly.
TOPNOTCH MEDICAL BOARD PREP SURGERY MAIN DIGITAL HANDOUT BY LOUBOMIR ANTONIO, MD
For inquiries visit www.topnotchboardprep.com.ph or https://www.facebook.com/topnotchmedicalboardprep/
This handout is only valid for the September 2021 PLE batch. This will be rendered obsolete for the next batch since we update our handouts regularly.
BETHESDA SYSTEM FOR REPORTING THYROID CYTOPATHOLOGY
Risk for
Diagnostic Category Usual management
Malignancy (%)
Nondiagnostic or Unsatisfactory
Cyst fluid only Repeat FNA with ultrasound
1-4
Virtually acellular specimen guidance
Other (obscuring blood, clotting artifact etc.)
Benign
Consistent with a benign follicular nodule (includes adenomatoid nodule, colloid
nodule, etc.) 0-3 Critical follow-up
Consistent with lymphocytic (Hashimoto) thyroiditis in the proper clinical context
Consistent with granulomatous (subacute) thyroiditis other
Atypia of Undetermined Significance or Follicular Lesion of Undetermined Significance ~5-15 Repeat FNA
Follicular neoplasm or Suspicious for a Follicular Neoplasm
15-30 Surgical lobectomy
Specify if Hürthle cell (oncocytic) type
Suspicious for Malignancy
Suspicious for papillary carcinoma
Suspicious for medullary carcinoma Near-total thyroidectomy or
60-75
Suspicious for metastatic carcinoma surgical lobectomy
Suspicious for lymphoma
Other
Malignant
Papillary thyroid carcinoma
Poorly differentiated carcinoma
Medullary thyroid carcinoma
Undifferentiated (anaplastic) carcinoma
97-99 Near-total thyroidectomy
Squamous cell carcinoma
Carcinoma with mixed features (specify)
Metastatic carcinoma
Non-Hodgkin lymphoma
Other
• Ultrasound MALIGNANT THYROID DISEASE
o To detect non-palpable thyroid nodules, differentiate solid PAPILLARY CARCINOMA
versus cystic lesions and identify cervical lymphadenopathies
• Most common type of thyroid malignancy (~80%)
o Can identify nodules at risk for being malignant (TI-RADS)
Ultrasound Features Ultrasound Features Suggesting
o Predominant in children, childhood radiation exposure and
Suggesting Malignancy in Malignancy in a Cervical Lymph iodine sufficient areas
a Thyroid Nodule Node o Seen in women 30-40 years old
• Hypoechogenicity • Complex echo pattern or irregular o Has the best prognosis
• Microcalcifications hyperechoic small intranodular • Presentation: slow growing, painless mass, palpable
• Irregular or blurred structures lymphadenopathy
margins • Irregular diffuse intranodular blood flow • Age – most important prognosticating factor
• Increased nodular blood • Solbiati index (ratio of largest to • Diagnosis: FNAB
flow visualized by smallest diameter of a lymph node)
o (+) Orphan Annie nuclei (large round cells with a dense
Doppler = 1 (lymph node is rounder than
• Evidence of tumor long) nucleus and clear cytoplasm)
invasion or regional • Taller than wide o (+) Psammoma bodies (concentric lamellated calcified structures)
lymph node metastases • Loss of fatty hilum
• Thyroid Function test (TSH)
o First test that must be requested in the evaluation of
thyroid nodules
o Euthyroid/Hypothyroid: Perform FNAB
o Hyperthyroid: Perform RAI scan
Management
• Malignant: Perform thyroidectomy
• Simple thyroid cysts:
o Perform aspiration (up to 3x)
o Perform unilateral lobectomy: >3 attempts, cysts >4cms,
complex cysts • Treatment:
• Colloid adenoma o High-risk tumors/ bilateral: perform TT/NTT
o <3cms: observe +/- administer L-thyroxine o Low-risk tumors: controversial, perform TT/NTT
o 3cms or more: lobectomy + isthmusectomy o Minimal papillary cancer: perform lobectomy + isthmusectomy
American Thyroid Association Guidelines 2015
Classification of Thyroid Cancer Based on Histology
Lobectomy + isthmusectomy can be done for papillary thyroid
• Papillary cancer
carcinoma if:
o Classic morphology
o Encapsulated variant • No prior radiation • Tumor size <4 cm
o Follicular variant • No distant metastases • No familial disease
o Aggressive variants (diffuse • No cervical lymph node metastases • Unifocal distribution
Differentiated
sclerosing variant, tall cell • No extrathyroidal extensions
thyroid cancer
variant, columnar cell variant)
Follicular FOLLICULAR CARCINOMA
• Follicular Cancer
epithelial • 10% of thyroid malignancies
o Classic morphology (called
cell o More common on iodine deficient areas, women 50 y/o
“Follicular Carcinoma”)
o Hürthle cell variant • Presentation: solitary nodule with history of rapid size increase
Poorly and long standing goiter, hematogenous spread is more common
differentiated • Insular carcinoma • Diagnosis: cannot be diagnosed by FNAB (capsular and
thyroid cancer
vascular invasion not seen in FNAB)
Undifferentiated
cancer
• Anaplastic carcinoma • Treatment
Parafollicular cell or C Cell • Medullary carcinoma o Follicular neoplasm: Perform lobectomy + isthmusectomy
Surgery Platinum. 1st ed. 2018. p. 242 o Older and >4cms: Perform TT/NTT

TOPNOTCH MEDICAL BOARD PREP SURGERY MAIN DIGITAL HANDOUT BY LOUBOMIR ANTONIO, MD Page 28 of 101
For inquiries visit www.topnotchboardprep.com.ph or https://www.facebook.com/topnotchmedicalboardprep/
This handout is only valid for the September 2021 PLE batch. This will be rendered obsolete for the next batch since we update our handouts regularly.
TOPNOTCH MEDICAL BOARD PREP SURGERY MAIN DIGITAL HANDOUT BY LOUBOMIR ANTONIO, MD
For inquiries visit www.topnotchboardprep.com.ph or https://www.facebook.com/topnotchmedicalboardprep/
This handout is only valid for the September 2021 PLE batch. This will be rendered obsolete for the next batch since we update our handouts regularly.
HÜRTHLE CELL CARCINOMA COMPLICATIONS OF THYROID SURGERY
• A subtype of Follicular CA, cannot be diagnosed by FNAB, relies • Hematoma formation (paratracheal area) secondary to massive
on the presence of capsular and vascular invasion bleeding → needs immediate surgery to prevent airway compromise
• Distinct features: • RLN injury and airway compromise
o More often multifocal and bilateral • Injury to the parathyroid glands – transient and permanent
o Not avid for RAI hypocalcemia
o More likely to metastasize to LN and to distant sites • Injury to nearby structures ( trachea, esophagus, etc.)
o Higher mortality
• Treatment: similar to Follicular cell CA ✔ GUIDE QUESTIONS
Regarding salivary gland tumors, which one of the following statements
POSTOPERATIVE MANAGEMENT OF
is true?
DIFFERENTIATED THYROID CANCER
a. The majority of malignant salivary gland tumors arise in the
Radioactive • Reduces the recurrence of the disease, and parotid gland
iodine therapy improves survival b. Most parotid neoplasms are malignant
• Used for unresectable, locally invasive, or c. Fine needle aspiration biopsy is recommended for all suspected
External beam
recurrent disease salivary gland malignancies
radiotherapy
• Also used for those with bone metastases d. Minor salivary gland tumors occur most commonly in the floor
• Used for replacement therapy after TT/NTT of the mouth
L-thyroxine • Also suppresses TSH (growth factor of the The likelihood of a given tumor’s being malignant is lowest in the parotid
thyroid gland) via negative feedback gland (approximately 20%), followed by the submandibular salivary
gland (approximately 50%) and sublingual glands (nearly 100%).
POSTOPERATIVE SURVEILLANCE OF PATIENTS WITH
However, because more than 75% of all salivary gland tumors occur in the
DIFFERENTIATED THYROID CANCER
parotid gland, the parotid gland accounts for the majority of the
• Elevated levels is highly suggestive of malignant salivary gland tumors. The diagnostic evaluation of a salivary
metastatic or persistent normal thyroid gland mass depends on the location and clinical scenario. FNAB is not
Thyroglobulin
tissue indicated for all parotid tumors, since a tissue diagnosis does not change
measurement
• Levels are usually requested at 6 months the treatment plan for a patient with a small, mobile mass clearly within
initially then annually thereafter the gland. When the location is uncertain, the history suggests the
possibility of metastatic disease, or the tumor size or location indicates a
MEDULLARY THYROID CARCINOMA difficult facial nerve dissection, FNAB may be helpful. Biopsy, usually a
punch or excisional biopsy, should be performed for suspected minor
• Arises from the neuroendocrine parafollicular cells (C cells) salivary gland tumors, the most common site of which is the palate, usually
located at the superolateral lobes of the thyroid gland at the junction of the hard and soft palate. Like FNAB, imaging studies (CT
o 75% sporadic or MRI) should be used when they are likely to augment the clinical
o 25% familial assessment of staging and affect treatment planning.
• Secondary to germ line mutation at the ret protooncogene Dr. Cocos
A 40-year-old woman comes to the clinician’s office with a thyroid mass,
o MEN 2A: MTC, pheochromocytoma, primary HPT which is confirmed on FNA and UTZ to be unilateral, 3.2 cm follicular
o MEN 2B: MTC, pheochromocytoma, Marfanoid habitus, neoplasm. She has been completely asymptomatic. What will the next
mucocutaneous ganglioneuromatosis intervention be?
o Familial MTC: MTC a. Total thyroidectomy
• Produces: calcitonin, CEA, CGRP, histaminidase, serotonin b. Hemithyroidectomy or isthmusectomy
• Pathology: c. Excisional biopsy
d. Core-needle biopsy
o (+) unilateral in sporadic cases, bilateral in familial cases
e. Thyroid suppression via T3 or T4 analogues
o (+) for amyloidosis (stains with Congo red)
• Diagnosis: FNAB (all must be screened for RET point The presence of a follicular neoplasm as confirmed by FNA mandates
further evaluation, since FNA does not provide enough information
mutations, pheochromocytoma, primary HPT)
about tissue architecture to differentiate between a benign follicular
• Treatment: adenoma and a follicular carcinoma. Vascular or capsular invasion
o Operation of choice: TT + routine bilateral central neck confirms the presence of carcinoma. The management of small,
dissection unilateral follicular lesions is controversial (total versus hemi-
§ TT before age 6 for MEN 2A thyroidectomy with frozen section). However, lesions larger than 4 cm
§ TT before age 1 for MEN 2B should be treated with total thyroidectomy, since multicentricity
• Postoperative follow-up: annual calcitonin and CEA becomes more common as tumor size increases. Total thyroidectomy
also facilitates the effectiveness of postoperative radioactive iodine,
• Survival: best survival is seen in non-MEN familial MTC
since no residual thyroid tissue remains to serve as a sink for the
• For MEN syndromes, address pheochromocytoma and HPT first radioisotope. Another form of biopsy (excisional or CNB) is not
warranted and thyroid suppression is not an option in this case.
ANAPLASTIC CARCINOMA Dr. Cocos
During a total thyroidectomy for papillary cancer, the clinician observes
• 1% of all thyroid malignancies, seen in women in the 7th-8th an intact recurrent laryngeal nerve on the right side and a completely
decade transected nerve on the left, with both ends in view. What should
• Long standing neck mass which rapidly enlarges, with symptoms management of this patient at this point entail?
of local invasion, with LN and distant metastases at presentation a. Complete the operation and evaluate the vocal cords
• Diagnosis: postoperatively via flexible bronchoscopy
o FNAB: (+) giant multinucleated cells b. Perform intraoperative flexible bronchoscopy to evaluate vocal cords
o Incision biopsy may occasionally be needed c. Repair the nerve using 8.0 monofilament sutures
d. None of the above
• Treatment: thyroidectomy (still with minimal survival)
If the recurrent laryngeal nerve is injured or transected during an
otherwise uncomplicated operation, it should be repaired using loupes
THYROID LYMPHOMA or an operating microscope to visualize the field, and 8.0 or 9.0
• Most are non-Hodgkin B-cell type monofilament sutures to anastomose the cut ends of the nerves. There
• Usually from Hashimoto thyroiditis, symptoms similar to is no role for flexible bronchoscopy either intraoperatively or
anaplastic carcinoma although often painless postoperatively unless there is uncertainty about the injury or the
function of the contralateral nerve.
• Diagnosis: core-needle/open biopsy Dr. Cocos
• Treatment: R-CHOP regimen (Rituximab, cyclophosphamide,
doxorubicin, vincristine and prednisone) PARATHYROID
EMBRYOLOGY
METASTATIC CARCINOMA TO THE THYROID • Superior parathyroid glands arise from 4th branchial pouch
• Renal cell CA: most common malignancy that metastasize to • Inferior parathyroid glands arise from 3rd branchial pouch
the thyroid o Superior parathyroids are more consistent in position –
• Treatment: treat accordingly (lobectomy may be needed) posterior aspect of upper and middle thyroid lobes at the level
of the cricoid cartilage
o Inferior parathyroids – usually located 1cm from where
inferior thyroid artery crosses the RLN
TOPNOTCH MEDICAL BOARD PREP SURGERY MAIN DIGITAL HANDOUT BY LOUBOMIR ANTONIO, MD Page 29 of 101
For inquiries visit www.topnotchboardprep.com.ph or https://www.facebook.com/topnotchmedicalboardprep/
This handout is only valid for the September 2021 PLE batch. This will be rendered obsolete for the next batch since we update our handouts regularly.
TOPNOTCH MEDICAL BOARD PREP SURGERY MAIN DIGITAL HANDOUT BY LOUBOMIR ANTONIO, MD
For inquiries visit www.topnotchboardprep.com.ph or https://www.facebook.com/topnotchmedicalboardprep/
This handout is only valid for the September 2021 PLE batch. This will be rendered obsolete for the next batch since we update our handouts regularly.
ANATOMY (A) Metastasis from a parathyroid carcinoma
(B) Osteitis fibrosa cystica (brown tumor) and subperiosteal
• Blood supply: mostly inferior thyroid artery resorption of the phalanges
• Thymus: most common location of supernumerary glands (C) Dermoid cyst
• Paraesophageal: most common location of ectopic parathyroids (D) Eosinophilic granuloma
(E) Chondroma
PRIMARY HYPERPARATHYROIDISM (PHPT) Osteitis fibrosa cystica is a skeletal disorder caused by excess
• ↑ PTH production with resultant increase in the level of Ca parathyroid hormone from overactive parathyroid glands. This
stimulates the activity of osteoclasts, cells that breakdown bone, in a
• Etiology: parathyroid adenoma (80%) > parathyroid
process known as osteoclastic bone resorption.
hyperplasia (15-20%), parathyroid carcinoma (1%) Dr. Cocos
• Clinical Manifestations (most are asymptomatic)
o Pentad: kidney stones, painful bones, abdominal groans,
psychic moans, fatigue overtones
BREAST
• Physical Findings: usually non-palpable EMBRYOLOGY
• Diagnosis: • Appearance of mammary ridges (milk lines) at the 5th or 6th
o ↑ serum Ca, ↑ intact PTH or two-site PTH levels, ↓↓serum weeks of gestation – extends from the axillary to inguinal area
phosphate • Responds to ovarian estrogen and progesterone at puberty
o Elevated 24-hour urine Ca • Complete development at the time of pregnancy
• Treatment: if symptomatic - Parathyroidectomy ANOMALIES OF THE BREAST
o Indications for Parathyroidectomy (asymptomatic) Polymastia • Presence of accessory breasts
§ Serum Ca >1mg/dL above upper limits of normal Polythelia • Presence of accessory nipples
§ Life threatening hypercalcemic crisis Amastia • Absence of the breast
§ Creatinine clearance reduced by 30% Athelia • Absence of the nipple
§ (+) kidney stones • Hypoplasia or complete absences of breast
§ Markedly elevated 24-h urine Ca (>400mg/d)
Poland • Costal cartilage and rib defects
§ Decreased bone mineral density (>2.5 SD) syndrome • Hypoplasia of subcutaneous tissues of chest wall
§ Age less than 50 years • Brachysyndactyly
o Preoperative localization: Sestamibi scan
• Webbing between the breasts across the
o Adenoma: perform resection of the involved gland Symmastia
midline
o Hyperplasia: perform 3.5 parathyroidectomy or total
parathyroidectomy + auto transplantation FUNCTIONAL ANATOMY
o Carcinoma: perform en bloc resection + ipsilateral thyroid lobe • Composed of 15 to 20 lobes
• Cooper suspensory ligament - provides structural support
HYPERCALCEMIC CRISIS • Extends from the 2nd or 3rd rib superiorly to the 6th or 7th rib
• Presents acutely with nausea, vomiting, fatigue, muscle inferiorly at the inframammary fold, from the lateral border of
weakness, confusion and decreased level of consciousness sternum to anterior axillary line (axillary tail of Spence)
• Usually from parathyroid carcinoma and familial HPT cases • The upper outer quadrant contains a greater volume of tissue
compared to the other quadrant (most common location of
• Tx:
o IV 0.9% saline hydration – mainstay treatment malignant mass)
o Surgery only when serum calcium is normal

HYPERCALCEMIA
https://qrs.ly/1wcl0pz

SECONDARY HYPERPARATHYROIDISM
• Occurs among patients with chronic renal failure
• Treatment: directed to the underlying disorder BLOODY SUPPLY
o Phosphate binding antacids • Perforating branches of the internal mammary artery
o Oral calcium and vitamin D • Lateral branches of the posterior intercostal arteries
o Surgery for uncontrolled symptoms: 3.5 parathyroidectomy or • Branches from the axillary artery, including the highest thoracic,
total parathyroidectomy + auto-transplantation lateral thoracic, and pectoral branches of the thoracoacromial artery
VENOUS DRAINAGE
TERTIARY HYPERPARATHYROIDISM • Follows the course of the arteries
• Persistent hyperparathyroidism and hypercalcemia following o Perforating branches of the internal thoracic vein
successful renal transplant or resolution of underlying o Perforating branches of the posterior intercostal veins
disorder o Tributaries of the axillary vein
• Due to irreversible parathyroid gland hyperplasia with • Batson vertebral plexus
autonomous PTH production o Extends from the base of the skull of the sacrum
• Treatment: surgery for symptomatic patients o Provide a route for breast cancer metastases
o 3 ½ parathyroidectomy or total parathyroidectomy + auto INNERVATION
transplantation • Lateral cutaneous branches from the 3rd-6th intercostal nerves
• Intercostobrachial nerve – (2nd intercostal nerve)
• Supraclavicular nerve from the cervical plexus
HYPERPARATHYROIDISM
LYMPHATICS
https://qrs.ly/5ccl0q5
• Lateral to the • Scapular group
Level
pectoralis minor • External mammary group
I
muscle • Axillary vein group
✔ GUIDE QUESTION • Deep to the
A 22-year-old student is scheduled to undergo parathyroidectomy for Level • Central group
pectoralis minor
hyperparathyroidism associated with familial multiglandular II • Interpectoral group (Rotter nodes)
muscle
syndrome. His sister developed peptic ulcer disease secondary to a Level • Medial to the
Zollinger-Ellison (hypergastrinemia) tumor of the pancreas. On • Subclavicular group
III pectoralis minor
examination, a swelling was noted over the posterior aspect of the
patient’s fifth rib. What is the most likely finding?
TOPNOTCH MEDICAL BOARD PREP SURGERY MAIN DIGITAL HANDOUT BY LOUBOMIR ANTONIO, MD Page 30 of 101
For inquiries visit www.topnotchboardprep.com.ph or https://www.facebook.com/topnotchmedicalboardprep/
This handout is only valid for the September 2021 PLE batch. This will be rendered obsolete for the next batch since we update our handouts regularly.
TOPNOTCH MEDICAL BOARD PREP SURGERY MAIN DIGITAL HANDOUT BY LOUBOMIR ANTONIO, MD
For inquiries visit www.topnotchboardprep.com.ph or https://www.facebook.com/topnotchmedicalboardprep/
This handout is only valid for the September 2021 PLE batch. This will be rendered obsolete for the next batch since we update our handouts regularly.
MASTITIS
• Epidemic puerperal mastitis
o Due to MRSA transmitted via the suckling neonate
o Treatment: cessation of breastfeeding, antibiotics, incision
and drainage
• Non-epidemic (sporadic) mastitis
o Involvement of the interlobular connective tissue
o Nipple fissuring and milk stasis
o Treatment: emptying the breast using suction pumps;
antibiotics as needed
ZUSKA’S DISEASE
BREAST DEVELOPMENT AND FUNCTION
• Also called recurrent periductal mastitis
• Orchestrated by hormones:
Estrogen • For ductal development
• Recurrent retroareolar infections and abscesses
Progesterone • For lobular development
• Smoking is a risk factor
Prolactin • For lactogenesis • Treatment: administer antibiotics and perform I&D
LH/FSH • For regulation of estrogen and progesterone release MONDOR DISEASE
• Variant of thrombophlebitis that involves the superficial veins of
GYNECOMASTIA the anterior chest wall and breast (involving the lateral thoracic
• Enlargement of the male breast vein, thoracoepigastric vein, superficial epigastric vein)
• Breast tissue measuring at least 2 cm in diameter • Presentation:
• Excess of circulating estrogens in relation to testosterone o Acute pain at the lateral aspect of the breast
o Physiologic – neonatal, adolescence, senescence o Tender, firm cord following the distribution of the veins
• Does not predispose the male breast to cancer except when it is • Benign and self-limited, resolves in 4 to 6 weeks
due to hypoandrogenic state (Klinefelter syndrome) • Treatment
• Clinical Grade o Warm compress
Grade I • Mild breast enlargement • (-) skin o Anti-inflammatory medications
Grade IIa redundancy o Restriction of motion
• Moderate breast enlargement
Grade IIb o Brassiere support
• (+) skin
• Marked breast enlargement o Surgery – excision of the involved vein segment
Grade III redundancy
with ptosis
• Treatment ✔ GUIDE QUESTION
• For androgen deficiency: testosterone administration A patient presents 1 month after a benign right breast biopsy with a
Non- lateral subcutaneous cord felt just under the skin and causing pain. The
• Discontinuation of offending drugs with estrogenic activity
surgical etiology of this condition is?Fat necrosis
• Danazol (but androgenic side effects are considerable)
(A) Infection
• Considered when: (B) Superficial thrombophlebitis
o Unresponsive to medical treatment (C) Suture granuloma
o Does not resolve spontaneously (D) Misdiagnosed breast cancer
o Causing discomfort
Surgical
o Causing psychological distress Superficial thrombophlebitis
o Long-standing is also known as Mondor
• Includes: local excision, liposuction, subcutaneous disease, classically described
mastectomy as painful superficial
Surgery Platinum. 1st ed. 2018
subcutaneous cord-like
✔ GUIDE QUESTION
structure (vessel). Also
A 25-year-old nonalcoholic man has noticeable right gynecomastia known as subacute
since age 20. He is most uncomfortable and reluctant to swim or subcutaneous trunculitis of
exercise at a gym for fear of being an object of derision. He should be the anterolateral chest wall,
advised to have which of the following? it is a rare superficial
(A) Right mastectomy thrombosis most often
(B) Observation affecting the subcutaneous
(C) Needle biopsy of the breast A: Lateral thoracic vein
vessels of the anterior or B: Thoraco epigastric vein
(D) Endocrine workup and right subcutaneous mastectomy lateral chest wall.
(E) Testosterone therapy by transdermal patch C: Superior epigastric vein
Dr. Cocos
Work-up for gynecomastia should include liver function tests and assays for
BENIGN DISORDERS AND DISEASES
follicle-stimulating hormone, luteinizing hormone, human chorionic
gonadotropin, thyroid-stimulating hormone, thyroxine, estrogen, estradiol, and NON- PROLIFERATIVE ATYPICAL
testosterone levels in children and in individuals with progressive disease. PROLIFERATIVE DISORDERS PROLIFERATIVE
Subcutaneous mastectomy (removal of most of the breast tissue with LESIONS* WITHOUT ATYPIA LESIONS**
preservation of skin and the nipple-areola complex) is a feasible option if (No increased risk) (No or slightly
mammography is negative for carcinoma, otherwise an oncologic resection is increased risk)
warranted (MRM, BCS, etc.). there is no need to do a biopsy since there is no • Cysts and • Sclerosing adenosis
• Atypical lobular
palpable mass. Observation is not the best option for unilateral gynecomastia. apocrine hyperplasia
• Radial and complex
Dr. Cocos metaplasia • Atypical ductal
sclerosing lesions
INFECTIOUS AND INFLAMMATORY DISORDERS • Duct ectasia hyperplasia
• Moderate or florid
• Mild ductal ductal epithelial
BACTERIAL INFECTION epithelial hyperplasia
• Breast abscess hyperplasia • Intraductal
o Staphylococcal: (+) localized and deep • Calcifications papillomas
o Streptococcal: (+) diffuse and superficial • Fibroadenomas
• Diagnosis: history and physical examination, ultrasound *Non-proliferative lesions account for most benign conditions
• Treatment: **Atypical proliferative lesions have some features of carcinoma in situ
(CIS) but either lack a major defining feature of CIS or have the features in
o Antibiotic therapy + surgical drainage
less than fully developed form
o Needle aspiration: accepted initial approach when the skin is intact
o Operative drainage: reserved for unresponsive or when
overlying skin is compromised BENIGN BREAST LESION
RECURRENT ABSCESS https://qrs.ly/m9cl0r1
• Cultures for acid-fast bacilli, anaerobic and aerobic bacteria and
fungi (e.g. blastomycosis or sporotrichosis)
• Long term antibiotic therapy
• Biopsy of the abscess cavity to rule out cancer

TOPNOTCH MEDICAL BOARD PREP SURGERY MAIN DIGITAL HANDOUT BY LOUBOMIR ANTONIO, MD Page 31 of 101
For inquiries visit www.topnotchboardprep.com.ph or https://www.facebook.com/topnotchmedicalboardprep/
This handout is only valid for the September 2021 PLE batch. This will be rendered obsolete for the next batch since we update our handouts regularly.
TOPNOTCH MEDICAL BOARD PREP SURGERY MAIN DIGITAL HANDOUT BY LOUBOMIR ANTONIO, MD
For inquiries visit www.topnotchboardprep.com.ph or https://www.facebook.com/topnotchmedicalboardprep/
This handout is only valid for the September 2021 PLE batch. This will be rendered obsolete for the next batch since we update our handouts regularly.
CANCER RISK • (+) family history of breast cancer: the greater the number of
Non-proliferative lesions None relatives affected, the closer the genetic relationship, the younger the
Sclerosing adenosis None age at diagnosis, and the presence of bilateral versus unilateral
Intraductal papilloma None disease all increased the likelihood of development of breast cancer
Florid hyperplasia 1.5 to 2-fold in an individual.
Atypical lobular hyperplasia 4-fold
Atypical ductal hyperplasia 4-fold
BREAST CANCER RISK ASSESSMENT
Ductal involvement by cells of atypical ductal • Gail model
--
hyperplasia o Age
LCIS 10-fold o Age at menarche
DCIS 10-fold o Age at first live birth
✔ GUIDE QUESTIONS o Number of breast biopsy specimens
A 35-year-old professional dancer presents with a well-defined, tense, o History of atypical hyperplasia
smooth mass in the upper outer quadrant of the left breast. She states o Number of first-degree relatives with breast cancer
that the mass becomes larger just before onset of her periods.
Aspiration yields a clear yellow fluid and the mass disappears. The most BREAST CANCER RISK MANAGEMENT
likely diagnosis is: • Screening mammogram:
(A) Fibroadenoma in a cyst o Baseline mammogram at age 35
(B) Fibrocystic disease of the breast o Annual mammogram beginning at age 40 with annual clinical
(C) Carcinoma in a cyst
breast exam and breast cancer awareness
(D) Lipoma
(E) Galactocele o Biennial screening mammography for women age 50 to 74 years
Fibrocystic breast disease is the most common benign type of breast • Chemoprevention
disease. The etiology of benign breast disease has demonstrated a o Selective estrogen receptor modulators (SERMs): Tamoxifen
strong clinical association with women receiving estrogen and anti- and Raloxifene
estrogen treatment. It is associated with the menstrual cycle and may o Aromatase inhibitors: Exemestane
present as cyclic breast pain, tenderness, lumps (cysts) and clear • Prophylactic Mastectomy for BRCA mutation carriers
nipple discharge. The rest of the choices don’t vary with the menstrual
cycle and don’t disappear completely after aspiration. BRCA1 BRCA2
Dr. Cocos Chromosome 17 Chromosome 13
A 36-year-old woman complains of a 3-month history of bloody Autosomal dominant – tumor suppressor genes
discharge from the nipple. At examination, a small nodule is found, • Involved in 45% of hereditary • 85% lifetime risk for Breast
deep to the areola. Careful palpation of the nipple areolar complex
breast CA and 80% of CA
results in blood appearing at the 3 o’clock position. Mammogram
findings are normal. What is the likeliest diagnosis?
hereditary ovarian CA • 6% increased risk for Breast
(A) Intraductal papilloma (D) Carcinoma in situ • 90% lifetime risk for breast CA CA in men
(B) Breast cyst (E) Fat necrosis BRCA1 BRCA2
(C) Intraductal carcinoma • Associated CA are poorly • Associated CA are well
Most common cause of bloody nipple discharge is intraductal differentiated and are differentiated and express
papilloma. It is a small, benign tumor that forms in a milk duct usually hormone receptor negative hormone receptors
presenting as a nodule below the areola. C and D choices would result • Early onset, bilateral disease,
in abnormal mammogram findings. A breast cyst typically has clear CA at other sites
yellowish discharge. Fat necrosis is a lump of dead or
damaged breast tissue that appears after breast surgery, radiation, or Risk Management for BRCA Mutation Carriers
trauma which are all not apparent in this case. • Prophylactic mastectomy and reconstruction
Dr. Cocos
• Prophylactic oophorectomy and HRT
An 18-year-old presents with a well circumscribed 2-cm mass in her
• Intensive surveillance for breast and ovarian CA
right breast. The mass is painless and has a rubbery consistency and
discrete borders. It appears to move freely through the breast tissue. o Clinical Breast Exam (CBE) every 6 months
What is the likeliest diagnosis? o Mammogram yearly beginning at age 25 years
(A) Carcinoma (D) Cystosarcoma phyllodes o Breast MRI
(B) Cyst (E) Intramammary lymph nodes o Transvaginal ultrasound (TVS) and CA-125 yearly beginning at
(C) Fibroadenoma age 25
Classic definition of fibroadenoma: well-circumscribed, painless, • Chemoprevention
rubbery, discrete borders, movable mass in young females. A palpable
breast carcinoma would usually present with hard irregular borders
SUPPLEMENT: BREAST CANCER SIGNS AND SYMPTOMS
and is usually fixed especially if in advanced stage. A breast cyst is
usually described as round or oval lump with distinct edges, but is not • Palpable mass (most common)
rubbery. A phyllodes can present with similar PE findings of • Breast enlargement or asymmetry
fibroadenoma but usually they manifest as larger masses and display • Nipple changes - retraction, or discharge
rapid growth and is usually found in much older age group. • Skin dimpling- due to shortening of Cooper's suspensory ligament
Intramammary lymph nodes are nodes surrounded by breast tissue • Ulceration / erythema of the skin
which are usually discovered incidentally on imaging. • Axillary mass or nodes
Dr. Cocos o Firm or hard
o Involved sequentially from the low (level I) to the central (level II)
BREAST CANCER to the apical (level III) lymph node groups.
RISK FACTORS o Axillary lymph node status: most important prognostic correlate of
HORMONAL NON-HORMONAL disease-free and overall survival
• Female gender • Genetic (BRCA 1 or 2, • Peau d'orange (Localized edema): blocked drainage of lymph fluid
Ataxia-telangiectasia, • Musculoskeletal discomfort
• Early menarche (<12y)
Li-Fraumeni, Cowden syndrome) • Distant metastases: most common cause of death in breast cancer
• Obesity
patients
• Nulliparity • Radiation
o Due to neovascularization (hematogenous spread) → cancer cells
• Late menopause (>55y) • Alcohol consumption shed directly to axillary and intercostals veins or vertebral column
• Age >30y/o at 1st pregnancy • Dietary fat via Batson’s plexus of veins.
SUPPLEMENT: RISK FACTORS o Metastatic foci occurs after the primary ca exceeds 0.5 cm in
diameter
• increased exposure to estrogen: early menarche, nulliparity, late
o Most common first site of distant spread is bone (51%) followed by
menopause, older age at first live birth (after the age of 30 y/o),
lung (17%), brain (16%), and liver (6%)
Hormone Replacement Therapy (HRT), obesity, (major source of
estrogen in postmenopausal women is the conversion of PRIMARY BREAST CANCER
androstenedione to estrone by adipose tissue)
• radiation exposure: patients with multiple fluoroscopies, mantle CIS • Cancer cells have not invaded the basement membrane
radiation for treatment of Hodgkin’s lymphoma • Originates from the terminal duct lobular units
• increased alcohol intake (leads to increased estradiol levels) • Develops only in the female breast
• high fat diet (increased serum estrogen levels) LCIS • Cytoplasmic mucoid globules are distinctive
• prolonged use of OCPs (particularly estrogen-plus-progesterone) • Presence of neighboring calcifications: marked increase
and HRT for invasive carcinoma
TOPNOTCH MEDICAL BOARD PREP SURGERY MAIN DIGITAL HANDOUT BY LOUBOMIR ANTONIO, MD Page 32 of 101
For inquiries visit www.topnotchboardprep.com.ph or https://www.facebook.com/topnotchmedicalboardprep/
This handout is only valid for the September 2021 PLE batch. This will be rendered obsolete for the next batch since we update our handouts regularly.
TOPNOTCH MEDICAL BOARD PREP SURGERY MAIN DIGITAL HANDOUT BY LOUBOMIR ANTONIO, MD
For inquiries visit www.topnotchboardprep.com.ph or https://www.facebook.com/topnotchmedicalboardprep/
This handout is only valid for the September 2021 PLE batch. This will be rendered obsolete for the next batch since we update our handouts regularly.
• Also called intra-ductal carcinoma •
10% of all invasive breast cancers
o Characterized by the proliferation of the epithelium •
Frequently multifocal, multicentric, bilateral
(papillary – cribriform – solid – comedo) Invasive •
(+) absence of e-cadherin – diagnostic marker
DCIS • Calcification occurs in the areas of necrosis lobular •
(+) Indian file configuration – small cells arranged
• Regarded as true anatomic precursor of invasive carcinoma carcinoma in a single file orientation
• Classified based on nuclear grade and presence or • (+) Signet ring cell carcinoma – presence of
absence of necrosis intracytoplasmic mucin, displacing the nucleus
Favorable breast cancer histologic types: tubular, mucinous and papillary types
LOBULAR Dr. Cocos
DUCTAL CARCINOMA
CARCINOMA IN SITU ✔ GUIDE QUESTION
IN SITU (DCIS)
(LCIS) A 65-year-old mother presents with a slight change in color of the
Age (in years) • 54-58 years • 44-47 years areola of her left breast. An eczematous rash of the left areola has
• Incidental findings, persisted for the last 3 months. Biopsy of the nipple reveals Paget
mammographic • Incidental on biopsy disease. In Paget disease of the nipple which of the following is TRUE?
Presentation abnormality, nipple • Usually no clinical (A) Carcinoma of the breast is rarely found
discharge, Paget signs (B) Surgical therapy often fails to cure Paget disease
disease, palpable mass (C) The diagnosis should be made by nipple biopsy when
Predominant suspected
• Ducts • Lobules
location (D) The underlying carcinoma when present is very large
Cell size • Medium or large • Small (E) Paget disease of the bone is commonly encountered
• Comedo, cribriform, A punch biopsy is used to confirm a suspected diagnosis of Paget's
Pattern • Solid
micropapillary, solid disease of the nipple. Other statements are false because (A and D)
Calcifications • Present or absent • Usually absent Paget disease is occasionally associated with underlying breast cancer
Relationship to and carcinoma-in-situ (small lesion). (B) Surgery is curative and (E)
invasive breast • Precursor • Risk factor Paget disease of the bone is very rare.
cancer Dr. Cocos

Risk of Four molecular subtypes of breast cancer


subsequent • Higher • Lower
SUBTYPES ER PR HER2 Ki-67
invasive cancers
Luminal A + +/– – <14%
Location of
• Ipsilateral • Bilateral Luminal B + +/– +/– ≥14%
subsequent cancer
Surgery Platinum. 1st ed. 2018. p. 215 HER2+ – – + ≥14%
TNBC – – – ≥14%
INVASIVE BREAST CARCINOMA
DIAGNOSIS
• Chronic, eczematous, eruption of the nipple
• Usually associated with DCIS and invasive cancer
History and PE
• (+) Paget cells • Risk factors, breast mass, enlargement or asymmetry
o Pathognomonic histologic finding • Nipple changes, retraction or discharge, ulceration or
Paget
Disease
o Large, pale, vacuolated cells in the rete pegs of erythema of the skin
epithelium • Axillary mass, musculoskeletal discomfort
• Must be differentiated from melanoma o *Breast pain is usually associated with benign disease
o (+) CEA – Paget disease o *50% of breast complaints have no physical signs
o (+) S-100 – Melanoma
Mammography
• Most common type (80%)
• Screening mammography – used to detect unexpected breast
Invasive • Solitary, firm mass, with poorly defined margins
ductal • Seen in peri/postmenopausal women in their 5th to 6th cancer in asymptomatic women
carcinoma decade of life • Diagnostic mammography – used to evaluate women with
• (+) central stellate configuration with streaks abnormal findings
• Frequent phenotype among BRCA1 o *False positive rate = 10%
• 4% of all invasive breast cancers o *False negative rate = 7%
• Bilateral in 20%, associated with DCIS in 50% • Specific mammographic features that suggest a diagnosis of
Medullary • (+) soft, hemorrhagic and bulky in nature breast cancer
carcinoma • (+) dense lymphoreticular infiltrate of lymphocytes o Solid mass with or without stellate features
and plasma cells o Asymmetric thickening of tissues
• (+) large pleomorphic nuclei
o Clustered microcalcifications (occurs in as many as 50% of
• (+) poorly differentiated sheet-like growth pattern
non-palpable cancers)
• 2% of all invasive breast cancers
Mucinous • More commonly seen in the elderly population
• *33%reduction in mortality
(colloid) • (+) extracellular pools of mucin surrounding low- • *30% increase in overall survival
carcinoma grade cancer cells • Disadvantage: young women with dense breast (do ultrasound
• (+) 90% displays hormone receptors instead)
• 2% of all invasive breast cancers
Papillary • Presents at the 7th decade
carcinoma • Has low rate of axillar node metastasis BREAST SCREENING
• (+) papillae formation with fibrovascular stalks https://qrs.ly/3scl0t0
• 2% of all invasive breast cancers
Tubular
• Presents in perimenopausal or early menopausal periods
carcinoma
• (+) haphazard array of small tubular elements

BIRADS
ASSESSMENT DESCRIPTION OR RECOMMENDATION
CATEGORY
0 • Incomplete • Additional imaging recommended prior mammogram for comparison
1 • Negative (normal finding) • Routine screening recommended
2 • Benign finding • Routine screening recommended
• Very high probability of benign finding
• Probably benign finding
3 o Short interval follow-up recommended (6 months) to establish stability
• <2% risk of malignancy
o Then every 6-13 months for 1-2 years
4A • Low suspicion for malignancy (3-10%)
4B • Intermediate risk for malignancy (11 to 50%) • Perform biopsy, preferably needle biopsy
4C • Moderate risk for malignancy (51-94%)
5 • Highly suggestive of malignancy (> 95%) • Appropriate action should be taken
6 • Biopsy proven malignancy • Assure that treatment is completed
Additional imaging for BIRADS 0 is usually breast ultrasonography
Dr. Cocos

TOPNOTCH MEDICAL BOARD PREP SURGERY MAIN DIGITAL HANDOUT BY LOUBOMIR ANTONIO, MD Page 33 of 101
For inquiries visit www.topnotchboardprep.com.ph or https://www.facebook.com/topnotchmedicalboardprep/
This handout is only valid for the September 2021 PLE batch. This will be rendered obsolete for the next batch since we update our handouts regularly.
TOPNOTCH MEDICAL BOARD PREP SURGERY MAIN DIGITAL HANDOUT BY LOUBOMIR ANTONIO, MD
For inquiries visit www.topnotchboardprep.com.ph or https://www.facebook.com/topnotchmedicalboardprep/
This handout is only valid for the September 2021 PLE batch. This will be rendered obsolete for the next batch since we update our handouts regularly.
Ductography ✔ GUIDE QUESTION
• Primary indication for ductography: bloody nipple discharge, A 35-year-old premenopausal woman whose mother had breast cancer
particularly when the fluid contains blood comes into your office and has been told that she has fibrocystic breasts.
• Contrast media is injected into 1 or more of the major ducts & On examination she has multiple areas of thickening but no discrete
mass. Of the following diagnostic tests, which should be performed?
mammography is performed
(A) Re-examination in 6 months
• Some findings: (B) Bilateral breast ultrasound
o Intraductal papillomas: seen as small filling defects (C) Thermography
surrounded by contrast media (D) Bilateral breast magnetic resonance imaging (MRI) with
o Cancers: may appear as irregular masses or as multiple gadolinium
intraluminal filling defects (E) Spot compression views if an area of discrete asymmetry or
Ultrasound concerning calcifications is seen
• Important in resolving equivocal mammographic findings, The diagnostic study of choice for premenopausal women (more dense
breasts) is breast ultrasonography. Reexamination after 6 months is
defining cystic masses, and demonstrating the echogenic
not an option especially since this patient has a family history of breast
qualities of specific solid abnormalities cancer. Thermography uses a special camera to measure the
• Used to guide fine-needle aspiration biopsy, core-needle biopsy, temperature of the skin on the breast’s surface and has been around
& needle localization of breast lesions for decades but there is no evidence that it’s an effective screening tool.
MRI A breast MRI is overkill and is not recommended as routine screening.
• MRI screening: recommended in women with BRCA mutations A spot compression view or focal compression view is an
• Some scenarios where MRI may be useful: additional mammographic view performed by applying the
compression to a smaller area resulting in better visualization in that
o Evaluation of a patient who presents with nodal metastasis area. Mammograms are usually advised for >40 yrs old since
from breast CA without an identifiable primary tumor premenopausal women have denser breasts.
o Assess response to therapy in the setting of neoadjuvant Dr. Cocos

systemic treatment
o To select patients for partial breast irradiation techniques
o Evaluation of the treated breast for tumor recurrence
BREAST BIOPSY
ADVANTAGE(S) DISADVANTAGE(S)
TECHNIQUE
• Fast, efficient • Inaccurate or inadequate sample
Fine needle
• Office procedure • Dependent on pathologist experience
aspiration
• Inexpensive • Difficult or unable to evaluate in situ disease
biopsy
• Local or without anesthesia • ER/PR/Her2 not available
• Rapid
• Relatively painless
• No incision
Core needle • May give false-negative results due to sampling error
• Can be read by any pathologist
biopsy • Incomplete lesion characterization can occur
• Markers (ER/PR/Her2) routinely available
• Diagnostic technique of choice for patients who
will receive preoperative systemic therapy
• Relatively fast, frozen section possible • Surgical procedure
Incisional biopsy • ER/PR flow studies obtainable • May underestimate extent of in situ disease
• Extent on in situ diseases evaluable • Will require additional surgical procedure to treat
• Same as incisional biopsy • Larger procedure
Excisional
• More complete evaluation of the situ disease • May still underestimate true extent of tumor requiring re-excision
biopsy
• Can evaluate margins of excisions • May make future conservative treatment more difficult
Wire or needle- • Dependent upon skill of radiologist
directed • Done in the operating room if needed
• Used for nonpalpable abnormalities in imaging
excisional • May underestimate extent of tumor
biopsy • May require re-excision
• Accurate image-guidance • Require special mammography equipment
Stereotactic • Cost efficient • Limited sample size
biopsy • Accurate histology • ER/PR may not be available
• Not done in the operating room • May underestimate in situ disease
Surgery Platinum. 1st ed. 2018. p. 223
STAGING M Description
• Clinical staging Mx Mets cannot be assessed
• Pathologic staging M0 No distant metastasis
• Parameters: TNM status, grade of tumor, biomarkers (ER PR M1 Distant metastasis
HER2/neu status, Ki-67)
• Triple negative breast cancers (ER- PR- HER2-) and those with STAGE T N M
higher histologic grades have poorer prognosis 0 Tis N0 M0
I T1 N0 M0
T Description
T0 N1 M0
Tx Cannot be assessed
IIA T1 N1 M0
T0 No evidence
T2 N0 M0
Tis Ca in situ
T1 Tumor 2 cms or less in diameter T2 N1 M0
IIB
T2 Tumor greater than 2 cm but not greater than 5cm T3 N0 M0
T3 Tumor greater than 5 cm T0 N2 M0
Any size with direct extension to the chest wall (4a) T1 N2 M0
T4 IIIA T2 N2 M0
or skin (4b)
T3 N1 M0
N Description T3 N2 M0
Nx Cannot be assessed T4 N0 M0
N0 No regional LN mets IIIB T4 N1 M0
N1 Mets to movable ipsilateral ALN T4 N2 M0
Mets in ipsilateral ALN fixed or matted or ipsilateral internal IIIC Any T N3 M0
N2
mammary LN IV Any T Any N M1
Surgery Platinum. 1st ed. 2018. p. 224
Mets in ipsilateral ALN and internal mammary LN, or
N3
infraclavicular LN, or supraclavicular LN
TOPNOTCH MEDICAL BOARD PREP SURGERY MAIN DIGITAL HANDOUT BY LOUBOMIR ANTONIO, MD Page 34 of 101
For inquiries visit www.topnotchboardprep.com.ph or https://www.facebook.com/topnotchmedicalboardprep/
This handout is only valid for the September 2021 PLE batch. This will be rendered obsolete for the next batch since we update our handouts regularly.
TOPNOTCH MEDICAL BOARD PREP SURGERY MAIN DIGITAL HANDOUT BY LOUBOMIR ANTONIO, MD
For inquiries visit www.topnotchboardprep.com.ph or https://www.facebook.com/topnotchmedicalboardprep/
This handout is only valid for the September 2021 PLE batch. This will be rendered obsolete for the next batch since we update our handouts regularly.
Note: it is important to familiarize yourself with breast cancer staging. If TYPES OF MASTECTOMY
may imememorize kayo na staging, ito na yun! J Personally ang TOTAL MASTECTOMY
minememorize ko lang ay stage IIB: T2N1M0 and T3N0M0. Basta N1, at
least stage IIA. N2, at least stage IIIA. T4, at least stage IIIB, and N3, at
least stage IIIC.
Dr. Cocos
BREAST CANCER MANAGEMENT
• Observation
LCIS • Chemoprevention (Tamoxifen)
• Prophylactic Bilateral Total Mastectomy (TM)
• Low grade: lumpectomy alone
• Intermediate grade: lumpectomy +
DCIS
radiotherapy
• High grade: mastectomy
• Stages I, IIa, IIb
o Breast conservation surgery +/- ± Adjuvant AUTOLOGOUS – TRAM FLAP
chemo/RT
o Lumpectomy
o ALN status assessment (SLNB)
Early o Radiotherapy
invasive • Absolute contraindications to breast
breast conservation surgery
cancer o Prior radiotherapy
o Pregnant (1st and 2nd trimester)
o Persistently positive margins
o Multicentric lesions
o Diffuse microcalcifications AUTOLOGOUS – LATISSIMUS DORSI FLAP
o Connective tissues disorders (scleroderma)

IMPLANTS

SENTINEL LYMPH NODE BIOPSY (SLNB) RECOMMENDATIONS


SLNB
• No further axillary lymph node dissection
negative
• Only micro metastases (approximately 200 cells,
larger than 0.2 mm, but none larger than 2.0 mm) BREAST RECONSTRUCTION
– No further lymph node dissection https://qrs.ly/kkcl0u7
• Meets ALL of the following ACOSOG Z11 criteria: -
SLNB No further lymph node dissection
positive o T1 or T2 tumor TYPE OF
o 1 or 2 positive sentinel lymph nodes (out of 3) DESCRIPTION
MASTECTOMY
o Breast-conserving surgery • Removal of entire breast, including the nipple and
o Whole-breast RT planned Simple
the areola
o No preoperative chemotherapy (or total)
• Does not remove axillary lymph nodes or muscle
mastectomy
SLNB not tissue
• Axillary level I/II lymph node dissection
identified • Preserves the pectoralis major muscle with
removal of axillary lymph nodes
• Combines a simple mastectomy +removal of
SLNB axillary LN (axillary LN dissection)
https://qrs.ly/pscl0tx • Common subtypes:
o Patey – removes the pectoralis minor to allow
Modified
complete dissection of the axillary LN (I, II, and
radical
III)
LEVEL III DISSECTION mastectomy
o Madden and Auchincloss – preserves the
• Level III dissection to the thoracic inlet should be performed pectoralis minor with dissection of axillary
only in cases with gross disease in level II and/or III. lymph node levels I and II
• In the absence of gross disease in level II nodes, lymph node o Scanlon – transects the pectoralis minor muscle
dissection should include tissue inferior to the axillary vein from for complete axillary LN dissection (I, II, and III)
followed by repair of the transected muscle
the latissimus dorsi muscle laterally to the medial border of the
Radical • Removes entire breast, axillary lymph nodes, and
pectoralis minor muscle (level I/II) (Halsted) both pectoral muscles
mastectomy • Rarely done at present
• Removal of the cancerous part of the breast tissue
Partial
and some normal tissue (e.g., lumpectomy,
mastectomy
quadrantectomy)

TOPNOTCH MEDICAL BOARD PREP SURGERY MAIN DIGITAL HANDOUT BY LOUBOMIR ANTONIO, MD Page 35 of 101
For inquiries visit www.topnotchboardprep.com.ph or https://www.facebook.com/topnotchmedicalboardprep/
This handout is only valid for the September 2021 PLE batch. This will be rendered obsolete for the next batch since we update our handouts regularly.
TOPNOTCH MEDICAL BOARD PREP SURGERY MAIN DIGITAL HANDOUT BY LOUBOMIR ANTONIO, MD
For inquiries visit www.topnotchboardprep.com.ph or https://www.facebook.com/topnotchmedicalboardprep/
This handout is only valid for the September 2021 PLE batch. This will be rendered obsolete for the next batch since we update our handouts regularly.
• Removes all pf the breast tissue, the nipple-areola Breast cancer affects about 1 in 3,000 women who are pregnant. MRM
complex (NAC), and the skin overlying a previous is preferred during the 1st and 2nd trimesters of pregnancy and BCS
Skin-sparing biopsy site is only offered during the last trimester because radiotherapy (which
mastectomy • Most of the skin over the breast (other than is part of BCS) should only be done after delivery. Chemotherapy is not
nipple) is left intact given during the first trimester because of increased risk of abortion
• May not be suitable for larger tumors but if warranted can be given during the latter part of pregnancy.
• Combines a skin-sparing mastectomy with Radiotherapy, hormonal therapy (e.g. Tamoxifen) and targeted
Nipple- preservation of the NAC, intraoperative therapy (e.g. Trastuzumab) are reserved after delivery.
Dr. Cocos
sparing pathological assessment of the nipple tissue core,
mastectomy and immediate reconstruction INFLAMMATORY BREAST CARCINOMA (IIIB)
• Permits better cosmesis • Characterized by skin changes of brawny induration,
• A type of skin-sparing mastectomy which erythema with raised edge, and edema (peau d’ orange)
removes tissue through an incision under the
• Skin biopsy reveals permeation of dermal lymphatics with
breast, leaving the skin, areola, and nipple intact
• A bilateral subcutaneous mastectomy is mainly a tumor
prophylactic operation, indicated in patients with • High rate of axillary LN mets (75%) and distant mets (25% )
Subcutaneous widespread fibrocystic disease and in high risk • Treatment:
mastectomy patients with widespread fibrocystic disease o Neoadjuvant chemo with adriamycin-based regimen
• A unilateral subcutaneous mastectomy is o MRM
indicated in patients who have already had a o Adjuvant chemo + RT + hormone therapy
mastectomy for carcinoma and whose remaining
breast has an increased risk also for developing a
carcinoma
MALE BREAST CANCER
Surgery Platinum. 1st ed. 2018. p. 226 • Less than 1% of all breast cancers
✔ GUIDE QUESTION • Preceded by gynecomastia in 20%
After undergoing modified radical mastectomy for cancer of the right • Same survival rate as women stage by stage but do worse
breast, a 52-year-old female teacher becomes aware that the medial
because of advanced stage at presentation
end of her scapula becomes prominent in protraction movements at
the shoulder. She also complains of some weakness in complete • Risk factors: radiation, estrogen therapy, testicular
abduction of the same shoulder. What nerve was injured? feminizing syndromes
(A) Long Thoracic (D) Median • Treatment: MRM+ Chemo + RT + Endocrine therapy
(B) Thoracodorsal (E) Intercostobrachial
(C) Ulnar FOLLOW-UP AFTER TREATMENT
Long thoracic: serratus anterior- protraction of scapular
Thoracodorsal: latissimus dorsi- extension and adduction of shoulder • Every 4 mos. for the first 2 years
Medial pectoral nerve: pectoralis minor and major • Every 6 mos. for the next 3 years
Lateral pectoral nerve: pectoralis major • Every 12 mos. thereafter
Intercostobrachial: sensation of medial upper arm • Monthly self-examination is recommended
Dr. Cocos
• Mammogram yearly to the contralateral breast
ADVANCED LOCOREGIONAL BREAST CANCER & DISTANT
METASTASES PHYLLODES TUMORS
• Neoadjuvant chemotherapy (adriamycin-
Stages
containing agents) then either: • benign, borderline or malignant
IIIa, • Sharply demarcated from surrounding breast tissue
o BCS or MRM + Adjuvant chemo/RT
IIIb • Connective tissue composes the bulk of the tumor
o Antiestrogen Tx (Luminal A and B subtypes)
• Palliative approach • Stroma has greater cellular activity than fibroadenoma
o Antiestrogen o *Evaluation of the number of mitoses and presence or absence
o Chemotherapy of invasion aids in the diagnosis of malignancy
Stage
IV
o Surgery • Treatment:
o Radiotherapy o Excision with a margin of normal tissue (1cm)
o Breast-conserving surgery o May require mastectomy
o Clinical trials o No need for axillary dissection
SUPPLEMENT:
LOCOREGIONAL RECURRENCE MANAGEMENT
SURGICAL TECHNIQUES FOR BREAST SURGERY
If s/p Excision + chemotherapy and hormone therapy + 1. Skin-Sparing Mastectomy: removes ALL breast tissue, the nipple-
MRM radiotherapy areola complex and scars from any prior biopsy procedure
If s/p BCS MRM + chemotherapy and hormone therapy 2. Total (Simple) Mastectomy: removes ALL breast tissue, the
nipple-areola complex and skin
BREAST CANCER TREATMENT DURING PREGNANCY
3. Extended Simple Mastectomy: removes ALL breast tissue, the
1st and 2nd trimester MRM nipple-areola complex, skin and level I axillary lymph nodes
3rd trimester MRM or BCS 4. Modified Radical Mastectomy: removes ALL breast tissue, nipple-
areola complex, skin and the level I and II axillary lymph nodes
✔ GUIDE QUESTIONS
5. Halsted Radical Mastectomy: removes ALL breast tissue and skin,
A 39-year-old patient presents to your office with a left 3.5cm breast
the nipple-areola complex, the pectoralis major and pectoralis
tumor, which on core needle biopsy, is shown to be an invasive ductal
minor muscles and the level I, II and III axillary lymph nodes
cancer. On left axillary examination, she has a hard nonfixed lymph
node. A biopsy of a left supraclavicular node is positive for malignancy.
Her stage is currently classified as? ✔ GUIDE QUESTIONS
(A) IIIC A 58 y/o woman presents with chronic, erythematous, oozing,
(B) IV eczematoid rash involving the left nipple and areola. There are no
(C) IIB breast masses palpable, and her mammogram is normal. Which of the
(D) IIIB following recommendations is appropriate?
(E) IIA a. Referral to a dermatologist
Metastasis to the supraclavicular node is N3, and automatically the b. Oral vitamin E and topical aloe and lanolin
stage is at least IIIC. The complete staging is T2N3Mx. c. Biopsy
Dr. Cocos d. Non allergenic brassiere
An 18-week pregnant, 35-year-old woman presents after undergoing a e. Standard treatment that includes breast conservation
modified radical mastectomy for a 2-cm ductal cancer with one out of
This is a case of Paget’s disease of the breast a rare, primary ductal
fifteen positive axillary lymph nodes.
carcinoma that secondarily invades the epithelium of the nipple and
What should she be informed of regarding breast cancer during
areola. Biopsy of any chronic nipple rash is mandatory and will show
pregnancy?
the distinctive pagetoid cells. Because of the possible invasion of the
(A) She cannot undergo chemotherapy until after she delivers
tumor on the underlying rich lymphatics of the nipple areolar complex,
(B) She should have a therapeutic abortion in order to proceed
mastectomy is usually indicated. In selected cases, breast conservation
with radiotherapy
therapies can also be employed.
(C) Breast cancer is the most common cancer during pregnancy Dr. Cocos
(D) Radiotherapy is indicated
(E) Most of these cancers are ER+
TOPNOTCH MEDICAL BOARD PREP SURGERY MAIN DIGITAL HANDOUT BY LOUBOMIR ANTONIO, MD Page 36 of 101
For inquiries visit www.topnotchboardprep.com.ph or https://www.facebook.com/topnotchmedicalboardprep/
This handout is only valid for the September 2021 PLE batch. This will be rendered obsolete for the next batch since we update our handouts regularly.
TOPNOTCH MEDICAL BOARD PREP SURGERY MAIN DIGITAL HANDOUT BY LOUBOMIR ANTONIO, MD
For inquiries visit www.topnotchboardprep.com.ph or https://www.facebook.com/topnotchmedicalboardprep/
This handout is only valid for the September 2021 PLE batch. This will be rendered obsolete for the next batch since we update our handouts regularly.
If patient with metastatic breast ca is ER (+), which of the following Cells of the Skin
statements are appropriate? Keratinocytes • 90% of the cells of the epidermis
a. Bilateral oophorectomy
b. Antiestrogen drugs (Tamoxifen) Langerhans • Antigen presenting cells (APCs) mostly
c. Hypophysectomy cells found at the stratum spinosum
d. Adrenalectomy • Found at the stratum basale
Melanocytes
e. Aromatase inhibitor • Produces melanin
Patients with high ER & PR levels (based on immunohistochemical • Found at the stratum basale
Merkel cells
stains) have better. The goal is to decrease circulating amounts of • For slow-adapting mechanosensation
estrogen, usually with a receptor-blocking agent (tamoxifen). • Mostly present as effector memory T-
Bilateral oophorectomy in premenopausal women is considered a Lymphocytes
cells
reasonable option if the patient is amenable and has completed family
size. Surgical hypophysectomy & adrenalectomy were at one point • Found in the epidermis of the nipple
considered forms of hormonal manipulation (but risks outweigh their Toker cells • Possible source of Paget’s disease of the
benefits), and are now being replaced “medically” in the form of nipple
GNRH agonists e.g., goserelin and aromatase inhibitors e.g.
anastrozole, which inhibit conversion of androgens to estrogens in the
adrenal gland and peripherally.
Dr. Cocos
A 39-year-old woman presents with an ill-defined 2 cm mass in the
outer quadrant of her breast. Mammography shows very dense tissue
but no discrete lesion. Ultrasound examination shows a solid lesion. An
ultrasound-guided fine needle aspiration (FNA) is performed, and the
aspirate is plated, fixed, and sent to the laboratory for cytologic study.
A highly cellular monomorphic pattern is seen, with poorly cohesive
intact cells, nuclear “crowding” with a variation in nuclear size, radial
dispersion and clumping of the chromatin, and prominent nucleoli.
Which of the following management choices is/are appropriate?
a. MRM
b. Reassuring the patient that the process is benign
c. Lumpectomy, sentinel lymph node biopsy and irradiation
d. Excision of a fibroadenoma with narrow margins
e. Lumpectomy and sentinel lymph node biopsy without irradiation
Aspiration biopsy with a 22-gauge needle is an effective and safe way of
assessing palpable breast lesions. Performing the aspiration under INJURIES TO THE SKIN
ultrasound guidance ensures that the lesion has been sampled • Clean/Clean-Contaminated
thoroughly while under direct vision. Although a smaller volume of • Contaminated/Dirty
tissue is obtained than the core needle biopsy, FNA frequently yields
results that may be equal to core biopsy if read by an experienced
• Bite wounds
cytopathologist. A fibroadenoma would show broad sheets of cohesive o Human bites (Eikenella corrodens)
cells with nuclei that are uniform in size and shape. The chromatin o Animal bites (Pasteurella multocida)
pattern would be finely granular and large numbers of bare nuclei
would be present. The cytologic findings described in this question EXPOSURE TO CAUSTIC SUBSTANCES
is diagnostic of carcinoma. Appropriate management, therefore, • Acid: coagulation necrosis (less severe due to coagulation)
includes either a modified radical mastectomy or lumpectomy, axillary
evaluation by either a sentinel lymph node biopsy or an axillary nodal
• Alkali: liquefactive necrosis (more severe and widespread)
dissection, and whole-breast irradiation. o Examples of alkali – sodium hydroxide (drain decloggers),
Dr. Cocos calcium hydroxide (cement)
• IVF extravasations
SKIN AND SUBCUTANEOUS TISSUE o Dorsum of hand – most common site of extravasation injury in
an adult
ANATOMY AND PHYSIOLOGY OF THE SKIN o Management
Layers • Acidic injury – dilution of the offending agent
• Stratum corneum Initial
(using distilled water or saline) for 30 minutes
• Stratum lucidum – only seen in thick skin (i.e. • Alkaline injury – dilution of the offending agent
management
palms & soles) for 2 hours
Epidermis of caustic
• NEVER NEUTRALIZE THE OFFENDING AGENT
• Stratum granulosum injury
• Prophylactic antibiotic is GENERALLY
• Stratum spinosum AVOIDED
• Stratum basalis
• Papillary layer – upper layer PRESSURE INJURY
Dermis
• Reticular layer – lower layer • Threshold: at least one hour of 60mmHg pressure
Hypodermis • Subcutaneous tissue o Sacral pressure at 150 mmHg when in supine position
Mnemonic: Californians Like Girls in String Bikinis o Ischial pressure at 300 mmHg when sitting
*S. lucidum only present in thick skin like palms and soles
Dr. Cocos
o Sensitivity to ischemia: muscles > skin
• Ischial tuberosity (28%)
Most common
• Greater trochanter (19%)
sites of
• Sacrum (17%)
pressure injury
• Heel (9%)
Stage Non-blanching erythema over intact
1 skin
Stage Partial-thickness injury with
2 blistering or exposed dermis
Full-thickness injury extending
Stages of
Stage down to, but not including, fascia
pressure ulcers
3 and without undermining any
adjacent tissue
Full-thickness skin injury with
Stage
destruction or necrosis of muscle,
4
bone, tendon or joint capsule
• Treatment: avoidance of prolonged pressure to at-risk areas
o Wound care
o Nutritional build-up
o Debridement +/- grafting
TOPNOTCH MEDICAL BOARD PREP SURGERY MAIN DIGITAL HANDOUT BY LOUBOMIR ANTONIO, MD Page 37 of 101
For inquiries visit www.topnotchboardprep.com.ph or https://www.facebook.com/topnotchmedicalboardprep/
This handout is only valid for the September 2021 PLE batch. This will be rendered obsolete for the next batch since we update our handouts regularly.
TOPNOTCH MEDICAL BOARD PREP SURGERY MAIN DIGITAL HANDOUT BY LOUBOMIR ANTONIO, MD
For inquiries visit www.topnotchboardprep.com.ph or https://www.facebook.com/topnotchmedicalboardprep/
This handout is only valid for the September 2021 PLE batch. This will be rendered obsolete for the next batch since we update our handouts regularly.
RADIATION EXPOSURE ACTINOMYCOSIS
• Solar (UV) radiation • Granulomatous suppurative bacterial disease caused by
o UVA (400-320nm) – low energy, more abundant (95%) Actinomyces
o UVB (320-290nm) – responsible for acute and chronic skin • Present as nodules and spread to form draining tracts
effects • Infection usually results following tooth extraction, odontogenic
o UVC (290-200nm) – absorbed by ozone layer infection, or facial trauma
• Acute Radiation • Diagnosis: (+) yellow (sulfur) granules
o Erythema, edema, alopecia • Treatment: combination of penicillin + surgical debridement
o Reepithelialized
VIRAL INFECTIONS - HUMAN PAPILLOMAVIRUS
• Chronic Radiation
o loss of capillaries via thrombosis and fibrinoid necrosis • Morphologic types
o leads to poor perfusion and ulceration o Common wart: verruca vulgaris
o Plantar: verruca plantaris
• For minor skin radiation: use moisturizers for remaining
o Flat: verruca plana
healthy skin
o Venereal: Condyloma acuminata
• For severe skin radiation: surgical excision of damaged tissues

INFECTIONS OF THE SKIN


AND SUBCUTANEOUS TISSUES
• Uncomplicated Skin Infections – cellulitis, folliculitis,
furuncles (boils), carbuncles (cluster of boils)
o Treatment: adequate hygiene, ward compress, oral
antibiotics, incision and drainage
• Complicated Skin Infections - necrotizing Soft Tissue Infections
o Most common sites: external genitalia, perineum, abdominal wall
o Most common organisms (polymicrobial > monomicrobial)
§ Gram (+): group A Streptococci, enterococci, CNS, S. aureus,
S. epidermidis and Clostridium sp. • Histologic findings: papillomatosis, acanthosis, hyperkeratosis
§ Gram (-): E coli, Enterobacter, Pseudomonas, Proteus, • Treatment:
Serratia, Bacteroides o First line – topical salicylic acid, AgNO3, glutaraldehyde
o Risk Factors: DM, CLL, malnutrition, steroid use, chronic o Alternative line – cryoablation, electrodessication, pulsed dye
alcoholism, renal failure, peripheral vascular disease, laser therapy
cirrhosis, autoimmune states
o Classification: INFLAMMATORY DISEASES OF THE SKIN AND
• Polymicrobial
Type I
• Gram (+) cocci, Gram (-) rods, anaerobes, Clostridium
SUBCUTANEOUS TISSUES
• Monomicrobial PYODERMA GANGRENOSUM
Type II
• (b-hemolytic Streptococcus or Staphylococcus) • Rapidly enlarging, necrotic lesion with undermined border and
Type • Vibrio vulnificus infection surrounding erythema
III • Exposure of wound to saltwater • Linked to underlying disease in 50% - IBD, RA, hematologic
o Treatment: malignancy and IgA gammopathy
§ Broad-spectrum IV antibiotics • Diagnosis: recognition of the underlying disease
§ Source control and wide surgical debridement • Treatment: steroids, cyclosporine (for medical treatment) →
§ intensive care and resuscitation surgery (if persistent)
§ IVF, inotropes, ventilator support, etc.
STAPHYLOCOCCAL SCALDED SKIN SYNDROME (SSSS)
HIDRADENITIS SUPPURATIVA/ACNE INVERSA AND TOXIC EPIDERMAL NECROLYSIS (TEN)
• Painful skin condition affecting apocrine gland bearing areas • Presents with skin erythema, bullae formation and skin loss
of the body SSSS TEN
• Located at the axilla, perineum, inframammary and inguinal folds • Exfoliative toxins A
Etiology • Immune-mediated
• Tender deep nodules that expand, coalesce, spontaneously and B
drain, and form persistent sinus tracts Site • Stratum granulosum • Dermo-epidermal junction
• Superimposed bacterial infection during episodic flares • (+) Nikolsky sign – separation of epidermis from dermis with
• More common among women lateral skin pressure
• Associated with smoking and obesity • Both are treated similar to burn (IVF and Biologic dressings)
• Hurley Classification • 1st principle of management in TEN – discontinue offending agent
o Stage I – single/multiple nodules or abscesses without sinus (anti-convulsant, sulfonamides, allopurinol, oxicams, nevirapine)
tracts or scarring
o Stage II – abscess, sinus tracts, scarring SEBORRHEIC AND SOLAR KERATOSES
o Stage III – diffuse interconnected sinus tracts and abscesses • Seborrheic keratoses
• Treatment: o Benign lesions of the epidermis due to clonal keratinocyte and
o Stages I and II – administer clindamycin melanocyte expansion
o Stage III – radical excision, laser treatment, biologic agents o Occurs on the chest, back and abdomen of older individuals
o (+) “stuck on” yellow or light brown, papules/plaques with
✔ GUIDE QUESTION
velvety, greasy texture
A 49-year-old male postman had undergone several operations to excise
recurrent infections in both axillary lesions and perianal region. The • Solar (Actinic ) keratoses
lesions are Hidradenitis Suppurativa. Which is TRUE of these? o (+) atypical keratinocytes, arising in sun-exposed areas
(A) They arise from stratum corneum of skin. o Serves as premalignant lesion (SCC) – serves as predictor of
(B) They are noninflammatory conditions. development of other squamous and basal cell carcinomas
(C) They always require surgical intervention. o Treatment: excision, 5-FU, cautery destruction, dermabrasion
(D) They frequently involve the scalp.
(E) They are usually caused by Staphylococci and Streptococci
NEVI (ACQUIRED, CONGENITAL)
• Acquired melanocytic nevi – junctional, compound, dermal
Hidradenitis suppurativa (HS) is a chronic inflammatory skin disease
brought about by obstruction of apocrine glands (not from stratum
o most undergo involution
corneum) defined by recurrent nodules, tunnels (sinus tracts) and
• Congenital nevi
scarring involving the intertriginous regions (not scalp). First line of
o Premalignant lesions seen in melanoma
treatment would be diet modification, weight reduction, hygiene and
antibiotics if infected (not surgical intervention).
o Giant hairy nevi: occur in a swimming trunk distribution
Dr. Cocos • Treatment: excision
TOPNOTCH MEDICAL BOARD PREP SURGERY MAIN DIGITAL HANDOUT BY LOUBOMIR ANTONIO, MD Page 38 of 101
For inquiries visit www.topnotchboardprep.com.ph or https://www.facebook.com/topnotchmedicalboardprep/
This handout is only valid for the September 2021 PLE batch. This will be rendered obsolete for the next batch since we update our handouts regularly.
TOPNOTCH MEDICAL BOARD PREP SURGERY MAIN DIGITAL HANDOUT BY LOUBOMIR ANTONIO, MD
For inquiries visit www.topnotchboardprep.com.ph or https://www.facebook.com/topnotchmedicalboardprep/
This handout is only valid for the September 2021 PLE batch. This will be rendered obsolete for the next batch since we update our handouts regularly.
GLOMUS TUMOR • Pathology:
• Benign neoplasm of the o SCC arise from the basal layer of epidermis
extremity o Arise in association with an area of preexisting skin damage
• More commonly affects the o Epidermal cytokeratin antibody: specific for squamous
hand epithelium
• Arise from glomus bodies o Tumor grade: determined by the degree of cellular
(neuromyoarterial apparatus) differentiation based on the ratio of atypical cells to normal
• Presents with blue, subungual epithelium
discoloration associated with • Clinical Manifestations:
severe pain, point tenderness o Enlarging bumps that may have an irregular or reddened
and cold sensitivity surface
• Treatment: excision o Classic appearance: (+) hollow ulcer with ‘‘heaped up’’ edges
o SCC has a higher metastatic potential (than BCC)
o Cranial nerve dysfunction (particularly the facial
MALIGNANT TUMORS OF THE SKIN trigeminal distribution): ominous sign of perineural invasion
• Skin cancers can be broadly categorized into two groups: non- that indicates an aggressive tumor
melanoma and melanoma Diagnosis of Nonmelanoma Skin Cancers (BCC and SCC)
• Shave or incisional biopsy techniques: adequate for
nonmelanoma skin cancer
• Unlike melanoma, staging will not be affected by the depth of
the lesion
• Re-excision: strongly recommended for positive specimens
because of the high likelihood of residual microscopic tumor
even if the biopsy site appears healed and free of tumor.
• Vigilance of biopsy is particularly important in:
o Old burn wounds
o Chronic nonhealing wounds
o Scars that undergo acute changes to monitor for malignant
transformation (Marjolin ulcer)
MANAGEMENT OF NONMELANOMA SKIN CANCERS (BCC, SCC)
• Surgical resection with 4-6 mm margins (if possible)
• Moh’s surgery – excellent for face, ears, and locations where
BASAL CELL CARCINOMA
preservation of uninvolved tissues is critical for cosmesis
• Most common type of skin cancer
• Others: curettage and electrodessication, cryosurgery,
• Slow-growing, rarely metastasize, yet locally invasive
radiation
• Most common malignant eyelid tumor and usual for upper lip
malignancy MALIGNANT MELANOMA
• Melanoma occurs in all ethnic groups
VARIANTS REMARKS
• Associated with increased sun exposure and decreased melanin
• Forms a waxy or pearly appearing papule
content
Nodular BCC with raised, well-demarcated borders
• Responsible for the majority of deaths from skin cancer
(most common • Telangiectasias over the translucent surface
variant) or central ulceration • correlates more with the intensity of prior sun exposure than
• Crusting that bleeds with minor trauma cumulative exposure (in contrast to nonmelanoma)
• Mistaken for melanoma because of the • Risk Factors:
increased content of brown or black pigment o A history of five or more sunburns in early life has been
Pigmented BCC (in addition to the features of nodular BCC) associated with a doubled risk of developing malignant
• Occur more frequently in darker skinned melanoma later in life
people o Fair skin and light eyes
• Marked by bluish or gray cystic nodules o Dysplastic nevi and multiple (>50) nevi
Cystic BCC o Prior history of melanoma or family history of melanoma
• May be misidentified as benign cysts
• Scaly patch-like or papule form that varies in o Immunosuppression (transplant patient)
color from pink to red or brown o Xeroderma pigmentosum – impaired nucleotide excision repair
Superficial BCC
• Rarely become invasive • Pathology
• Psoriasis and eczema: can be mistaken diagnoses o (+) migration of melanocytes upward to the dermo-epidermal
• Aggressive variant junction
Micronodular o Depth of invasion dictates management and prognosis
• Well-defined border
BCC
• Less prone to ulceration HISTOLOGIC
• Ill-defined borders that extend beyond DESCRIPTION
Morpheaform SUBTYPES
clinically visible margins Superficial • Most common subtype of melanoma (70%)
and
• Scar-like appearance in a plaque or papule spreading • Generally arise in a preexisting nevus
infiltrating BCC
formation that can in fact be mistaken for • Worst prognosis because of the prominence of
(most Nodular
scar tissue the vertical growth phase
aggressive
• Ulceration, bleeding, and crusting are • Occur in an older patient population
variant)
uncommon (uncommon in individuals younger than 50
Surgery Platinum. 1st ed. 2018. p. 199 Lentigo
years)
maligna
• Grows slowly and superficially
SQUAMOUS CELL CARCINOMA
• Large (>3cm at diagnosis), flat lesions
• 15-20% of all skin cancers • Located in the subungual regions and glabrous
• Men>women Acral skin of palms and soles
• Risk factors: lentiginous • Easy to miss diagnosis, leading to a poor
o Most important risk factor: solar exposure prognosis based on delay in treatment
o Immunosuppression (transplant and AIDS patients • Rare but aggressive subtype of melanoma
o Human papilloma virus (HPV) infection (16 and 18) • propensity toward perineural invasion and
o Old scars or burn sites (Marjolin ulcer) recurrence
o Exposures to arsenic, tar, and other polycyclic aromatic Desmoplastic
• Occur mostly on the head and neck of elderly
hydrocarbons men (6th-7th decade)
o Previous diagnosis of any skin malignancy • Often deep at the time of diagnosis
Surgery Platinum. 1st ed. 2018. p. 2018

TOPNOTCH MEDICAL BOARD PREP SURGERY MAIN DIGITAL HANDOUT BY LOUBOMIR ANTONIO, MD Page 39 of 101
For inquiries visit www.topnotchboardprep.com.ph or https://www.facebook.com/topnotchmedicalboardprep/
This handout is only valid for the September 2021 PLE batch. This will be rendered obsolete for the next batch since we update our handouts regularly.
TOPNOTCH MEDICAL BOARD PREP SURGERY MAIN DIGITAL HANDOUT BY LOUBOMIR ANTONIO, MD
For inquiries visit www.topnotchboardprep.com.ph or https://www.facebook.com/topnotchmedicalboardprep/
This handout is only valid for the September 2021 PLE batch. This will be rendered obsolete for the next batch since we update our handouts regularly.
• Clinical Manifestations • administer high doses of chemotherapy to an
o Variegated color, irregular raised surface, irregular perimeter, affected extremity while the drug and the limb is
Isolated
and surface ulceration heated to 41°C for 90 minutes
limb
o Most common sites of metastases – lung and liver • Both the therapeutic effect and adverse side effects
perfusion
are localized to the limb of interest
o Most common non-cutaneous site – ocular melanoma
• Melphalan: drug most commonly used
o Mucosal melanomas – worse prognosis
• Some success in treating patients with nodal or in-
• Diagnosis (Malignant melanoma) Interferon
transit metastasis or node-negative thick melanoma
o ABCDE Rule (evaluates lesions suspicious for melanoma): alpha-2b
(>4mm)
o Asymmetry Surgery Platinum. 1st ed. 2018. p. 203

o Border irregularity
o Color variation
o Diameter >6mm SLNB MELANOMA
o Evolution (change in lesion over time) https://qrs.ly/n3cl0vs

ABDOMINAL WALL HERNIAS


HERNIA
• The protrusion of tissue through a defect in fascial and/or
muscular layer(s) that normally contain it

• Biopsy:
o Complete excision with a 1-3 mm margin of normal skin
down to subcutaneous fat should be performed (whenever
possible)
o Incisional or punch biopsy: for large lesions or cosmetically
sensitive areas (e.g., face)
o Fine-needle aspiration biopsy: best diagnostic when an
enlarged lymph node is palpated
• Staging (AJCC)

INGUINAL HERNIAS
Pathophysiology
• 75% of all abdominal wall hernias are found in the groin
• 95% of groin hernias are hernias of the inguinal canal (with
the remainder being femoral hernia)
• Main concern: strangulation – blood supply to that part of the
intestine is interrupted, causing necrosis

LAYERS OF THE ABDOMINAL WALL AND CORRESPONDING


LAYERS IN THE SCROTUM
Note: it is important to obtain full thickness biopsy if melanoma is
ABDOMINAL WALL SCROTUM
suspected because the depth of invasion dictates management and
prognosis. • Skin • Skin
Dr. Cocos • Superficial fatty (Camper • Dartos muscle
✔ GUIDE QUESTION fascia)
A 38-year-old female undergoes removal of a 2 × 1-cm skin lesion shown to • Deep membranous (Scarpa • Colles fascia
be a melanoma. It is reported as Clark level 1, which implies what? fascia)
(A) It is superficial to the basement membrane. • External oblique fascia and • External spermatic fascia
(B) It is 1 mm in thickness.
(C) It has nodal involvement.
aponeurosis (Gallaudet fascia)
(D) It involves the papillary layer. • Internal oblique fascia and • Cremasteric fascia
(E) It involves the reticular dermis. aponeurosis
• Does not reach the
Clark Level 1 is otherwise known as melanoma-in-situ. It does not • Transversus abdominis
invade the dermal layers (papillary and reticular) and is rarely scrotum
associated with nodes. See previous illustration.. • Transversalis fascia • Internal spermatic fascia
Dr. Cocos
• Properitoneal space and fat • Properitoneal space and fat
• Mainstay of treatment: wide surgical excision with • Parietal peritoneum • Tunica vaginalis
Surgery
1-2 cm margin depending on the tumor thickness Surgery Platinum. 1st ed. 2018. p. 417.

Lymph node • Lesions <1 mm: no lymph node dissection (LND)


dissection • Lesions 1-4 mm: sentinel lymph node biopsy (SLNB)
(LND) • Clinically positive node: complete LND
Sentinel • Standard staging procedure to evaluate the
lymph node regional nodes for patients with clinically node-
biopsy negative melanoma
(SLNB) • Identifies the first draining lymph node

TOPNOTCH MEDICAL BOARD PREP SURGERY MAIN DIGITAL HANDOUT BY LOUBOMIR ANTONIO, MD Page 40 of 101
For inquiries visit www.topnotchboardprep.com.ph or https://www.facebook.com/topnotchmedicalboardprep/
This handout is only valid for the September 2021 PLE batch. This will be rendered obsolete for the next batch since we update our handouts regularly.
TOPNOTCH MEDICAL BOARD PREP SURGERY MAIN DIGITAL HANDOUT BY LOUBOMIR ANTONIO, MD
For inquiries visit www.topnotchboardprep.com.ph or https://www.facebook.com/topnotchmedicalboardprep/
This handout is only valid for the September 2021 PLE batch. This will be rendered obsolete for the next batch since we update our handouts regularly.
INGUINAL CANAL TRIANGLE OF DOOM
• ~ 4-6 cm in length • An anatomic area of interest seen in laparoscopic hernia repair
• Begins on the posterior
abdominal wall at the deep
(internal) inguinal ring (oval
hole in the transversalis
fascia)
• Terminates medially at the
superficial (external) inguinal
ring (triangular defect in the
external oblique aponeurosis)
• Where the spermatic cord in
males and the round ligament
in females pass through the
internal inguinal ring
Figure 38-9a. Schwartz’s Principles of Surgery. 11th ed. 2019

WALLS OF THE INGUINAL CANAL STRUCTURE(S)


WALL COMPONENT Medial border • Vas deferens
Anterior • External oblique aponeurosis Lateral border • Vessels of the spermatic cord
• Medial: conjoint tendon • External iliac vessels
Posterior • Lateral: transversalis fascia and • Deep circumflex iliac vein
Contents
transversus abdominis muscle • Femoral nerve
Lateral • Internal oblique • Genital branch of the genitofemoral nerve
WALL COMPONENT TRIANGLE OF PAIN
Medial • Conjoint tendon • An anatomic area of interest seen in laparoscopic hernia repair
• Arching fibers of internal oblique and
Superior (roof)
transversus abdominis muscles
Inferior (floor) • Inguinal and lacunar ligaments
Surgery Platinum. 1st ed. 2018. p. 419

INGUINAL CANAL
https://qrs.ly/bnbjerd

STRUCTURES UNDER THE INGUINAL LIGAMENT

Figure 38-9b. Schwartz’s Principles of Surgery. 11th ed. 2019

STRUCTURE(S)
Inferolateral
• Iliopubic tract
border
Lateral border • Reflected peritoneum
Superomedial
• Gonadal vessels
border
• Lateral femoral cutaneous nerve
• Femoral branch of the genitofemoral
Contents
nerve
• Femoral nerve
Mnemonic: The triangle of FAIN contains the FEMORAL nerve
structures
THE INGUINAL RINGS Dr. Rubio

INGUINAL LOCATION AND REMARKS CIRCLE OF DEATH


RING • An anatomic area of interest seen in laparoscopic hernia repair
• Lies immediately above and medial to the • It serves as a vascular continuation formed by the following
Superficial
(external)
pubic tubercle vessels:
• Triangular opening that forms the exit of the • Common iliac vessels
inguinal ring • Inferior epigastric vessels
inguinal canal Contents • Internal iliac vessels
• External iliac vessels
Deep • Located above and halfway between the pubic • Obturator vessels
(internal) tubercle and the anterior superior iliac spine
inguinal ring • Entrance to the inguinal canal
ANATOMY OF THE SPERMATIC CORD
• The spermatic cord passes through the inguinal canal
• Content :
• Testicular artery
3 arteries • Cremasteric artery
• Artery of vas deferens
• Pampiniform plexus or testicular veins
3 veins • Cremasteric vein
• Deferential vein PREPERITONEAL ANATOMY
• Autonomic nerves (sympathetic nerves) https://qrs.ly/jucl0wx
2 nerves
• Genital branch of genitofemoral nerve
• Vas deferens
Other
• Lymphatic vessels
structures
• Vestige of processus vaginalis
Remember: The ilioinguinal nerve is NOT found inside the spermatic
cord. The ilioinguinal nerve only accompanies the spermatic cord through
the inguinal canal
Dr. Rubio

TOPNOTCH MEDICAL BOARD PREP SURGERY MAIN DIGITAL HANDOUT BY LOUBOMIR ANTONIO, MD Page 41 of 101
For inquiries visit www.topnotchboardprep.com.ph or https://www.facebook.com/topnotchmedicalboardprep/
This handout is only valid for the September 2021 PLE batch. This will be rendered obsolete for the next batch since we update our handouts regularly.
TOPNOTCH MEDICAL BOARD PREP SURGERY MAIN DIGITAL HANDOUT BY LOUBOMIR ANTONIO, MD
For inquiries visit www.topnotchboardprep.com.ph or https://www.facebook.com/topnotchmedicalboardprep/
This handout is only valid for the September 2021 PLE batch. This will be rendered obsolete for the next batch since we update our handouts regularly.
INGUINAL TRIANGLE OF HESSELBACH
STRUCTURE(S)
Medial border • Rectus abdominis muscle
Superolateral border • Inferior epigastric vessels
Inferolateral border • Inguinal ligament

RISK FACTORS FOR INGUINAL HERNIATION


• Coughing • Ascites
• Chronic obstructive • Upright position
pulmonary disease • Congenital connective tissue
• Obesity disorders
• Straining during constipation • Defective collagen synthesis
• Straining during prostatism • Previous right lower
• Pregnancy quadrant incision
• Birthweight <1500 grams • Arterial aneurysms
• Family history of a hernia • Cigarette smoking
• Valsalva’s maneuver • Heavy lifting
CLASSIFICATION OF GROIN HERNIAS
RELATIONSHIP TO COVERED BY
GENERAL INFERIOR INTERNAL
REMARKS ONSET AND ETIOLOGY
CLASSIFICATION EPIGASTRIC SPERMATIC
VESSELS (IEV) FASCIA?
• Occurs in the floor of the inguinal canal,
through Hesselbach triangle • Adulthood (associated with
Direct inguinal • Medial or inferior
• Hernia does not traverse the internal ring (it • No chronic straining →
hernia to the IEV
traverses directly through abdominal wall) weakness in musculature)
• Rarely strangulates
• Most common subtype of groin hernia in
MEN and WOMEN (Schwartz) • Congenital (usually due to
• Lateral or
Indirect inguinal • Hernia that passes through the internal patent processus
superior to the • Yes
hernia inguinal ring and sown the inguinal canal vaginalis)
IEV
• Can extend into scrotum • Adulthood
• Commonly strangulates
• situated in the femoral ring (not inguinal canal)
Femoral inguinal • Occurs when intraabdominal contents protrude along the femoral sheath into the femoral canal
hernia • 70% of femoral hernia repairs are performed in women; however the most common subtype of groin hernia is
still indirect inguinal hernia (Schwartz)
Surgery Platinum. 1st ed. 2018. p. 425
EHS Groin Hernia Classification IIIA Direct hernia
P = primary hernia R = recurrent hernia Indirect hernia large enough to encroach upon
0 = no hernia detectable TYPE III IIIB posterior inguinal wall; Includes indirect
1 = < 1.5 cm (one finger) sliding, scrotal, and pantaloon hernias
IIIC Femoral hernia
2=<3 cm (two fingers)
IVA Recurrent hernia: indirect
3=>3 cm (more than two fingers)
IVB Recurrent hernia: direct
x = not investigated TYPE IV
IVC Recurrent hernia: femoral
L = lateral/ indirect hernia M = medial/ direct hernia F = Femoral IVD Recurrent hernia: mixed
hernia Schwartz’s Principles of Surgery, 11th edition

e.g. primary indirect hernia 2cm internal ring is PL2 DIAGNOSIS OF INGUINAL HERNIAS
CLASSIFICATION
• History and physical examination: gold standard
BASED ON DESCRIPTION • Physical examination: 75% sensitive and 96% specific in the
SYMPTOMS diagnosis of inguinal hernia
Asymptomatic • Inguinal hernia without pain or discomfort • Examination done with the patient standing and supine
inguinal hernia • Mx: watchful waiting • Inspect for presence or absence of a bulge in the groin (both
Minimally • Inguinal hernia with complaints that do sides) - palpate for both testicles in the scrotum
symptomatic not interfere with daily normal activities Percussion and Auscultation
hernia • Mx: watchful waiting • A resonant hernia is more likely to contain loops of bowel
Symptomatic • Inguinal hernia which causes symptoms • Auscultate over the hernia for presence of bowel sounds
inguinal hernia • Mx: elective surgery • Transillumination test
Reducible • Displaced organ or hernia can be returned o By holding a light behind the scrotum one can easily determine
hernia to their usual anatomic site whether the mass is cystic (light shines through) or solid
Irreducible • Hernia in which contents of sac cannot be (light blocked by the mass)
(incarcerated) reduced into abdominal cavity
inguinal hernia • May cause intestinal obstruction Compressibility test
• Hernia which is nonreducible • At the root of the scrotum place the thumb in front and the index
(incarcerated) and shows symptoms of finger behind the bulge
strangulation (vascular disorders of the • Try to reach above the swelling:
Strangulated o Inguinal hernia: cannot get above, non-compressible
hernia content) and/or ileus
inguinal hernia o Pure scrotal swelling: will be able to get above, compressible
• Incarcerated hernia with ischemia and
bowel necrosis Cough impulse test
• Mx: urgent surgery • Patient is asked to cough
Surgery Platinum. 1 ed. 2018. p. 425
st

• (+) visible & palpable cough impulse at the area of the defect
NYHUS
DECRIPTION
CLASSIFICATION Location of the neck in relation to the pubic tubercle
TYPE I
Indirect hernia; internal abdominal ring normal Inguinal • Neck is ABOVE AND MEDIAL to the pubic
Typically in infants, children, small adults hernia tubercle
Indirect hernia; internal ring enlarged without Femoral • Neck is BELOW AND LATERAL to the
TYPE II impingement on the floor of the inguinal canal;
Does not extend to the scrotum
hernia pubic tubercle

TOPNOTCH MEDICAL BOARD PREP SURGERY MAIN DIGITAL HANDOUT BY LOUBOMIR ANTONIO, MD Page 42 of 101
For inquiries visit www.topnotchboardprep.com.ph or https://www.facebook.com/topnotchmedicalboardprep/
This handout is only valid for the September 2021 PLE batch. This will be rendered obsolete for the next batch since we update our handouts regularly.
TOPNOTCH MEDICAL BOARD PREP SURGERY MAIN DIGITAL HANDOUT BY LOUBOMIR ANTONIO, MD
For inquiries visit www.topnotchboardprep.com.ph or https://www.facebook.com/topnotchmedicalboardprep/
This handout is only valid for the September 2021 PLE batch. This will be rendered obsolete for the next batch since we update our handouts regularly.
Ring occlusion test ✔ GUIDE QUESTIONS
• Reduce the hernia and occlude the deep ring by the thumb In repair of a femoral hernia, the structure most vulnerable to major
• While holding the thumb in position ask the patient to stand injury lies:
then cough (A) Medially (D) Posteriorly
(B) Laterally (E) Superficially
o (-) bulging: indirect hernia
(C) Anteriorly
o (+) bulging: direct hernia
Lateral to the femoral hernia are the following vital structures prone
Reducibility test to injury during repair (lateral to medial): femoral nerve, femoral
• Ask patient to reduce hernia themselves while in supine position artery and femoral vein.
o Thigh of the affected side should be flexed, adducted and Dr. Cocos

internally rotated A 70-year-old cigarette smoker presents with a right inguinal mass that
o Deep ring is stabilized by thumb and index finger and then the has enlarged and has caused discomfort in recent months. He
complains of recent difficulty with micturition and nocturia. The
spermatic cord is elongated while scrotum and its content are swelling, which does not extend to the scrotum, reduces when resting.
gently squeezed What is the likely diagnosis?
• Contraindicated if suspecting presence of strangulation (A) Direct inguinal hernia
• Hernia is incarcerated after 1-2 failed attempts (B) Strangulated indirect inguinal hernia
(C) Hydrocele
DIAGNOSTICS (D) Aneurysm of the femoral artery
(E) Cyst of the cord
• Rarely needed
• Used as an adjunct to history and physical examination for Points to a direct inguinal hernia: old age, adult onset, cigarette smoker,
ambiguous diagnosis symptomatic BPH which could cause straining, does not extend to scrotum
(unlike most indirect hernias). A strangulation presents with severe pain,
• should be performed only in patients with obscure pain and/or tenderness and erythema in addition to obstruction which is not present in
swelling this case. A hydrocele (usually extend to the scrotum and is positive for
• The flow chart recommended in these cases: transillumination), aneurysm of femoral artery (pulsatile), and cyst of the
o Ultrasound - if expertise is available cord (does not reduce with resting) are usually not associated with difficulty
o If ultrasound negative → do MRI (with Valsalva maneuver) in micturition and nocturia.
Dr. Cocos
o If MRI negative → consider herniography
A 65-year-old female requires emergency surgery for a strangulated
TOOL DESCRIPTION inguinal hernia. Which of the following is correct?
• Identifying movement of hernia contents (A) The sac is formed by an unobliterated processus vaginalis.
through the canal is essential to making the (B) The hernia is direct rather than indirect.
Ultrasound (C) Such hernia never contains small intestine.
diagnosis, absence of movement can lead to
false negative reading (D) Strangulation never results in bowel ischemia and gangrene
requiring resection.
• Has a sensitivity and specificity of more than
(E) Indirect inguinal hernia is never found in female patients.
MRI 94% and is also useful to reveal other
musculotendineal pathology A strangulated inguinal hernia is usually an indirect hernia, the
• Has high sensitivity and specificity but is invasive pathophysiology of which is a patent processus vaginalis. The hernia
sac may contain small intestine and strangulation results in bowel
• Indicated in suspected sports hernia, recurrent
Herniography ischemia and gangrene which may require resection.
hernia, possible hydrocele, and surgical Dr. Cocos
complications especially chronic groin pain At surgery for a right inguinal hernia, a 72-year-old man is found to
Surgery Platinum. 1st ed. 2018. p. 427
have a hernia sac that is not independent of the bowel wall. The cecum
forms part of the wall of the sac. Such a hernia is properly referred to
MANAGEMENT OF INGUINAL HERNIAS as which of the following?
OPERATIVE TECHNIQUE IN ADULT MALES (A) Incarcerated
PRESENTATION TECHNIQUE (B) Irreducible
• Mesh repair: Lichtenstein or (C) Sliding
Primary unilateral (D) Richter
laparoscopic repair
(E) Interstitial
Primary bilateral • Mesh repair: ideally laparoscopic repair
• Open preperitoneal mesh or Sliding hernia is a type of hernia wherein a part of an internal organ
Recurrent inguinal (bowel, urinary bladder etc.) forms a part of the hernial sac.
laparoscopic approach
hernia Dr. Cocos
• Anterior mesh (Lichtenstein)

HERNIA REPAIRS
https://qrs.ly/zucl0xk

FEMORAL HERNIAS
BASSINI REPAIR
• Treatment: McVay repair Figure 37-15b. Schwartz’s Principles of Surgery. 11th ed. 2019

• Borders of the femoral canal:


STRUCTURE(S)
Medial border • Lacunar ligament
Lateral border • Femoral vein
Anterior border • Inguinal ligament
• Pectineal ligament, superior ramus of
Posterior border
the pubic bone, and pectineus muscle
SHOULDICE REPAIR
Figure 37-16. Schwartz’s Principles of Surgery. 11th ed. 2019

TOPNOTCH MEDICAL BOARD PREP SURGERY MAIN DIGITAL HANDOUT BY LOUBOMIR ANTONIO, MD Page 43 of 101
For inquiries visit www.topnotchboardprep.com.ph or https://www.facebook.com/topnotchmedicalboardprep/
This handout is only valid for the September 2021 PLE batch. This will be rendered obsolete for the next batch since we update our handouts regularly.
TOPNOTCH MEDICAL BOARD PREP SURGERY MAIN DIGITAL HANDOUT BY LOUBOMIR ANTONIO, MD
For inquiries visit www.topnotchboardprep.com.ph or https://www.facebook.com/topnotchmedicalboardprep/
This handout is only valid for the September 2021 PLE batch. This will be rendered obsolete for the next batch since we update our handouts regularly.

MCVAY COOPER’S LIGAMENT REPAIR TAPP: TRANSABDOMINAL PREPERITONEAL


Figure 37-17.. Schwartz’s Principles of Surgery. 11th ed. 2019

DESARDA REPAIR
• described in 2001
LICHTENSTEIN HERNIOPLASTY • mesh-free repair using a strip of external oblique aponeurosis
Figure 37-19. Schwartz’s Principles of Surgery. 11th ed. 2019

✔ GUIDE QUESTION
Following laparoscopic preperitoneal repair of an inguinal hernia, a 50-
year-old male complains of severe burning pain, which radiates down
the lateral side of the ipsilateral thigh. The most likely cause is injury to
which of the following:
(A) Ilioinguinal nerve
(B) Iliohypogastric nerve
(C) Genitofemoral nerve
(D) Femoral nerve
(E) Lateral femoral cutaneous nerve

PLUG AND PATCH HERNIOPLASTY (RUTKOW AND ROBINS) Ilioinguinal nerve-L1) innervates the anterior surface of scrotum and
labia majora, root of the penis and mons pubis, small portion of upper
anteromedial thigh. Iliohypogastric (T12-L1) nerve innervates the
inguinal, pubic and gluteal region. Genitofemoral nerve (L1-L2)
innervates the spermatic cord/round ligament and the skin of
scrotum/labia majora. Femoral nerve (L2-L4) innervates anterior
thigh and anteromedial aspect of the leg.
Dr. Cocos

HERNIA EPONYMS
• Herniation of the antimesenteric border of
Richter hernia
the bowel through the fascial defect
Littre hernia • Herniation of a Meckel diverticulum
Pantaloon • Ipsilateral concurrent, direct and indirect
hernia inguinal herniation
GPRVS (STOPPA PROCEDURE) Maydl hernia • Herniation of two loops of bowel in the sac
Amyand
• Herniation of the vermiform appendix
hernia
Grynfeltt- • Herniation of abdominal contents through
Lesshaft the back (superior lumbar triangle)
hernia
• Herniation of abdominal contents through
Petit hernia
the back (inferior lumbar triangle)
• Herniation of abdominal contents through
Morgagni
the anterior aspect of the diaphragm (via
hernia
foramen of Morgagni)
Bochdalek • Herniation of abdominal contents through
hernia the posterolateral aspect of the diaphragm
• Also known as lateral ventral hernia
Spigelian
• Herniation through the linea semilunaris
hernia
(Spigelian line)

SUPPLEMENT: HERNIA TYPES


PRIMARY VENTRAL HERNIAS
• Non incisional or true ventral hernias
• Examples:
TEP: TOTAL EXTRAPERITONEAL • Epigastric hernias: congenital due to defective midline fusion of
lateral abdominal wall; occurs in multiples and are small; located in
the midline between the xiphoid process and the umbilicus; found
to contain omentum or a portion of the falciform ligament.

TOPNOTCH MEDICAL BOARD PREP SURGERY MAIN DIGITAL HANDOUT BY LOUBOMIR ANTONIO, MD Page 44 of 101
For inquiries visit www.topnotchboardprep.com.ph or https://www.facebook.com/topnotchmedicalboardprep/
This handout is only valid for the September 2021 PLE batch. This will be rendered obsolete for the next batch since we update our handouts regularly.
TOPNOTCH MEDICAL BOARD PREP SURGERY MAIN DIGITAL HANDOUT BY LOUBOMIR ANTONIO, MD
For inquiries visit www.topnotchboardprep.com.ph or https://www.facebook.com/topnotchmedicalboardprep/
This handout is only valid for the September 2021 PLE batch. This will be rendered obsolete for the next batch since we update our handouts regularly.
o Umbilical hernias: due to a patent umbilical ring; more common
in premature infants; spontaneous closure can occur at age of 5,
no closure by that time, do elective surgical repair
o Spigelian hernias: occur anywhere along the length of the
Spigelian line or zone—an aponeurotic band of variable width at
the lateral border of the rectus abdominis
RECTUS ABDOMINIS DIASTASIS
• aka diastasis recti
• clinically evident separation of the rectus abdominis muscle pillars
resulting to a characteristic bulging of the abdominal wall in the
epigastrium (sometimes mistaken for a ventral hernia)
• may be congenital
• associated with advancing age, in obesity, or after pregnancy
• In the postpartum setting, rectus diastasis tends to occur in women
who are of advanced maternal age, who have a multiple or twin
pregnancy, or who deliver a high-birth-weight infant.
• Diagnosis:
o CT scan: differentiate rectus diastasis from a true ventral hernia
• Treatment: surgery
✔ GUIDE QUESTIONS
In the setting of an equivocal examination, which of the ff has the Figure 25-3. Schwartz’s Principles of Surgery. 11th ed. 2019
greatest sensitivity in diagnosing an inguinal hernia?
DIAGNOSTIC TESTS FOR ESOPHAGEAL
a. Repeat examination by a second surgeon SUPPLEMENT:
FUNCTION
b. Ultrasound
c. CT scan Tests to detect structural abnormalities
d. MRI 1. Barium swallow
o 1st diagnostic test in patients with suspected esophageal disease
Although CT scan is useful in ambiguous clinical presentations, little o can reveal anatomic problems
data exist to support its routine use in diagnosis. The use of MRI in o if patient complains of dysphagia and no obstructing lesion seen in
assessing groin hernias was examined in a group of 41 patients barium swallow → use a barium-impregnated marshmallow,
scheduled to undergo laparoscopic inguinal hernia repair. barium-soaked bread or barium hamburger
Preoperatively, all patients underwent US and MRI. Laparoscopic § will bring out the functional disturbance in the esophageal
confirmation of the presence of inguinal hernia was deemed as gold transport that can be missed when liquid barium is used.
standard. Physical examination was found to be the least sensitive. 2. Endoscopic evaluation
False positives were low on physical examination and MRI (one o endoscopy is indicated in patients complaining of dysphagia even
finding), but higher with US (four findings). With further refinement with a normal radiographic study
of technology, radiologic techniques will continue to improve Tests to detect functional abnormalities
sensitivity and specificity rates of diagnosis, thereby serving a 1. Manometry
supplementary role in cases of uncertain diagnosis. Magnetic o indicated when a motor abnormality of the esophagus is
resonance imaging is by far the most sensitive, specific, and reliable considered on the basis of complaints (dysphagia, odynophagia, or
modality to diagnose inguinal hernias. This compares with the very noncardiac chest pain) and barium swallow and endoscopy does
low performance of CT. Ultrasonography historically has been the first not show a structural abnormality
line of approach for evaluation of inguinal hernias. o essential tool in preoperative evaluation of patients before anti-
Dr. Cocos
,
reflux surgery
Tests to detect increased exposure to gastric juice
ESOPHAGUS 1. 24-hour ambulatory pH monitoring
o most direct method of measuring increased esophageal exposure
✔ GUIDE QUESTION to gastric juice (not reflux)
A chest CAT scan is done to further delineate an abnormality seen on a o sensitivity and specificity of 96%
chest x-ray. The superior mediastinum at the level of T4 is evaluated. o gold standard for the diagnosis of GERD
Which structure is remote from the esophagus? 2. Radiographic exposure of gastroesophageal reflux
(A) Trachea (D) Azygous vein o radiographic demonstration of spontaneous regurgitation of
(B) Recurrent laryngeal nerves (E) Brachiocephalic vein barium into the esophagus in the upright position is a reliable
(C) Aorta indicator that reflux is present but failure to see this does not
The cervical part of the esophagus is near the trachea and recurrent indicate absence of disease
laryngeal nerves. The thoracic part is adjacent to the azygous vein and
aorta. The brachiocephalic vein is remote from its course.
Dr. Cocos GASTROESOPHAGEAL REFLUX DISEASE
ANATOMY • Most common esophageal pathology
• Starts at the level of cricoid cartilage (C6) and ends at the cardia • Defined by symptoms, endoscopic findings and exposure to
of stomach (T11) gastric juice
• Has three regions: • Loss of high pressure zone at the esophagogastric junction
ARTERIAL VENOUS (collar sling fibers and clasp fibers)
REGION LENGTH
SUPPLY DRAINAGE • Lower Esophageal Sphincter (LES) - not a true anatomic sphincter
Cervical • ~5 cm • Inferior • Inferior thyroid
region LOWER ESOPHAGEAL SPHINCTER VALUES
long thyroid artery vein
Total length 3-5 cm
• Bronchial veins
• ~ 20 Abdominal length 2-4 cm
Thoracic • Bronchial • Azygous vein
cm Resting pressure 6-26 mmHg
region arteries • Hemi-azygous
long
vein LOWER ESOPHAGEAL SPHINCTER (LES)
• Ascending Three components • Mechanically effective LES
branch of left of the antireflux • Efficient esophageal clearance
Abdominal • ~ 2 cm • Coronary vein/
gastric artery mechanism • Adequately functioning gastric reservoir
region long Left gastric vein
• Inferior phrenic • Average LES mean resting pressure <6 mmHg
Characteristics of
artery • Overall sphincter length <2 cm
a mechanically
• Intraabdominal sphincter length of <1cm
THREE NORMAL AREAS OF NARROWING defective LES
(most consistent characteristic)
Level of cricopharyngeus • Narrowest point of the
muscle (C6) esophagus (1.5 cm )
Level of left main stem
• 1.6 cm
bronchus and aortic arch (T4)
• 1.6 to 1.9 cm
Level of the hiatus of the • Caused by
diaphragm (T10) gastroesophageal
sphincter mechanism
TOPNOTCH MEDICAL BOARD PREP SURGERY MAIN DIGITAL HANDOUT BY LOUBOMIR ANTONIO, MD Page 45 of 101
For inquiries visit www.topnotchboardprep.com.ph or https://www.facebook.com/topnotchmedicalboardprep/
This handout is only valid for the September 2021 PLE batch. This will be rendered obsolete for the next batch since we update our handouts regularly.
TOPNOTCH MEDICAL BOARD PREP SURGERY MAIN DIGITAL HANDOUT BY LOUBOMIR ANTONIO, MD
For inquiries visit www.topnotchboardprep.com.ph or https://www.facebook.com/topnotchmedicalboardprep/
This handout is only valid for the September 2021 PLE batch. This will be rendered obsolete for the next batch since we update our handouts regularly.
COMPLICATIONS ASSOCIATED WITH GERD (A) Sliding hiatal hernia with GERD
• Esophagitis • Progressive pulmonary fibrosis (B) Paraesophageal hernia
• Stricture • Barrett esophagus (C) Benign esophageal stricture
(D) Squamous carcinoma of the esophagus
• Repetitive aspiration • Esophageal adenocarcinoma
(E) Adenocarcinoma arising in a Barrett esophagus
BARRETT ESOPHAGUS
GERD is a risk factor for Barrett esophagus which is a risk factor for the
• Columnar lined epithelium of the esophagus rather than development of esophageal adenocarcinoma. Since there is already an
squamous epithelium irregular and ulcerated area with mucosal disruption and overhanging
• Any length of endoscopically identifiable columnar mucosa with edges it is safe to assume that a malignant lesion is already present which
intestinal metaplasia eliminates choices A, B and C . Adenocarcinoma usually arises from the distal
3rd of the esophagus and GERD is a risk factor. Squamous carcinoma usually
• Hallmark of intestinal metaplasia: (+) intestinal goblet cells
arises from the middle 3rd of the esophagus.
• May lead to ulcer, stricture formation and dysplasia- Dr. Cocos
adenocarcinoma
• 30 to 125x increased risk of developing adenocarcinoma DIAPHRAGMATIC (HIATAL) HERNIA
ETIOLOGY
DIAGNOSIS OF GERD
• Structural deterioration of the phrenoesophageal membrane
• (+) symptoms - start with empiric 12-week antacid treatment
• Persistence of symptoms – perform endoscopy, upper GI
contrast study, 24-hour pH monitoring (gold standard),
esophageal monitoring
TREATMENT OF GERD
• Gold standard treatment: proton pump inhibitors (PPI)
• Anti-Reflux Surgery
o Indications for Anti reflux surgery
• Symptomatic patients with or • Severe esophagitis Figure 25-7. Schwartz’s Principles of Surgery. 11th ed. 2019
without esophagitis • Presence of stricture
• Structurally defective LES • Uncomplicated Barrett CLINICAL MANIFESTATIONS
• Young patients with esophagus • Sliding hernia: symptoms of GERD (heartburn, regurgitation)
documented reflux • Paraesophageal hernia: dysphagia, postprandial fullness, and
o Principles of Anti reflux surgery some with massive bleeding, gastric volvulus, and infarction
§ Restore LES pressure to a level 2x the resting gastric • Borchardt triad (chest pain, retching with inability to vomit,
pressure and its length to at least 3cms and inability to pass a nasogastric tube) is indicative of
§ Place an adequate length of the LES in the (+) pressure incarcerated intra thoracic stomach
environment of the abdomen (not <1cm) TYPE I • Most common type
§ Allow the reconstructed cardia to relax on deglutition (SLIDING HERNIA) • Upward dislocation of GE junction
§ Fundoplication should not increase the resistance of the LES to a
and cardia into the thorax through
level greater than the peristaltic power of the esophageal body
the esophageal hiatus of diaphragm
§ Repair can be placed in the abdomen without undue tension
• Symptoms: reflux, dysphagia,
o Anti-reflux procedures aspiration
PROCEDURE DESCRIPTION
• Usually medical treatment
• Most common antireflux surgical procedure
• Involves 360° fundoplication around the lower TYPE II
Nissen (ROLLING / • Upward dislocation of the gastric
esophagus for a distance of 4-5cm, without
fundoplication
division of the short gastric blood vessels (wraps PARAESOPHAGEAL fundus alongside a normally
fundus of stomach all the way around esophagus) HERNIA) positioned cardia
• Alternative to the Nissen Fundoplication that • Herniation of part of the stomach
Toupet seeks to decrease its adverse effects without displacement of GE junction
fundoplication • 270° posterior fundoplication around the distal • Symptoms: obstructive symptoms,
4 cm of esophagus dysphagia, gastric ulcer,
Dor • 180° anterior fundoplication of the distal strangulation
fundoplication esophagus • Requires surgical treatment
• Use a stapler to divide the cardia and upper stomach,
parallel to the lesser curvature of the stomach TYPE III
Collis
• Results in a gastric tube continuous with the (COMBINED HERNIA)
gastroplasty
esophagus thereby extending length of
esophagus by several centimeters
Belsey Mark • 240-270° fundoplication performed through a • Combined herniation of the cardia
IV thoracic approach and fundus
• Arcuate ligament repair (close the large esophageal
Hill posterior
hiatus) + gastropexy to diaphragm (suture stomach
gastropexy
to the diaphragm)
Surgery Platinum. 1st ed. 2018. p.253
SUPPLEMENT: • Includes an additional organ that
Principles of surgical therapy in reflux disease TYPE IV
herniates, aside from the stomach
1. The operation should restore the pressure of the distal esophageal • Also referred to as paraesophageal
sphincter to a level twice the resting gastric pressure
TYPES II-IV hernias – as a portion of the stomach
2. The operation should place an adequate length of the distal
is situated adjacent to the esophagus
esophageal sphincter in the positive-pressure environment of the Surgery Platinum. 1st ed. 2018. p. 254
abdomen by a method that ensures its response to changes in intra- DIAGNOSIS OF HIATAL HERNIAS
abdominal pressure
3. The operation should allow the reconstructed cardia to relax on TOOL REMARKS
deglutition Plain upright • Air-fluid level behind cardiac shadow (this is
4. The fundoplication should not increase the resistance of the lateral chest usually caused by a PEH or an intrathoracic
relaxed sphincter to a level that exceeds the peristaltic power of the radiograph stomach)
body of the esophagus • Findings:
5. The operation should ensure that the fundoplication can be placed Upper GI o Outpouching of barium at the lower and of
in the abdomen without undue tension, and maintained there by barium esophagus
approximating the crura of the diaphragm above the repair swallow o Wide hiatus through which gastric folds are
✔ GUIDE QUESTION (or Upper GI seen in continuum with those in the
A 46-year-old man has a long history of heartburn (GERD). His barium series) stomach
study shows an irregular, ulcerated area in the lower third of his o Free reflux of barium
esophagus. There is marked mucosal disruption and overhanging • More accurate for paraesophageal hiatal
edges. What is the most likely diagnosis? hernia
TOPNOTCH MEDICAL BOARD PREP SURGERY MAIN DIGITAL HANDOUT BY LOUBOMIR ANTONIO, MD Page 46 of 101
For inquiries visit www.topnotchboardprep.com.ph or https://www.facebook.com/topnotchmedicalboardprep/
This handout is only valid for the September 2021 PLE batch. This will be rendered obsolete for the next batch since we update our handouts regularly.
TOPNOTCH MEDICAL BOARD PREP SURGERY MAIN DIGITAL HANDOUT BY LOUBOMIR ANTONIO, MD
For inquiries visit www.topnotchboardprep.com.ph or https://www.facebook.com/topnotchmedicalboardprep/
This handout is only valid for the September 2021 PLE batch. This will be rendered obsolete for the next batch since we update our handouts regularly.
Upper GI • This is because a sliding hernia can often
barium spontaneously reduce
swallow
(cont)
• allows visualization of esophageal mucosa
from upper esophageal sphincter to
esophagogastric junction
Flexible
• Sliding hernia: gastric pouch extending
esophagoscopy
above crural impression (at least 2 cm)
• Rolling or PEH: separate orifice adjacent the
GEJ
Surgery Platinum. 1st ed. 2018. p. 254
TREATMENT OF HIATAL HERNIAS
• Medical treatment → surgical repair (if
Sliding hernia
refractory to medical management)
• Surgical repair (i.e. reduction of hernia,
Paraesophageal
repair of diaphragmatic defect, may include
hernia
fundoplication)
✔ GUIDE QUESTIONS
A 64-year-old woman presents with severe upper abdominal pain and
MIDTHORACIC DIVERTICULA
retching of 1-day duration. Attempts to pass a nasogastric tube are • Located 5 cm above and below the level of the carina
unsuccessful. X-rays show an air-fluid level in the left side of the chest • Associated with both traction and pulsion
in the posterior mediastinum. An incarcerated Paraesophageal hernia • Traction diverticula
and gastric volvulus is diagnosed. What is the next step in o Usually due to granulomatous diseases (TB, histoplasmosis)
management?
o Almost all are asymptomatic
(A) Insertion of a weighted bougie to untwist the volvulus
(B) Elevation of the head of the bed o Treatment: medical treatment of underlying inflammation
(C) Placing the patient in the Trendelenburg position with the head of • Pulsion Diverticula
the bed lowered o More common
(D) Laparotomy and vagotomy o Results from diffuse motility disorders of the esophagus
(E) Surgery, reduction of the gastric volvulus, and repair of the o Diagnosis: manometry, barium swallow, endoscopy
hernia o Treatment: reserved for symptomatic diverticula -
Gastric volvulus is an absolute indication for emergent surgical esophagomyotomy + diverticulectomy/diverticulopexy
intervention and is classically described by the Borchardt triad:
inability to pass an NGT, retching without actual food regurgitation
and epigastric pain. After reduction of the gastric volvulus (not
necrotic) or resection if necrotic, a paraesophageal repair is done
which generally includes four steps: hernia sac dissection and
resection, esophageal mobilization, crural repair, and fundoplication.
A vagotomy is unnecessary.
Dr. Cocos
A 64-year-old man has symptoms of reflux esophagitis for 20 years. The
barium study demonstrates a sliding hiatal hernia. Which is TRUE in
sliding hiatal hernia?
(A) A hernia sac is absent.
(B) The cardia is displaced into the posterior mediastinum.
(C) Reflux esophagitis always occur.
(D) A stricture does not develop.
(E) Surgery should always be avoided.
Sliding hiatal hernia or type 1 hiatal hernia occurs when the GEJ and
cardia dislocates upward thru the esophageal hiatus located in the
posterior mediastinum. A hernia sac may be present in cases of sliding
hiatal hernia. Reflux esophagitis usually occurs (but not always) and
might lead to esophageal strictures. Refractory GERD might warrant
surgical intervention.
Dr. Cocos EPIPHRENIC DIVERTICULA
ESOPHAGEAL DIVERTICULA • Pulsion diverticula that occurs at the distal 10cms of esophagus
• Usually single and occurs on the right side
• Mucosa-lined pouches protruding from the esophageal lumen
• Diagnosis: manometry, barium swallow, endoscopy
• Classified according to location:
• Treatment: similar to midthoracic diverticulum
o Pharyngoesophageal (Zenker diverticulum)
o Midthoracic (Parabronchial) ✔ GUIDE QUESTIONS
o Epiphrenic A 79-year-old retired opera singer presents with dysphagia, which has
• True diverticula: contains all layers of esophageal wall become progressively worse during the last 5 years. He states that he
• False diverticula: contains only submucosa and mucosa (more is sometimes aware of a lump on the left side of his neck and that he
common) hears gurgling sounds during swallowing. He sometimes regurgitates
ZENKER DIVERTICULA (FALSE DIVERTICULUM) food during eating. What is the likely diagnosis?
(A) Carcinoma of the esophagus
• Most common esophageal diverticula
(B) Foreign body in the esophagus
• Results from high pressure generated in the hypopharynx (C) Plummer-Vinson (Kelly-Patteson) syndrome
• Killian triangle: area of potential weakness situated behind the (D) Zenker (pharyngoesophageal) diverticulum
esophagus at the level of the cricopharyngeus muscle (E) Scleroderma
• Symptoms: dysphagia, regurgitation of undigested food, Zenker diverticulum is a pseudo diverticulum (not involving all layers)
halitosis, choking, and aspiration and occurs mainly in the elderly. Presenting symptoms
include dysphagia, regurgitation of undigested food, halitosis,
• Diagnosis: barium swallow, esophagogram
hoarseness, and chronic cough. Although this is an eldergly male with
• Treatment: 5 year history of dysphagia, a diagnosis of carcinoma is insufficient
o 2cms or less: pharyngomyotomy but should be ruled out. There is no history of foreign body ingestion
o >2 cm: diverticulectomy/diverticulopexy and is unlikely considering the chronic course. Plummer Vinson
o wide-based: diverticulopexy syndrome is more common in middle-aged women and is a rare
condition characterized by the classic triad of dysphagia, iron-
deficiency anemia and esophageal web. Scleroderma can cause
secondary esophageal motility and therefore dysphagia. Choices A, B,
C and E would not explain the gurgling sounds during swallowing.
Dr. Cocos

TOPNOTCH MEDICAL BOARD PREP SURGERY MAIN DIGITAL HANDOUT BY LOUBOMIR ANTONIO, MD Page 47 of 101
For inquiries visit www.topnotchboardprep.com.ph or https://www.facebook.com/topnotchmedicalboardprep/
This handout is only valid for the September 2021 PLE batch. This will be rendered obsolete for the next batch since we update our handouts regularly.
TOPNOTCH MEDICAL BOARD PREP SURGERY MAIN DIGITAL HANDOUT BY LOUBOMIR ANTONIO, MD
For inquiries visit www.topnotchboardprep.com.ph or https://www.facebook.com/topnotchmedicalboardprep/
This handout is only valid for the September 2021 PLE batch. This will be rendered obsolete for the next batch since we update our handouts regularly.
A 30-year-old psychiatric patient has a barium swallow after removal performed to prevent reflux from the stomach into the esophagus
of a foreign body to rule out a small perforation of the esophagus. No following the myotomy. An esophagectomy is not indicated unless
perforation is seen, but an epiphrenic diverticulum is visualized. An there is evidence of malignancy. Surgical myotomy should extend
epiphrenic diverticulum may be associated with which of the following? proximal and distal to the LES. A repeat pneumatic dilation is no
(A) Duodenal ulcer longer an option and would likely not alleviate his symptoms. A
(B) Gastric ulcer fundoplication would worsen achalasia if a proper myotomy is not
(C) Cancer of the tongue done prior.
(D) Cancer of the lung Dr. Cocos

(E) Hiatal hernia


Many patients with epiphrenic diverticula have an associated sliding ESOPHAGEAL PERFORATION
hiatal hernia (30%), and in these patients, reflux-induced esophageal
neuromotor dysfunction may be the cause of the pouch. Epiphrenic • True surgical emergency
diverticulum is a pulsion diverticula of the distal esophagus arising • usually following a therapeutic or diagnostic procedure
just above the lower esophageal sphincter, more frequently on the • Pain: striking and consistent symptom of esophageal
right posterolateral wall. rupture
Dr. Cocos
• Boerhaave syndrome
o Spontaneous rupture of the esophagus
MOTILITY DISORDERS o Usual history of resisting vomiting
ACHALASIA o High mortality due to misdiagnosis and delay in treatment
• Primary motility disorder characterized by loss of peristaltic o Usually occurs into the left pleural cavity or just above the
waveform in the esophageal body and failure of the LES to gastroesophageal junction
relax • Diagnostics: Water soluble contrast esophagogram
• Pathogenesis: (Gastrografin)
• Neurogenic degeneration • Treatment: Key to optimum management is early diagnosis
• Hypertension of LES o Closure within 24 hours: 90% survival
• Failure of the LES to relax • Within 24 hours – perforation is debrided, closed primarily and
Pathogenesis • Elevation of intraluminal esophageal reinforced with a serosal/pleural patch or fundoplication
pressure • >24 hours – Survival not influenced by type of repair
• Dilatation of the esophageal body o Resection of the diseased segment
• Loss of peristalsis in the esophageal body o Remainder of the esophagus brought out as end cervical
• Hypertensive LES esophagostomy
o CTT on contaminated chest
Triad • Aperistalsis of the esophagus
o Feeding jejunostomy
• Failure of the LES to relax
o Subsequent reconstruction (colonic interposition/gastric pull-up)
• Manometry (gold standard)
• Non operative
• (+) bird’s beak esophagus – seen in
o If perforation is contained within the mediastinum and drain
Diagnostics esophagogram well back into the esophagus
• Endoscopy – to rule out malignancy or o Treatment: NPO, hyperalimentation, antibiotics, antisecretory
strictures drugs, DAT resumed after 7-14 days (with no extravasation
• Heller myotomy + partial confirmed by esophagogram)
fundoplication
• Pneumatic dilatation – most effective ✔ GUIDE QUESTION
Treatment
non-surgical management A patient is diagnosed with Boerhaave syndrome. Management
• Medications – calcium channel blockers, involves which of the following?
nitrates, botulinum toxin (A) Administration of intravenous antibiotics and TPN
(B) Administration of intravenous antibiotics and TPN, and
DIFFUSE AND SEGMENTAL ESOPHAGEAL SPASM (DES) insertion of a chest tube and a nasogastric tube
(C) Administration of intravenous antibiotics and TPN, and
• Simultaneous waveforms and multi- insertion of a nasogastric tube
Manometric
peaked contractions (D) Resuscitation and emergency surgery either by laparotomy
findings
• Intermittent normal peristalsis or thoracotomy
Esophagogram • Corkscrew deformity (E) Resuscitation, administration of intravenous antibiotics,
replacement of fluids and electrolytes; elective surgical
NUTCRAKER ESOPHAGUS intervention when the general status of the patient improves
• Most common primary esophageal motility disorder Boerhaave syndrome or esophageal rupture is a surgical emergency.
• Mean peristaltic amplitude in distal After initial resuscitation, surgery is warranted to address the
esophagus > 180 mmHg perforation (A, B, and C are incomplete). Delay in operation leads to
Manometric high mortality (hence E is wrong).
• Increased mean duration of contraction
findings Dr. Cocos
>7 seconds
• Normal peristalsis
CAUSTIC INJURIES
HYPERTENSIVE LES TYPE OF
REMARKS OR DESCRIPTION
SUBSTANCE
• Elevated LES pressure >26 mmHg
Manometric • Causes liquefactive necrosis
• Normal LES relaxation
findings (saponification of fats)
• Normal peristalsis
Alkali agents • Penetrates deeply
(pH >7) • Includes: NaOH (drain openers,
✔ GUIDE QUESTION
detergents, Clinitest. Tablets, hair
A patient has been diagnosed with achalasia. He refused surgery
initially, preferring to try nonoperative therapy. He tried life style
relaxers, button batteries)
modification, calcium channel blockers, botulin toxin injection, and • Causes coagulation necrosis
endoscopic pneumatic dilatation. None of the treatments alleviated his • More superficial
symptoms. What are his surgical options? Acids • Includes:
(A) Esophagectomy (pH <7) o HCl (pool and metal cleaners)
(B) Surgical esophagomyotomy proximal to the LES o Hydrofluoric acid (anti-rust products)
(C) Modified Heller myotomy and partial fundoplication
o Sulfuric acid (battery fluid)
(D) Repeat pneumatic dilation using pressures of loops
(E) Nissen fundoplication Bleaches
• Example: sodium hypochlorite
(pH = 7)
Heller’s myotomy is a surgical procedure which opens the tight lower Surgery Platinum. 1st ed. 2018. p.
esophageal sphincter by performing a longitudinal incision across the
LES to relieve the dysphagia. Further, a Dor or Toupet fundoplication
(a partial wrapping of the stomach around the esophagus) is
TOPNOTCH MEDICAL BOARD PREP SURGERY MAIN DIGITAL HANDOUT BY LOUBOMIR ANTONIO, MD Page 48 of 101
For inquiries visit www.topnotchboardprep.com.ph or https://www.facebook.com/topnotchmedicalboardprep/
This handout is only valid for the September 2021 PLE batch. This will be rendered obsolete for the next batch since we update our handouts regularly.
TOPNOTCH MEDICAL BOARD PREP SURGERY MAIN DIGITAL HANDOUT BY LOUBOMIR ANTONIO, MD
For inquiries visit www.topnotchboardprep.com.ph or https://www.facebook.com/topnotchmedicalboardprep/
This handout is only valid for the September 2021 PLE batch. This will be rendered obsolete for the next batch since we update our handouts regularly.
CLINICAL MANIFESTATIONS
PHASE
OF DESCRIPTION MANIFESTATIONS
INJURY
• Pain in the mouth and
• Acute necrotic substernal region
1st phase
phase (1 to 4 days) • Hypersalivation
• Odynophagia and dysphagia
• Ulceration and • Disappearance of symptoms
2nd phase granulation phase (3 • Period when esophagus
to 5 days) is the weakest
• Reappearance of
• Cicatrization and dysphagia due to
3rd phase scarring stricture formation
(3rd week) (60% in 1st month,
80% in 2 months)
Surgery Platinum. 1st ed. 2018. p.
INITIAL TESTS
• Gas shadow in the mediastinum: suggests
Chest esophageal perforation
radiograph • Gas under the diaphragm: indicates gastric
perforation
• May assess depth and boundary of injury
CT Scan
• Can diagnose impending perforation ESOPHAGEAL CARCINOMA
Surgery Platinum. 1st ed. 2018. p.
• Early Endoscopy SQUAMOUS CELL
ADENOCARCINOMA
o Done within 24 hours of ingestion CARCINOMA
o Mid esophagus: most common site of perforation • Increasing in
• Most common type
incidence
worldwide
TREATMENT • Most common in
• Men are affected 3 to
developed
• Maintain NPO 4 times as often as
Epidemiology countries
• Diagnostics: endoscopy, radiograph to women
• Men are affected 6
rule out perforation • Lymphatic metastasis
Nonsurgical to 8 times more
• IVF hydration are found in 30-70%
frequently than
• Prevention of stress ulcers with IV PPI of surgical specimens
women
• Broad spectrum IV antibiotics Usual • Middle 3rd of thoracic
• Do not induce emesis (may cause re- • Distal esophagus
location esophagus
Contraindicated exposure esophagus to substance) • Barrett metaplasia:
treatment • Avoid neutralizers (heat can be generated precursor lesion
modalities from neutralization) • Plaque-like, erosive,
• Columnar
• Do not insert a nasogastric tube papillary
epithelium is seen
• Emergent surgery indicated for • Classified as
Early lesion with mucosal
perforation, mediastinitis, peritonitis intraepithelial,
glands often
Surgical • Esophagectomy: required for severe intramucosal or
containing
strictures submucosal
intestinal goblet
• Feeding jejunostomy cells
Surgery Platinum. 1st ed. 2018. p. Surgery Platinum. 1st ed. 2018. p. 260

RISK FACTORS FOR THE


DEVELOPMENT OF ESOPHAGEAL CARCINOMA
CAUSTIC INJURY SQUAMOUS CELL
https://qrs.ly/wwcl0yt RISK FACTOR ADENOCARCINOMA
CARCINOMA
Tobacco use +++ ++
Alcohol use +++ -
Barrett esophagus - ++++
Zargar’s classification (endoscopic classification) Weekly reflux (GERD)
- +++
GRADE FEATURES symptoms
0 Normal Obesity - ++
Superficial mucosal Poverty ++ -
1
edema and erythema Achalasia +++ -
Mucosal and submucosal Caustic injury to the
2 ++++ -
ulceration esophagus
Superficial ulceration, Plummer-Vinson
2A ++++ -
erosion, exudate syndrome
Deep discrete or History of head and
2B ++++ -
circumferential ulcer neck cancer
Transmural ulceration History of breast
3
with necrosis cancer with +++ +++
3A Focal necrosis radiotherapy
Surgery Platinum. 1st ed. 2018. p. 260
3B Extensive necrosis
4 Perforation
CLINICAL MANIFESTATIONS
• Most common presenting symptom – dysphagia (60% of
esophageal lumen is infiltrated)
• stridor, coughing, choking, • anorexia and weight loss
aspiration pneumonia • upper body edema if with
• bleeding superior vena cava
• hoarseness syndrome
• jaundice and bone pain • malignant pleural effusion

TOPNOTCH MEDICAL BOARD PREP SURGERY MAIN DIGITAL HANDOUT BY LOUBOMIR ANTONIO, MD Page 49 of 101
For inquiries visit www.topnotchboardprep.com.ph or https://www.facebook.com/topnotchmedicalboardprep/
This handout is only valid for the September 2021 PLE batch. This will be rendered obsolete for the next batch since we update our handouts regularly.
TOPNOTCH MEDICAL BOARD PREP SURGERY MAIN DIGITAL HANDOUT BY LOUBOMIR ANTONIO, MD
For inquiries visit www.topnotchboardprep.com.ph or https://www.facebook.com/topnotchmedicalboardprep/
This handout is only valid for the September 2021 PLE batch. This will be rendered obsolete for the next batch since we update our handouts regularly.
FUNCTIONAL GRADES OF DYSPHAGIA Three-field • Carried out through separate laparotomy,
INCIDENCE AT (McKeown) right thoracotomy, and cervical incisions
GRADE DEFINITION
DIAGNOSIS • oblique incision from the midpoint
I • Eating normally 11% between the xiphoid & umbilicus across
II • Requires liquids with meals 21% the costal arch to the tip of the scapula
Left
• Able to take semisolids but • Abdomen is opened, the costal arch is
III 30% thoracoabdomin
unable to take any solid food divided, and the chest is entered through
al (Akiyama)
40% the seventh intercostal space
IV • Able to take liquids only • Diaphragm is opened in a circumferential
(most common)
• Unable to take liquids, but able manner along the chest wall
V 7% • Include laparoscopic, hand-assisted,
to swallow saliva Minimally
VI • Unable to swallow saliva 12% thoracoscopic, and robot-assisted
invasive
Surgery Platinum. 1 ed. 2018. p. 261
st esophagectomy
Surgery Platinum. 1st ed. 2018. p. 262
DIAGNOSTIC EVALUATION
TOOL REMARKS
• Essential for all patients suspected of
Endoscopy
having esophageal cancer; gold standard
(EGD) with
• Determines histology, location, degree of
biopsy
obstruction and extent of the lesion
• Crucial to visualize the mucosa, luminal
distensibility, motility and any anatomic
Barium abnormalities
swallow • Early cancers appear as small intraluminal
plaques, polypoid lesions, or areas of discrete
ulcerations
• Determines the local extent and relationship
CT scan of TRANSTHORACIC ESOPHAGECTOMY - IVOR LEWIS PROCEDURE
to adjacent structures and distant metastases
chest &
(e.g., lung liver)
abdomen
• Not useful for determining T-stage
• Provides more accurate T- and N-staging
Endoscopic • Able to identify five distinct layers in the
ultrasound esophageal wall, thus providing an
assessment of the depth of tumor invasion
Thoracoscopy • Offers direct visualization and histopathologic
and diagnosis for nodal status & extent of local
laparoscopy invasion and metastatic disease
Surgery Platinum. 1st ed. 2018. p. 261

ESOPHAGEAL CARCINOMA TREATMENT


STAGE MANAGEMENT
• Upper endoscopy + four-quadrant biopsy KOCHER MANEUVER: MOBILIZATION OF THE FIXED PORTIONS
• (+) 2 consecutive negative biopsies: follow- OF THE DUODENUM
Barrett
up EGD every 3 years
metaplasia
• Surgery: best chance for cure and for high-
grade dysplasia
• Esophagectomy
Locoregional • Others: endoscopic therapy (e.g., mucosal
Disease resection), preoperative chemoradiation
(Stage I-III) • Chemoradiation: standard for non-surgical
management of locally advanced disease
Metastatic • Unresectable
Disease • ECOG score ≤ 2: chemotherapy + palliative care TRANSHIATAL ESOPHAGECTOMY- ORRINGER PROCEDURE
(Stage IV) • ECOG score >2: palliative care
*Eastern Cooperative Oncology Group (ECOG) Score (measures performance
status used to determine if a patient can retrieve chemotherapy): a score of
0 pertains to “perfect health” whereas a score of 5 pertains to “death”
Surgery Platinum. 1st ed. 2018. p. 262

Exclusion Criteria (Contraindications) for Curative Surgery


or Resection
• Age >75 y • Locally advance tumor (with
• FEV1 <1.25 and EF <40% signs of invasion)
• >20% weight loss • Distant metastases
APPROACH REMARKS
• if esophageal cancer limited to the
Vagal-sparing
intramucosal layer without the need for AKIYAMA TECHNIQUE - (LEFT THORACOABDOMINAL
esophagectomy
lymphadenectomy APPROACH)
• Performed via upper midline laparotomy Bypass Approaches (see table below)
and left cervical incision o Palliative resection or bypass surgery (for those who cannot
Transhiatal • Gastric conduit based on the right undergo curative surgery)
(Orringer and gastroepiploic artery is used to establish • Done to decrease local recurrence and produce good
Sloan) gastrointestinal continuity (if the stomach symptomatic results
is unstable, a colonic interposition can be
• Performed substernally (most common), transthoracically, or
performed)
subcutaneously (rare)
• Upper midline abdominal incision and a
right thoracotomy is done
• Esophagus is dissected out of its bed;
Transthoracic stomach is pulled up into the thorax
(Ivor-Lewis) through esophageal hiatus & divided at its
cardiac portion to allow creation of an
anastomosis between the distal end of the
esophagus and stomach
TOPNOTCH MEDICAL BOARD PREP SURGERY MAIN DIGITAL HANDOUT BY LOUBOMIR ANTONIO, MD Page 50 of 101
For inquiries visit www.topnotchboardprep.com.ph or https://www.facebook.com/topnotchmedicalboardprep/
This handout is only valid for the September 2021 PLE batch. This will be rendered obsolete for the next batch since we update our handouts regularly.
TOPNOTCH MEDICAL BOARD PREP SURGERY MAIN DIGITAL HANDOUT BY LOUBOMIR ANTONIO, MD
For inquiries visit www.topnotchboardprep.com.ph or https://www.facebook.com/topnotchmedicalboardprep/
This handout is only valid for the September 2021 PLE batch. This will be rendered obsolete for the next batch since we update our handouts regularly.
ROUTE ADVANTAGES DISADVANTAGES
• Offers the best direct conduit to the neck when
primary tumor is left in situ • Requires a more tortuous and
• Reduced possibility of recurrent malignant hazardous course for the
Substernal
dysphagia or malignant fistulization substrate than does the
• Allows creation of a portal for palliative transthoracic route
radiation therapy
• Allow better maintenance of an esophageal • Requires excision of the
substitute over time segment of the native
Transthoracic
• Utilizes shortest and most direct route between esophagus involved by
the neck and the abdominal cavity primary tumor GEJ CARCINOMA
https://qrs.ly/q6cl0zc
• Any leakage will be superficial, and if the • Previous cardiac or
bypass should demonstrate infarction, this can mediastinal surgery
Subcutaneous be recognized quickly and bypass easily removed • Utilizes the longest route
• Allows creation of a portal for palliative between the neck and the
radiation therapy abdominal cavity
Surgery Platinum. 1st ed. 2018. p. 263

ESOPHAGEAL SARCOMA
• Smooth polypoid lesion on barium swallow is distinctive
Remain superficial to muscularis layer thus rarely metastasize
to lymph nodes
• Treatment: esophagectomy

MISCELLANEOUS LESIONS
PLUMMER-VINSON SYNDROME
• Triad of iron-deficiency anemia, dysphagia, esophageal webs
• Can be a premalignant lesion for SCCA
SCHATZKI RING
• Thin submucosal ring in the lower esophagus
• Treatment: esophageal dilatation/antireflux procedure LYMPHATIC DRAINAGE
MALLORY-WEISS TEAR DRAINED STRUCTURE
• Mucosal tears located at the gastroesophageal junction Superior gastric lymph nodes • Proximal lesser curvature
• Presentation: UGIB after repeated vomiting Suprapyloric lymph nodes • Distal lesser curvature
o Treatment: supportive management (heals spontaneously) Pancreaticosplenic lymph
• Proximal greater curvature
nodes
Infrapyloric lymph nodes • Distal greater curvature
STOMACH
REGIONS
DESCRIPTION
• Connects to the esophagus
• First region of the stomach into which material
Cardia
(food) passes
• Important for the hydrochloric acid (HCI) secretion
• Superior-most
• Crucial role in capacitance by undergoing receptive
relaxation
Fundus • Site of the autonomic pacemaker responsible for
initiating gastric motor activity
• Inferior extent defined by horizontal plane of the
gastroesophageal junction
Body
• From the cardiac orifice to the incisura angularis
(Corpus)
• From the incisura angularis to the pylorus STOMACH INNERVATION
Antrum LOCATION AND DISTRIBUTION
• Forms the distal 25-30% of stomach
• Distal tubular part with the pyloric sphincter as its • Lies on the anterior surface
Pylorus Left vagal trunk
muscular wall • Gives hepatic branch
(anterior)
Surgery Platinum. 1st ed. 2018. p. 267 • Continues as the nerve of Latarjet
BLOOD SUPPLY • Lies on the posterior surface
ARTERIAL SUPPLY Right vagal trunk • Gives celiac branch
Less • Right gastric artery ← common hepatic artery (posterior) • Continues as the criminal nerve of
curvature • Left gastric artery ← celiac trunk Grassi
• Right gastroepiploic artery ←
Greater
gastroduodenal artery
curvature
• Left gastroepiploic artery ← splenic artery
Fundus • Short gastric arteries ← splenic artery
VENOUS DRAINAGE
VENOUS DRAINAGE
Drain to the portal • Right gastric vein
circulation • Left gastric vein or coronary vein
• Short gastric vein
Drain to the splenic
• Left gastroepiploic vein
vein
• Coronary vein (occasionally)
Drain to the superior
• Right gastroepiploic vein
mesenteric vein

Figure 26-5. Schwartz’s Principles of Surgery. 11th ed. 2019

TOPNOTCH MEDICAL BOARD PREP SURGERY MAIN DIGITAL HANDOUT BY LOUBOMIR ANTONIO, MD Page 51 of 101
For inquiries visit www.topnotchboardprep.com.ph or https://www.facebook.com/topnotchmedicalboardprep/
This handout is only valid for the September 2021 PLE batch. This will be rendered obsolete for the next batch since we update our handouts regularly.
TOPNOTCH MEDICAL BOARD PREP SURGERY MAIN DIGITAL HANDOUT BY LOUBOMIR ANTONIO, MD
For inquiries visit www.topnotchboardprep.com.ph or https://www.facebook.com/topnotchmedicalboardprep/
This handout is only valid for the September 2021 PLE batch. This will be rendered obsolete for the next batch since we update our handouts regularly.
STOMACH HISTOLOGY ✔ GUIDE QUESTION
CELLS PRODUCTS A 50-year-old man is involved in a major motor vehicle collision and
Surface mucous • Protective mucus layer that contains suffers multiple trauma. He is admitted to the intensive care unit. After
cells bicarbonate and glycoprotein 2 days of hospital admission he bleeds massively from the stomach.
• Less alkaline mucus layer that contains What is the probable cause?
(A) Gastric ulcer
Mucus neck cells glycoprotein
(B) Duodenal ulcer
• Pepsinogen (C) Hiatal hernia
Parietal (oxyntic) • Hydrochloric acid (D) Mallory-Weiss tear
cells • Intrinsic factor (for absorption of Vit B12) (E) Erosive gastritis
• Pepsinogen
Chief (zymogenic) Acute erosive gastritis (stress gastritis or reactive gastritis) has a number
• Gastric lipase of causes, including certain drugs; alcohol; bile; ischemia; bacterial, viral,
cells
• Leptin and fungal infections; acute stress (shock); radiation; allergy and food
Enterochromaffin poisoning; and direct trauma. The common mechanism of injury is an
• Serotonin
cells imbalance between the aggressive and the defensive factors that maintain
Enterochromaffin- the integrity of the gastric lining (mucosa). A gastric and duodenal ulcer
• Histamine
like cells can present with massive bleeding and should be ruled out (but the most
D cells • Somatostatin plausible cause is gastritis). A hiatal hernia usually presents with GERD-
G cells • Gastrin symptoms not bleeding. A history of forceful retching is needed to suspect
Surgery Platinum. 1st ed. 2018. p. 269 a Mallory-Weiss tear.
Dr. Cocos

SUPPLEMENT: DIAGNOSTIC TESTS FOR THE STOMACH


1. EGD
§ patients with one or more of the alarm symptoms must undergo
immediate upper endoscopy
§ requires an 8-hour fasting
§ more sensitive than double contrast upper GI series
§ most serious complication: esophageal perforation
2. Radiologic tests
§ Plain abdominal x-ray
- helpful in the diagnosis of gastric perforation
(pneumoperitoneum) or delayed gastric emptying (large air-
fluid level)
§ Double contrast upper GI series
- better than EGD in detecting the ff: diverticula, fistula,
tortuosity or stricture location, and size of hiatal hernia
3. CT and MRI
§ is part of routine staging work-up for most patients with a
Figure 26-6. Schwartz’s Principles of Surgery. 11th ed. 2019 malignant gastric tumor
4. Gastric secretory analysis
✔ GUIDE QUESTION § maybe useful in the evaluation of patients with
A 55-year-old man complains of anorexia, weight loss, and fatigue. AUGI hypergastrinemia, including Zollinger-Ellison syndrome,
study demonstrates an ulcerated lesion at the incisura. patients with refractory ulcer or GERD or recurrent ulcer after
Where is the incisura? operation
(A) Cardia 5. Tests for Helicobacter pylori
(B) Fundus § Serologic test for H. pylori
(C) Greater curvature - a positive test is a presumptive evidence of active infection if
(D) Lesser curvature the patient has never been treated for H. pylori infection
(E) Gastrocolic ligament § Histologic examination of antral mucosal biopsy (with special
Angularis incisura refers to the sharp angular depression in the stains)
lesser curvature of the stomach at the junction of the body with - gold standard for H. pylori
the pyloric canal. Review the regions of the stomach as illustrated § Urease breath test
in the earlier part of the stomach discussion. - standard test to confirm eradication of H. pylori post-
Dr. Cocos treatment
- basis: the patient ingests urea labeled with nonradioactive 13C
STOMACH – CONTROL OF ACID SECRETION → labeled urea is acted upon by the urease present in H. pylori
→ converts urea into ammonia and carbon dioxide →
• Acetylcholine radiolabeled carbon dioxide is excreted from lungs and is
Induces acid
• Histamine detected in expired air.
secretion
• Gastrin § H. pylori fecal antigen test
Inhibits acid - sensitive and specific for active H. pylori infection
• Somatostatin - can also be used to confirm cure
secretion
PEPTIC ULCER DISEASE
• Focal defects in the gastric or duodenal mucosa which extends
into the submucosa or deeper

PATHOPHYSIOLOGY

Adapted from Schwartz’s Principles of Surgery. 11th ed. 2019

Adapted from Schwartz’s Principles of Surgery. 11th ed. 2019

TOPNOTCH MEDICAL BOARD PREP SURGERY MAIN DIGITAL HANDOUT BY LOUBOMIR ANTONIO, MD Page 52 of 101
For inquiries visit www.topnotchboardprep.com.ph or https://www.facebook.com/topnotchmedicalboardprep/
This handout is only valid for the September 2021 PLE batch. This will be rendered obsolete for the next batch since we update our handouts regularly.
TOPNOTCH MEDICAL BOARD PREP SURGERY MAIN DIGITAL HANDOUT BY LOUBOMIR ANTONIO, MD
For inquiries visit www.topnotchboardprep.com.ph or https://www.facebook.com/topnotchmedicalboardprep/
This handout is only valid for the September 2021 PLE batch. This will be rendered obsolete for the next batch since we update our handouts regularly.
✔ GUIDE QUESTION
A 45-year-old man complains of burning epigastric pain that wakes him up
at night. The pain is relieved by eating or using over-the counter antacids
and H2 blockers. Diagnosis is best confirmed by which of the ff?
(A) Urea breath test
(B) Serum gastrin levels
(C) Barium meal examination
(D) Upper endoscopy
(E) Upper endoscopy and biopsy
Symptoms of this patient point to a possible ulcer, hence, endoscopy is
the best confirmatory test. A biopsy is warranted routinely if a
stomach ulcer is diagnosed to rule out possible malignancy. Urea
breath test is used for H. pylori diagnosis. Serum gastrin level is a
diagnostic test for patients suspected to have gastrinoma. Barium
meal examination is a screening tool for dysphagia but will not
PEPTIC ULCER DISEASES CAUSES confirm the diagnosis of ulcer.
Adapted from Schwartz’s Principles of Surgery. 11th ed. 2019
Dr. Cocos

PEPTIC ULCER DISEASE MANIFESTATIONS COMPLICATIONS


DUODENAL ULCER Bleeding
GASTRIC ULCER (GU) • Most common complication
(DU)
• First portion of the • Presentation: melena or hematemesis with or without shock
• At the incisura, near
Usual location duodenum (within • Diagnosis: NGT and Early endoscopy
the lesser curvature
3 cm of pylorus) • Treatment:
Age group • Older age group • Younger age group o Non-operative: NPO, acid suppression, endoscopic treatment
Sex (epinephrine/cautery)
• M:F 1:1 • M:F 2:1
prediction o Indications for Surgery
Pain in • Pain occurs while • Pain 2-3 hours § Persistent bleeding /rebleeding after endoscopic therapy
relation to eating after meal § Significant hemorrhage (>4 units/24 hours)
meals • Food worsens pain • Food relieves pain § Elderly patients with co-morbidities
Pain in § Ulcers located at posterior duodenal bulb and ulcers located
• Less likely to • Awakens the
relation to at the gastric lesser curvature
awaken patient patient from sleep
sleep § High risk of rebleeding based on endoscopic findings (active
Risk of • Common (should be pulsatile bleeding, visible vessel)
• Extremely rare
malignancy biopsied)
• H. pylori, NSAID- • H. pylori, NSAID-
THE FORREST CLASSIFICATION FOR ENDOSCOPIC FINDINGS
Usual etiology AND REBLEEDING RISKS
induced injury induced injury
Classification Rebleeding Risk
• Due to increased
• Due to decreased production of Grade Ia: active, pulsatile bleeding High
(gastric) gastric aid Grade Ib: active, non-pulsatile bleeding High
Patho- Grade IIa: nonbleeding visible vessel High
cytoprotection • Bicarbonate
physiology Grade IIb: adherent clot Intermediate
• Gastric acid output secretion is
normal of decreased significantly Grade IIc: black dot Low
decreased Grade III: no signs of recent bleeding Low
• Decrease Grade 1a ulcers have >85% risk of rebleeding. Grade III lesions on the
• Gastrectomy other hand have <3% risk of rebleeding.
Goal of production of
• Rule out cancer with Dr. Cocos
management gastric acid (e.g.,
biopsy Perforation
vagotomy)
• Usually presents as acute abdomen with marked abdominal
• Bleeding (most common): melena or coffee-
guarding and rebound tenderness
ground emesis
• Penetrating ulcer: pain becomes constant, no
• Upright chest radiograph: pneumoperitoneum in 80%
Complications • Treatment: Non-operative (sealed leak w/o signs of
longer relieved by antacids, radiates to the back
(for both peritonitis)
• Perforation: sudden severe, generalized
ulcer types) Gastric outlet obstruction
abdominal pain
• Gastric outlet obstruction: pain worsening with • Presentation: nonbilious vomiting
meals, nausea or vomiting of undigested food o Leads to profound hypokalemic hypochloremic metabolic
Surgery Platinum. 1st ed. 2018. p.270 alkalosis
PEPTIC ULCER DIAGNOSIS o Treatment: Non-operative
DIAGNOSTIC REMARKS ✔ GUIDE QUESTION
• Young patients with symptoms: may start A 63-year-old woman is admitted to the hospital with severe abdominal
empiric treatment pain of 3-hour duration. Abdominal examination reveals board-like rigidity,
Upper
• Endoscopy indicated in: guarding, and rebound tenderness. Her blood pressure is 90/50 mm Hg,
endoscopy pulse 110 bpm (beats per minute), and respiratory rate is 30 breaths per
o All patients >45 years with symptoms
(EGD) minute. After a thorough history and physical, and initiation of fluid
o With alarm symptoms: weight loss, bleeding,
recurrent vomiting, anemia, dysphagia resuscitation, what diagnostic study should be performed?
(A) Supine abdominal x-rays
• All gastric ulcers and areas of gastritis should
(B) Upright chest x-ray
Biopsy be biopsied to rule out carcinoma and H pylori (C) Gastrografin swallow
infection (D) CT scan of the abdomen
• Serology: test of choice when endoscopy is not (E) Abdominal sonogram
indicated
An upright chest x-ray is probably the most sensitive plain radiograph
• Urea breath test: preferred for confirming
for the detection of free intraperitoneal gas (pneumoperitoneum)
H. pylori • Histologic test: ascertain presence of H pylori which is an indirect marker for bowel perforation. It appears as a
testing • Rapid urease test: simplest method when lucency beneath the right hemidiaphragm (above the right lobe of the
endoscopy is necessary liver). Supine abdominal x-rays and gastrograffin swallow are helpful
• Culture: done for repeated failure of medical in identifying a possible bowel obstruction but will not rule out
treatment pneumoperitoneum. An abdominal sonogram may demonstrate free
Other • CBC, chemistry, bleeding, or clotting parameters fluid, but not free air. Although a CT scan will show both free fluid and
diagnostics • Baseline serum gastrin level free air, it will take longer to perform and may delay the definitive
Surgery Platinum. 1st ed. 2018. p.271 treatment. The combination of an acute abdomen and an upright
chest x-ray with free air under the diaphragm provides enough
information to take the patient to the operating room for exploration.
Dr. Cocos

TOPNOTCH MEDICAL BOARD PREP SURGERY MAIN DIGITAL HANDOUT BY LOUBOMIR ANTONIO, MD Page 53 of 101
For inquiries visit www.topnotchboardprep.com.ph or https://www.facebook.com/topnotchmedicalboardprep/
This handout is only valid for the September 2021 PLE batch. This will be rendered obsolete for the next batch since we update our handouts regularly.
TOPNOTCH MEDICAL BOARD PREP SURGERY MAIN DIGITAL HANDOUT BY LOUBOMIR ANTONIO, MD
For inquiries visit www.topnotchboardprep.com.ph or https://www.facebook.com/topnotchmedicalboardprep/
This handout is only valid for the September 2021 PLE batch. This will be rendered obsolete for the next batch since we update our handouts regularly.
MEDICAL TREATMENT OF PUD
Inhibition of acid secretion NSAID withdrawal
Smoking cessation Eradicate H pylori
Alcohol avoidance Repeat EGD and biopsy at 6-8 wks
✔ GUIDE QUESTION
A 30-year-old executive learns that he has a duodenal ulcer. His
gastroenterologist prescribes and outlines medical therapy. The
patient worries that he may need surgery if medical therapy fails. Which
of the following is the best indication for elective surgical therapy for
duodenal ulcer disease?
(A) An episode of melena
(B) Repeated episodes of pain
(C) Pyloric outlet obstruction due to scar formation from an ulcer
(D) Frequent recurrences of ulcer disease
(E) Referral of pain to the back, suggestive of pancreatic penetration
Surgical intervention for peptic ulcer disease is uncommon. It is
indicated by four clinical situations—intractable pain, hemorrhage,
perforation, and obstruction. Noncompliance with medication is often
the cause of recurrence. Patients with gastric decompression need a
nasogastric tube and fluid and electrolyte correction prior to surgery.
The most common surgical procedures performed for gastric outlet
obstruction related to PUD are vagotomy and antrectomy, vagotomy GASTRIC ULCER TYPES
Figure 26-30. Schwartz’s Principles of Surgery. 11th ed. 2019
and pyloroplasty, truncal vagotomy and gastrojejunostomy, and
pyloroplasty.
Dr. Cocos

SURGICAL TREATMENT OF PUD


INDICATION DUODENAL GASTRIC
1. Oversewa 1. Oversew and biopsya
Bleeding 2. Oversew, V + D 2. Oversew, biopsy, V + D
3. V + A 3. Distal gastrectomyb
1. Patcha 1. Biopsy and patcha
Perforation 2. Patch, HSV 2. Wedge excision, V + D
3. Patch, V + D 3. Distal gastrectomyb
1. HSV + GJ 1. Biopsy; HSV + GJ
Obstruction
2. V + A 2. Distal gastrectomyb
1. HSVb 1. HSV and wedge excision 2.
Intractability/
2. V + D Distal gastrectomy
nonhealing
3. V + A
aUnless the patient is in shock or moribund, a definitive procedure should be

considered.
bOperation of choice in low-risk patient.

GJ = gastrojejunostomy; HSV = highly selective vagotomy; V+A = vagotomy


and antrectomy; V+D = vagotomy and drainage.
Schwartz’s Principles of Surgery, 11th edition, p.1133
Type Feature
Type I Antral lesser curvature (most common)
Type II Type I ulcer + duodenal ulcer
Type III Prepyloric ulcer
Type IV High in the lesser curvature
Type V NSAID-induced
Mnemonic: “one is less (lesser curvature), two has two (ulcers), three is
pre (prepyloric), four is by the door, five is NSAID (5 letters)”
Dr. Cocos

PUD
https://qrs.ly/tocl102

Adapted from Schwartz’s Principles of Surgery. 11th ed. 2019

Adapted from Schwartz’s Principles of Surgery. 11th ed. 2019

TOPNOTCH MEDICAL BOARD PREP SURGERY MAIN DIGITAL HANDOUT BY LOUBOMIR ANTONIO, MD Page 54 of 101
For inquiries visit www.topnotchboardprep.com.ph or https://www.facebook.com/topnotchmedicalboardprep/
This handout is only valid for the September 2021 PLE batch. This will be rendered obsolete for the next batch since we update our handouts regularly.
TOPNOTCH MEDICAL BOARD PREP SURGERY MAIN DIGITAL HANDOUT BY LOUBOMIR ANTONIO, MD
For inquiries visit www.topnotchboardprep.com.ph or https://www.facebook.com/topnotchmedicalboardprep/
This handout is only valid for the September 2021 PLE batch. This will be rendered obsolete for the next batch since we update our handouts regularly.
✔ GUIDE QUESTIONS
A healthy 75-year-old man bleeds from a duodenal ulcer. Medical
management and endoscopic measures fail to stop the bleeding. What
is the next step in management?
(A) Continued transfusion of 8 U of blood
(B) Administration of norepinephrine
(C) Oversewing of the bleeding point
(D) Oversewing of the bleeding point, vagotomy, and
pyloroplasty
(E) Hepatic artery ligation
Since the cause of duodenal ulcers is usually due to acid
hypersecretion, a vagotomy is warranted plus a drainage procedure
(pyloroplasty) IF the patient is stable enough to tolerate these
procedures. Otherwise, oversewing of the bleeder is acceptable.
OMENTAL PATCHING
Continued transfusion is futile if the main bleeding ulcer is not
addressed. Administration of norepinephrine will likely not be
successful in stopping the bleed. Hepatic artery ligation might be
effective for bleeding liver lesion but not for a duodenal ulcer.
Dr. Cocos
A 44-year-old was admitted to the hospital with a 1-day history of
hematemesis caused by a recurrent duodenal ulcer. He has shown
considerable improvement following operative treatment by a truncal
vagotomy and pyloroplasty, 10 years prior to this incident. Which is
TRUE of truncal vagotomy?
(A) It is performed exclusively via the thorax.
(B) It can be performed in the neck.
BILLROTH I GASTRODUODENOSTOMY (C) If complete, it will result in increased acid secretion.
Figure 26-39. Schwartz’s Principles of Surgery. 11th ed. 2019
(D) It requires a gastric drainage procedure
(E) It has been abandoned as a method to treat ulcer disease.
Truncal vagotomy, unlike HSV, also denervates the antrum and the
pylorus hence, a drainage procedure via a pyloroplasty or
gastrojejunostomy is required to prevent gastric stasis or outlet
obstruction. This can be done via a laparotomy approach but not
through a neck incision. If complete truncal vagotomy is done this will
result in decreased acid secretion and is still being done today
(although rarely) to treat ulcer disease.
Dr. Cocos

BILLROTH II ANTECOLIC GASTROJEJUNOSTOMY A 42-year-old executive has refractory chronic duodenal ulcer disease.
Figure 26-40. Schwartz’s Principles of Surgery. 11th ed. 2019 His physician has suggested several surgical options. The patient has
chosen a parietal (highly selective) vagotomy instead of a truncal
vagotomy and antrectomy because?
(A) It results in a lower incidence of ulcer recurrence.
(B) It benefits patients with antral ulcers the most.
(C) It reduces acid secretion to a greater extent.
(D) The complication rate is lower.
(E) It includes removal of the ulcer.
HSV has the least morbidity but also has the highest recurrence rate.
It benefits patients with duodenal ulcers the most but reduces acid
secretion to a lesser extent compared to truncal vagotomy. This
TRUNCAL VAGOTOMY HIGHLY SELECTIVE VAGOTOMY procedure does not resect the ulcer itself.
Figure 26-34 and Figure 26-33. Schwartz’s Principles of Surgery. 11th ed. 2019 Dr. Cocos
Three months after recovery from an operation to treat peptic ulcer
disease, a patient complains that she has difficulty eating a large meal.
A 99m Tc-labeled chicken scintigraphy test confirms a marked delay in
gastric emptying. A delay in gastric emptying may be due to which of
the following?
(A) Zollinger-Ellison syndrome (ZES)
(B) Steatorrhea
(C) Massive small-bowel resection
(D) Previous vagotomy
(E) Hiatal hernia
Vagotomy results in denervation of the stomach hence marked delay
in gastric emptying without the proper drainage procedure. The rest
of the choices would not cause delayed gastric emptying.
Dr. Cocos
A 68-year-old woman has been diagnosed with a benign ulcer on the
greater curvature of her stomach. After 3 months of standard medical
therapy, she continues to have guaiac positive stool, anemia, and
abdominal pain with failure of the ulcer to heal. Biopsies of the gastric
ulcer have not identified a malignancy. The next step in management is
HEINEKE-MIKULICZ PYLOROPLASTY
Figure 26-36. Schwartz’s Principles of Surgery. 11th ed. 2019 which of the following?
(A) Treatment of the anemia and repeat all studies in 6 weeks
(B) Endoscopy and bipolar electrocautery or laser photocoagulation
of the gastric ulcer
(C) Admission of the patient for total parenteral nutrition (TPN),
treatment of anemia, and endoscopic therapy
(D) Surgical intervention, including partial gastric resection
(E) Surgical intervention, including total gastrectomy
A symptomatic gastric ulcer that fails to heal despite medical therapy
should be excised hence surgical intervention is already warranted in
this patient (A, B and C are wrong) but removing the whole stomach
(total gastrectomy) is not necessary. Vagotomy is not necessary
because gastric ulcers are usually not associated with acid
hypersecretion.
Dr. Cocos

FINNEY PYLOROPLASTY JABOULEY PYLOROPLASTY


Figure 26-37 and Figure 26-38. Schwartz’s Principles of Surgery. 11th ed. 2019

TOPNOTCH MEDICAL BOARD PREP SURGERY MAIN DIGITAL HANDOUT BY LOUBOMIR ANTONIO, MD Page 55 of 101
For inquiries visit www.topnotchboardprep.com.ph or https://www.facebook.com/topnotchmedicalboardprep/
This handout is only valid for the September 2021 PLE batch. This will be rendered obsolete for the next batch since we update our handouts regularly.
TOPNOTCH MEDICAL BOARD PREP SURGERY MAIN DIGITAL HANDOUT BY LOUBOMIR ANTONIO, MD
For inquiries visit www.topnotchboardprep.com.ph or https://www.facebook.com/topnotchmedicalboardprep/
This handout is only valid for the September 2021 PLE batch. This will be rendered obsolete for the next batch since we update our handouts regularly.
POST-GASTRECTOMY PROBLEMS § Diversion of bile flow from the gastric mucosa
• Dumping syndrome • Gallstones § Preferred option: creation of a long-limb (45 cm) Roux-en-Y
• Diarrhea • Weight loss gastrojejunostomy
• Gastric stasis • Anemia
• Bile reflux gastritis • Bone disease
• Roux syndrome
DUMPING SYNDROME
• Caused by destruction of the pyloric sphincter causing abrupt
delivery of hyperosmolar load to the small intestines
• Early dumping (15-30 mins post-prandial)
o peripheral and splanchnic vasodilatation leads to shock-like
symptoms (sweating, light-headedness, tachycardia) that is
relieved by saline or recumbency
o diarrhea then follows
• Late dumping ( 2-3 hours post-prandial) ROUX STASIS SYNDROME
o due to hyperinsulinemia with reactive hypoglycemia • Pathophysiology
o relieved by glucose administration o Results from functional obstruction due to disruption of
• Treatment normal propagation of pacesetter potentials in the Roux limb
• Octreotide: for early dumping from the proximal duodenum, as well as altered motility in
Nonsurgical • Alpha-glucosidase inhibitors: for late gastric remnant
dumping • Clinical Manifestations
Surgical • Conversion to Roux-en-Y anastomosis o Characterized by chronic abdominal pain, nausea, and
Surgery Platinum. 1st ed. 2018. p. 274 vomiting that is aggravated by eating
BILE OR ALKALINE REFLUX GASTRITIS • Diagnosis
• Pathophysiology o Upper GI series confirms these findings and may show delayed
o Most commonly associated with Billroth II gastrojejunostomy gastric emptying
o Most often requires operative treatment o Endoscopy may show retained food or bezoars, dilation of the
• Clinical Manifestations gastric remnant, and/or dilation of the Roux limb
o Triad: constant epigastric pain, nausea, and bilious emesis o GI motility testing shows abnormal motility in the Roux limb,
o Vomiting does not relieve the pain and is not associated with with propulsive activity toward, rather than away from, the
meals stomach
• Diagnosis • Management
o Endoscopy: reveals inflamed, beefy red, friable gastric mucosa o Nonoperative: use of pro-motility agents
• Management o Surgery: Near-total or Total gastrectomy to remove the atonic
o Non-operative: frequent meals, antacids, and cholestyramine stomach
o Surgery:
LOOP SYNDROMES
AFFERENT LOOP SYNDROME EFFERENT LOOP SYNDROME
• Results from intermittent obstruction of the efferent
• Caused by acute bowel kink, volvulus, or internal herniation
limb of the gastrojejunostomy
Pathophysiology • Duodenal stump blowout results from progressive afferent limb
• Obstructions are usually due to internal herniation of
dilation, leading to peritonitis, abscess, or fistula formation
distal intestine behind the efferent limb
• Severe abdominal pain, and nonbilious emesis
• Chronic form present with postprandial right upper quadrant
pain relieved by bilious emesis that is not mixed with food • Abdominal pain and bilious emesis months to years
Manifestations
• Stasis can lead to bacterial overgrowth and bile salt after surgery
deconjugation causing blind loop syndrome (steatorrhea,
vitamin B12, folate, & iron deficiency)
• Upper gastrointestinal contrast study (UGIS) usually
• CT scan shows dilated afferent limb
reveals signs of a small bowel obstruction
Diagnosis • HIDA scan can show excretion by the biliary system but then
• CT scan can occasionally reveal internal herniation as
stagnation upon entering the afferent limb
a right-to-left herniation through the mesenteric defect
• Conversion to a Roux-en-Y anastomosis or creation of a
Management • Prompt surgical intervention to relieve obstruction
jejunojejunostomy (braun)
Surgery Platinum. 1st ed. 2018. p. 275
Note: After Billroth II reconstruction, the small intestine loop has 2 loops. The loop going to the upper stomach is the afferent loop. The loop carrying food away
from the stomach is called the efferent loop. After a roux-en-y gastrojejunostomy, the roux limb is the segment of jejunum attached directly to the remnant
stomach (sometimes also called the efferent limb since it transmits food away from the stomach).
Dr. Cocos

ZOLLINGER-ELLISON SYNDROME (1) Cystic duct and


• Uncontrolled secretion of gastrin by a pancreatic or duodenal CBD junction
neuroendocrine tumor (2) 2nd and 3rd part of
• 80% sporadic; 20% familial (MEN1) the Duodenum
• Gastrinoma; most common pancreatic tumor in patients with junction
(3) Junction of the
MEN1 - 50% are malignant with distant metastases at time of
neck and body of
diagnosis
Pancreas
• Presentation: epigastric pain, GERD and diarrhea (>90% with Mnemonic: Peptic-Causing Disease
peptic ulcer) • Elevated fasting serum gastrin and BAO
• Most ulcers are in typical location (proximal duodenum) (Basal Acid Output)
• Majority of gastrinomas are found in this • Confirmatory test: secretin stimulation test
Passaro
triangle Diagnosis (gastrin >200 pg/mL after IV secretin
Triangle
• Formed by three points: administration)
• Rule out the presence of MEN1 – Ca and PTH levels
• Preoperative imaging/localization: octreotide scan
• Sporadic cases: resection
Treatment • Familial cases: surgical debulking to ameliorate
symptoms

TOPNOTCH MEDICAL BOARD PREP SURGERY MAIN DIGITAL HANDOUT BY LOUBOMIR ANTONIO, MD Page 56 of 101
For inquiries visit www.topnotchboardprep.com.ph or https://www.facebook.com/topnotchmedicalboardprep/
This handout is only valid for the September 2021 PLE batch. This will be rendered obsolete for the next batch since we update our handouts regularly.
TOPNOTCH MEDICAL BOARD PREP SURGERY MAIN DIGITAL HANDOUT BY LOUBOMIR ANTONIO, MD
For inquiries visit www.topnotchboardprep.com.ph or https://www.facebook.com/topnotchmedicalboardprep/
This handout is only valid for the September 2021 PLE batch. This will be rendered obsolete for the next batch since we update our handouts regularly.

MALIGNANT NEOPLASMS OF THE STOMACH Physical Examination:


• Left supraclavicular
• Adenocarcinoma (95%) – most common malignant tumor Virchow node
lymphadenopathy
• Lymphoma (4%)
Krukenberg tumor • Ovarian metastasis
• Gastrointestinal tumor (GIST)
Sister Mary Joseph nodule • Metastatic umbilical nodule
• Rare primary malignancies:
• Cul-de-sac tumor palpable on
Carcinoid Blumer shelf
rectal examination
Angiosarcoma
EARLY GASTRIC ADENOCARCINOMA
Squamous carcinoma
• limited to the mucosa and submucosa
• Metastatic
• Treatment
GASTRIC ADENOCARCINOMA o Endoscopic Mucosal / Submucosal Resection (EMR/ ESR)
Risk factors § tumor <2cms
• Family history • Atrophic gastritis § limited to the mucosa or submucosa
• Diet (e.g., preserved, smoked, • Cigarette smoking o Surgery
and cured foods) and salt • Adenomatous gastric polyps § the only curative treatment
• Diet low in vitamins A and C • Menetrier disease § offers best palliation
• Familial polyposis and • Pernicious anemia § more accurate staging
adenomas • Type A blood o Goal: Resection of all tumor (R0 resection) with 4-6cms grossly
• Hereditary nonpolyposis • H. pylori infection negative margins + en-bloc removal of adjacent lymph nodes
colon cancer (HNPCC) and involved organs
• Previous gastric surgery
DIAGNOSTIC TOOLS FOR GASTRIC ADENOCARCINOMA
DESCRIPTION
Pathology
• Surveillance laboratory finding in gastric
• H pylori and autoimmune gastritis: most common etiologic
CEA cancer
lesions that create an environment for gastric inflammation to
• Useful for postoperative surveillance
develop
• Diagnostic gold standard
COURSE DESCRIPTION OR REMARKS
Upper • Allows direct visualization of the tumor,
• Universal precursor to adenocarcinoma (these endoscopy extent of mucosal involvement, and biopsy for
Dysplasia patients with dysplasia should be considered for (EGD) + tissue diagnosis
gastric resection) biopsy • At a minimum, 4-quadrant biopsies should be
• Occurs when dysplastic proliferation produces performed
macroscopic protruding lesions described as
Abdominal • To assess direct local invasion, regional
o Tubular adenoma
or pelvic CT lymphadenopathy, and possible liver
o Tubulovillous adenoma
Adenoma with IV and metastasis
o Villous adenoma
oral contrast • Most frequently used modality for staging
• Tend to occur in the distal stomach
• More accurate staging by assessing gastric
• Has a prolonged precancerous phase and Endoscopic
wall invasion and determining presence or
expanding growth pattern ultrasound
absence of lymph node metastases
• Early gastric adenocarcinoma: malignant cells
• detect occult metastases in up to 31% of cases
limited to the mucosa and submucosa, regardless Staging
• Reserved for medically fit patients with
Carcinoma of lymph node status laparoscopy
potentially resectable disease, specifically
• Carcinoma is diagnosed when the tumor invades with
when considerations are made for
the lamina propria or muscularis mucosae peritoneal
chemotherapy or surgery
Surgery Platinum. 1st ed. 2018. p. 276
fluid
Histology (Lauren Classification) • Role: to rule out peritoneal implants and to
cytology
o Diagnosis requires histopathologic assessment of tissue or evaluate for liver metastasis
cytologic assessment of gastric brushing or washes • Low detection rate due to low tracer
o Lauren Classification describes tumors based on microscopic accumulation in diffuse and mucinous
configuration and growth pattern tumor (30%)
PET scan
INTESTINAL TYPE DIFFUSE TYPE • Lower sensitivity compared to CT in the
• Composed of small detection of local lymph node involvement (56
• Well-delineated versus 78%)
cells that grow diffusely
or differentiated Surgery Platinum. 1st ed. 2018. p. 278
into surrounding gastric
Histology glandular STANDARD SURGICAL TREATMENT FOR GASTRIC
wall
structure ADENOCARCINOMA
• No glands & poorly
• Slow growth • For tumors of the gastric antrum (distal
differentiated
• More common in • Presents at a younger subtotal gastrectomy
less-developed age • Removal of:
nations • Less associated with Radical o Distal 75% of the stomach
Epidemiology subtotal o Pylorus + 2cm of proximal duodenum
• Predominant environment, more
subtype in high- associated with gastrectomy o Greater and lesser omentum
risk populations genetics o Associated lymph nodes
• Distal stomach • Reconstruction is by Billroth II gastrojejunostomy
Location • Cardia of stomach or Roux-en-Y gastrojejunostomy
• Often ulcerative
• Worse prognosis • Removal of stomach + Roux-en-y limb sewn in
esophagus
• Linitis plastica:
• Preceded by long • Done for:
characterized by
precancerous Radical o All lesions in the midbody of fundus (proximal
Prognosis extensive or complete
phase total of the stomach)
infiltration of the
• Better prognosis gastrectomy o Linitis plastica
entire stomach
(leather bottle) o Cancer associated with Menetrier disease,
Surgery Platinum. 1st ed. 2018. p. 277 gastric remnant carcinoma, and multiple
Presentation: diffuse gastric polyps
Surgery Platinum. 1st ed. 2018. p. 280
• Most common: weight loss + anorexia + early satiety
EXTENT OF LYMPH NODE RESECTION
• Abdominal pain, nausea, vomiting, bloating REMARKS REMARKS
• UGIB, dysphagia D0 • Gastrectomy + incomplete resection of N1 nodes
• Paraneoplastic syndrome D1 • Gastrectomy + complete dissection of the N1 nodes
o Trousseau syndrome (migratory thrombophlebitis) D2 • Gastrectomy + resection of N1 and N2 nodes
o Acanthosis nigricans D3 • Gastrectomy + resection of N1 to N3 nodes
D4 • Most expensive, includes all lymph node levels
TOPNOTCH MEDICAL BOARD PREP SURGERY MAIN DIGITAL HANDOUT BY LOUBOMIR ANTONIO, MD Page 57 of 101
For inquiries visit www.topnotchboardprep.com.ph or https://www.facebook.com/topnotchmedicalboardprep/
This handout is only valid for the September 2021 PLE batch. This will be rendered obsolete for the next batch since we update our handouts regularly.
TOPNOTCH MEDICAL BOARD PREP SURGERY MAIN DIGITAL HANDOUT BY LOUBOMIR ANTONIO, MD
For inquiries visit www.topnotchboardprep.com.ph or https://www.facebook.com/topnotchmedicalboardprep/
This handout is only valid for the September 2021 PLE batch. This will be rendered obsolete for the next batch since we update our handouts regularly.
Surgery Platinum. 1st ed. 2018. p. 281
• Pathology
NODAL
REMARKS TYPES REMARKS
GROUPS
• Histologic confirmation of lymphoma without any
• Perigastric nodes along greater and lesser
N1 Primary evidence of peripheral lymphadenopathy or
curvatures (Levels 1-6)
hepatosplenomegaly
• Adjacent to the celiac axis and its major branches
N2 • Indicates the involvement of the stomach by a
(Levels 7, 8a, 9, 10, 11p, 11d, 12a, 14v) Secondary
diffuse lymphoma elsewhere
• Hepatoduodenal ligament, retropancreatic region,
N3 • A recurrence within the stomach after the
celiac plexus, and superior mesenteric artery Tertiary
treatment of lymphoma of the peripheral basin
N4 • Paraaortic nodes Surgery Platinum. 1st ed. 2018. p. 282
Surgery Platinum. 1st ed. 2018. p. 280
PRIMARY GASTRIC LYMPHOMA
LN LOW-GRADE MALT HIGH-GRADE
LOCATION
GROUP LYMPHOMA LYMPHOMA
1 R cardiac • Indolent
2 L cardiac Behavior • Aggressive
• Not a surgical lesion
3 Lesser curvature • Nearly always arise
4 Greater curvature • Non-MALT type,
from mucosa or
5 Suprapyloric although many may
submucosa, from the
6 Infrapyloric Histology have been initially
mucosa-associated
7 L gastric artery MALT lymphomas that
lymphoid tissues
8 Common hepatic artery progressed
(MALT)
9 Celiac artery
• Associated with
10 Splenic hilar • Although the
inactivation of p53
11 Splenic artery stomach is devoid of
tumor-suppressor
12 Hepatic pedicle lymphoid tissues,
Patho- gene expression of the
13 Retroduodenal MALT can arise in
physiology replication error
14 Mesenteric root response to chronic
phenotype, and c-myc
15 Middle colic artery inflammation
proto-oncogene
16 Paraaortic (H. pylori infection)
mutation
Around lower
17 Surgery Platinum. 1st ed. 2018. p. 282
esophagus • Clinical Manifestation
18 Supradiaphragmatic o Nonspecific upper GI symptoms: abdominal pain, anorexia,
Gastrectomies for gastric adenocarcinoma usually include D2 nausea, vomiting, and weight loss
lymphadenectomy (n1 and n2). No need to memorize, just familiarize! J o Bleeding: more common complication (rather than perforation)
Dr. Cocos • Diagnosis
✔ GUIDE QUESTION o Need to rule out extra gastric disease
A 60-year-old man has been having vague symptoms of upper o Endoscopy (with biopsy) alone or in combination with barium
abdominal discomfort, early satiety, and fatigue. He is referred to a radiography
gastroenterologist, who performs an upper endoscopy. Although a o CT scans of the chest, abdomen and pelvis
discrete mass is not visualized, the stomach looks abnormal. It does not o Bone marrow biopsy
distend easily with insufflation. A biopsy shows signet ring cells. Which • Management
of the ff is TRUE? • Nonsurgical lesion
(A) Signet ring cells are typically found in intestinal type gastric
• H. pylori eradication represents the first line of
adenocarcinoma.
treatment
(B) Signet ring cell cancer is the most common type of gastric cancer.
(C) “Leather bottle stomach” is a term used to describe a non- Low-grade • Persistent localized lesions second course of H.
distensible stomach infiltrated by cancer. gastric pylori eradication + low doses (30 Gy) of external
(D) The gross appearance of the stomach always shows classic lymphoma beam radiation (EBRT)
findings of linitis plastica. • Advanced lesions: Initial H. pylori eradication +
(E) Linitis plastica has an excellent prognosis. EBRT with chemotherapy, then endoscopic
surveillance
Linitis plastica is a diffuse type of gastric adenocarcinoma and has
poor prognosis. It is described as leather bottle stomach for the entire • Chemoradiation for most cases
stomach is involve and it is no longer distensible to insufflation. The High-grade • Surgery: reserved for urgent situations of tumor
Lauren classification divides gastric adenoccarcinomas into two gastric complications like acute hemorrhage,
histologic types—an intestinal type and a diffuse type. The intestinal lymphoma obstruction not relieved by steroids, or failure of
type is more common and usually forms a discrete lesion. The diffuse response to chemoradiation
infiltrating type is less common and a mass may not be seen. In the Surgery Platinum. 1st ed. 2018. p. 282

intestinal type, cells form glandular structures; in the diffuse type, cells GASTROINTESTINAL STROMAL TUMOR (GIST)
are poorly organized and full of mucin (signet ring cells). The diffuse • Arise from the interstitial cells of Cajal
type may extensively infiltrate the muscles of the stomach, thus
leading to rigidity.
• Prognostic Factors: tumor size and mitotic count
Dr. Cocos • Submucosal in location, metastasize hematogenously
• Tumor Marker: c-KIT (CD117) and CD34
GASTRIC LYMPHOMA
• Presentation:
• Pathophysiology o Usually asymptomatic
o More than 50% of gastrointestinal lymphomas arise from the o Bleeding, early satiety, abdominal pain/fullness
stomach • Diagnosis: endoscopy + biopsy, CT scan
o Over 95% are non-Hodgkin type • Treatment: wedge resection with negative margins
o Over half of patients with non-Hodgkin lymphoma have o Unresectable, metastatic GIST: Imatinib (Gleevec)
involvement of the GI tract
o Most are B-cell type, thought to arise in mucosa associated GASTRIC CARCINOIDS
lymphoid tissue (MALT) • Rare location for carcinoids (1%)
• Risk factors: • Arise from gastric ECL cells (Schwartz)
o Chronic gastritis due to H. pylori infection (culprit)
SUBTYPES REMARKS
o Gene mutations
• Carcinoid associated with type A chronic atrophic
o Transplant-related immunosuppression
gastritis, with or without pernicious anemia
o Inflammatory bowel disease
• Presumably autoimmune in etiology caused by
o HIV infection
antibodies to parietal cells and intrinsic factor
Type I
• Most common variant
• Occurs in patients with hypergastrinemia
• Usually affects women
• Has an often-benign course
TOPNOTCH MEDICAL BOARD PREP SURGERY MAIN DIGITAL HANDOUT BY LOUBOMIR ANTONIO, MD Page 58 of 101
For inquiries visit www.topnotchboardprep.com.ph or https://www.facebook.com/topnotchmedicalboardprep/
This handout is only valid for the September 2021 PLE batch. This will be rendered obsolete for the next batch since we update our handouts regularly.
TOPNOTCH MEDICAL BOARD PREP SURGERY MAIN DIGITAL HANDOUT BY LOUBOMIR ANTONIO, MD
For inquiries visit www.topnotchboardprep.com.ph or https://www.facebook.com/topnotchmedicalboardprep/
This handout is only valid for the September 2021 PLE batch. This will be rendered obsolete for the next batch since we update our handouts regularly.
• Carcinoid-associated with Zollinger-Ellison MISCELLANEOUS LESIONS
syndrome
Type II HYPERTROPHIC GASTROPATHY (Menetrier Disease)
• Occur almost exclusively in patients with multiple
endocrine neoplasia type 1 syndrome • Associated with protein losing enteropathy & hypochlorhydria
• Carcinoid tumor of sporadic form or • Presentation: middle-aged men with epigastric pain, weight
neuroendocrine carcinoma loss, diarrhea and hypoproteinemia
• Not associated with hypergastrinemia • Increased risk of cancer
• More common in men • Biopsy: diffuse hyperplasia of surface mucus-secreting cells
Type III and decrease parietal cells
• Usually, solitary
• Has the worse prognosis • Treatment: Cetuximab (EDGF receptor blockade) or resection
• Patients usually have larger tumors and present at WATERMELON STOMACH (Gastric Antral Vascular Ectasia)
a more advanced stage
Surgery Platinum. 1st ed. 2018. p. 283 • Dilated mucosal blood vessels in the distal stomach
• Clinical Presentation • Resembles portal gastropathy (but proximal stomach)
o Caused by hormonal excess, local tumor growth, or metastases • Presentation:
o Maybe incidental at the time of surgery o Elderly women with chronic GI blood loss
o Only 10% of carcinoids present with carcinoid syndrome, o Associated autoimmune connective tissue disorder
which is usually associated with hepatic metastases o Chronic liver disease
• Carcinoid Syndrome • Treatment: estrogen and progesterone, endoscopic
Secretory diarrhea Pellagra treatment, antrectomy, TIPS if with portal hypertension
Flushing Cramping DIEULAFOY LESION
Telangiectasia Edema
• Congenital AV malformation, unusually large tortuous
Valvular heart disease Bronchial constriction
submucosal artery
• Presentation: men with UGIB which is usually intermittent
• Dx: • Diagnosis: Endoscopy
o endoscopy + biopsy
• Treatment: Endoscopic hemostasis, angiographic
o EUS
embolization oversew or resection
o Plasma chromogranin A and NSE
o 24-hour urine 5-HIAA (by product of serotonin) ✔ GUIDE QUESTIONS
o Localizing test (somatostatin analogue like indium octreotide) A 54-year-old man presents with a massive UGI bleed. After
resuscitation, endoscopy is performed. No esophageal varices, gastritis,
• Management
or gastric ulcers are seen. After copious irrigation, a pinpoint lesion is
o Carcinoid syndrome: debulking surgery + somatostatin seen near the GE junction. What can be said about this lesion?
analogue (Octreotide) (A) It is a carcinoid.
o Other agents that have been used for symptomatic treatment (B) It is related to alcohol use.
include histamine H1 and H2 receptor antagonists, methoxamine, (C) It is exclusively a mucosal lesion.
cyproheptadine, and diphenoxylate sodium with atropine. (D) Surgery if first-line therapy.
o resistant to most chemotherapeutic agents (E) Bleeding is from a submucosal vessel.
The lesion is likely a Dieulafoy lesion which is a vessel from a submucosal
SUBTYPES MANAGEMENT
artery and normal-appearing gastric mucosa is usual. First line treatment
• Endoscopic polypectomy is therapeutic endoscopy, it is not a carcinoid (bleeding is not a usual
Type I
• Antrectomy for recurrence presentation), and it is not related to NSAID or alcohol use.
• Treatment of the gastrinoma Dr. Cocos

Type II • Somatostatin analogues An alert nursing home patient is unable to swallow because of a
• Possible local excision or antrectomy neurologic disease and has lost a significant amount of weight. What
• En-bloc resection with regional lymph nodes treatment should be offered?
Type III (A) Central hyperalimentation
• Possible chemotherapy or radiation
Surgery Platinum. 1st ed. 2018. p. 284 (B) Intralipids
(C) Percutaneous endoscopic gastrostomy (PEG)
✔ GUIDE QUESTIONS
(D) Nasogastric feeding
A 36-year-old man presents with weight loss and a large palpable tumor (E) Cervical esophagostomy
in the upper abdomen. Endoscopy reveals an intact gastric mucosa
PEG insertion is warranted for neurologic patients no longer capable
without signs of carcinoma. Multiple biopsies show normal gastric
of swallowing and are in need of upbuilding. NGT feeding is temporary
mucosa. A UGI study shows a mass in the stomach. At surgery, a 3-kg
and puts the patient at risk for aspiration pneumonia. Cervical
mass is removed. It is necessary to remove the left side of the transverse
esophagostomy is a type of diversion and is not done for feeding
colon. What is the most likely diagnosis?
purposes. As much as possible, the enteral route is preferred hence
(A) Gastric cancer
central alimentation (e.g. TPN) and intralipids are of last resort.
(B) Gastrointestinal stromal tumor (GIST) Dr. Cocos
(C) Choledochoduodenal fistula
(D) Eosinophilic gastroenteritis
(E) Linitis plastica
GIST is a mesenchymal tumor (submucosal) hence it is not uncommon
GASTROSTOMY
for endoscopic biopsy results to show normal gastric mucosa. This is https://qrs.ly/ktcl114
an uncommon occurrence for gastric cancer (A) and linitis plastic (E).
choledochoduodenal fistula (C) and eosinophilic gastroenteritis (D)
would not present as palpable tumor mass in the upper abdomen.
Dr. Cocos PERCUTANEOUS
A 63-year-old woman is admitted to the hospital with a UGI bleed that ENDOSCOPIC
subsides spontaneously within a short time after admission. A barium
GASTROSTOMY
study shows a gastric ulceration that is described by the radiologist as
having a “doughnut sign.” What is the most likely diagnosis? (PEG)
(A) Lipoma (D) GIST
(B) Gastric ulcer (E) Carcinoma
(C) Ectopic pancreas
A GIST can occur in any part of the stomach. Most commonly they are
found in the submucosa and grow towards the lumen. Ulceration may
occur and give rise to the characteristic “doughnut sign” or “target
sign” on barium studies. Hematemesis and/or melena may sometimes STAMM
be massive. Local resection is curative.
GASTROSTOMY
A gastric ulcer and carcinoma is a close differential but the doughnut sign is Figure 26-65. Schwartz’s
classically described for GIST ulceration. A lipoma does not usually ulcerate Principles of Surgery. 11th ed.
2019
and an ectopic pancreatic tissue if present in the stomach is very rare.
Dr. Cocos

TOPNOTCH MEDICAL BOARD PREP SURGERY MAIN DIGITAL HANDOUT BY LOUBOMIR ANTONIO, MD Page 59 of 101
For inquiries visit www.topnotchboardprep.com.ph or https://www.facebook.com/topnotchmedicalboardprep/
This handout is only valid for the September 2021 PLE batch. This will be rendered obsolete for the next batch since we update our handouts regularly.
TOPNOTCH MEDICAL BOARD PREP SURGERY MAIN DIGITAL HANDOUT BY LOUBOMIR ANTONIO, MD
For inquiries visit www.topnotchboardprep.com.ph or https://www.facebook.com/topnotchmedicalboardprep/
This handout is only valid for the September 2021 PLE batch. This will be rendered obsolete for the next batch since we update our handouts regularly.

LIVER & GALLBLADDER


LIVER
ANATOMY OF THE LIVER
• Cantlie’s line – plane from gallbladder fossa to IVC separating
the TRUE left and right lobes of the liver;
• Falciform ligament – separates left lateral and left medial
segments
JANEWAY GASTROSTOMY • Anatomically divided by the falciform ligament (not true
Figure 26-67. Schwartz’s Principles of Surgery. 10th ed. 2015 division) into:
o Small left lobe
o Large right lobe
• Right lobe further divided by gall bladder, fissure of ligamentum
teres, IVC, fissure of ligamentum venosum into:
o Quadrate lobe
o Caudate lobe

Frank H. Netter Atlas of Human Anatomy. 5th ed. 2010


WITZEL GASTROSTOMY
Figure 26-66. Schwartz’s Principles of Surgery. 11th ed. 2019 LIVER – PORTA HEPATIS
PYLORIC STENOSIS • Posteroinferior
surface of liver
• Upper part lesser
omentum attached
to margin
• Contains – common
hepatic duct,
proper hepatic
artery, portal vein

Figure 31-2. Schwartz’s Principles of Surgery. 11th ed. 2019


• Characteristics:
Regurgitation of feedings Olive-shaped mass in right LIVER – SEGMENTAL ANATOMY (Couinaud Segments)
Projectile non-bilious emesis upper epigastrium • Based on portal vein blood supply and hepatic venous outflow
Peristaltic gastric waves Hypokalemic hypochloremic Segment 1 • Caudate lobe
metabolic alkalosis Segments 2 and 3 • Left lateral segment
Segment 4 • Left medial segment
• Diagnosis: history and physical examination, abdominal
Segments 5 and 8 • Right anterior lobe
ultrasound, abdominal radiograph
Segments 6 and 7 • Right posterior lobe
• Medical management: correct metabolic abnormalities
• Surgical management: Fredet-Ramstedt pyloromyotomy
Fredet-Ramstedt pylomyotomy - Incision is carried down to the level of
submucosa starting about 1 cm proximal to pylorus and runs across the
thickening to about 1 cm distal to the pylorus on the first part of the
duodenum.
Dr. Cocos

Figure 31-21. Schwartz’s Principles of Surgery. 11th ed. 2019

TOPNOTCH MEDICAL BOARD PREP SURGERY MAIN DIGITAL HANDOUT BY LOUBOMIR ANTONIO, MD Page 60 of 101
For inquiries visit www.topnotchboardprep.com.ph or https://www.facebook.com/topnotchmedicalboardprep/
This handout is only valid for the September 2021 PLE batch. This will be rendered obsolete for the next batch since we update our handouts regularly.
TOPNOTCH MEDICAL BOARD PREP SURGERY MAIN DIGITAL HANDOUT BY LOUBOMIR ANTONIO, MD
For inquiries visit www.topnotchboardprep.com.ph or https://www.facebook.com/topnotchmedicalboardprep/
This handout is only valid for the September 2021 PLE batch. This will be rendered obsolete for the next batch since we update our handouts regularly.
BLOOD SUPPLY In performing hepatic resection, a knowledge of the different lobes and
Portal vein • 75% of arterial supply to the liver segments of the liver is mandatory. The right and left lobes of the liver
are separated by an imaginary plane (Cantlie line) that passes between
Hepatic artery • 25% of arterial supply to the liver
the inferior vena cava (IVC) and which of the following?
(A) Portal vein
VENOUS DRAINAGE (B) Falciform ligament
Right hepatic vein • Drains segments 5-8 (C) Left margin of the quadrate lobe
Middle hepatic vein • Drains segments 4, 5 and 8 (D) Gallbladder
(E) Left margin of the caudate lobe
Left hepatic vein • Drains segments 2 and 3
IVC • Drains the caudate lobe Note that the falciform ligament does not divide the liver into true right
and left lobes but divides the left lobes into lateral and medial segments.
Dr. Cocos

LIVER FUNCTION TESTS


Hepatocellular injury • ALT, AST
Abnormal Synthetic • Albumin
Functions • Clotting factors except factor VIII
• Total Bilirubin
• B2
Cholestasis
• Alkaline phosphatase
• GGTP

• Measures liver’s synthetic function


• prothrombin time and INR: best test among
the 3 to measure the liver’s synthetic function
• PT is prolonged with conditions such as
Serum vitamin K deficiency or warfarin therapy
albumin, (because vitamin K is involved in the Y-
prothrombin carboxylation of factors used to measure
time & clotting prothrombin time)
factors (except ***factor VIII: not synthesized exclusively in the
Figure 31-8. Schwartz’s Principles of Surgery. 11th ed. 2019
factor VIII) liver – not an indicator of liver function
BILE DRAINAGE Clotting factor (in general) has the shortest half-
Bile canaliculi lives hence useful for determining liver
↓ failure/synthetic functions. Factor VII has the
Intrahepatic ducts shortest halflife (3-6 hours).
↓ • Indicators of integrity of hepatocellular
Common hepatic duct membranes; increased levels reflect
Cystic duct --> ↓ hepatocellular injury with leakage
Common bile duct • AST: can also be found in liver, cardiac
AST muscle, skeletal muscle, kidney, brain,
(formerly SGOT) pancreas, lungs, and red blood cells and thus
& ALT is less specific
(formerly SGPT) • ALT: more specific for liver disease
• AST:ALT ratio of >2:1 → alcoholic liver disease
• moderate increases: viral hepatitis
• in the thousands → ischemia, toxin ingestion
(acetaminophen), fulminant hepatitis
• elevations point to intrahepatic cholestasis,
Indirect hemolytic disorders (hemolytic anemia,
(unconjugated) resorption of hematomas), bilirubin defects
bilirubin in hepatic uptake or conjugation (acquired
or inherited)
• elevations point to extrahepatic or
Direct
obstructive cholestasis, inherited or
(conjugated)
acquired disorders of intrahepatic excretion
bilirubin
✔ GUIDE QUESTIONS or extrahepatic obstruction
A 20-year-old man is brought to the emergency department with a • found in liver and bones;
gunshot wound to the abdomen. His blood pressure is 70 systolic and • indicative of biliary obstruction
Alkaline
his heart rate is 140 beats per minute (bpm). He is taken directly to the • since half-life of AP is 7 days, it may take
phosphatase
operating room for an exploratory laparotomy. A large, actively several days for the levels to normalize even
bleeding liver laceration is found. A pringle maneuver is performed as after resolution of biliary obstruction
part of the procedure to control his bleeding. The pringle maneuver • early marker and sensitive test for
compresses which structures? hepatobiliary disease
(A) Portal vein, hepatic vein, and hepatic artery
• nonspecific; can also be elevated in overdose of
(B) Portal vein, hepatic artery, and cystic artery
certain medications, alcohol abuse, pancreatic
(C) Portal vein and hepatic artery GGTP
(D) Portal vein, hepatic artery, and CBD disease, myocardial infarction, renal failure, &
(E) Cystic artery, cystic duct, and CBD obstructive pulmonary disease
• interpret GGTP elevations with other
During a pringle maneuver, a large atraumatic hemostat is used to clamp enzyme abnormalities
the hepatoduodenal ligament (free border of the lesser omentum)
interrupting the flow of blood through the hepatic artery and the portal JAUNDICE
vein and thus helping to control bleeding from the liver. If the bleeding
• Detectable when serum bilirubin > 2.5 to 3 mg/dL
persists, the bleeding might be coming from the hepatic veins.
Dr. Cocos ETIOLOGIES OF JAUNDICE
Prehepatic • Acquired/inherited Hemolytic anemias
pathology • Protein loss
• Gilbert disease
Intrahepatic • Crigler-Najjar syndrome
pathology • Dubin-Johnson/Rotors syndrome
• Hepatitis (alcohol, viral, sepsis, drugs)
Posthepatic
• Obstruction to bile flow
pathology
TOPNOTCH MEDICAL BOARD PREP SURGERY MAIN DIGITAL HANDOUT BY LOUBOMIR ANTONIO, MD Page 61 of 101
For inquiries visit www.topnotchboardprep.com.ph or https://www.facebook.com/topnotchmedicalboardprep/
This handout is only valid for the September 2021 PLE batch. This will be rendered obsolete for the next batch since we update our handouts regularly.
TOPNOTCH MEDICAL BOARD PREP SURGERY MAIN DIGITAL HANDOUT BY LOUBOMIR ANTONIO, MD
For inquiries visit www.topnotchboardprep.com.ph or https://www.facebook.com/topnotchmedicalboardprep/
This handout is only valid for the September 2021 PLE batch. This will be rendered obsolete for the next batch since we update our handouts regularly.
✔ GUIDE QUESTION o Progressive hyperbilirubinemia
A 45-year-old man with hepatitis C undergoes an uneventful percutaneous o Hepatic encephalopathy
liver biopsy. About 6-weeks later, he complains of RUQ pain, is clinically o Decreased synthetic function
jaundiced, with a hemoglobin of 9.2 mg/dL and is fecal occult blood positive. o Spontaneous bacterial peritonitis
Which diagnosis best explains this patient’s symptoms?
(A) Hepatocellular carcinoma
(B) Chronic hepatitis
(C) Colon carcinoma with liver metastasis
(D) Haemobilia
(E) Symptomatic cholelithiasis
The most common causes of haemobilia are iatrogenic, traumatogenic,
and neoplastic. This should be suspected in patients who underwent prior
liver biopsy who later develops abdominal pain and gastrointestinal
bleeding. The classic presentation of haemobilia is formally known as
Quincke's triad: jaundice, right upper quadrant abdominal pain,
and upper gastrointestinal hemorrhage, but the presentation of all three
together only occur in 22%–35% of cases. Malignancy (choices A and C)
are less likely considering the subacute course of this patient. Chronic
hepatitis (B) and symptomatic cholelithiasis (E) would not explain the
gastrointestinal bleeding.
Dr. Cocos

CIRRHOSIS
• Final sequelae of chronic liver injury characterized by the
presence of fibrous septa that leads to hepatic nodules
• Complications
o Portal HTN
o Ascites
o Malnutrition
o Hepatocellular CA

GENERAL FINDINGS DESCRIPTION ETIOLOGY


• Yellow discoloration of the skin, • Compromised hepatocyte excretory function
Jaundice
cornea, and mucous membrane • Occurs when serum bilirubin >2mg/dl
• Central arteriole with tiny radiating • Elevated estradiol
Spider angiomata
vessels, mainly on trunk and face • Decreased estradiol degradation in the liver
• Fibrosis
Nodular liver • Irregular hard surface on palpation
• Irregular regeneration
• Portal hypertension
Splenomegaly • Enlarged on palpation or ultrasound
• Splenic congestion
• Prominent veins radiating from • Portal hypertension
Caput medusa
umbilicus • Re-opening of umbilical vein that shunts blood from the portal vein
• Proteinaceous fluid in abdominal
Ascites cavity • Portal hypertension
• Clinically detected when >1.5L
Cruveilhier-Baumgarten • Shunts from portal vein to umbilical vein branches
• Epigastric vascular murmur
Syndrome • Can be present even without caput medusa
• Erythema sparing central portions of • ↑ estradiol
Palmar erythema
the palm • ↓ estradiol degradation in the liver
• Horizontal white bands and or
White nails • Hypoalbuminemia
proximal white nail plate
Hypertrophic
• Painful proliferative • Hypoxemia due to right→left shunting
osteoarthropathy/
osteoarthropathy of long bones • Portopulmonary hypertension
finger clubbing
• Fibrosis and contraction of the • Enhanced oxidative stress
Dupuytren contracture
palmar fascia • ↑hypoxanthine (alcohol exposure or diabetes)
Gynecomastia/ Loss of male • Benign proliferation of glandular • Enhanced conversion of androstenedione to estrone & estradiol
hair pattern male breast tissue • ↓estradiol degradation in the liver
• Mainly in alcoholic cirrhosis and
Hypogonadism • Direct toxic effect of iron or alcohol
hemochromatosis
• Asynchronous flapping motions of • Hepatic encephalopathy
Flapping Tremor/ Asterixis
dorsiflexed hand • Disinhibition of motor neurons
• Volatile dimethyl-sulfide, especially in portosystemic shunting and
Fetor hepaticus • Sweet, pungent smell
liver failure
Anorexia, fatigue weight • Occurs in greater than 50% of
• Catabolic metabolism by diseases liver, secondary to anorexia
loss, muscle wasting cirrhotics
Type II diabetes • Occurs in 15-30% of cirrhotics • Disturbed glucose utilization and/or decreased insulin removal of liver

ASSESSMENT OF HEPATIC RESERVE CLASS


A B C
CHILD-TURCOTTE – PUGH SCORE
Serum bilirubin
• Predicts the surgical risks of intra-abdominal operations <2 2-3 >3
(mg/dL)
performed on patients with cirrhosis Serum albumin
>3.5 2.8-3.5 <2.8
(g/dL)
CLASS Prothrombin time
A B C >70 40-70 <40
(% of control)
Nutritional status Excellent Good Poor
MORTALITY
Minimal, Moderate to
Ascites None Class A = 5-6 points 10%
controlled severe
Minimal, Moderate to Class B = 7-9 points 30%
Encephalopathy None Class C = 10-15 points 80%
controlled severe
TOPNOTCH MEDICAL BOARD PREP SURGERY MAIN DIGITAL HANDOUT BY LOUBOMIR ANTONIO, MD Page 62 of 101
For inquiries visit www.topnotchboardprep.com.ph or https://www.facebook.com/topnotchmedicalboardprep/
This handout is only valid for the September 2021 PLE batch. This will be rendered obsolete for the next batch since we update our handouts regularly.
TOPNOTCH MEDICAL BOARD PREP SURGERY MAIN DIGITAL HANDOUT BY LOUBOMIR ANTONIO, MD
For inquiries visit www.topnotchboardprep.com.ph or https://www.facebook.com/topnotchmedicalboardprep/
This handout is only valid for the September 2021 PLE batch. This will be rendered obsolete for the next batch since we update our handouts regularly.
SPONTANEOUS BACTERIAL PERITONITIS This figure illustrates the intra-abdominal venous flow pathways
leading to engorged veins (varices) from portal hypertension.
• Diagnosis: Dr. Cocos
o Neutrophil count > 250/mL
CAUSATIVE FACTORS OF PORTAL HYPERTENSION
o Culture negative (60%) Extrahepatic
o Culture positive o Portal vein thrombosis
• Treatment: empiric antibiotics o Splenic vein thrombosis
o Cefotaxime – 1st line medication Presinusoidal Intrahepatic
o Amoxicillin/Clavulanic, Quinolones obstruction o Congenital hepatic fibrosis
• Ascites: management o Primary biliary cirrhosis
o Na restriction o Sarcoidosis
o Diuretics (Spironolactone) o Schistosomiasis
o Serial paracentesis CAUSATIVE FACTORS OF PORTAL HYPERTENSION
• Cirrhosis: Laboratory Findings Sinusoidal • Steatohepatitis
o Mild normocytic normochromic anemia obstruction • Wilson disease
o Reduced WBC and platelet counts Extrahepatic
o Prolonged prothrombin time (PT) o Budd-Chiari syndrome
o Decreased serum albumin o Right heart failure
Post-
Intrahepatic
sinusoidal
o Hemochromatosis
PORTAL HYPERTENSION obstruction
o Laennec (alcoholic) cirrhosis
• Normal portal pressure = 5-10mmHg o Secondary biliary cirrhosis
• Portal HTN o Posthepatitic cirrhosis
o Direct portal venous pressure >5mmHg of IVC pressure High-flow • Arteriovenous fistula
o Splenic pressure > 15mmHg states • Banti’s syndrome (Massive splenomegaly)
• Pressure > 12mmHg is necessary for varices to form and bleed ESOPHAGEAL VARICES
• Hepatic venography: most accurate method of determining • Most significant manifestation of portal HTN
portal HTN • Leading cause of morbidity and mortality
• HVPG (sinusoidal pressure) = FHVP (free hepatic venous • Natural History
pressure) minus WHVP (wedge hepatic venous pressure) o 1/3 with varices will bleed
• Etiology o Each bleed = 20 to 30% mortality
o Pre-sinusoidal o 70% of survivors (1st bleed) will re-bleed within 1 year
o Sinusoidal
o Post- sinusoidal PREVENTION OF VARICEAL BLEEDING
• Presentation • Improvement of liver function
o Gastroesophageal varices o Abstention from alcohol
o Splenomegaly o Avoidance of hepatotoxic medications
o Re-canalization of the umbilical vein • Administration of beta–blockers
o Ascites o Reduces index bleed by 45%
o Ano-rectal varices o Reduces mortality by 50%
• Prophylactic endoscopic variceal ligation (EVL)
MANAGEMENT OF ACUTE VARICEAL BLEEDING
• Pharmacologic therapy
o Vasopressin
o Octreotide ( current preferred agent )
• Early endoscopy and variceal ligation or sclerotherapy
• 80% stop bleeding
• Refractory bleeding
o Surgical shunt: Child A and B
o TIPS: Child C

This diagram shows the histologic changes in cirrhosis.


Dr. Cocos
PORTO-SYSTEMIC COLLATERALS
VARICEAL
BLEEDING
https://qrs.ly/xpcl129

TRANSJUGULAR INTRAHEPATIC PORTOSYSTEMIC SHUNT (TIPS)

TOPNOTCH MEDICAL BOARD PREP SURGERY MAIN DIGITAL HANDOUT BY LOUBOMIR ANTONIO, MD Page 63 of 101
For inquiries visit www.topnotchboardprep.com.ph or https://www.facebook.com/topnotchmedicalboardprep/
This handout is only valid for the September 2021 PLE batch. This will be rendered obsolete for the next batch since we update our handouts regularly.
TOPNOTCH MEDICAL BOARD PREP SURGERY MAIN DIGITAL HANDOUT BY LOUBOMIR ANTONIO, MD
For inquiries visit www.topnotchboardprep.com.ph or https://www.facebook.com/topnotchmedicalboardprep/
This handout is only valid for the September 2021 PLE batch. This will be rendered obsolete for the next batch since we update our handouts regularly.
CLASSIFICATION OF PORTOSYSTEMIC SURGICAL SHUNTS Bleeding varices
NONSELECTIVE
• End-to-side portacaval shunt (Eck fistula) Resuscitation and
• Side-to-side portacaval shunt medical measures
• Large diameter interposition shunts (e.g., mesocaval, Drapanas)
• Central splenorenal shunt
EGD
• Proximal Splenorenal shunt (Linton) Sclerotherapy-banding
SELECTIVE
Bleeding controlled Bleeding not controlled
• Distal splenorenal (Warren)
• Left gastric vena caval shunt (Inokuchi)
• Partial Repeat Balloon
• Small-diameter portacaval H graft shunt (Sarfeh) EGD tamponade

Bleeding stops Recurrent bleeding

Child’s C
Assess TIPS
Child’s A or B

Surgical shunt

Stable Decompensation

Observe OLT

BUDD-CHIARI SYNDROME
• Congestive hepatopathy characterized by obstruction to hepatic
venous outflow
• Primary: endoluminal hepatic venous thromboses
• Secondary: veins compressed / invaded by a neighboring lesion
o Affects relatively young women with myelofibrosis
o Factor V Leiden mutation
o Anti-cardiolipin antibodies
o Oral contraceptives
o Hyperhomocystinemia
NON-SHUNT SURGICAL PROCEDURES • Diagnosis
PROCEDURE REMARKS o Abdominal US: initial investigation
• Involves: o Hepatic venography: definitive study
o Ligation of venous branches entering the distal • Treatment: systemic anti coagulation
esophagus and the proximal stomach from the
level of the inferior pulmonary vein ✔ GUIDE QUESTION
Sugiura- A 43-year-old man with chronic hepatitis and liver cirrhosis is admitted
o Transection of esophagus, splenectomy,
Fukugawa with upper GI bleeding. He has marked ascites and shows multiple
truncal vagotomy, & pyloroplasty
procedure telangiectasias, liver palmar erythema, and clubbing. A diagnosis of
• Reserved for recurrent variceal bleeding
bleeding esophageal varices secondary to portal hypertension is made.
despite endoscopic & medical treatment
Portal pressure is considered elevated when it is above which of the ff?
who are not candidates for TIPS & have
(A) 0.15 mm Hg (C) 40 mm Hg
vasculature unsuitable for shunt operations (B) 1.5 mm Hg (D) 105 mm Hg
• Consists of: (C) 12 mm Hg
o Splenectomy
o Perihiatal devascularization of the lower Normal portal venous pressure is at 5-10 mmHg.
Dr. Cocos
esophagus
Hassab PYOGENIC LIVER ABSCESS
o Litigation of the left gastric vessels
procedure • More frequent on the right lobe
o Devascularization of the proximal half of
the stomach o 40% monomicrobial
o Separation of stomach from its bed o 40% polymicrobial
through the abdominal approach o 20% culture negative
• Most definitive form of therapy for • Gram negative organisms: most common (E coli in 2/3)
Orthotopic complications of portrayal hypertension • Presentation
liver • Limited application because of scarcity of o RUQ pain and fever
transplantation donors and the need for life-long o Jaundice in 1/3 of patients
immunosuppression after transplant o Elevated WBC, ESR, alkaline phosphatase
Surgery Platinum. 1st ed. 2018. p 351
o US: (+) hypoechoic lesions with well-defined borders and
variable internal echoes
o CT: (+) hypodense lesions with or without air-fluid levels and
peripheral enhancement
• Treatment

TOPNOTCH MEDICAL BOARD PREP SURGERY MAIN DIGITAL HANDOUT BY LOUBOMIR ANTONIO, MD Page 64 of 101
For inquiries visit www.topnotchboardprep.com.ph or https://www.facebook.com/topnotchmedicalboardprep/
This handout is only valid for the September 2021 PLE batch. This will be rendered obsolete for the next batch since we update our handouts regularly.
TOPNOTCH MEDICAL BOARD PREP SURGERY MAIN DIGITAL HANDOUT BY LOUBOMIR ANTONIO, MD
For inquiries visit www.topnotchboardprep.com.ph or https://www.facebook.com/topnotchmedicalboardprep/
This handout is only valid for the September 2021 PLE batch. This will be rendered obsolete for the next batch since we update our handouts regularly.
o Correction of underlying cause • Presentation
o Broad spectrum antibiotics at least 8 weeks o RUQ pain, fever and hepatomegaly
o Percutaneous aspiration (80% respond) o Leukocytosis is common
o Failure of medical management → surgical drainage or resection o Elevated transaminase and jaundice are unusual
o Mildly elevated alkaline phosphatase – most common
AMEBIC LIVER ABSCESS o (+) fluorescent antibody test
• Entamoeba histolytica o US and CT findings similar to pyogenic liver abscess
o Infective stage: cyst • Treatment
o Pathogenic stage: trophozoite o Metronidazole 750mg TID for 7-10 days
• Usually located at superior – anterior aspect of right lobe o Indication for aspiration
• Necrotic central portion containing reddish brown pus-like § Large abscesses
material (anchovy paste) § Failure of medical management
• Most common form of liver abscess worldwide § Superinfection
§ Abscesses of the left lobe (could perforate into the
pleuropericardial space)
HYDATID DISEASE

• Echinococcus granulosus
• MOT: ingestion of ova containing feces (dog)
• Ova at intestinal mucosa
• Adult cyst in the liver

• Presentation
o commonly affects the antero-inferior or posteroinferior
portions of the right lobe
o dull RUQ pain or abdominal distention
o allergic or anaphylactic reaction with cyst rupture
• Diagnosis
o ELISA for echinococcal antigens
o US/CT
§ Well defined hypodense lesions
§ Ring-like calcification
• Treatment
o Surgical based
§ Complete cyst removal with instillation of scolicidal
agent (preferred)
§ Formal anatomic liver resection
o Small asymptomatic cysts: Albendazole
Echinococcosis. http://www.cdc.gov

✔ GUIDE QUESTION
A 45-year-old male is suspected of having an amebic abscess of the liver.
Serum bilirubin is mildly elevated. The WBC is 11,000 but there is
eosinophilia. The initial line of treatment involves which of the
following?
(A) Cortisone
(B) Metronidazole (Flagyl)
(C) Surgical excision
(D) Sulfonamides and penicillin
(E) Colon resection
Although eosinophilia (parasitism) is not a feature of amoebic liver
abscess, if there is suspicion that this is an amebic abscess then
metronidazole is the initial treatment of choice. Amebic liver abscess
almost always responds to treatment with metronidazole (Flagyl).
Occasionally, percutaneous aspiration is required when there is no Adapted from Schwartz’s Principles of Surgery. 11th ed. 2019

response to Flagyl or if the abscess is secondarily infected.


HEPATIC CYSTS
Sulfonamides, penicillin and cortisone are not given to amebic
abscesses. There is no need to excise the amebic abscess or do colonic • Most frequently encountered liver lesion overall
resection. o Primary (congenital)
Dr. Cocos o Secondary (trauma, infection, neoplasm)
EVALUATION OF INCIDENTAL LIVER MASS • Treatment
o Asymptomatic: observation
o Symptomatic: percutaneous aspiration and sclerotherapy
HEMANGIOMA
• Most common benign solid mass in the liver
• More common in women
• Pain: most common symptom and indication for resection
• Spontaneous bleeding may occur but is rare
• Diagnosis:
o Dual-phase CT: asymmetrical peripheral enhancement with
progressive centripetal enhancement
o MRI: hypointense on TI and hyperintense in T2
• Treatment: enucleation or resection

TOPNOTCH MEDICAL BOARD PREP SURGERY MAIN DIGITAL HANDOUT BY LOUBOMIR ANTONIO, MD Page 65 of 101
For inquiries visit www.topnotchboardprep.com.ph or https://www.facebook.com/topnotchmedicalboardprep/
This handout is only valid for the September 2021 PLE batch. This will be rendered obsolete for the next batch since we update our handouts regularly.
TOPNOTCH MEDICAL BOARD PREP SURGERY MAIN DIGITAL HANDOUT BY LOUBOMIR ANTONIO, MD
For inquiries visit www.topnotchboardprep.com.ph or https://www.facebook.com/topnotchmedicalboardprep/
This handout is only valid for the September 2021 PLE batch. This will be rendered obsolete for the next batch since we update our handouts regularly.
ADENOMA A 35-year-old woman complains of RUQ pain after meals with nausea
and vomiting. An ultrasound reveals cholelithiasis and an anechoic 3-
• Benign solid neoplasm cm mass on the inferior surface of the right lobe of the liver. Select the
• Occurs in young women, typically solitary most likely diagnosis.
• Prior or current use of oral contraceptives - clearest risk factor (A) Congenital cyst
• Complications: spontaneous rupture and hemorrhage, (B) Hydatid cyst
malignant degeneration (C) Hamartoma
• Diagnosis: CT/MRI/nuclear scan (D) Adenoma
(E) Focal nodular hyperplasia
• Treatment: resection
Congenital cysts are usually asymptomatic, appear anechoic on
FOCAL NODULAR HYPERPLASIA ultrasound and are the most common liver lesions. They can be multiple or
single and usually contain clear, watery fluid. These cysts are more
• Occurs in young women
common in the right lobe. Hydatid cysts, caused by Echinococcus, are also
• Link to oral contraceptives is not as clear as adenoma more common in the right lobe. The colorless fluid in the cyst is under high
• Usually: no spontaneous rupture, no malignant degeneration pressure, unlike parasitic cysts. Ultrasound will show internal echoes.
• Abdominal pain: usual indication for surgery Hemangiomas can have a variable echogenic pattern on ultrasound; focal
• Diagnosis nodular hyperplasia and adenoma is often hypodense. A biliary
o CT/MRI: central scar hamartoma has heterogenous appearance on imaging, not anechoic.
Dr. Cocos
o Nuclear scan: hot A 32-year-old diabetic woman who has taken contraceptive pills for 12
• Treatment years develops RUQ pain. CT scan of the abdomen reveals a 5-cm
o Asymptomatic: observe hypodense lesion in the right lobe of the liver consistent with a hepatic
o Symptomatic: resection adenoma. What should the patient be advised to do?
(A) Undergo segmentectomy
MALIGNANT LIVER TUMORS (B) Stop oral contraceptives only
Primary tumor of (C) Stop oral hypoglycemic medication
• Hepatocellular carcinoma (D) Undergo right hepatectomy
hepatocytes
Primary tumor of bile (E) Have serial CT scans every 6 months
• Cholangiocarcinoma A diagnosis of hepatic adenoma warrants surgical intervention (even
ducts
when asymptomatic) since it can degenerate into a malignant lesion and
• Most common malignant liver tumor
Liver metastasis has a risk for rupture, unlike FNH wherein surgical intervention is
• Usually from colorectal carcinoma
indicated only if symptomatic. Therefore, serial Ct scans is not an option.
HEPATOCELLULAR CARCINOMA However, doing a formal right hepatectomy for a 5cm benign lesion is also
not correct. Oral contraceptives (not oral hypoglycemic) should be
• Risk Factors
stopped, but as stated prior, resection should be done as well.
Cirrhosis Hemochromatosis Dr. Cocos
Hepatitis B or C Non-alcoholic steatohepatitis A 35-year-old woman is seen in the office with focal nodular hyperplasia.
Alcoholic This condition is similar to hepatic adenoma, in that it does what?
• Typically hypervascular, derived from hepatic artery (A) Frequently causes symptoms
• Diagnosis - CT/MRI, AFP (B) Tends to lead to liver rupture
(C) LFT and alpha fetoprotein (AFP) are normal
• Treatment (D) Easily detected by CT scan of the liver
o Resection – o Indications for transplantation (E) Tends to undergo malignant changes
treatment of choice § One nodule < 5cms Unlike hepatic adenomas, FNH do not usually cause symptoms. Unlike
§ Non cirrhotic § 2 or 3 nodules <3 cms hepatic adenomas, it does not tend to cause intramural bleeding with
§ Child A § (-) vascular invasion rupture into the peritoneal cavity. CT or US scan may frequently miss
§ Single lesion § (-) extrahepatic spread the lesion, because it is so dense. There is no definite relationship with
§ No metastasis § Child A, B, C oral contraceptives. Focal nodular hyperplasia lesions are not well
Standard Hepatic Resections encapsulated and have a central stellate scar. Malignant changes have
not been reported. LFT and AFP are normal in both conditions
Dr. Cocos
A 64-year-old man has mild upper abdominal pain. On contrast CT scan,
a 5-cm lesion in the left lobe of the liver enhances and then decreases
over a 10-minute period from without to within. The most likely lesion
is which of the following?
(A) Congenital cyst
(B) Hemangioma
(C) Fungal abscess
(D) Focal nodular hyperplasia
(E) Hepatic adenoma
The CT scan description points to a hemangioma with classic
centripetal enhancement followed by decrease in dye over 10 minutes
from without to within. Hemangiomas occur more frequently in
women. Most lesions are asymptomatic, discovered incidentally, and
require no treatment. Larger hemangiomas may cause pain because
of stretching of liver capsule or thrombocytopenia due to platelet
trapping. These tumors may occasionally require resection.
Dr. Cocos
A healthy 64-year-old woman had a cancer of the left colon resected 4
Note: To memorize the Couinaud segments easier, number the segments
years previously. During follow-up, an increased carcinoembryonic
from 2 to 8 in a clockwise manner. And remember that the right lobe has
antigen (CEA) level lead to a CT scan of the abdomen, which revealed
anterior (8 & 5) and posterior (7 & 6) segments, while the left lobe has
two discrete lesions in the left lateral lobe of the liver. Liver biopsy
lateral (2 & 3) and medial (4a & 4b) segments.
Dr. Cocos confirms that this is metastatic colon cancer. What is the most
appropriate plan?
✔ GUIDE QUESTIONS (A) Inform the patient that there is no treatment, and that her
A 40-year-old female alcoholic is suspected of having a hepatic mass. expectation of life is limited.
Percutaneous ultrasound guided liver biopsy is contraindicated in which of the ff? (B) Irradiation is recommended.
(A) Hepatocellular carcinoma. (C) Cirrhosis (C) Local cauterization of the cancer is recommended.
(B) Metastatic carcinoma (D) Hepatitis C (D) Liver resection is recommended.
(E) Hepatic adenoma (E) Chemotherapy is recommended.
Contraindications to liver biopsy is increased risk for bleeding, massive
ascites, etc. Tru-cut liver biopsy allows pathologic diagnosis of liver Metastasectomy in stage 4 colorectal cancer has been shown to
lesions. Needle biopsy is contraindicated if hemangioma is suspected increase survival, hence a complete R0 resection of all the metastatic
and in adenomas, because there is high risk of bleeding. Other lesions is recommended. Irradiation and local cauterization is not
potential complications of percutaneous needle biopsy are pain, recommended. Chemotherapy may be recommended but metastatic
pneumothorax, and bile peritonitis. Needle biopsy can diagnose resection should be done prior.
Dr. Cocos
posthepatic and postnecrotic cirrhosis, malignant tumors, and
hepatitis, and can determine the need for treatment in hepatitis C.
Dr. Cocos

TOPNOTCH MEDICAL BOARD PREP SURGERY MAIN DIGITAL HANDOUT BY LOUBOMIR ANTONIO, MD Page 66 of 101
For inquiries visit www.topnotchboardprep.com.ph or https://www.facebook.com/topnotchmedicalboardprep/
This handout is only valid for the September 2021 PLE batch. This will be rendered obsolete for the next batch since we update our handouts regularly.
TOPNOTCH MEDICAL BOARD PREP SURGERY MAIN DIGITAL HANDOUT BY LOUBOMIR ANTONIO, MD
For inquiries visit www.topnotchboardprep.com.ph or https://www.facebook.com/topnotchmedicalboardprep/
This handout is only valid for the September 2021 PLE batch. This will be rendered obsolete for the next batch since we update our handouts regularly.
CHOLANGIOCARCINOMA ANATOMY OF THE BILE DUCT
• Intrahepatic – presents as a hepatic mass REMARKS
• Extrahepatic – can be proximal (hilar) or distal Dimensions of the • Length: 1-4 cm
• Klatskin tumor – occurs at hepatic duct confluence; proximal hepatic duct • Diameter: 4 mm
• Treatment: resection (gold standard) Dimensions of the • Length: 7-11 cm
common bile duct • Diameter: 5-10 mm (<6 mm in Asians)
RADICAL BILE DUCT RESECTION • Gastroduodenal artery
Blood supply
• Right hepatic artery

• Terminal CBD usually


joins pancreatic duct
• Opens into the Ampulla
of Vater 10cm distal to
pylorus surrounded by
the sphincter of Oddi

Figure 32-3. Schwartz’s Principles of Surgery. 11th ed. 2019

ANOMALIES
• Classic anatomy applies only to 1/3 of population
• Gallbladder: intrahepatic gallbladder
• Cystic artery: originates from SMA
• Cystic duct: runs long and parallel with the CBD
ROUX-EN-Y HEPATICOJEJUNOSTOMY
• Accessory ducts: ducts of Luschka
GALLBLADDER
ANATOMY OF THE GALLBLADDER
REMARKS
• 7-10 cm long
Dimensions
• 30-50 mL capacity
Location • Inferior surface of the liver
Parts • Fundus, corpus, infundibulum, neck
• Lacks muscularis mucosa and
Histologic feature
submucosa
• Cystic artery ← right hepatic artery
Blood supply
• Found inside the triangle of Calot
• Drains to the portal vein
Venous drainage
• Some direct to the liver
Innervation • Autonomic nervous system

ENTERO-HEPATIC CIRCULATION
• Formed by the:
o Cystic duct to the right The primary bile salts (cholate and chenodeoxycholate) are synthesized
in the liver from cholesterol. They are conjugated there with taurine and
o Common hepatic duct to the left
glycine. Bile salts are excreted into the bile by the hepatocyte and aid in
o Margin of the right lobe of the liver superiorly
Hepatocystic the digestion and absorption of fats in the intestines. In the intestines,
• Confused with what is presently known as the about 80% of the conjugated bile acids are absorbed in the terminal
triangle of
triangle of Calot ileum. The remainder is dehydroxylated (deconjugated) by gut bacteria,
Budde
• In most individuals (96%), the cystic artery is forming secondary bile acids deoxycholate and lithocholate. These are
found in the hepatocystic triangle, which is used absorbed in the colon, transported to the liver, conjugated, and secreted
to locate it during laparoscopic into the bile. Eventually, about 95% of the bile acid pool is reabsorbed
cholecystectomy and returned via the portal venous system to the liver, the so-called
• The triangle originally described by Calot defined enterohepatic circulation. Five percent is excreted in the stool.
Dr. Cocos
the superior boundary as the cystic artery
NEUROHORMONAL REGULATION
• Borders:
o Cystic duct Vagal stimulation • Gallbladder contraction
Triangle of • Gallbladder contraction
o Common hepatic duct CCK
Calot • Relaxation of sphincter of Oddi
o Inferior surface of the liver
• Calot node: Lymph node located within the VIP • Inhibits gallbladder contraction
triangle & is frequently enlarged during Somatostatin • Inhibits gallbladder contraction
cholecystitis or cholangitis
SPHINCTER OF ODDI
• Circular area, 30 mm in diameter that fits into
the hepatocystic duct angle • Regulates flow of bile and pancreatic juice into the duodenum
Moosman • About 85% of all variations in the hepatic • Prevents regurgitation of duodenal contents into the biliary tree
Area pedicle are found in Moosman area, and 50% of • Diverts bile into the GB
these variations are all potential hard during • Basal Pressure = 13 mmHg above duodenal pressure
cholecystectomy
Surgery Platinum. 1st ed. 2018. p. 365
TOPNOTCH MEDICAL BOARD PREP SURGERY MAIN DIGITAL HANDOUT BY LOUBOMIR ANTONIO, MD Page 67 of 101
For inquiries visit www.topnotchboardprep.com.ph or https://www.facebook.com/topnotchmedicalboardprep/
This handout is only valid for the September 2021 PLE batch. This will be rendered obsolete for the next batch since we update our handouts regularly.
TOPNOTCH MEDICAL BOARD PREP SURGERY MAIN DIGITAL HANDOUT BY LOUBOMIR ANTONIO, MD
For inquiries visit www.topnotchboardprep.com.ph or https://www.facebook.com/topnotchmedicalboardprep/
This handout is only valid for the September 2021 PLE batch. This will be rendered obsolete for the next batch since we update our handouts regularly.
✔ GUIDE QUESTION
In attempting to minimize complications during cholecystectomy, the
surgeon defines the triangle of Calot during the operation. The
boundaries of the triangle of Calot (modified) are the common hepatic
duct medially, the cystic duct inferiorly, and the liver superiorly. Which
structure courses through this triangle?
(A) Left hepatic artery
(B) Right renal vein
(C) Right hepatic artery
(D) Cystic artery
(E) Superior mesenteric vein
The cystic artery is usually found within the hepatocystic triangle
which is bordered by the cystic duct, common hepatic duct and inferior
edge of the liver.
Dr. Cocos

DIAGNOSTIC STUDIES
• Initial investigation
Ultrasound • >90% specificity and sensitivity in detection of
gallstones CHRONIC CHOLECYSTITIS (BILIARY COLIC)
• Acoustically dense
Stones in • Most common presentation of gallstone disease
• Produces posterior sonic shadow
ultrasound characterized by recurrent biliary colic
• Moves with changes in position
• Symptoms are due to partial obstruction of the cystic duct
Acute • Stones can be present
cholecystitis • Gallbladder thickening
• Biliary colic
in • Pericholecystic fluid o Lasts 1 to 5 hours
ultrasound • Sonographic Murphy sign o Epigastric in location or RUQ
• Oral cholecystography
o Radiates to the back/shoulders
• HIDA scan
o Episodic, subsides spontaneously
• CT scan • Diagnosis: History, PE, and abdominal US
Other • Treatment: symptomatic → elective lap cholecystectomy
• Percutaneous Transhepatic Cholangiography (PTC)
diagnostic
• Endoscopic Retrograde PROPHYLACTIC VERSUS INCIDENTAL CHOLECYSTECTOMY
tools
Cholangiopancreatography (ERCP) • Removal of the gallbladder to prevent future
Prophylactic
• Endoscopic US (EUS) complications of gallstones in an asymptomatic
cholecystectomy
• MRCP patient
• Removal of the gallbladder during an operation
GALLSTONE DISEASE Incidental
for an otherwise unrelated diagnosis (e.g.,
cholecystectomy
Risk Factors gallbladder polyp, liver metastasectomy)
Surgery Platinum. 1st ed. 2018. p. 369
Female gender Gastric and terminal ileal surgery
TRADITIONAL INDICATIONS FOR CHOLECYSTECTOMY
Obesity (Vagotomy/Ileal resection)
(ASYMPTOMATIC)
Pregnancy Hemolytic disorders
Crohn disease Biliary stasis (prolonged fasting, TPN • Elderly with diabetes
use, Spinal Cord Injury) • Isolation from medical care for extended periods
• Increased risk of GB cancer (porcelain gallbladder?)
Natural History
RECOMMENDATIONS FOR PROPHYLACTIC CHOLECYSTECTOMY
• Most remain asymptomatic
Prophylactic Cholecystectomy Indicated
• 3% of asymptomatic individuals will become symptomatic per year
• Hemoglobinopathies (sickle cell disease)
• 3 to 5% of symptomatic individuals will develop a complication • Hereditary spherocytosis and thalassemia at the time of splenectomy
of gallstone disease • Transplant recipients (cardiac and lung)
GALLSTONE FORMATION Prophylactic Cholecystectomy Not Indicated
• Diabetic patients
• Cirrhotic patients
• Transplant recipients (kidney and pancreas)
• Result of solid • Porcelain gallbladder (incidence rate of gallbladder cancer is almost 0)
settling out of • Patients receiving prolonged TPN
solution • Spinal cord injury
• Major organic Prophylactic Cholecystectomy Remains Controversial
solutes – • Morbid obesity
cholesterol, bile • After bariatric surgery
Surgery Platinum. 1st ed. 2018. p. 369
salts,
CRITICAL VIEW OF SAFETY IN LAPAROSCOPIC CHOLECYSTECTOMY
phospholipids
1) the triangle of Calot must be dissected free of fat (without
exposing the common bile duct)
2) the base of the gallbladder (at least 1/3) must be dissected off
The area labeled “1 phase” shows the range of concentrations found consistent
with a clear micellar solution (single phase), where cholesterol is fully
the liver bed (or cystic plate)
solubilized. The green area directly above this region corresponds to a 3) two structures (and only two, the cystic duct and artery) enter
metastable zone, supersaturated with cholesterol. Bile with a composition the gallbladder and these can be seen circumferentially (360-
that falls above the green area has exceeded the solubilization capacity of degree view)
cholesterol and precipitation of cholesterol crystals occurs, hence gallstones. ABSOLUTE CONTRADICTIONS
Dr. Cocos
TYPES OF GALLSTONES • Inability to tolerate general anesthesia or laparotomy
• Refractory coagulopathy
• Result from cholesterol supersaturation
Cholesterol • Diffuse peritonitis with hemodynamic compromise
• Most are radiolucent (>90%) • Cholangitis
stones
• Pure cholesterol stones are uncommon • Potentially curable gallbladder cancer
• Small, brittle, speculated RELATIVE CONTRADICTIONS
Black
• Supersaturation of calcium bilirubinate, • Previous upper abdominal with extensive adhesions
pigment
carbonate, phosphate • Cirrhosis
stones
• Associated with hemolytic disorder and cirrhosis • Portal hypertension
Brown • Soft and mushy • Severe cardiopulmonary disease
pigment • Form either in gallbladder or bile ducts • Morbid obesity
stones • Associated with bacterial infections of bile stasis • Pregnancy
Surgery Platinum. 1st ed. 2018. p. 369
Pathophysiology of calculous cholecystitis
TOPNOTCH MEDICAL BOARD PREP SURGERY MAIN DIGITAL HANDOUT BY LOUBOMIR ANTONIO, MD Page 68 of 101
For inquiries visit www.topnotchboardprep.com.ph or https://www.facebook.com/topnotchmedicalboardprep/
This handout is only valid for the September 2021 PLE batch. This will be rendered obsolete for the next batch since we update our handouts regularly.
TOPNOTCH MEDICAL BOARD PREP SURGERY MAIN DIGITAL HANDOUT BY LOUBOMIR ANTONIO, MD
For inquiries visit www.topnotchboardprep.com.ph or https://www.facebook.com/topnotchmedicalboardprep/
This handout is only valid for the September 2021 PLE batch. This will be rendered obsolete for the next batch since we update our handouts regularly.

ACUTE CHOLECYSTITIS DIAGNOSTIC CRITERIA FOR ACUTE CHOLECYSTITIS BY TG18:


A. Local Signs of inflammation etc.:
• 90-95% due to gallstones (1) Murphy’s sign, (2) RUQ mass / pain tenderness
• Acalculous cholecystitis B. Systemic signs of inflammation etc.:
o Critically ill (1) Fever, (2) elevated CRP, (3) elevated WBC count
o Biliary stasis C. Imaging findings:
• Obstruction of the cystic duct is the initiating event Imaging findings characteristic of acute cholecystitis
• Leads to GB distention, inflammation, and edema Suspected diagnosis:
One item in A + one item in B
PRESENTATION Definite diagnosis:
• Symptoms One item in A + one item in B + C
Surgery Platinum. 1st ed. 2018. p. 370
o Main symptom of uncomplicated cholelithiasis is biliary colic
IMAGING
caused by the obstruction of the gallbladder for several days
• Ultrasound
o RUQ pain unremitting, persists for several days
o Diagnostic test of choice for acute cholecystitis
o Fever, anorexia, nausea, vomiting
o Findings:
o Complications: abscess, perforation, choledocholithiasis,
§ Enlarged gallbladder
gallstone ileus
§ Thickening of the gallbladder wall
• Signs
§ Gallbladder stoned Debris echo
o Murphy sign: examiner hooks fingers under right costal
§ Direct tenderness when probe is pushed against the
margin and asks the patient to deeply inhale – positive test is
gallbladder (ultrasonographic Murphy sign)
noted if the patient stops inhaling suddenly due to pain
o In gangrenous cholecystitis and emphysematous cholecystitis:
o Boas sign: hyperesthesia in the RUQ or right infrascapular region
irregular thickening of gallbladder wall and imaging of the
o Collins sign: present when the patient points to the right
ruptured gallbladder
scapular tip with a fist and thumb pointing upwards to
• Hepatobiliary scintigraphy (Tc-HIDA scan)
describe the pain
Surgery Platinum. 1st ed. 2018. p. 370 o Involved IV injection of technetium-labeled analogues of
DIAGNOSIS iminodiacetic acid, which are excreted in the bile
• Diagnosis of acute cholecystitis is made as follows according o Failure of gallbladder to fill within 60 minutes after
to the 2018 Tokyo Guidelines administration of tracer indicates that the cystic duct is
• When acute cholecystitis is suspected from clinical signs and obstructed
results of blood tests, a definite diagnosis is made after it has o Rim sign: blush of increased pericholecystic radioactivity in
been confirmed by diagnostic imaging the cholecystitis

SEVERITY GRADING FOR ACUTE CHOLECYSTITIS


GRADE SEVERITY REMARKS
• Associated with dysfunction of any one of the following organs or systems:
o Cardiovascular: hypotension requiring treatment with dopamine >µg/kg/min or any dose of norepinephrine
o Neurological: decreased level of consciousness
Severe acute
III o Respiratory: PaO2/Fio2 ratio <300
cholecystitis
o Renal oliguria, creatinine > 2.0 mg/dL
o Hepatic: PT-INR >1.5
o Hematological: platelet count<100,000/mm3
• Cholecystitis without organ dysfunction but with risk
• Cholecystectomy and biliary drainage should be carried out immediately
• Associated with any one of the following conditions:
Moderate
o Elevated WBC (>18,000/mm3)
II acute
o Palpable tender mass in RUQ
cholecystitis
o Duration of >72 hours
o Marked local inflammation (gangrenous or emphysematous cholecystitis, pericholecystic abscess, hepatic
abscess, biliary peritonitis)
Mild acute • Does not meet the criteria of “Grade III” or “Grade II”
I
cholecystitis • Acute cholecystitis in a healthy patient with no organ dysfunction and mild inflammatory changes in the gallbladder
Surgery Platinum. 1st ed. 2018. p. 371
MANAGEMENT ✔ GUIDE QUESTIONS
• NPO, IV fluids A 48-year-old woman is admitted with acute cholecystitis. The
• IV antibiotics for severe acute cholecystitis: guided bilirubin level is elevated, as are the serum and urinary amylase levels.
by the most common organisms likely to be present Which radiologic sign indicates biliary obstruction in pancreatitis?
Non-
(E. coli, Klebsiella spp., and Streptococcus spp.) (A) Pancreatic intraductal calcification
surgical
• Meperidine or NSAIDs: usually employed for (B) Smooth narrowing of the distal CBD
analgesia because they may produce less spasm of (C) Stomach displaced anteriorly
the Sphincter of Oddi than drugs such as Morphine (D) Calcified gallstone
• Definitive management: early cholecystectomy (2-3 days) (E) Air in the biliary tree
• If patient unfit for surgery: cholecystostomy plus Gallstone pancreatitis is presumed if a patient presents with signs and
elective lap cholecystectomy symptoms of pancreatitis with a concomitant gallstone on ultrasound.
• Management based on grade of cholecystitis The passage of small stones through Vater’s ampulla often results in
Grade I • Early laparoscopic cholecystectomy pancreatitis. It is important to perform cholecystectomy after
(Mild) (within 72 hours) is preferred pancreatitis has subsided but during the same hospital stay in patients
with documented gallstone pancreatitis (to avoid recurrence of
• Early cholecystectomy (laparoscopic
symptoms). Smooth tapering of the common duct is usually seen with
Surgical Grade II or open) is recommended; early stones obstructing the common duct. Pancreatic intraductal
(Moderate) gallbladder drainage (percutaneous or calcification is consistent with chronic pancreatitis, and air in the
surgical) if with severe local inflammation biliary tree is consistent with gallstone ileus.
• Urgent management of organ Dr. Cocos

dysfunction and gallbladder drainage A 38-year-old male lawyer develops abdominal pain after having a
Grade III fatty meal. Examination reveals tenderness in the right
(delayed elective cholecystectomy 2-3
(Severe) hypochondrium and a positive Murphy sign. Which test is most likely
months later when general conditions
are improved) to reveal acute cholecystitis?
Surgery Platinum. 1st ed. 2018. p. 371 (A) HIDA scan (D) CT scan of the abdomen
Note: for high-risk patients, we can just do early gallbladder drainage i.e. (B) Oral cholecystogram (E) ERCP
an ultrasound-guided or open tube cholecystostomy (insert a tube to the (C) Intravenous cholangiogram
gallbladder and drain bile). This is the quickest possible procedure with Initial best test would be ultrasound. A HIDA scan (rarely used) is an
least morbidity. imaging test used to examine the gallbladder and the ducts leading
Dr. Cocos

TOPNOTCH MEDICAL BOARD PREP SURGERY MAIN DIGITAL HANDOUT BY LOUBOMIR ANTONIO, MD Page 69 of 101
For inquiries visit www.topnotchboardprep.com.ph or https://www.facebook.com/topnotchmedicalboardprep/
This handout is only valid for the September 2021 PLE batch. This will be rendered obsolete for the next batch since we update our handouts regularly.
TOPNOTCH MEDICAL BOARD PREP SURGERY MAIN DIGITAL HANDOUT BY LOUBOMIR ANTONIO, MD
For inquiries visit www.topnotchboardprep.com.ph or https://www.facebook.com/topnotchmedicalboardprep/
This handout is only valid for the September 2021 PLE batch. This will be rendered obsolete for the next batch since we update our handouts regularly.
into and out of the gallbladder. The patient receives an intravenous • No sedation usually required
injection of a radioactive material called hydroxy iminodiacetic acid • No iodinated intravenous contrast (avoids iodine anaphylaxis
(HIDA). The material is taken up by the liver and excreted into the and contrast nephropathy)
biliary tract. When the test chemical fails to appear in the gallbladder
(non-visualization of the gallbladder), but is detected in the intestine,
• Rapid scan time
there probably is an obstruction of the cystic duct leading to and • No ionizing radiation (safe in pregnancy and children)
from the gallbladder. (Obstruction is most commonly caused • Delineates ductal anatomy proximal to obstructions
by gallstones and, less commonly, by tumors.) • Delineates anatomy post-biliary-enteric anastomosis
Dr. Cocos
• Define extraductal structures (useful in staging malignancy)
A 60-year-old diabetic man is admitted to the hospital with a diagnosis
of acute cholecystitis. The WBC count is 28,000, and a plain film of the
abdomen and CT scan show evidence of intramural gas in the
gallbladder. What is the most likely diagnosis?
(A) Emphysematous gallbladder (D) Sclerosing cholangitis
(B) Acalculous cholecystitis (E) Gallstone ileus
(C) Cholangiohepatitis
Emphysematous cholecystitis is caused by infection with gas-forming
anaerobes such as Clostridium perfringens. Diabetic patients are at
risk, and the disease can progress quickly to profound sepsis.
Emergent cholecystectomy is indicated. Air within the biliary tree (not
• Indications for IOC during laparoscopic cholecystectomy
gallbladder wall) may be seen in gallstone ileus, after biliary-enteric o jaundice or history of jaundice, or history of pancreatitis
anastomosis or after sphincterotomy. o elevated liver function tests
Dr. Cocos o a common bile duct larger than 5-7mm in diameter
A 48-year-old man is admitted to the hospital with severe abdominal o a cystic duct larger than 3mm in diameter
pain, tenderness in the right hypochondrium, and a WBC count of
o multiple small gallbladder stones
12,000. A HIDA scan fails to show the gallbladder after 4 hours. Acute
cholecystitis is established. After diagnosis, cholecystectomy should be o unclear anatomy
performed within which of the following? o common bile duct stones visualized on preoperative
(A) 3–60 minutes ultrasound
(B) The first 2–3 days following hospital admission o palpable common bile duct stones intraop
(C) 8 days o possible bile duct injury or leak
(D) 3 weeks o short cystic duct
(E) 3 months
• Normal Cholangiogram
According to the Tokyo guidelines, this patient falls under the Grade I
(mild) category, hence cholecystectomy within 72 hours is
o Complete opacification of
recommended (same hospital admission). Between the seventh and biliary tree
fourteenth day after admission, surgery may be extremely difficult o No filling defects
because of resolving infection and adhesions. o No contrast extravasation
Dr. Cocos
o Unimpeded duodenal egress of
contrast
CHOLEDOCHOLITHIASIS
• Present in 6-12% of patients with GB stones • Alternatives:
• Incidence increases with age (20-25% above age 60) o Laparoscopic Ultrasound
§ an alternative to IOC and attempts to improve upon some of
• Types:
REMARKS
the potential drawbacks of IOC including radiation exposure,
Primary • Formed in the CBD
operative time, and failure rate
choledocho- • Usually brown pigment type o Near-Infrared Fluorescent cholangiography
lithiasis • Seen in biliary stasis and infection (ascariasis) § involves the preoperative IV injection of indocyanine green
Secondary • Formed in the GB and migrated to the CBD (ICG) (excreted exclusively into the bile)
choledocho- • More common § protein-bound ICG emits light with a peak wavelength of 830
lithiasis • Usually cholesterol stones nm when illuminated with near-infrared light
• Treatment
TYPE OF CHOLEDOCHOLITHIASIS BASED ON THE TIMING OF • ERCP plus sphincterotomy + ductal
DIAGNOSIS (AFTER CHOLECYSTECTOMY) clearance of stones + laparoscopic
• Stoned identified by cholangiography shortly after If known diagnosis cholecystectomy (same admission)
cholecystectomy previously • Laparoscopic cholecystectomy +
Retained
• Missed during the operation IOC + CBDE or sphincterectomy
• Prevalence of about 1-7% (next day)
• Stones that are found later (<2 years after • Failure of endoscopic management
cholecystectomy) Open CBDE indications
• Unavailability of endoscope
Residual • Same composition (black pigment or cholesterol) as Roux-En-Y • If stones are impacted at the
the gallbladder stones choledochojejunostomy ampulla of Vater
• Prevalence of about 2-5%
• Assumed to be primary common duct stoned (usually
Recurrent of brown pigment type)
• Stones discovered > 2 years after the cholecystectomy
Surgery Platinum. 1st ed. 2018. p. 373

• Presentation
o Asymptomatic (silent)
o Symptomatic – pain similar to biliary colic, jaundice
o Elevation of bilirubin, alkaline phosphatase, transaminase
• Diagnosis
o Abdominal US: initial test
§ GB stones, Dilated CBD (>8mm)
ROUX-EN-Y CHOLEDOCHOJEJUNOSTOMY
Figure 32-26. Schwartz’s Principles of Surgery. 11th ed. 2019
o ERCP (shown on right):
gold standard - provides
therapeutic option at the
time of diagnosis

MRCP (magnetic resonance cholangiopancreatography)


• Non-invasive (avoids complications of diagnostic ERCP or PTC)
TOPNOTCH MEDICAL BOARD PREP SURGERY MAIN DIGITAL HANDOUT BY LOUBOMIR ANTONIO, MD Page 70 of 101
For inquiries visit www.topnotchboardprep.com.ph or https://www.facebook.com/topnotchmedicalboardprep/
This handout is only valid for the September 2021 PLE batch. This will be rendered obsolete for the next batch since we update our handouts regularly.
TOPNOTCH MEDICAL BOARD PREP SURGERY MAIN DIGITAL HANDOUT BY LOUBOMIR ANTONIO, MD
For inquiries visit www.topnotchboardprep.com.ph or https://www.facebook.com/topnotchmedicalboardprep/
This handout is only valid for the September 2021 PLE batch. This will be rendered obsolete for the next batch since we update our handouts regularly.
MIRIZZI SYNDROME
• Common hepatic
duct obstruction
caused by an extrinsic
compression from an
impacted stone in the
cystic duct or
Hartmann's pouch of
the gallbladder

SPHINCTEROTOMY/SPHINCTEROPLASTY Mirizzi syndrome types


Figure 33-48 and 49. Schwartz’s Principles of Surgery. 11th ed. 2019
TYPE FIGURE DESCRIPTION
External compression of the
common hepatic duct due to a
CHOLEDOCHOLITHIASIS I
stone impacted at the neck of the
https://qrs.ly/1scl14k
gallbladder or at the cystic duct
The fistula involves less than one-
Retained Stones II third of the circumference of the
• With T-tube: T-tube cholangiogram common bile duct
o Stone extraction through the T-tube tract at 2-4 weeks
Involvement of between one-third
• Without T-tube: ERCP plus sphincterotomy and two-thirds of the
✔ GUIDE QUESTIONS
III
circumference of the common bile
A 42-year-old woman with a history of chronic alcoholism is admitted to the duct
hospital because of acute pancreatitis. The bilirubin and amylase levels are
in the normal range. An ultrasound reveals cholelithiasis. The symptoms
Destruction of the entire wall of
abate on the fifth day after admission. What should she be advised IV
(A) To start on a low-fat diet. the common bile duct
(B) To increase the fat content of her diet.
(C) To undergo immediate cholecystectomy.
(D) To undergo cholecystectomy during the same hospital stay Cholecystoenteric fistula together
as well as an assessment of her bile ducts V
with any other type of MS
(E) That she will be discharged and now should undergo elective
cholecystectomy after 3 months.
Aside from alcoholism, a likely cause for the pancreatitis of this patient
GALLSTONE ILEUS
is gallstone which made its way to the pancreatic duct. Hence, aside
from cholecystectomy, intraoperative cholangiography (to assess if
there are stones in the biliary tree) is indicated for this patient.
Elective cholecystectomy should be avoided during the actual phase of
pancreatitis.
Dr. Cocos
A 73-year-old woman is evaluated for obstructive jaundice after an injury to
the CBD, 7 months previously at laparoscopic cholecystectomy. The alkaline
phosphatase is elevated. In obstructive jaundice, which of the following
statements is true regarding alkaline phosphatase?
(A) Its level increases before that of bilirubin.
(B) Its level is unlikely to be increased in pancreatic malignancy.
(C) Its elevation indicates bone metastasis.
(D) Its elevation excludes hepatic metastasis.
(E) Its level falls after that of the bilirubin, following surgical
• Bouveret syndrome: gallstone ileus of the duodenum
intervention.
o Uncommon form of gallstone ileus characterized by gastric
Alkaline phosphatase level usually is more sensitive than the bilirubin outlet obstruction caused by gallstone impaction in the pylorus
level for indicating cholestatic jaundice. It is likely to increase in
or proximal duodenum after its passage through a
pancreatic malignancy if obstruction is present (pancreatic head). It
also is more likely to fall before the bilirubin level when the obstruction
cholecystoduodenal fistula
has been relieved. If an unexplained alkaline phosphatase elevation o Patients with gallstone ileus may present with radiographic
exists (even in the presence of a normal bilirubin), biliary pathology findings of Rigler’s triad (i.e., pneumobilia, small-bowel
must be excluded. Elevation of the alkaline phosphatase from a obstruction, and an ectopic gallstone)
possible source in bone disease can be excluded by measuring
isoenzymes and does not always indicate bone metastasis. Its CHOLANGITIS
elevation does not exclude hepatic metastasis.
Dr. Cocos • Ascending bacterial infection of the biliary tree in association
A 43-year-old woman undergoes open cholecystectomy. Intraoperative with partial or complete blockage of the bile duct
cholangiogram revealed multiple stones in the CBD. Exploration of the • Combination of bacterial contamination and obstruction
CBD was performed to extract gallstones. The CBD was drained with a • Gallstones are most common cause
#18 T-tube. After 10 days, a T-tube cholangiogram reveals a retained
CBD stone. This should be treated by which of the following? • Usually due to gram negative aerobes
(A) Laparotomy and CBD exploration
(B) Subcutaneous heparinization
(C) Antibiotic therapy for 6 months and then reevaluation
(D) Extraction of the stone through the pathway created by the
T-tube (after 6 weeks)
(E) Ultrasound crushing of the CBD stone
If a stone is detected, the T tube should be left in place for 6 weeks to
allow the tract to mature. At this time, the T tube can be removed, and
the stone can be extracted by using a basket under fluoroscopy or
choledochoscopy. This approach is indicated only when a T-tube
larger than 16 has been inserted. If this approach is not feasible, the
stone can be extracted by retrograde endoscopic techniques or CBD
exploration.
Dr. Cocos

TOPNOTCH MEDICAL BOARD PREP SURGERY MAIN DIGITAL HANDOUT BY LOUBOMIR ANTONIO, MD Page 71 of 101
For inquiries visit www.topnotchboardprep.com.ph or https://www.facebook.com/topnotchmedicalboardprep/
This handout is only valid for the September 2021 PLE batch. This will be rendered obsolete for the next batch since we update our handouts regularly.
TOPNOTCH MEDICAL BOARD PREP SURGERY MAIN DIGITAL HANDOUT BY LOUBOMIR ANTONIO, MD
For inquiries visit www.topnotchboardprep.com.ph or https://www.facebook.com/topnotchmedicalboardprep/
This handout is only valid for the September 2021 PLE batch. This will be rendered obsolete for the next batch since we update our handouts regularly.
DIAGNOSTIC CRITERIA FOR ACUTE CHOLANGITIS BY TG18: (A) Infusion of cryoprecipitate
A. Systemic inflammation: (B) Oral vitamin K tablets to decrease prolonged INR
A-1. Fever and/or shaking chills (C) Parenteral vitamin K to decrease prolonged INR
A-2. Lab data: Evidence of inflammatory response (D) Demonstration that urobilinogen is increased in the urine
B. Cholestasis (E) Demonstration that stercobilinogen is increased in the stool
B-1. Jaundice Vitamin K requires bile salts for efficient absorption from the gut, as
do the other fat-soluble vitamins—A, D, and E. Therefore, Vitamin K is
B-2. Lab data: abnormal liver function tests
given intravenously (not orally) and is shown to improve INR and
C. Imaging
prothrombin time. Infusion of fresh frozen plasma is done
C-1. biliary dilatation perioperatively (during or minutes prior to the procedure).
C-2 evidence of etiology on imaging (stricture, stone, stent, etc.) Urobilinogen usually is absent in the urine in obstructive jaundice,
Suspected diagnosis: One item in A + one item in either B or C because its presence depends on a patent biliary–enteric circulation.
Definite diagnosis: One item in A + one item in B & one item in C Stercobilinogen will be absent in fecal examination.
Dr. Cocos

GRADING FOR ACUTE CHOLANGITIS


GRADE CRITERIA CHOLEDOCHAL CYSTS
• Severe associated dysfunction at least in one of the • Congenital cystic dilatations of the biliary tree
following organ systems: • More common in females
o Cardiovascular: hypotension requiring dopamine >5 • Usually presents during childhood
µg/kg/min or any dose of norepinephrine
• >90% with anomalous pancreatobiliary duct junction
III o Nervous: decreased level of consciousness
o Respiratory: PaO2/FiO2 ratio <300 • Long common channel – pathognomonic sign seen in ERCP
o Renal oliguria, creatinine > 2.0 mg/dL TYPE FEATURE
o Hepatic: PT-INR >1.5 • Fusiform diverticulum
Type 1
o Hematological: platelet count<100,000/mm3 • Most common type
• Moderate does not respond to the initial medical Type 2 • Saccular diverticulum
treatment but is not associated with organ dysfunction Type 3 • Choledochoceles, multiple cysts
• Risk of increased severity without early biliary drainage • Extrahepatic and intrahepatic duct
Type 4a
involvement
(May progress if without immediate intervention)
Type 4b • Extrahepatic duct involvement only
• Associated with any two of the following:
II • Intrahepatic duct involvement only
o Abnormal WBC count (>12,000/mm3 or Type 5
• Caroli disease
<4,000mm3)
o High fever (>39°C)
o Age >75 years old
o Hyperbilirubinemia
o Hypoalbuminemia
• Mild responds to initial medical treatment
I • Does not meet the criteria for grade II (moderate) or
III (severe)
Surgery Platinum. 1st ed. 2018. p. 375

ACUTE CHOLANGITIS: MANAGEMENT


• Initial medical therapy with antibiotics
• For non-responders: biliary drainage should be
Grade considered
I • Intervention for etiology (e.g. choledocholithiasis,
pancreato-biliary malignancy): endoscopic,
percutaneous, or operative intervention after workup
• Early endoscopic or percutaneous drainage
Grade
• Emergency operative drainage with a T-tube
II
• Definitive procedure to remove cause of cholangitis
• Appropriate organ support
• Urgent biliary drainage
o ERCP (gold standard) + papillotomy
Grade
o PTC with catheter drainage Figure 32-27. Schwartz’s Principles of Surgery. 11th ed. 2019
III • Presentation: abdominal pain, jaundice, mass
o Laparotomy with decompression of the bile duct
with a T-tube • Diagnosis: US/CT/MRCP
• Definitive procedure to remove cause of cholangitis • Treatment
Surgery Platinum. 1st ed. 2018. p. 375
TYPE MANAGEMENT
✔ GUIDE QUESTIONS Types 1
• Excision + Roux-en-Y hepaticojejunostomy
A 40-year-old man underwent laparoscopic cholecystectomy 2 years and 4B
earlier. He remains asymptomatic until 1 week before admission, when Type 2 • Excision; defect in the CBD is closed over a T-tube
he complains of RUQ pain and jaundice. He develops a fever and has Type 3 • Sphincterotomy
several rigor attacks on the day of admission. An ultrasound confirms • Segmental liver resection, excision & Roux-en-Y
the presence of gallstones in the distal CBD. The patient is given Type 4 A
hepaticojejunostomy
antibiotics. Which of the following should be undertaken as the next
Type 5 • Liver transplantation
step in therapy? Surgery Platinum. 1st ed. 2018. p. 377
(A) Should be discharged home under observation
(B) Should be observed in the hospital ✔ GUIDE QUESTION
(C) Undergo surgical exploration of the CBD A 15-year-old female presents with RUQ abdominal pain. Workup
(D) ERCP with sphincterotomy and stone removal reveals a choledochal cyst. Which of the following statements is TRUE?
(E) Anticoagulants (A) Choledochal cysts are more common in men.
(B) Laparoscopic cholecystectomy is the recommended treatment.
Since this patient already underwent cholecystectomy prior, it would (C) Patients with a choledochal cyst have an increased risk of
be better to do a minimally invasive procedure through an ERCP with cholangiocarcinoma.
sphincterotomy for CBD stones. Otherwise, there is a need to do (D) All patients with a choledochal cyst have abdominal pain, a RUQ
surgical exploration and address both the gallbladder and the stones. mass, and jaundice.
Observation is not an option especially since this patient is already in (E) The etiology is infectious.
mild cholangitis.
Dr. Cocos The etiology of choledochal cysts is probably multifactorial but the
A 43-year-old man is admitted with jaundice of 6-week duration. An presence of a long common channel between the common bile duct
ultrasound shows multiple small stones in the gallbladder and the and pancreatic duct is usually demonstrated. The risk of having
presence of a CBD stone. A preoperative ERCP followed by a laparoscopic malignant transformation from a choledochal cyst is as high as 11%
cholecystectomy is planned. The international normalization ratio (INR) therefore complete surgical excision is recommended, whether it be
is elevated to 3.1 What is the next step in management? open or laparoscopic. The classic triad of jaundice, RUQ mass and
TOPNOTCH MEDICAL BOARD PREP SURGERY MAIN DIGITAL HANDOUT BY LOUBOMIR ANTONIO, MD Page 72 of 101
For inquiries visit www.topnotchboardprep.com.ph or https://www.facebook.com/topnotchmedicalboardprep/
This handout is only valid for the September 2021 PLE batch. This will be rendered obsolete for the next batch since we update our handouts regularly.
TOPNOTCH MEDICAL BOARD PREP SURGERY MAIN DIGITAL HANDOUT BY LOUBOMIR ANTONIO, MD
For inquiries visit www.topnotchboardprep.com.ph or https://www.facebook.com/topnotchmedicalboardprep/
This handout is only valid for the September 2021 PLE batch. This will be rendered obsolete for the next batch since we update our handouts regularly.
abdominal pain is found in less than a third of patients. For most types T DESCRIPTION MANAGEMENT
of choledochal cyst, excision of the cyst with a Roux-en-Y biliary • Tumor invades
enteric anastomosis is recommended. Laparoscopic cholecystectomy T1a • Simple cholecystectomy
lamina propria
alone is not sufficient. Choledochal cysts are more common in females. • Extended cholecystectomy (includes
Dr. Cocos
segments IVB and V of the liver) plus
SCLEROSING CHOLANGITIS (SC) • Tumor invades
T1b lymphadenectomy of nodes in the
muscle layer
• Inflammatory strictures involving both extra and intrahepatic porta hepatis, gastrohepatic
biliary tree ligament, and retroduodenal space
• Progressive leading to secondary biliary cirrhosis • Extended cholecystectomy (includes
• Tumor invades
REMARKS segments IVB and V of the liver) plus
perimuscular
• Associated with other sclerosing lesions T2 lymphadenectomy of nodes in the
connective
Primary SC • (UC, Riedel thyroiditis and retroperitoneal porta hepatis, gastrohepatic
tissue
fibrosis) ligament, and retroduodenal space
• More common • Extended right hepatectomy
• Tumor
• May degenerate into cholangiocarcinoma (includes segments IV to VIII of
perforates
the liver) plus en bloc resection of
Secondary SC • More common in men serosa and/or
T3 the common bile duct for grossly
• Most commonly involves hepatic duct invades the
positive periportal lymph nodes
bifurcation liver or
followed by Roux-en-Y
• Weight loss and fatigue adjacent organs
Presentation hepaticojejunostomy
• Intermittent jaundice, pruritus, abdominal pain • Extended right hepatectomy
• ERCP/PTC: (+) multiple dilation and • Tumor invades
Diagnosis (includes segments IV to VIII of
strictures or “beading pattern” main portal
the liver) plus en bloc resection of
Treatment • Liver transplantation vein or hepatic
T4 the common bile duct for grossly
artery, or
positive periportal lymph nodes
multiple extra
GALLBLADDER POLYPS followed by Roux-en-Y
hepatic organs
Natural History of Gallbladder Polyps hepaticojejunostomy
Surgery Platinum. 1st ed. 2018. p. 380
• Most gallbladder polyps are of cholesterol type and have no ✔ GUIDE QUESTION
malignancy potential (local form of cholesterolosis) A 55-year-old white female undergoes a laparoscopic cholecystectomy
• There is an 8-11% prevalence of gallbladder cancer in for symptomatic cholelithiasis. The operation went well, and the
specimens with gallbladder polyps patient was discharged home. One week later, she comes to your office
• Almost all malignant polyps are more than 1cm in size (88- for a routine postoperative follow-up. The final pathology report shows
100%) an incidental finding of a gallbladder carcinoma confined to the mucosa.
In further advising the patient, you should inform her that:
• Factors associated with malignancy in gallbladder polyps: (A) She should undergo radiation therapy.
Presence of single polyp Rapid growth (B) She should undergo right hepatectomy to remove locally
Size of polyp >1cm Sessile in morphology infiltrating disease.
Age >50 years Adenomatous in histology (C) She should undergo regional lymphadenectomy.
(D) She requires systemic chemotherapy.
GALLBLADDER CARCINOMA (E) She does not require any further therapy.
Only T1a lesions, as in this case, can be addressed by just a
• Rare aggressive tumor affecting elderly population cholecystectomy. Otherwise, a reoperation is needed.
• More common among women Dr. Cocos

• Risk Factors:
BILE DUCT CARCINOMA
Gallstones >3cm (most important) Choledochal cysts
Gallbladder polyps >1cm Sclerosing cholangitis • Rare tumor arising from the biliary epithelium
Porcelain gallbladder Exposure to, nitrosamines • Most commonly involves the hepatic duct bifurcation
Although Schwartz mentions Porcelain GB as premalignant, that is no • Risk Factors:
longer regarded as true as per latest updates. Near zero ang Primary sclerosing cholangitis Biliary-enteric anastomosis
malignancy risk ng porcelain GB. Choledochal cysts Biliary tract infections (Clonorchis)
Dr. Antonio
Choledocholithiasis Exposure to nitrosamines
• Adenocarcinoma (90%)
• >95% adenocarcinoma (nodular type as most common)
• At the time of diagnosis, 40% have distant metastasis
• May occur distal, proximal, and perihilar
• Presentation
• Klatskin Tumor (perihilar cholangiocarcinoma)
o Similar to cholecystitis/cholelithiasis
o Weight loss, anorexia and jaundice are uncommon Bismuth-Corlette classification of biliary tract cancers
o Lab studies are non-diagnostic
• Diagnosis
o Imaging studies are suggestive
o Usually incidental after a cholecystectomy
• Discontinuous mucosa, echogenic mucosa, and
submucosal echolucency (seen in early
malignancy more commonly compared with
Ultrasound
benign gallbladder disease)
• most typical finding is an inhomogeneous mass
replacing all or part of the gallbladder
• Most common finding: mass involving all or part
of the gallbladder
CT scan • Other findings: asymmetric wall thickening,
intraluminal mass, and extension to nearby
structures
Modified from de Groen PC, Gores GJ, LaRusso NF, et al. N Engl J Med 1999; 341:1368. UpToDate.com
• Usually incidental after a routine
• Presentation
cholecystectomy
o Painless jaundice (most common)
Others • Laboratory studies are not helpful
o Weight loss, fatigue, anorexia, abdominal pain
• CEA and CA 19-9: tumor markers with variable
specify and sensitivity
o Elevated CA 125, CEA, CA 19-9
Surgery Platinum. 1st ed. 2018. p. 380 • Diagnosis
• Treatment: Palliative treatment o US/CT
o ERCP
o PTC: defines proximal extent (most important factor
determining resectability)

TOPNOTCH MEDICAL BOARD PREP SURGERY MAIN DIGITAL HANDOUT BY LOUBOMIR ANTONIO, MD Page 73 of 101
For inquiries visit www.topnotchboardprep.com.ph or https://www.facebook.com/topnotchmedicalboardprep/
This handout is only valid for the September 2021 PLE batch. This will be rendered obsolete for the next batch since we update our handouts regularly.
TOPNOTCH MEDICAL BOARD PREP SURGERY MAIN DIGITAL HANDOUT BY LOUBOMIR ANTONIO, MD
For inquiries visit www.topnotchboardprep.com.ph or https://www.facebook.com/topnotchmedicalboardprep/
This handout is only valid for the September 2021 PLE batch. This will be rendered obsolete for the next batch since we update our handouts regularly.
• Treatment PART FEATURES
o Unresectable: palliative drainage; percutaneous route preferred • Disc-shaped
Head
o Resectable: Excision plus reconstruction • Within concavity of the C-shaped duodenum
o Roux-en-Y hepaticojejunostomy Uncinate • Projection to the left; from lower part to head
process behind the superior mesenteric arteries
✔ GUIDE QUESTIONS
A 67-year-old woman is evaluated for obstructive jaundice. The • Narrow; connects the head to the body
Neck
cholangiographic findings indicate that she has a cancer of the lower end of the • Found in front of the origin of the portal vein
CBD. Clinical examination would most likely reveal which of the following? Body • Upward, to the left across the midline
(A) Enlarged gallbladder • Extends to the hilum of the spleen and
(B) Shrunken gallbladder Tail
splenoreneal ligament
(C) Enlarged pancreas PANCREATIC DUCTS
(D) Shrunken pancreas
(E) Palpable tumor
An enlarged gallbladder associated with painless jaundice (known as
Courvoisier sign) is usually brought about by build-up of bile due to a
distal obstruction from a periampullary (distal CBD, pancreatic head,
duodenum, ampullary) malignancy. In obstructive jaundice
attributable to gallstones, the gallbladder is usually shrunken, owing
to the previous inflammatory condition affecting the gallbladder.
Dr. Cocos
A 57-year-old presents with gradually increasing obstructive jaundice.
An ultrasound of the liver shows dilated intrahepatic ducts, but the CBD
is normal. An ERCP shows a filling defect at the level of the common
hepatic duct. Endoscopic brush biopsies are taken, and histology DUCT FEATURES
confirms cholangiocarcinoma. In discussing these findings, the surgeon • Opens to the second part of duodenum
should inform the patient that Main pancreatic via ampulla of Vater (with common bile
(A) This tumor affects men more commonly than women. duct of Wirsung duct)
(B) The tumor is a result of gallstones. • Derived from the ventral anlage
(C) The tumor is best treated with a stent to relieve obstructive jaundice.
(D) Weight loss is common in this condition.
• Often absent
Accessory duct of
(E) The most common location of these tumors is at the ampulla of Vater. • Drains the upper half of the pancreas
Santorini
Cholangiocarcinoma is most common at the perihilar area (Klatskin). • Derived from the dorsal anlage
Having gallstones is not a risk factor. he male-to-female ratio for
cholangiocarcinoma is 1:2.5 in patients in their 60s and 70s and 1:15
in patients younger than 40 years. The best treatment option is
excision but palliation can be done through inserting a percutaneous
tube for biliary drainage for unresectable tumors.
Dr. Cocos
A patient presents with biliary colic. On ultrasound there are multiple
small gallstones in the gallbladder and the common bile duct measures
9mm in diameter. No stone is visualized in the common bile duct.
Which of the following is the most reasonable next step?
a. Repeat UTZ in 24-48 hours
b. MRCP with contrast Figure 33-3. Schwartz’s Principles of Surgery. 11th ed. 2019

c. Percutaneous cholangiography
ANOMALIES OF THE PANCREAS
d. Laparoscopic cholecystectomy and intraoperative
cholangiography ANOMALY FEATURE
For patients with symptomatic gallstones and suspected CBD stones, either • Most common congenital anomaly of the
preoperative endoscopic cholangiography or an intraoperative pancreas
cholangiogram is needed to document the presence of bile duct stones. If • Predisposes to acute and chronic pancreatitis
Pancreas
an endoscopic cholangiogram reveals stones, sphincterotomy and ductal • Functional obstructions of the duct of Santorini
divisum
clearance of the stones is appropriate, followed by a laparoscopic • Treatment: operative or endoscopic
cholecystectomy. An intraoperative cholangiogram at the time of sphincteroplasty of the minor papilla and
cholecystectomy will also document the presence or absence of bile duct accessory duct in symptomatic patients
stones. Laparoscopic common bile duct exploration via the cystic duct or • Second part of the duodenum is surrounded by a
Annular
with formal choledochotomy allows the stones to be retrieved in the same rim of pancreatic tissue
pancreas
setting. If the expertise and/or the instrumentation for laparoscopic • (+) proximal small intestinal obstruction
common bile duct exploration are not available, a drain should be left
adjacent to the cystic duct and the patient scheduled for endoscopic
sphincterotomy the following day. An open common bile duct exploration
is an option if the endoscopic method has already been tried or is, for some
reason, not feasible. There is no point in repeating the ultrasound and
routine MRCP for gallstones is controversial.
Dr. Cocos
Which hepatic cells provides the primary defense against
lipopolysaccharide (LPS)?
a. Hepatocytes
b. Kupffer cells
c. Bile duct epithelial cells
d. Intrahepatic endothelial cells
The complications of Gram-negative sepsis is initiated by the
endotoxin LPS. The liver is the main organ in the clearance of LPS in
the bloodstream and plays a critical role in the identification and
processing of LPS. Kupffer cells are the resident macrophages in the
liver and have been shown to participate in LPS clearance.
Figure 33-20. Schwartz’s Principles of Surgery. 11th ed. 2019
Dr. Cocos

PANCREAS
ANATOMY OF THE PANCREAS
• Most unforgiving organ of the body (Schwartz)
• retroperitoneal pistol-shaped organ
• Extends transversely across the upper abdomen behind the stomach
• The pancreas is an endocrine and exocrine organ about 15-20
cm long and weighs about 75 to 100 g in an adult
• Formed by the fusion of a ventral bud and a larger, dorsal bud,
as a result of gut rotation
TOPNOTCH MEDICAL BOARD PREP SURGERY MAIN DIGITAL HANDOUT BY LOUBOMIR ANTONIO, MD Page 74 of 101
For inquiries visit www.topnotchboardprep.com.ph or https://www.facebook.com/topnotchmedicalboardprep/
This handout is only valid for the September 2021 PLE batch. This will be rendered obsolete for the next batch since we update our handouts regularly.
TOPNOTCH MEDICAL BOARD PREP SURGERY MAIN DIGITAL HANDOUT BY LOUBOMIR ANTONIO, MD
For inquiries visit www.topnotchboardprep.com.ph or https://www.facebook.com/topnotchmedicalboardprep/
This handout is only valid for the September 2021 PLE batch. This will be rendered obsolete for the next batch since we update our handouts regularly.
BLOOD SUPPLY ✔ GUIDE QUESTION
• Splenic artery ← celiac trunk A 1-week-old infant is brought to the hospital because of vomiting. An
• Superior pancreaticoduodenal artery ← upper gastrointestinal (GI) series reveals duodenal obstruction. On
gastroduodenal artery laparotomy, annular pancreas is found. Which of the following
Arterial supply statements about annular pancreas is TRUE?
• Inferior pancreaticoduodenal artery ←
(A) Resection is the treatment of choice
superior mesenteric artery (B) It is associated with Down syndrome
• Dorsal, great and caudal pancreatic arteries (C) Symptoms usually begin with back pain
Venous drainage • Pancreatic vein → portal vein (D) It is most likely due to abnormal rotation encircling the third
part of the duodenum
(E) Symptoms begin in childhood
Annular pancreas is a rare congenital abnormality characterized by a
ring of pancreatic tissue surrounding the descending portion of the
duodenum (D2) not D3. It is thought to originate from incomplete
rotation of the ventral pancreatic bud. Different chromosomal
diseases (for example Trisomy 21 and, with a minor
frequency, Trisomy 18 and Trisomy 13) are present in about 33% of
subjects affected by annular pancreas. Down syndrome (trisomy 13) is
associated with annular pancreas as well as duodenal atresia and
stenosis. The treatment to relieve the obstruction is a bypass
(duodenojejunostomy in neonates or gastrojejunostomy in adults.
Resection is not an acceptable choice due to the high incidence of
fistula. It may manifest in adults, presenting with abdominal pain,
nausea, and vomiting.
Dr. Cocos

ACUTE PANCREATITIS
Figure 33-4. Schwartz’s Principles of Surgery. 11th ed. 2019
• Etiology
o Gallstone
o Alcohol
o Miscellaneous
• Clinical Presentations
o May vary from mild abdominal discomfort to profound shock,
hypotension/hypoxemia
o Epigastric pain radiating to the back with nausea and vomiting
o Discomfort may be relieved by sitting and aggravated by lying
o In severe cases may have rigid abdomen, epigastric guarding
and rebound tenderness
o Severe pancreatic inflammation and necrosis may cause
retroperitoneal hemorrhage, which leads to:
§ Large 3rd space fluid loss
§ Hypovolemia
§ Hypotension
Figure 33-5. Schwartz’s Principles of Surgery. 11th ed. 2019 § Tachycardia
LYMPHATICS o Blood dissection along different planes:
• Celiac nodes • Blood extends to the flank tissues → flank
Grey Turner sign
• Superior mesenteric nodes ecchymosis
• Blood dissects to the falciform ligament →
Cullen sign
periumbilical ecchymosis
Fox sign • Ecchymosis of the inguinal ligament
Bryan sign • Bluish discoloration of the scrotum
o History of intake of heavy meal with consumption of large
quantities of alcohol
o Pain typically begins 1-4 hours after meal
Theory Description
• Acute pancreatitis arises from a secretory block
Secretary preventing exocytosis, leading to protein
block accumulation within the cytoplasm of acinar
theory cells leading to fusion of zymogen granules:
cause tissue damage & inflammation
• Reflux of either bile or duodenal fluid with
Reflux enterokinase into the pancreatic duct can
theory activate pancreatic proenzymes within the
Figure 33-6. Schwartz’s Principles of Surgery. 11th ed. 2019
pancreas
INNERVATION • Toxic substances and increased triglycerides of
• Celiac plexus – sympathetic innervation Direct
ischemic damage of the pancreas may cause
damage of
• Vagus nerve – parasympathetic innervation direct damage to pancreatic acinar cells leading
pancreas
to acute pancreatitis
Surgery Platinum. 1st ed. 2018. p. 388

TYPICAL POSTURE TO
REDUCE PANCREATIC-
TYPE PAIN
Kalloo AN, et al. 2003. Chronic Pancreatitis:
Introduction. Johns Hopkins University.
Hopkinsmedicine.org

Figure 33-7. Schwartz’s Principles of Surgery. 11th ed. 2019

TOPNOTCH MEDICAL BOARD PREP SURGERY MAIN DIGITAL HANDOUT BY LOUBOMIR ANTONIO, MD Page 75 of 101
For inquiries visit www.topnotchboardprep.com.ph or https://www.facebook.com/topnotchmedicalboardprep/
This handout is only valid for the September 2021 PLE batch. This will be rendered obsolete for the next batch since we update our handouts regularly.
TOPNOTCH MEDICAL BOARD PREP SURGERY MAIN DIGITAL HANDOUT BY LOUBOMIR ANTONIO, MD
For inquiries visit www.topnotchboardprep.com.ph or https://www.facebook.com/topnotchmedicalboardprep/
This handout is only valid for the September 2021 PLE batch. This will be rendered obsolete for the next batch since we update our handouts regularly.
• Diagnosis is based on two or more of the following criteria: Biliary sludge or stones in gallbladder
§
1. Severe characteristic abdominal pain Increased pancreatic volume
§
2. Serum amylase or lipase more than three times (3x) Changes of the pancreatic parenchyma
§
higher than the upper limit Peripancreatic fluid collections
§
• Remains increased for Mild pancreatitis: no sonographic abnormalities may be
§
Amylase
Peak within the first 24 36 hours appreciable
hours of symptoms • Sustained for a much § If inflammation is diffuse and more severe, pancreas may be
Lipase
longer period uniformly hypoechoic
3. Contrast-enhanced computed tomography (CECT) § Pancreatic necrosis or hemorrhage: cannot be determined
findings of acute pancreatitis reliably with ultrasound
• Radiology § Pseudocysts: well-marginated ovoid or spherical anechoic
o Abdominal radiograph: relatively insensitive masses with posterior acoustic enhancement
o Significant findings o Abdominal CT scan – diagnostic gold standard
§ Calcification in the lesser sac and the pancreas § Mild pancreatitis – interstitial edema
§ Gas formation in the lesser sac (abscess formation) § Severe pancreatitis – (+) necrosis
§ Blurred psoas shadow (retroperitoneal pancreatic necrosis) § Pancreatic swelling, peripancreatic infiltrates, fluid collections
§ Cut-off sign (colonic spasm adjacent to an inflamed pancreas) and fat necrosis and areas of nonenhancement of the pancreas
§ Reversed-3 or inverted-3 sign (focal duodenal and jejunal § Abnormal extraluminal gas bubbles: pathognomonic of
ileus in the area of the head of the pancreas) pancreatic infection
o Ultrasound
Type of Collection Time (Weeks) Location Imaging Appearance
Interstitial edematous
• Homogenous, fluid attenuation,
pancreatitis • Adjacent to pancreas,
• ≤4 no liquefaction,
Acute peri-pancreatic extrapancreatic only
not encapsulated
fluid collection
• Homogenous, fluid attenuation,
• Adjacent or distant to
Pseudocyst • >4 no liquefaction,
pancreas
encapsulated
• Heterogenous, non-liquefied
Necrotizing pancreatitis • In parenchyma and/or
• ≤4 material, variably loculated,
Acute necrotic collection extrapancreatic
not encapsulated
• Heterogenous, non-liquefied
• In parenchyma and/or
Walled-off necrosis • >4 material, variably loculated,
extrapancreatic
encapsulated
Adapted from Thoeni RF. Radiology 2012;262:751-764, with permission from Radiology Society of North America.*
REVISED ATLANTA CRITERIA FOR ACUTE PANCREATITIS RANSON’S PROGNOSTIC SIGNS OF PANCREATITIS
Severity Criteria Criteria for acute pancreatitis not due to gallstones
• No organ failure At admission During the initial 48 h
• No local complications (e.g., peripancreatic fluid • Age >55 y • Hematocrit fall >10 points
Mild collections, Pancreatic necrosis) • WBC >16,000/mm³ • BUN elevation >5 mg/dL
• No systemic complications • Blood glucose >200 mg/dL • Serum calcium <8 mg/dL
• Typically resolves in first week • Serum LDH >350 IU/L • Arterial PO2 <60 mmHg
• Transient organ failure (≤ 48 hours) • Serum AST >250 U/dL • Base deficit >4 mEq/L
or • Estimated fluid
Moderate • Local complications sequestration >6L
or
• Exacerbation of comorbid disease Criteria for acute gallstone pancreatitis
Severe • Persistent organ failure (> 48 hours) At admission During the initial 48 h
PRINCIPLES OF MANAGEMENT OF ACUTE PANCREATITIS • Age > 70 y • Hematocrit fall > 10 points
MANAGEMENT DESCRIPTION • WBC >18,000/mm³ • BUN elevation >2 mg/dL
• Adequate fluid resuscitation to restore • Blood glucose >220 mg/dL • Serum calcium <8 mg/dL
Resuscitation normal blood volume, blood pressure, and • Serum LDH >400 IU/L • Base deficit >5 mEq/L
urine output • Serum AST >250 U/dL • Estimated fluid
• Vital signs, oxygenation, hemodynamic, sequestration >4L
Monitoring Schwartz’s Principles of Surgery, 11th edition
kidney, metabolic parameters
• Intravenous narcotics, patient controlled • Prognosis
anesthesia, NSAIDs RANSON SIGNS MORTALITY
Analgesia <2 points 0%
• No clinical evidence that Morphine can
induce spasm of the Sphincter of Oddi 3-5 signs 10-20%
>7 signs 50%
MANAGEMENT DESCRIPTION
• Mild and moderate pancreatitis: NPO and Indications for Surgical Intervention
bowel rest, advance to oral diet within 1 week • Diagnostic uncertainty
Nutritional
• Severe pancreatitis: early initiation of nutrition • Intraabdominal catastrophe unrelated to necrotizing
support
(<48 hours), enteral better than parenteral, pancreatitis (e.g. perforated viscus)
nasojejunal better than nasogastric • Infected necrosis documented by FNA or extraluminal gas on
Prophylactic • Reserved for documented infected pancreatic CT scan
antibiotics necrosis • Severe sterile necrosis
• Indicated in patients who have acute biliary • Symptomatic organized pancreatic necrosis
ERCP pancreatitis with cholangitis and retained bile
duct stones causing biliary obstruction
Surgery Platinum. 1st ed. 2018. p.392
INTERVENTION REMARKS TECHNIQUE
• Catheter is placed with a guidewire percutaneously through the (left)
• Least invasive technique for treating
retroperitoneum or transabdominally, but also through the wall of the
infected necrosis
Catheter Drainage stomach or duodenum
• Most effective if target lesions have a
• Approximately half will be successful and will not require surgical
significant fluid component
intervention

TOPNOTCH MEDICAL BOARD PREP SURGERY MAIN DIGITAL HANDOUT BY LOUBOMIR ANTONIO, MD Page 76 of 101
For inquiries visit www.topnotchboardprep.com.ph or https://www.facebook.com/topnotchmedicalboardprep/
This handout is only valid for the September 2021 PLE batch. This will be rendered obsolete for the next batch since we update our handouts regularly.
TOPNOTCH MEDICAL BOARD PREP SURGERY MAIN DIGITAL HANDOUT BY LOUBOMIR ANTONIO, MD
For inquiries visit www.topnotchboardprep.com.ph or https://www.facebook.com/topnotchmedicalboardprep/
This handout is only valid for the September 2021 PLE batch. This will be rendered obsolete for the next batch since we update our handouts regularly.
• Video-assisted retroperitoneal • Following near complete debridement, two large-bore surgical drains
Minimally invasive
debridement (VARD) procedure allows are placed into the empty cavity, continuously lavaged with increasing
necrosectomy
for the removal of large pieces of necrosis amounts (2, 4, then 6 L) of 0.9% saline
• Advantages:
Endoscopic • Transgastric internal fistula is created
o Requires no abdominal incision
transluminal • Disadvantage, need for repeated, multiple procedures to remove
o No external pancreatic fistula
necrosectomy sufficient amounts of necrosis
o No risk for incisional hernia
• Variations:
• Reference standard of treatment in
Open o Open necrosectomy with open or closed packing
patients with infected necrotizing
necrosectomy o Open necrosectomy with continuous closed postoperative lavage
pancreatitis
o Programmed open necrosectomy
Surgery Platinum. 1st ed. 2018. p. 392

✔ GUIDE QUESTIONS • Etiology


A 40-year-old alcoholic male is admitted with severe epigastric pain o Alcohol (most common)
radiating to the back. Serum amylase level is reported as normal, but o Idiopathic
serum lipase is elevated. The serum is noted to be milky in o Others: hypercalcemia, hyperlipidemia, familial, trauma,
appearance. A diagnosis of pancreatitis is made. The serum amylase is
iatrogenic, gallstones
normal because
(A) The patient has chronic renal failure
TIGAR-O CLASSIFICATION OF CHRONIC PANCREATITIS
(B) The patient has hyperlipidemia • Toxic and metabolic factors
(C) The patient has alcoholic cirrhosis • Alcohol – increasing incidence of CP with duration of
T
(D) The patient has alcoholic hepatitis alcohol use
(E) The diagnosis of pancreatitis is incorrect • Tobacco – etiology unknown
Hypertriglyceridemia compounds the diagnosis of acute pancreatitis • Idiopathic
by causing spuriously normal amylase levels. Plasma triglyceride • Early onset – mean age 20; pain predominant; calcification
levels higher than 500 mg/dL interfere with in vitro determination of rare
the actual amylase level by preventing the calorimetric reading of • Late onset – mean age 56; relatively painless; calcification;
the assay end point. Other causes of normal amylase in pancreatitis I endocrine and exocrine insufficiency present
include: increased urinary excretion of amylase and near complete • Minimal change – abdominal pain syndrome, minimal
destruction of pancreatic parenchyma as a result of chronic pancreatic changes on imaging studies
pancreatitis. On the other hand, the serum amylase level may be • Tropical pancreatitis – tropical calcific and fibrocalculous
elevated in the absence of pancreatitis (for example, perforated pancreatic diabetes
peptic ulcer, gangrenous cholecystitis, small-bowel strangulation or
• Genetic predisposition
chronic renal failure.) Therefore, serum lipase is more useful since it
is more specific for acute pancreatitis. • PRSSI – cationic trypsinogen cannot be inactivated
Dr. Cocos G • CFTR – cystic fibrosis transmembrane conductance
A 52-year-old woman is admitted to the hospital with abdominal pain. regulator mutation, cannot hydrolyze mucus
She reports that she drinks alcohol only at social occasions. The • SPINK I – inactivates trypsin inhibitor
amylase is elevated to 340 U. Which following x-ray finding would • Autoimmune
support a diagnosis of pancreatitis? A • Lymphoplasmacytic sclerosing pancreatitis; increased
(A) Hepatic lesion on CT scan
• IgG4; increased association with other autoimmune diseases
(B) Choledocholithiasis on ultrasound
(C) Anterior displacement of the stomach on barium upper GI • Recurrent and severe acute pancreatitis
R
series • Recurrent bouts of acute pancreatitis lead to chronic pancreatitis
(D) Large loop of colon in the RUQ • Obstructive
(E) Irregular cutoff of the CBD on cholangiogram • Stricture in main pancreatic duct leads to upstream (to the
The presence of choledocholithiasis on ultrasound supports a O tail) ductal dilation, acinar atrophy, and chronic pancreatitis
diagnosis of a possible cholangitis but may suggest a possible etiology • Caused by trauma, sphincter of Oddi dysfunction, pancreas
for pancreatitis. If a large pseudocyst is present (complication of divisum, sequelae of acute pancreatitis
pancreatitis), it may cause displacement of the transverse colon, Etemad B, Whitcomb DC. Chronic Pancreatitis. Gastroenterology; 120(3): 682-707; 2001

duodenum, or stomach (anteriorly). Other radiologic signs in


pancreatitis include pseudocyst on ultrasound or CT scan, downward • Clinical Manifestations
displacement of transverse colon, dilated pancreatic duct on SYMPTOM DESCRIPTION
pancreatogram, and smooth tapering of the CBD on cholangiogram • Most common symptom
(if the head of the pancreas is diseased). The irregular tapering of the • Highly variable pattern, constant and boring
common duct is suggestive of neoplasm. The looping of the colon in the • Epigastric or midepigastric in location, often
RUQ is seen with sigmoid volvulus. radiates to the back
Dr. Cocos
• Usually associated with anorexia, nausea,
Abdominal
CHRONIC PANCREATITIS and vomiting
pain
• A clinical entity rather than a pathologic process • Persist for hours or days with exacerbations
• Organ becomes small, indurated and nodular and edges • In late phases of the disease, pain may
disappear (“burnout”), a transition that is
becomes rounded
often associated with the development of
• Alternating areas of stricture and dilatation of the main diabetes and exocrine insufficiency
pancreatic duct
• Bloating, flatulence, diarrhea, steatorrhea
• Pathophysiology Malabsorption
• Weight loss and deficiencies in micronutrients
o Progressive inflammatory disorder that leads to irreversible and weight loss
and fat soluble vitamins A, D, E, K
destruction of the exocrine and endocrine tissue of the • Develops later in the course of the disease
pancreas when 90% of the parenchyma is replaced by
• Subtypes: Pancreatogenic fibrosis
o Chronic calcific pancreatitis diabetes • Global deficiency of all three glucoregulatory
o Chronic obstructive pancreatitis islet cell hormones: insulin, glucagon, and
pancreatic polypeptide
Surgery Platinum. 1st ed. 2018. p.396

TOPNOTCH MEDICAL BOARD PREP SURGERY MAIN DIGITAL HANDOUT BY LOUBOMIR ANTONIO, MD Page 77 of 101
For inquiries visit www.topnotchboardprep.com.ph or https://www.facebook.com/topnotchmedicalboardprep/
This handout is only valid for the September 2021 PLE batch. This will be rendered obsolete for the next batch since we update our handouts regularly.
TOPNOTCH MEDICAL BOARD PREP SURGERY MAIN DIGITAL HANDOUT BY LOUBOMIR ANTONIO, MD
For inquiries visit www.topnotchboardprep.com.ph or https://www.facebook.com/topnotchmedicalboardprep/
This handout is only valid for the September 2021 PLE batch. This will be rendered obsolete for the next batch since we update our handouts regularly.

CLINICAL PICTURE
STAGE MORPHOLOGY PANCREATIC FUNCTION DIAGNOSTICS
PAIN COMPLICATION
• Morphologic changes
detectable with imaging • Normal pancreatic
• Recurrent acute • EUS, ERP/MRP, CT scan,
Early • No complications procedure directed to endocrine and exocrine
attacks secretin
pancreatic parenchyma function
and ductal system
• Increasing
• Pseudocysts • Progressive morphologic • Impairment of • Transabdominal US,
number of
• Cholestasis changes detectable in pancreatic unction in ERP/MRP, EUS, CT, fasting
Moderate attacks and
• Segmental portal several imaging several degrees but blood glucose, oral glucose
increased
hypertension procedures rarely steatorrhea tolerance test
intensity
• Marked impairment of
• Pseudocysts • Transabdominal US,
• Decreasing pain pancreatic function,
• Cholestasis ERP/MRP, CT, FE-1, fasting
Advanced (burnout* of the • Calculi more often steatorrhea
• Segmental portal blood glucose, oral glucose
pancreas) than in other stages;
hypertension tolerance test
diabetes mellitus
Surgery Platinum. 1st ed. 2018. p. 396
• Diagnosis
o UGIS
o ERCP
o Ultrasound
o CT scan
*The presence of calcifications in the pancreas virtually establishes the
diagnosis

• Management
MANAGEMENT DESCRIPTION
• General: lifestyle changes, cessation of alcohol DUVAL’S CAUDAL PANCREATICOJEJUNOSTOMY
Figure 33-50. Schwartz’s Principles of Surgery. 11th ed. 2019
intake and smoking, diet modifications (small,
low-fat meals)
• Analgesia: NSAIDs, long and short acting
Medical narcotics
• Enzyme replacement: nonenteric and enteric-
coated preparations, at least 30,000 IU of
lipase per meal
• Antisecretory: somatostatin, octreotide
• Neuroablation (celiac plexus block,
Interventional
splanchnicectomy)
procedures for
• Desmopressin (endoscopic stenting, Puestow
pancreatic pain PUESTOW AND GILLESBY’S
procedure), resection (Whipple, Frey)
• Stricture dilatation, stone extraction, stent LONGITUDINAL PANCREATICOJEJUNOSTOMY
Figure 33-51. Schwartz’s Principles of Surgery. 11th ed. 2019
placement, shockwave lithotripsy, internal
Endoscopic
drainage (cystogastrostomy,
cystoduodenostomy or jejunostomy)
Surgery Platinum. 1st ed. 2018. p. 398
SURGICAL
DESCRIPTION
INTERVENTION
Drainage
Duval-Zollinger • Caudal pancreaticojejunostomy
• Longitudinal Roux en Y
pancreaticojejunostomy (invaginating
Puestow-
anastomosis between pancreas and jejunum
Gillesby
after amputating the tail of the pancreas and
opening the duct along its long axis) PARTINGTON-ROCHELLE LONGITUDINAL DOCHOTOMY
Figure 33-52. Schwartz’s Principles of Surgery. 11th ed. 2019
• Modification of the Puestow-Gillesby
Partington- procedural of longitudinal
Rochelle pancreaticojejunostomy that does not
include caudal pancreatectomy
Resectional
• 95% distal pancreatectomy (preserves the
rim of the pancreas in the
Fry and Child pancreaticoduodenal groove, along with its
associated blood vessels and distal common
bile duct)
Kaush-Whipple • Pancreaticoduodenectomy
Priestley • Total Pancreatectomy
Hybrid
• Duodenum-preserving pancreatic head KAUSCH-WHIPPLE PANCREATICODUODENECTOMY
Beger
resection (DPHR) Figure 33-54. Schwartz’s Principles of Surgery. 11th ed. 2019

• Local resection of the pancreatic head with


Frey and Smith
longitudinal pancreaticojejunostomy (LR-LPJ)
• Modification of the LR-LPJ procedure
(wider excavation of the pancreatic head is
Hamburg created in continuity with the dorsal
dochotomy followed by a single, side-to-
side pancreaticojejunostomy)
• Modification of the Berger procedure
(excavation of the central portion of the
Berne
head without formal division of the
pancreatic neck) HANS BEGER DUODENUM-PRESERVING
Surgery Platinum. 1st ed. 2018. p. 398 PANCREATIC HEAD RESECTION
Figure 33-55. Schwartz’s Principles of Surgery. 11th ed. 2019

TOPNOTCH MEDICAL BOARD PREP SURGERY MAIN DIGITAL HANDOUT BY LOUBOMIR ANTONIO, MD Page 78 of 101
For inquiries visit www.topnotchboardprep.com.ph or https://www.facebook.com/topnotchmedicalboardprep/
This handout is only valid for the September 2021 PLE batch. This will be rendered obsolete for the next batch since we update our handouts regularly.
TOPNOTCH MEDICAL BOARD PREP SURGERY MAIN DIGITAL HANDOUT BY LOUBOMIR ANTONIO, MD
For inquiries visit www.topnotchboardprep.com.ph or https://www.facebook.com/topnotchmedicalboardprep/
This handout is only valid for the September 2021 PLE batch. This will be rendered obsolete for the next batch since we update our handouts regularly.
A 40-year-old woman with severe chronic pancreatitis is scheduled to
undergo an operation, because other forms of treatment have failed.
The ultrasound shows no evidence of pseudocyst formation or
cholelithiasis and endoscopic retrograde cholangiopancreatogram
(ERCP) demonstrates dilated pancreatic ducts with multiple stricture
formation. Which operation is suitable to treat this condition?
(A) Pancreaticojejunostomy (Puestow procedure)
(B) Gastrojejunostomy
(C) Cholecystectomy
(D) Splenectomy
(E) Subtotal pancreatectomy
A Puestow procedure is indicated for the treatment of
symptomatic chronic pancreatitis patients with pancreatic ductal
obstruction and a dilated main pancreatic duct (>6mm). In this
operation, the pancreatic duct is slit open and anastomosed side-to-
FREY PROCEDURE side to the cut end of the divided jejunum with a Roux-en-Y
Figure 33-57c. Schwartz’s Principles of Surgery. 11th ed. 2019 anastomosis. Resection of the pancreas is reserved for patients
PSEUDOCYST OF THE PANCREAS without a dilated duct (<6 mm). In these cases, a distal
pancreatectomy is performed when the disease primarily involves the
• Most important causes: pancreatitis (75%), trauma (25%)
body and tail of the pancreas; whereas, a Whipple operation is
• Occasionally without demonstratable cause performed when the disease is confined to the head. Cholecystectomy,
• Pathology splenectomy and gastrojejunostomy choices are obviously wrong
o Fibrous wall surrounding a collection of pancreatic juice and (nothing to do with the pancreas).
necrotic or suppurative pancreatic tissue Dr. Cocos

o Unilocular an located in the lesser sac PANCREATIC TUMORS


o Fluid varies from the clear and colorless to brown or green
• Etiology
• Clinical Manifestation:
o Cigarette smoking
o Pain, fever and ileus appearing 2-3 weeks after an attack of
o Biliary lithiasis at CBD
pancreatitis or trauma to pancreas
o Pain radiating to the back • Pathology
o Jaundice o Cystadenoma and Cystadenocarcinoma
o Non-tender palpable mass § Rare with predilection for female
o Changing size of mass due to partial drainage § Comprise numerous cystic spaces from each other by thick
fibrous septa
• Diagnosis
§ Better prognosis than adenocarcinoma
o Continued elevation of serum amylase
§ Treatment: resection of mass
o CT scan is the most helpful examination
o Adenocarcinoma
o Ultrasound
§ 90% of cases of pancreatic malignancy
• Treatment
§ Usually at the head of the pancreas that compress the
o Surgery
pancreatic duct and CBD
§ To prevent complications such as infection, severe
§ Periampullary malignancy:
hemorrhage, rupture of adjacent viscus
- Pancreas (83%)
§ Pseudocyst that persist 4-6 weeks are unlikely to resolve
- Ampulla of Vater (10%)
spontaneously and should be treated surgically
o Types of surgery - Duodenum (3%)
§ External drainage - Distal CBD
§ Internal drainage (best treatment) – cystogastrostomy (can be § Metastasize to adjacent organs and regional LN
done endoscopically), cystoduodenostomy or cystojejunostomy • Clinical Manifestation
§ Small piece of cyst wall should be excised for histopathology to o Weight loss
rule out malignancy o Pain due to obstruction to the pancreatic duct and perineural
invasion
o Progressive jaundice - 75%
o Anorexia and weakness – 50%
o Pruritus
o Hepatomegaly
o Enlarged palpable gallbladder (Courvoisier gallbladder) not
tender and without fever is a reliable diagnostic criteria for
malignant choledochal obstruction
• Laboratory Work-up
o Blood exams: alkaline PO4, transaminases, B1 and B2
Acute Pancreatitis. Hopkinsmedicine.org
o UGIS
o CT scan
✔ GUIDE QUESTIONS o ERCP/ THC
A 24-year-old college student recovers from a bout of severe o FNAB: CT scan or ultrasound guided
pancreatitis. He has mild epigastric discomfort, sensation of bloating, • Treatment
and loss of appetite. Examination reveals an epigastric fullness that on
o Prior to surgery - improve nutrition, correct anemia, adequate
ultrasound is confirmed to be a pseudocyst. The swelling increases in
size over a 3-week period of observation. What should be the next step hydration
in management? o Whipple surgery
(A) Percutaneous drainage of the cyst § The only definitive and potentially curative treatment of
(B) Laparotomy and internal drainage of the cyst periampullary carcinoma
(C) Excision of pseudocyst
(D) Total pancreatectomy
(E) Administration of pancreatic enzymes
In most cases, pseudocyst resolves but if it enlarges, it may compress
the stomach anteriorly. An enlarging pseudocyst is an indication to
attempt percutaneous drainage. If unsuccessful, internal drainage (i.e.
cystogastrostomy) should be performed once the cyst wall is mature
enough. There is no need to excise the pseudocyst and total
pancreatectomy is too morbid a procedure for a benign entity.
Administration of pancreatic enzymes is done for chronic pancreatitis,
not for complications of acute pancreatitis.
Dr. Cocos
Whipple Procedure / pancreaticoduodenectomy
TOPNOTCH MEDICAL BOARD PREP SURGERY MAIN DIGITAL HANDOUT BY LOUBOMIR ANTONIO, MD Page 79 of 101
For inquiries visit www.topnotchboardprep.com.ph or https://www.facebook.com/topnotchmedicalboardprep/
This handout is only valid for the September 2021 PLE batch. This will be rendered obsolete for the next batch since we update our handouts regularly.
TOPNOTCH MEDICAL BOARD PREP SURGERY MAIN DIGITAL HANDOUT BY LOUBOMIR ANTONIO, MD
For inquiries visit www.topnotchboardprep.com.ph or https://www.facebook.com/topnotchmedicalboardprep/
This handout is only valid for the September 2021 PLE batch. This will be rendered obsolete for the next batch since we update our handouts regularly.

GLUCAGONOMA
• Presence of necrolytic migratory erythema
• Characteristics
o Serum glucagon 500pg/mL
o Usually at body and tail
o Metastatic at time of diagnosis
o Debulking as treatment
NON FUNCTIONING ISLET-CELL TUMOR
• Second most common islet-cell tumor
• Characteristics
o Elevated PP as marker
o Usually malignant
✔ GUIDE QUESTION
A 30-year-old male is admitted with frequent episodes of
hypoglycemia. Biochemical investigations confirmed an insulinoma.
Localization studies were carried out. ACT scan and magnetic
resonance imaging (MRI) of the abdomen failed to reveal a tumor in the
pancreas. An endoscopic ultrasound, however, localized a 2-cm
insulinoma in the tail of the pancreas. What should be the next step in
the management of this patient?
(A) Somatostatin receptor scintigraphy (SRS) to confirm the insulinoma
(B) Exploratory laparotomy and total pancreatectomy
(C) Distal pancreatectomy
(D) Whipple pancreaticoduodenectomy
WHIPPLE PROCEDURE (E) Enucleation of tumor
https://qrs.ly/23cl176 Most insulinomas are benign, solitary and small; therefore this can be
managed by simple enucleation. However, when the lesion is embedded
deep in the pancreatic tissue or is close to the main pancreatic duct,
✔ GUIDE QUESTION enucleation should be avoided and formal pancreatic resection (in this
A 66-year-old man with obstructive jaundice is found on ERCP to have case distal pancreatectomy or if in the head of the pancreas,
periampullary carcinoma. He is otherwise in excellent physical shape and pancreaticoduodenectomy) should be performed. Total pancreatectomy
there is no evidence of metastasis. What is the most appropriate treatment? is almost never required for the removal of insulinomas. Somatostatin
(A) Radical excision (Whipple procedure) where possible receptors are not always present on insulinoma cells, and, therefore, SRS
(B) Local excision and radiotherapy is less useful for localization of this tumor.
Dr. Cocos
(C) External radiotherapy
(D) Internal radiation seeds via catheter
(E) Stent and chemotherapy
SMALL INTESTINES
GROSS ANATOMY
Carcinoma of the head of the pancreas is treated with radical excision of
the head of the pancreas along with the duodenum. Most centers do not • Tubular structure about 6 m in adults
give irradiation routinely before or after surgery, because pancreatic • Regarded as the largest organ of the endocrine and immune
cancers do not respond well to radiotherapy. Endoscopically placed system (Schwartz) – due to the presence of neuroendocrine cells
stents alone are used only in palliative circumstances in patients with and presence of immune cells
limited life expectancy. See above QR code for discussion. J
Dr. Cocos
• Duodenum – most proximal and mostly retroperitoneal
• Jejunum – proximal 2/5 of jejunoileal segment
Palliation of Unresectable Pancreatic Adenocarcinoma
• Ileum - distal 3/5 of jejunoileal segment
• 80% of patients are not candidates for potentially curative
• Contains mucosal folds known as plicae circulares or valvulae
resection at the time of diagnosis
conniventes (valves of Kerckring)
• Major complications include:
o Biliary obstruction
o Gastric outlet obstruction
o Severe abdominal pain
• Main objectives; prevent and ameliorate patient suffering,
while ensuring optimal quality of life
INSULINOMA Figure 28-1. Schwartz’s Principles of Surgery. 11th ed. 2019
• Whipple Triad: MICROSCOPIC ANATOMY
o Symptomatic fasting hypoglycemia
o Serum glucose <50mg/dL
o Relief of symptoms with glucose administration
• Characteristics
o 90% benign, solitary and sporadic
o Evenly distributed throughout head, body and tail
o Simple enucleation as treatment EXCEPT if close to main
pancreatic duct and is more than 2 cm in size
GASTRINOMA
• Zollinger-Ellison Syndrome, can be part of MEN1 syndrome
• Presents as peptic ulcer disease
• 50% malignant
• Treatment: full thickness excision (duodenum), enucleation Figure 28-2. Schwartz’s Principles of Surgery. 11th ed. 2019

(pancreas), vagotomy (if tumor unresectable) EMBRYOLOGY

VIPOMA
• WDHA syndrome/VIPoma/Verner-Morrison syndrome
• Characteristics
o Usually located at the tail
o EUS as the most sensitive imaging modality
o Usually malignant
o Debulking as palliative treatment
TOPNOTCH MEDICAL BOARD PREP SURGERY MAIN DIGITAL HANDOUT BY LOUBOMIR ANTONIO, MD Page 80 of 101
For inquiries visit www.topnotchboardprep.com.ph or https://www.facebook.com/topnotchmedicalboardprep/
This handout is only valid for the September 2021 PLE batch. This will be rendered obsolete for the next batch since we update our handouts regularly.
TOPNOTCH MEDICAL BOARD PREP SURGERY MAIN DIGITAL HANDOUT BY LOUBOMIR ANTONIO, MD
For inquiries visit www.topnotchboardprep.com.ph or https://www.facebook.com/topnotchmedicalboardprep/
This handout is only valid for the September 2021 PLE batch. This will be rendered obsolete for the next batch since we update our handouts regularly.
Etiologies and Classification of Mechanical SBO
EXTRINSIC
Adhesions
• Congenital: Ladd or Meckel bands
• postoperative (most common)
• Postinflammatory (after pelvic inflammatory disease)
Hernias
• Abdominal wall
• Internal
• Incisional
Volvulus
External mass effect
• Abscess
• Annular pancreas
• Pancreatic pseudocyst
• Carcinomatosis
• Endometriosis
• Pregnancy
• SMA syndrome or Wilkie syndrome or Cast syndrome
• Tight fascial stoma opening
• Derived from endoderm INTRAMURAL
o Embryonic gut tube • Congenital (rare in adult)
§ Duodenum: foregut • Atresia, stricture, or stenosis
§ Jejunum and ileum: midgut • Web
o Initially connected with the yolk sac through the vitelline duct • Intestinal duplication
which later on obliterate (6th week) • Meckel diverticulum
• Mesodermally derived structures Inflammatory process
o Parietal and visceral peritoneum • Crohn disease
o Peritoneal cavity • Diverticulitis
AOG EVENT • Stricture from ischemia
5th wk AOG • Extracoelomic herniation • Radiation enteritis or stricture
• Medication induced (NSAIDs, KCI tablets)
• Retracted back into the abdominal cavity
Neoplasms
10th wk AOG • Bowel undergoes 270 degree counter-clockwise
• Primary intestinal or colon (malignant or benign)
rotation around the superior mesenteric artery
• Secondary (metastasis or carcinomatosis
Trauma (e.g. intramural hematoma)
SMALL BOWEL OBSTRUCTION (SBO) INTRALUMINAL
• Mechanical small bowel obstruction - most common surgical • Foreign bodies
disorder of the small intestines • Barium inspissation (colon)
• Bezoar
• Adhesions (75%)- most common cause
• Inspissated feces
• Gallstone ileus
• Meconium (cystic fibrosis)
• Parasites (Ascaris, Diphyllobothrium)
• Enterolith
• Intussusception
• Polypoid and exophytic lesions
Surgery Platinum. 1st ed. 2018. p. 289

CATEGORY CLASSIFICATION
Mechanism of Functional • Related to ineffective motility without any physical obstruction
obstruction Mechanical • Intestinal obstruction caused by a physical blockage of lumen
• Abrupt onset of symptoms
Acute
Duration of • Progressive & does not usually resolve without treatment
obstruction • Recurring symptoms with interval resolution
Chronic
• Usually seen in partial causes of obstruction
Partial • Intestinal lumen is narrowed but still permits the transit of some intestinal content partially
Extent of
• Lumen is totally obstructed
obstruction Complete
• None of the intestinal content can move distally
Proximal • Involve the pylorus, duodenum, and proximal jejunum
Location of
Intermediate • Involve the intestine from the mid-jejunum to the mid-ileum
obstruction
Distal • Obstruction arise in distal ileum, ileocecal valve, & even colon
• Bowel is occluded at a single point leading to proximal intestinal dilatation & distal intestinal
Simple
decompression without vascular compromise
Type of
• Bowel is occluded at two points, so that both the proximal and distal loops, as well as the bowel's mesentery
obstruction Closed-loop
are entrapped by a single constrictive lesion
Strangulated • Blood flow to the obstructed segment is compromised and tissue necrosis and gangrene are imminent
Surgery Platinum. 1st ed. 2018. p. 289
PATHOPHYSIOLOGY • Strangulated SBO
Obstruction o Abdominal pain disproportionate to PE findings
↓ o Tachycardia and leukocytosis
Gas and fluid accumulation o Marked acidosis and hyperkalemia

Rise in intraluminal/intramural pressure SBO Diagnosis
↓ • History and physical examination
Intestinal ischemia • Radiographs (abdominal series)
PRESENTATION • Upright chest radiograph
• Complete vs Partial SBO • Supine and Upright Abdominal x-rays
o Cardinal signs • TRIAD of radiographic findings in SBO (sensitivity 70-80%,
§ Vomiting specificity is low):
§ Obstipation o Dilated small bowel loops (>3cms)
§ Distention o Differential air-fluid levels
§ Crampy/colicky abdominal pain o Paucity of air in the colon
TOPNOTCH MEDICAL BOARD PREP SURGERY MAIN DIGITAL HANDOUT BY LOUBOMIR ANTONIO, MD Page 81 of 101
For inquiries visit www.topnotchboardprep.com.ph or https://www.facebook.com/topnotchmedicalboardprep/
This handout is only valid for the September 2021 PLE batch. This will be rendered obsolete for the next batch since we update our handouts regularly.
TOPNOTCH MEDICAL BOARD PREP SURGERY MAIN DIGITAL HANDOUT BY LOUBOMIR ANTONIO, MD
For inquiries visit www.topnotchboardprep.com.ph or https://www.facebook.com/topnotchmedicalboardprep/
This handout is only valid for the September 2021 PLE batch. This will be rendered obsolete for the next batch since we update our handouts regularly.
PHYSICAL EXAMINATION OF OBSTRUCTION • Of the patients successfully treated non-operatively, 85% to
• Vital signs, hydration status, abdominal 95% have substantial improvement in their symptoms within
inspection, auscultation, palpation, a search 48 hours
General • Contraindications to non-operative management include:
for potential hernia defects, and a rectal exam
examination
• Look closely for previous surgical incisions, o Suspected ischemia
including inguinal incisions o Large bowel obstruction
• More pronounced if the obstruction is located o Closed loop obstruction
Abdominal more distal o Strangulated hernia
distention • Distention may be absent in proximal small o Perforation
intestinal obstruction • Requires strict hemodynamic monitoring, serial abdominal
• May be hyperactive in the beginning radiographs (every 6 hours), and physical examination (every 3
• Later becomes absent, signaling intestinal hours)
fatigue & atony from long-standing
obstruction or the development of peritonitis Surgical
Bowel sounds
• Characteristic high-pitched metallic tinkling
• Varies according to the etiology of the obstruction
sounds ("water dripping into a large hollow
• Surgical procedures include:
container") indicative of dilated bowel with an
air-fluid interface o Adhesiolysis
• Slushing sound (heard through a stethoscope)
o Resection for tumors
after sudden movement of a patient, which o Reduction and repair of hernias
Succussion reflects gas and fluid in an obstructed organ • Assessment of bowel viability: color, peristalsis, marginal
splash • Usually present in SBO (regarded as an arterial pulsations, necrosis, perforations
important but often underappreciated sign of
bowel obstruction)
Surgery Platinum. 1st ed. 2018. p. 290

Radiographic findings in strangulated SBO:


• Thickened small bowel loops
• Mucosal ‘‘thumb-printing’’- indicate significant bowel edema
• Pneumatosis intestinalis- gas within the bowel wall
• Free peritoneal air (pneumoperitoneum)

SBO Diagnosis: Radiograms (Abdominal Series)


• False negative
o Closed-loop obstructions – bowel is filled with fluid but gas is
absent
• Proximal SBO
o (i.e., pyloric stenosis, annular pancreas, SMA syndrome)

SBO imaging: CT scan with water soluble contrast


• Limitation: in cases of partial SBO (sensitivity <50%)
• May reveal abscess, inflammatory processes, extraluminal
pathology resulting in obstruction, and mesenteric ischemia
• Can distinguish between ileus and mechanical SBO in
postoperative patients
• Preferred method in patients with history of abdominal
malignancy
• For Closed Loop Obstructions: Adapted from Schwartz’s Principles of Surgery. 10th ed. 2015

o Bowel is filled with fluid but gas is absent


o Characteristic CT findings (in closed loop): C- or U-shaped loop
of bowel with its mesenteric vessels converging toward the
point of constriction

SBO Imaging: Small Bowel Series / Enteroclysis


• Direct instillation of water-soluble contrast into the small
intestine through a tube placed in the duodenum
• Gold standard to differentiate partial from complete obstruction
but is not indicative of the etiology of obstruction
• Demonstrates a delay in passage of contrast & caliber change at
site of obstruction

SBO Initial Treatment


• NPO
• Aggressive fluid resuscitation: isotonic saline or lactated Ringer
solution
• Nasogastric tube (NGT) decompression
• Analgesics, antiemetics, and antibiotic prophylaxis
• Adequacy of resuscitation is judged by urine output & central
venous pressure

Nonsurgical (Conservative)
• 75% of partial SBO and 36% of complete SBO can be treated
non-operatively
• Provided there is no clinical deterioration and the patient shows
some evidence of improvement over the first 12-24 hours

Adapted from Schwartz’s Principles of Surgery. 11th ed. 2019


TOPNOTCH MEDICAL BOARD PREP SURGERY MAIN DIGITAL HANDOUT BY LOUBOMIR ANTONIO, MD Page 82 of 101
For inquiries visit www.topnotchboardprep.com.ph or https://www.facebook.com/topnotchmedicalboardprep/
This handout is only valid for the September 2021 PLE batch. This will be rendered obsolete for the next batch since we update our handouts regularly.
TOPNOTCH MEDICAL BOARD PREP SURGERY MAIN DIGITAL HANDOUT BY LOUBOMIR ANTONIO, MD
For inquiries visit www.topnotchboardprep.com.ph or https://www.facebook.com/topnotchmedicalboardprep/
This handout is only valid for the September 2021 PLE batch. This will be rendered obsolete for the next batch since we update our handouts regularly.
FEATURES AFFECTING SPONTANEOUS CLOSURE RATES
INTESTINAL FISTULAS FAVORABLE UNFAVORABLE
• Fistula: abnormal communication between two epithelialized • Continuity maintained • Complete disruption
surfaces • End fistula • Lateral fistula
• Most small bowel fistulas (75-80%) occur as a post-operative • No associated abscess • Associated abscess
complication (enterotomies, anastomotic leaks) • Healthy adjacent bowel • Diseased adjacent bowel
• May result in dehydration, electrolyte deficiency, malabsorption • Free flow distally • Distal obstruction
& malnutrition, infection, and skin irritation and excoriation • Duodenal stump • Lateral duodenal
• Jejunal • Ileal
Clinical Manifestations • Tract >2 cm • Tract <2 cm
• Internal fistulas: usually spontaneous and are difficult to • Defect <1 cm • Defect >1 cm
diagnose (high index of suspicion is necessary) • Patients with optimal • Patients with poor
• External fistulas: nutritional status nutritional status
o Easily recognized because drainage of enteric contents to the FACTORS THAT INHIBIT SPONTANEOUS
SUPPLEMENT
skin is obvious CLOSURE OF FISTULAS
o Febrile postoperative course with an erythematous wound Remember FRIEND (factors that inhibit spontaneous closure of
that begins to leak between 5th to 10th post-operative day at fistulas):
the wound or at the drains placed Foreign body within the fistula tract
Radiation enteritis
o May present with leukocytosis, prolonged ileus, and
Infection/Inflammation at the fistula origin
abdominal tenderness Epithelialization of the fistula tract
Neoplasm at the fistula origin
CATEGORY CLASSIFICATION Distal obstruction of the intestine
• Fistulous connection exists • Definitive therapy
Internal between two segments of bowel o If the fistula fails to close by 2-3 months, operative therapy
Fistula or between segment of intestine may be required
and another hollow viscus o Surgery should not take place until the patient is stable, not
Anatomic • A direct communication between septic, and in an adequate nutritional state
External
the intestine and the skin of the o Most favorable time to re-operate: either within 10 days of
Fistula
abdominal wall diagnosis or after 4 months
• Involve both internal and o The delay allows for adequate healing of any intra-abdominal
Mixed
external communication sepsis & for softening of adhesions, & a much easier operation
• Daily output does not exceed and hence, a better outcome
Low output
200mL • Healing
Moderate • Daily output ranges between 200 o Nutritional support (parenteral, enteral, or combined)
Physiologic
output and 500mL initiated early to prevent further nutritional losses and to
High replenish nutritional deficiencies
• Daily output exceeds 500mL
output
• Congenital ✔ GUIDE QUESTION
• Trauma A 69-year-old female is found to have an enterocutaneous fistula that
Etiologic • Infection arises from the proximal small intestine. Which of the following
• Perforation statements is TRUE concerning this fistula?
• Inflammation, radiation or tumor (A) If internal, it occurs mainly from iatrogenic causes
Surgery Platinum. 1st ed. 2018. p. 293 (B) It occurs more commonly after an anastomosis than
DIAGNOSIS AND MANAGEMENT spontaneously
(C) If internal, it always causes serious complications
• Stabilization (D) If external, it closes spontaneously in 10% of cases
o Accomplished in the first 24-48 hours (E) If external, it requires immediate closure in most cases
o Includes fluid resuscitation, control of infection, monitoring of
Most enterocutaneous fistula (80%) occur after bowel surgery (usually
fistula and urine output, and protection of the surrounding skin a leak from an anastomotic site). Other causes include infection,
• Investigation perforated peptic ulcer and inflammatory bowel diseases such as
o Takes place over the next 7 to 10 days Crohn's disease or ulcerative colitis. It may also develop from an
o Thorough evaluation of the GIT, definition of the anatomy of abdominal injury or trauma, such as a stabbing or gunshot. Crohn’s
the fistula, and identification of any complicating features such disease is the most common cause of internal small-bowel fistulas (A is
as abscess, stricture, or distal obstruction wrong), but neoplasia, lymphoma, and tuberculosis must be excluded.
• Fistulogram Internal fistula may be asymptomatic (C is wrong) or cause serious
malabsorption (proximal to distal fistulas) or infection (enterovesical
o Performed by inserting a small catheter through the drainage
fistulas). If external, it closes in most cases and therefore does not require
site into the fistula tract and then slowly injecting water- immediate closure (D and E choices are wrong).
soluble contrast under fluoroscopy guidance Dr. Cocos
o Performing the fistulogram first is prudent because contrast
from other imaging tests may make it difficult to interpret a SMALL BOWEL NEOPLASMS
fistulogram subsequently
EPIDEMIOLOGY
• CT Scan with Contrast
o Further anatomic details at the area of the fistula • Small bowel tumors (both benign and malignant) are uncommon
o Can show ongoing or unrecognized intraabdominal abscesses, • <5% of primary gastrointestinal malignancies arise from the
undrained fluid collections, as well as distal obstruction small intestine
• Diagnosis • Approximately 1/3 of primary small bowel neoplasms are
o CT with oral contrast benign and 2/3 are malignant
o Fistulogram • Small bowel neoplasms are less common in women than in men
• Decision • Most patients with small-bowel neoplasms present in their sixth
o The stage when different treatment options are considered to seventh decade of life.
o Timeline for conservative measures is determined • Except for adenocarcinoma, which has a predilection for the
o This is when the likelihood of spontaneous closure must be duodenum, malignant small bowel tumors are more common in
determined based on several factors the distal small bowel.
• Rates of spontaneous closure LOCATION OF TUMOR
o More than 90% in 1 month • 20% arise in the duodenum
o Less than 10% after 2 months • 30% in the jejunum
o None after 3 months • 50% in the ileum

TOPNOTCH MEDICAL BOARD PREP SURGERY MAIN DIGITAL HANDOUT BY LOUBOMIR ANTONIO, MD Page 83 of 101
For inquiries visit www.topnotchboardprep.com.ph or https://www.facebook.com/topnotchmedicalboardprep/
This handout is only valid for the September 2021 PLE batch. This will be rendered obsolete for the next batch since we update our handouts regularly.
TOPNOTCH MEDICAL BOARD PREP SURGERY MAIN DIGITAL HANDOUT BY LOUBOMIR ANTONIO, MD
For inquiries visit www.topnotchboardprep.com.ph or https://www.facebook.com/topnotchmedicalboardprep/
This handout is only valid for the September 2021 PLE batch. This will be rendered obsolete for the next batch since we update our handouts regularly.
RISK FACTORS MECKEL DIVERTICULUM
• Familial adenomatous • Neurofibromatosis • Most prevalent congenital anomaly of the GIT
polyposis (FAP) • Immunosuppression (IgA
• Most common true diverticulum
• Hereditary nonpolyposis deficiency, AIDS)
colon cancer (HNPCC) • Infection (Epstein- Barr virus,
• Usually found in the ileum within 100 cm from the ICV
• Gardner syndrome H. pylori) • 60% with heterotopic mucosa (gastric > pancreatic)
• Crohn disease • History of other primary cancer • “Rule of 2s”
• Peutz-Jeghers syndrome o 2% of the population
• Celiac disease o 2:1 male predominance
o 2 feet proximal to ICV
PRESENTATION o ½ of symptomatic are under 2 years old
o 2 types of mucosa: gastric and pancreatic
• No pathognomonic signs or symptoms
• Etiology: persistence of the vitelline/omphalomesenteric duct
• Malignant lesions: more symptomatic than benign lesions (e.g.,
abdominal pain, weight loss) PRESENTATION
• Benign tumors: more often present with acute hemorrhage as • Most often
the primary symptom or are identified as an incidental finding asymptomatic
on an imaging or during laparotomy • Children: bleeding
• Frequently present late in their course because of their ill- from ileal mucosal
defined symptoms ulceration
• Tumors are often diagnosed at the time of emergency surgical • Adults: intestinal
exploration for intestinal obstruction, perforation, or massive obstruction;
gastrointestinal hemorrhage inflammation mimics
appendicitis; may
DIAGNOSIS present as Littre
• Contrast radiography – small bowel series/Enteroclysis hernia
• Endoscopy DIAGNOSIS
• CT scan • Incidental finding during laparotomy
LESION MANAGEMENT • Technetium-99m pertechnetate scan: (+) only in the
presence of heterotopic gastric mucosa
• If <2cm: endoscopic polypectomy
Duodenal • If >2cm: transduodenal polypectomy or
adenoma segmental duodenal resection, or TREATMENT
pancreaticoduodenectomy • Incidental/Asymptomatic
Duodenal o Adult: Observe
• Endoscopic polypectomy
adenomas o Children: Diverticulectomy + Appendectomy
• Surveillance endoscopy at 6 month
in patients with • Symptomatic
interval, then annually
FAP o Diverticulectomy (wedge resection)
• Pancreaticoduodenectomy for proximal § Diverticulitis
lesions (right of superior mesenteric § Narrow base
Duodenal
artery, SMA) o Segmental resection
adenocarcinoma
• Segmental resection for distal duodenal § Bleeding
lesions (left of SMA) § Tumor
Jejunal or ileal • Segmental resection with 5 cm of tumor- § Wide base
tumors free proximal & dismal margins § Inflamed and perforated base
Localized small • Wide en bloc resection (includes
bowel carcinoid adjacent mesentery & lymph nodes)
Metastatic MESENTERIC ISCHEMIA
• Debulking surgery ACUTE
carcinoids
• Preoperative somatostatin or octreotide • Arterial embolus (most common)
• Debulking including resection of hepatic o 95% with cardiac disease
metastases o From left atrial thrombi
• Loperamide, diphenoxylate, or o Lodges to SMA distal to middle colic
Carcinoid cyproheptadine (a serotonin receptor • Arterial thrombosis
syndrome antagonist) for diarrhea o With atherosclerotic disease
• Type 1 and 2 histamine receptor o Thrombus along proximal SMA near origin
antagonists for the flushing • Vasospasm
• Albuterol and aminophylline for o Seen in critically ill patients receiving vasopressors
bronchospasm and wheezing
• Venous thrombosis
• Wide local excision of the primary
Localized or o Affects SMV in 95%
tumor with in continuity resection of
resectable GIST o Primary
adherent organs
o Secondary
Unresectable or
• Imatinib (Gleevec) • Acute mesenteric ischemia
metastatic GIST
• Palliative resection or bypass (bowel o Leads to full-thickness infarction in 6 hours
Advanced disease
diversion) CHRONIC
• Primary chemotherapy (CHOP regimen) • Develops insidiously
Small bowel
• Segmental resection for tumor
lymphoma • Rarely leads to intestinal infarction
complications
Surgery Platinum. 1st ed. 2018. p. 298 • Results from atherosclerotic lesions in the main splanchnic
arteries (celiac, SMA, and IMA)
SURGICAL OUTCOMES Diagnosis
5-yr survival
• Arterial: CT scan/Angiography
Duodenal adenocarcinoma 50-60%
• Venous: US duplex scan
Jejunum/ileal adenocarcinoma 5-30% (worst)
Localized carcinoid Treatment 75-95% (best)
Metastatic carcinoid 19-54%
• Standard treatment – surgical revascularization
Lymphoma 20-40%
• <12 hours – thrombolytics
GIST 35-60%
• Nonocclusive mesenteric ischemia (NOMI) – vasodilator
(papaverine) infusion
• Venous thrombosis – anticoagulation
TOPNOTCH MEDICAL BOARD PREP SURGERY MAIN DIGITAL HANDOUT BY LOUBOMIR ANTONIO, MD Page 84 of 101
For inquiries visit www.topnotchboardprep.com.ph or https://www.facebook.com/topnotchmedicalboardprep/
This handout is only valid for the September 2021 PLE batch. This will be rendered obsolete for the next batch since we update our handouts regularly.
TOPNOTCH MEDICAL BOARD PREP SURGERY MAIN DIGITAL HANDOUT BY LOUBOMIR ANTONIO, MD
For inquiries visit www.topnotchboardprep.com.ph or https://www.facebook.com/topnotchmedicalboardprep/
This handout is only valid for the September 2021 PLE batch. This will be rendered obsolete for the next batch since we update our handouts regularly.
✔ GUIDE QUESTION
A 64-year-old woman with a known history of cardiac disease is admitted Air in the bowel wall or pneumatosis intestinalis is a sign of possible
to the hospital with severe abdominal pain. Her blood pressure is 150/95 bowel ischemia. Air in the biliary tree (pneumobilia) can be caused by
mm Hg, and her pulse rate is 84 beats per minute (bpm). There are previous instrumentation, biliary enteric fistula etc. Air below the left
minimal signs of intravascular depletion. The possibility of small-bowel diaphragm is normal (magenblase or stomach bubble), however air
infarction is characterized by which of the following? below the right diaphragm is indicative of pneumoperitoneum and
(A) The stack-of-coins sign possible bowel perforation. Stack of coins sign is indicative of small
(B) Marked distention of loops of bowel bowel hematoma, not necessarily infarction.
(C) Air in the biliary tree Dr. Cocos

(D) Air in the bowel wall (intramural)


(E) Air below the left diaphragm
DISORDER MANIFESTATIONS DIAGNOSIS MANAGEMENT
• Abdominal X-ray: pigtail sign in • IV hydration, NG suction
mid-gut volvulus (bowel loops • Ladd procedure
• Intermittent vomiting, spiraling about the axis of the o Untwist the Bowels/derotation
Malrotation abdominal distention and mesenteric vessels) o Divide the Ladd bands
tenderness, Melena • UGIS = Abnormal C-loop of Duodenum o Incidental Appendectomy and
• Barium Enema = Cecum shown at rearrange the intestine inside
the RUQ o Bowel resection if necrosis is present
• Abdominal ultrasound
• Intermittent, colicky
o Pseudokidney Sign
abdominal pain, vomiting,
(Superimposed Hypoechoic and • Air then hydrostatic reduction- Barium
Lethargy, Currant-jelly
Hyperechoic Layers) enema
stools, sausage-shaped
o Donut/Target Sign (Two Rings • Laparotomy: reduction and assessment
Intussusception mass, Hyperactive bowel
of LOW Echogenicity separated of bowel viability
sounds,
by a HYPERECHOIC Ring) • Possible segmental resection and
• (+) Dance sign: absence of
• Abdominal X-ray primary anastomosis
bowel in the right lower
• Contrast Enema
quadrant
o Coiled spring sign
Intestinal atresia
Types
• I: Mucosal atresia with intact
bowel wall and mesentery
• II: Blind end separated by a
cord
• IIIA: Blind end separated by • Resection of proximal bulbous bowel
a V-shaped defect • Feeding intolerance, and atretic segment and primary end-
• Prenatal US: polyhydramnios
• IIIB: Apple-peel atresia Maternal polyhydramnios, to-end anastomosis
• Plain X-Rays = Dilated bowel
• IV: Multiple atresia (string Bilious emesis, Abdominal • Exteriorization of the bowel
loops with Differential air-fluid
of sausages) distention, Non-passage of (enterostomy) - when the vascular
levels
meconium in the first day integrity is in question or in severe
• Barium Enema: Microcolon
of life peritonitis, either a jejunostomy or
ileostomy or both may be performed

• Abdominal X-ray: soap bubble


• Feeding intolerance, sign or ground-glass • Hydration, Hyperosmolar, water-
Bilious emesis, Family appearance, eggshell pattern, soluble contrast enema, Ileostomy with
Meconium ileus
history of cystic fibrosis, gas filled loops mucus fistula, Ileostomy takedown after
Abdominal distention, • Barium enema: shows microcolon 2 to 3 weeks
and inspissated meconium

MALROTATION AND MIDGUT VOLVULUS

Ladd’s Procedure
Steps: 1) counterclockwise detorsion of the bowel, 2) surgical division
of Ladd's bands, 3) widening of the small intestine's mesentery, 4) appendectomy
(to avoid confusion in diagnosing appendicitis in the future), and 5) reorientation
of the small bowel on the right and the cecum and colon on the left.
Dr. Cocos

TOPNOTCH MEDICAL BOARD PREP SURGERY MAIN DIGITAL HANDOUT BY LOUBOMIR ANTONIO, MD Page 85 of 101
For inquiries visit www.topnotchboardprep.com.ph or https://www.facebook.com/topnotchmedicalboardprep/
This handout is only valid for the September 2021 PLE batch. This will be rendered obsolete for the next batch since we update our handouts regularly.
TOPNOTCH MEDICAL BOARD PREP SURGERY MAIN DIGITAL HANDOUT BY LOUBOMIR ANTONIO, MD
For inquiries visit www.topnotchboardprep.com.ph or https://www.facebook.com/topnotchmedicalboardprep/
This handout is only valid for the September 2021 PLE batch. This will be rendered obsolete for the next batch since we update our handouts regularly.
• The “double bubble sign” is seen: • Resection of up to 50% of the small bowel is well tolerated, with
o Duodenal obstruction SBS developing in patients with less than one-third
o Causes: (approximately 200 cm) of the remaining small intestine
§ duodenal atresia (most common) • Total parenteral nutrition (TPN) dependence:
§ duodenal web Adults lacking a functional • Less than 100 cm of residual
§ annular pancreas colon small intestine
Adults with an intact and • Less than 60 cm of residual
functional colon small intestine
Infants with short bowel • Less than 10 cm of residual
syndrome small intestine

Classification

FEATURES
• Most common and lethal gastrointestinal disorder affecting a • Small bowel resection with high-output
preterm neonate – necrotizing enterocolitis Type 1 SBS jejunostomy (end jejunostomy)
o Characteristics: • Most challenging to manage
§ prematurity and enteral alimentation increases the risk • Small bowel resection with partial colon resection
§ usually affects the terminal ileum and resulting enterocolonic anastomosis
Type 2 SBS
o Bell Stages (jejunocolic)
§ Stage 1- NEC scare • Most common type
§ Stage 2- Established NEC • Small intestine resection with small bowel
§ Stage 3- Advanced NEC anastomosis and intact colon (jejunoileal)
Type 3 SBS
o (+) pneumoperitoneum: indication for surgery • Best tolerated type with the most adaptive
potential
✔ GUIDE QUESTION
A previously healthy 2 1/2-year-old is admitted by the pediatrician for
• Jejunal resections are better tolerated than ileal resections
bilious vomiting and severe abdominal pain. Despite reports of the child o Ileum secretes peptide YY, absorbs B12, bile acids, and fatty acids
being inconsolable you find him sleeping very soundly in his mother’s
arms. His abdomen is soft with a suggestion of right upper quadrant Adaptation
fullness. He has heme-negative, soft stool in the rectum. He is afebrile and • Begins within 12 to 24 hours after resection and continues for 1
his white blood cell (WBC) is 7800. Abdominal x-ray shows dilated loops to 2 years.
of small bowel. You consider a diagnosis of intussusception;
(A) To be unlikely in the absence of “currant jelly” stools • Adaptive changes;
(B) And arrange prompt surgical exploration and reduction o elongation and dilation of the small bowel
(C) And order an ultrasound o hyperplasia of the mucosal epithelium
(D) And order an air contrast enema o increase in villous height, crypt depth, cell proliferation,
This is a typical presentation for intussusception. Between episodes of colicky and enzyme activity
pain, the exhausted child may rest comfortably. Although an ultrasound can o slowing in the rate of bowel transit
screen for intussusception (the presence of mesenteric lymph nodes within the • Glucagon-like peptide-2 (GLP-2) – most potent intestinotrophic
lumen of the intusscipiens is a highly specific ultrasound finding and a typical hormone
ileocolic intussusception has the appearance of a peripheral hypoechoic ring,
the target sign, with central echogenicity, the pseudokidney sign), a negative Medical management
study would not be definitive. An air contrast enema (or barium) with no more
than 120mmHg pressure would provide a definitive diagnosis and possibly PHASE MANAGEMENT
therapeutic as well. This should certainly be tried before subjecting a child to • Control and treatment of sepsis, maintenance of
surgical exploration (unless there are already signs of peritonitis). Currant jelly Acute fluid and electrolyte balance, and nutritional
stools are observed in only 50% of intussception cases. support (PN)
Dr. Cocos • Begin enteral nutrition
SHORT BOWEL SYNDROME Adaptation
• Goals: maximize intestinal adaptation, attempting
to discontinue PN, and preventing complications
• Anatomic definition: less than 200 cm of residual bowel in (1 to 2 years)
both of the underlying disease process and PN
adult patients
support
• Goals: provide adequate caloric nutrition, prevent
Common Etiologies: Maintenance complications, and provide effective medical
• Adults – multiple resections (Crohn disease, radiation enteritis, therapy for underlying disease
adhesions, malignancy), and massive resections due to mesenteric
ischemia and trauma Surgical management
• Children - necrotizing enterocolitis and congenital intestinal • Autologous Intestinal Reconstruction Surgery (AIRS)
anomalies (intestinal atresia) A. Improve Intestinal Function and Motility/Maximize Remnant
o Avoid resection
Pathophysiology o Restore continuity
• Massive intestinal loss result in loss of absorptive surface area o Recruit bypassed intestinal segments
o Relieve obstruction due to adhesions and strictures
and an increase in intestinal transit
o Slow intestinal transit
• Normal small bowel length = 300 to 600 cm (adults) and 200 to
o Taper dilated bowel segments
250 cm (full-term infants at birth)
TOPNOTCH MEDICAL BOARD PREP SURGERY MAIN DIGITAL HANDOUT BY LOUBOMIR ANTONIO, MD Page 86 of 101
For inquiries visit www.topnotchboardprep.com.ph or https://www.facebook.com/topnotchmedicalboardprep/
This handout is only valid for the September 2021 PLE batch. This will be rendered obsolete for the next batch since we update our handouts regularly.
TOPNOTCH MEDICAL BOARD PREP SURGERY MAIN DIGITAL HANDOUT BY LOUBOMIR ANTONIO, MD
For inquiries visit www.topnotchboardprep.com.ph or https://www.facebook.com/topnotchmedicalboardprep/
This handout is only valid for the September 2021 PLE batch. This will be rendered obsolete for the next batch since we update our handouts regularly.
B. Increase Absorptive Area • Most common location of the tip of the appendix:
§ Intestinal lengthening procedures retrocecal position
§ Longitudinal intestinal lengthening and tailoring • Immunologic organ (gut-associated lymphoid
Main
(LILT/Bianchi) tissue – GALT)
function
§ STEP • Secretes immunoglobulin A
§ Isolated bowel segment (Kimura/IOWA procedure)
• Intestinal transplantation

Frank H. Netter Atlas of Human Anatomy. 7th ed. 2019

ACUTE APPENDICITIS
• Inflammation of the appendix
o Incidence: most common acute surgical abdomen
o Most frequent in the 2nd and 3rd decade of life
o Rare in very young
o M/F ratio: 1:1 prior to puberty, 2:1 at puberty
• Etiology and pathogenesis
o Obstruction of the lumen→ increase intraluminal pressure
o Fecalith – the most common cause
o Hypertrophy of the lymphoid tissue
o Inspissated barium
o Vegetable and fruit seeds
o Intestinal worms (ascaris)

• Sequence of events following occlusion of the appendiceal lumen:


Closed loop obstruction (primary block)

Continuing normal secretion of appendiceal mucosa
SHORT BOWEL SYNDROME ↓
https://qrs.ly/bgcl1bq Rapid distention (stimulation of visceral nerve pain fibers)

Rapid bacterial multiplication
✔ GUIDE QUESTION ↓
Capillaries and venules occluded
Vitamin B12 deficiency can occur after
a. Gastrectomy ↓
b. Gastric bypass Vascular engorgement & congestion (reflux nausea and vomiting)
c. Ileal resection ↓
d. ALL OF THE ABOVE Inflammatory process involves serosa of the appendix (stimulation of
Vitamin B12 (cobalamin) malabsorption can result from a variety of somatic nerve)
surgical manipulations. The vitamin is initially bound by saliva-derived ↓
R protein. In the duodenum (bypassed during gastric bypass, choice B), RLQ pain
R protein is hydrolyzed by pancreatic enzymes, allowing free cobalamin ↓
to bind to gastric parietal cell-derived intrinsic factor (intrinsic factor is Absorption of necrotic tissue & bacterial toxin (fever, tachycardia, &
decreased during gastrectomy, choice A). The cobalamin-intrinsic leukocytosis)
factor complex is able to escape hydrolysis by pancreatic enzymes, ↓
allowing it to reach the terminal ileum (lost during ileal resection, Progressive distention
choice C), which expresses specific receptors for intrinsic factor. ↓
Subsequent events in cobalamin absorption are poorly characterized, Infarction (compromise blood supply)
but the intact complex probably enters enterocytes through ↓
translocation. Because each of these steps is necessary for cobalamin Perforation
assimilation, gastric resection, gastric bypass and ileal resection can
each result in Vitamin B 12 insufficiency.
• Clinical manifestation
o Right lower quadrant pain – prime symptom
Dr. Cocos
o Anorexia – most consistent clinical feature
o Vomiting – 75% of patients
APPENDIX • Sequence of symptoms
ANATOMY o Anorexia – 95%; 1st symptom
Length of vermiform appendix • 6-9 cm o Abdominal pain – 2nd symptom
Outer diameter of vermiform appendix • 3-8 mm o Vomiting
Luminal diameter of vermiform appendix • 1-3 mm o Murphy triad: abdominal pain + vomiting + fever
• Signs
• The three taenia coli (libera, omentalis, mesocolic) meet at the: o Vital signs:
base of the appendix and rectosigmoid junction § No change → uncomplicated
Arterial • Appendiceal artery ← posterior cecal branch of o Classic:
supply the ileocolic artery § Direct tenderness at McBurney point (lateral 1/3 from ASIS
• Usually found at the posteromedial wall of cecum to umbilicus)
Locations § Lanz point: right one-third point of the interspinal line
(just below the ileocecal valve of Bauhin)
§ Kummel point: right side below the umbilicus (approximately 2 cm)
TOPNOTCH MEDICAL BOARD PREP SURGERY MAIN DIGITAL HANDOUT BY LOUBOMIR ANTONIO, MD Page 87 of 101
For inquiries visit www.topnotchboardprep.com.ph or https://www.facebook.com/topnotchmedicalboardprep/
This handout is only valid for the September 2021 PLE batch. This will be rendered obsolete for the next batch since we update our handouts regularly.
TOPNOTCH MEDICAL BOARD PREP SURGERY MAIN DIGITAL HANDOUT BY LOUBOMIR ANTONIO, MD
For inquiries visit www.topnotchboardprep.com.ph or https://www.facebook.com/topnotchmedicalboardprep/
This handout is only valid for the September 2021 PLE batch. This will be rendered obsolete for the next batch since we update our handouts regularly.
• In retrocecal appendix: abdominal findings are less striking FEATURE SCORE INTERPRETATION
(flank maybe the most tender part) Migratory RLQ pain 1 • Score 9-10:
• Special signs on physical examination Anorexia 1 almost certain
• Referred pain or feeling of distress in Nausea or vomiting 1 • Score 7-8:
Aaron sign epigastrium or precordial region on continued RLQ (right iliac fossa) high likelihood of
2
firm pressure over the McBurney point tenderness appendicitis
• Sharp pain elicited by pinching appendix Rebound tenderness • Score 4-6:
1
Bassler sign between thumb of examiner and iliacus muscle right iliac fossa consider further
(chronic appendicitis) Elevation in temperature imaging
1
Blumberg (>36.3°C) • Score <3:
• Transient abdominal wall rebound tenderness Leukocytosis (>10 x 109 cells/L) 2 low likelihood of
sign
• Exacerbation of pain when the uterus is shifted Shift to the left of neutrophils 1 appendicitis
Surgery Platinum. 1st ed. 2018. p. 304
Bryan sign to the right side (acute appendicitis in
pregnancy) Appendicitis Inflammatory Response Score
Cutaneous • In area supplied by spinal nerves on the right PROBABILITY OF
hyperesthesia (T10, T11, T12) FEATURE SCORE
APPENDICITIS
Dunphy sign • Increased abdominal pain on coughing Vomiting 1
Kocher • Migration of pain from the umbilical region to RLQ (right inferior fossa)
1
(Kosher) sign the right iliac region pain
• RLQ pain on dropping from standing on toes to • Light 1 • Score 9-12:
Markle sign Rebound
heels • Medium 2 high probability –
tenderness
• Grimace when examiner performs a firm swish • Strong 3 explore
Massouh sign with index and middle finger across abdomen Elevated temperature • Score 5-8:
from epigastrium to right iliac fossa 1
(>38.5 C) intermediate –
• Tenderness in RLQ increases when patient • 70-84% 1 observe or diagnostic
Rosenstein PMN:
moves from supine position to a recumbent • >85% 2 lap
sign
posture on left side • 10-49 x 109 • Score 0-4:
• Pain at RLQ when palpatory pressure exerted 1 low probability –
Rovsing sign cells/L
at LLQ WBC count outpatient follow up
• >15 x 109
• Patient lies on left side, examiner then slowly 2
cells/L
Iliopsoas sign extends right thigh, stretching the iliopsoas • 10-49 g/L 1
muscle (positive if extension produces pain) CRP
• >50 g/L 2
• Performed by passive internal rotation of the Surgery Platinum. 1st ed. 2018. p. 305
Obturator flexed right thigh with the patient in supine IMAGING STUDIES
sign position (positive if with hypogastric pain on • Ultrasound
stretching the obturator internus muscle) o Inexpensive
• Increased abdominal muscle ton on o Does not require contrast
Summer sign exceedingly gentle palpation of right iliac fossa o Applicability among pregnant
(early appendicitis) o Signs of appendicitis– wall thickening, periappendiceal fluid
Ten Horn • Pain caused by gentle traction of spermatic
Sign cord
REAL-TIME US SIGNS OF ACUTE APPENDICITIS
• Course Direct signs • Indirect signs
o Progression to perforation is not predictable • Non-compressibility of the • Free fluid surrounding
o Spontaneous resolution is common appendix appendix
o Perforation: appendix might • Local abscess formation
o Perforation more common in the very young (< 5 years) and very
be compressible • Increased echogenicity of local
old (> 65 years)
• Diameter of the appendix > 6 mm mesenteric fat
o Rupture incidence is higher in the pediatric and geriatric age
• Single wall thickness ≥ 3 mm • Enlarged local mesenteric
groups
• Target sign: lymph nodes
o Hypoechoic fluid-filled • Thickening of the peritoneum
• Laboratory findings lumen • Signs of secondary small
o Urinalysis – to rule out UTI, bacteriuria generally not seen o Hypoechoic bowel obstruction
o Radiography – radiopaque fecalith present in the RLQ is nearly mucosa/submucosa
always associated with gangrenous acute AP o Hypoechoic muscularis layer
o Chest x-ray – to rule out right lower lobar pneumonia • Appendicolith: hyperechoic
with posterior shadowing
CLINICAL SCORING SYSTEMS • Color Doppler and contrast-
Alvarado Score enhanced US:
Feature Score o Hypervascularity in early
Migration of pain 1 stages of AA
o Hypo- to avascularity in
Anorexia 1
abscess and necrosis
Nausea 1
Tenderness in RLQ 2 • CT Scan criteria
Rebound pain 1 o Enlarged appendix >6mm in diameter
Elevated Temperature 1 o Appendiceal wall thickness >2mm (target sign)
Leukocytosis 2 o Periappendiceal inflammation (fat stranding)
Shift of WBC count to left 1 o (+) fecalith
TOTAL 10 o The presence of the arrowhead sign (thickened cecum
funneling contrast toward the appendiceal orifice)
• Accuracy of preoperative
diagnosis should be > 85%
• Rupture incidence is higher
in the pediatric and
geriatric age groups

Arrowhead sign

TOPNOTCH MEDICAL BOARD PREP SURGERY MAIN DIGITAL HANDOUT BY LOUBOMIR ANTONIO, MD Page 88 of 101
For inquiries visit www.topnotchboardprep.com.ph or https://www.facebook.com/topnotchmedicalboardprep/
This handout is only valid for the September 2021 PLE batch. This will be rendered obsolete for the next batch since we update our handouts regularly.
TOPNOTCH MEDICAL BOARD PREP SURGERY MAIN DIGITAL HANDOUT BY LOUBOMIR ANTONIO, MD
For inquiries visit www.topnotchboardprep.com.ph or https://www.facebook.com/topnotchmedicalboardprep/
This handout is only valid for the September 2021 PLE batch. This will be rendered obsolete for the next batch since we update our handouts regularly.
✔ GUIDE QUESTIONS
A 12-year-old boy complains of pain in the lower abdomen (mainly on the ✔ GUIDE QUESTION
right side). Symptoms commenced 12 hours before admission. He had A 79-year-old man has had abdominal pain for 4 days. An operation is
noted anorexia during this period. performed, and a gangrenous appendix is removed. The stump is
Examination revealed tenderness in the right iliac fossa, which was inverted. Why does acute appendicitis in elderly patients and in children
maximal 1 cm below Mc Burney point. In appendicitis, where does the have a worse prognosis?
pain frequently commence? (A) The appendix is retrocecal
(A) In the right iliac fossa and remains there (B) The appendix is in the preileal position
(B) In the back and moves to the right iliac fossa (C) The appendix is in the pelvic position
(C) In the rectal region and moves to the right iliac fossa (D) The omentum and peritoneal cavity appear to be less efficient
(D) In the umbilical region and then moves to the right iliac fossa in localizing the disease in these age group
(E) In the right flank (E) The appendix is longer in these age groups
The pain in appendicitis starts at the umbilical region (visceral pain) The omentum and peritoneal cavity seem to be less efficient in localizing
brought about by the distention of the appendiceal lumen, migrating to the disease in these age groups. Also, the often-atypical presentation and
the right lower quadrant (somatic pain) brought about by appendiceal delay in seeking medical help in elderly have been associated with delay
wall ischemia. in diagnosis and treatment resulting in high morbidity and mortality
Dr. Cocos
rates. In children, the delay in the diagnosis of acute appendicitis has
On examination, patients presenting with appendicitis typically show been attributed to nonspecific presentations, overlap of symptoms with
maximal tenderness over which of the following? many other common childhood illnesses, together with inability child to
(A) Inguinal region express and difficult abdominal examination in this age group.
(B) Immediately above the umbilicus Dr. Cocos
(C) At a point between the outer one-third and inner two-thirds of a
line between the umbilicus and the anterior superior iliac spine MANAGEMENT
(D) At a point between the outer two-thirds and inner one-third of a • Operative treatment of presumed
line between the umbilicus and the anterior superior iliac spine Uncomplicated uncomplicated appendicitis – remains as the
(E) At the midpoint of a line between the umbilicus and the anterior appendicitis standard of care
superior iliac spine • Non-operative management – may be considered
• Immediate appendectomy
• Non-operative management – may be
Complicated
considered for confined abscess/phlegmon,
appendicitis
limited peritonitis – antibiotics, fluids, bowel
rest, percutaneous drainage
• Laparoscopic appendectomy – for early
Appendicitis in appendicitis
the young • Urgent laparoscopic appendectomy – for
complicated appendicitis
• Laparoscopic appendectomy – safe and
Appendicitis in
This is known as the McBurney’s point, the location of the appendiceal might allow patients to reduce pain and their
older adults
base (not tip). hospital stay
Dr. Cocos
A 28-year-old man is admitted to the emergency department complaining
of pain in the umbilical region that moves to the right iliac fossa. Which is
a corroborative sign of acute appendicitis?
(A) Referred pain in the left side with pressure on the right (Rovsing)
sign
(B) Increase of pain with testicular elevation
(C) Relief of pain in lower abdomen with extension of thigh
(D) Relief of pain in lower abdomen with internal rotation of right thigh
(E) Hyperesthesia in the right lower abdomen
There is hyperesthesia in the cutaneous area supplied by spinal nerves
T10-T12. Choice A is not Rovsing sign, it should be referred pain in the
right side with pressure on the left. The rest of the choices are not signs OPEN APPENDECTOMY INCISIONS
of appendicitis. Review the table above re: special signs of physical
examination for appendicitis. J • If appendicitis is not found:
Dr. Cocos o Cecum and mesentery should be inspected
APPENDICITIS IN THE YOUNG o Retrograde evaluation of the small bowel
o Look for Crohn or Meckel diverticulitis
• Gangrene & rupture occur earlier during the course of acute AP
o Inspect reproductive organs (females)
o Inability to give accurate history
o Extend the incision if pus or bilious fluid is encountered
o Diagnostic delays
(Valentino appendicitis-RLQ pain from perforated peptic ulcer)
o High frequency of GI distress
o Underdeveloped greater omentum INCIDENTAL APPENDECTOMY
• Negative appendectomy rate = 25% • Indications:
• Perforation rate = 45% o Children about to undergo chemotherapy
o Disabled individuals
APPENDICITIS IN THE PREGNANCY o Crohn disease (cecum must be healthy)
• Most common extrauterine surgical emergency o Individuals about to travel to remote places
• Mostly in the first and second trimesters o Major operations such as urinary bowel diversions
• Negative appendectomy rate = 25% (2nd trimester) • Routinely performed in Ladd procedure
• Consider when there is new onset abdominal pain
TUMORS OF THE APPENDIX
• Imaging of choice: ultrasonography
CARCINOID
• Incidence of fetal loss = 4%
• Risk of earlier delivery = 7% • Most common site of
*Pregnancy does not alter the location of the appendiceal base more than 2cms GI carcinoid
from McBurney point • Best prognosis
o Usually small, firm,
APPENDICITIS IN ELDERLY circumscribed,
• Difficult diagnostic problem yellow brown tumor
o Atypical presentation o Location: usually
o Expanded differential diagnosis located at the tip
o Communication difficulty o Malignant potential
• Perforation rate = 50-70% related to size
• Higher morbidity and mortality
TOPNOTCH MEDICAL BOARD PREP SURGERY MAIN DIGITAL HANDOUT BY LOUBOMIR ANTONIO, MD Page 89 of 101
For inquiries visit www.topnotchboardprep.com.ph or https://www.facebook.com/topnotchmedicalboardprep/
This handout is only valid for the September 2021 PLE batch. This will be rendered obsolete for the next batch since we update our handouts regularly.
TOPNOTCH MEDICAL BOARD PREP SURGERY MAIN DIGITAL HANDOUT BY LOUBOMIR ANTONIO, MD
For inquiries visit www.topnotchboardprep.com.ph or https://www.facebook.com/topnotchmedicalboardprep/
This handout is only valid for the September 2021 PLE batch. This will be rendered obsolete for the next batch since we update our handouts regularly.
ADENOCARCINOMA LYMPHOMA
• Mucinous adenocarcinoma • Extremely uncommon, can presents as appendicitis
• Signet ring carcinoma (rarest, worst survival) • CT scan findings
• Treatment: right hemicolectomy o Appendiceal diameter > 2.5cm
o Surrounding soft tissue thickening
MUCOCELE • Treatment
• Cystic dilatation of the appendix containing mucoid material o Confined to the appendix: appendectomy
o Benign type – from non-inflammatory occlusion of the proximal o With cecal and mesoappendix involvement: right
lumen of the appendix hemicolectomy
o Malignant type – cystadenocarcinoma
COLON, RECTUM, ANUS
• Treatment COLON
o Appendectomy + wide resection of the mesoappendix and all
appendiceal lymph nodes GROSS ANATOMY
o Collection and cytologic examination of intraperitoneal mucus • Colon has a total length of 1-1.5 meters
• Special features:
PSEUDOMYXOMA PERITONEI o Haustrations
• Diffuse collection of gelatinous fluid and mucinous implants on o Plicae semilunares (vs plicae semicirculares of small
peritoneal surfaces and omentum intestine)
• More common in women – seen in appendiceal and ovarian tumors o Appendices epiploicae
o Three bands of longitudinal muscle called taeniae coli
• Abdominal pain, distention, mass
(taenia libera, taenia mesocolica, taenia omentalis)
• Diagnosis: CT scan is preferred
• Treatment: surgical debulking, appendectomy, omentectomy,
TAHBSO + HIPEC (hyperthermic intraperitoneal
chemotherapy)
SEGMENT LOCATION EMBRYOLOGY REMARKS
• 7.5 to 8.5 cm in diameter
• Blind pouch in the right iliac fossa
Cecum • Midgut • Widest portion, least likely to obstruct
• Retroperitoneal
• Thinnest wall, most common site of perforation
Ascending • Cecum to hepatic flexure
• Midgut • ~13 cm in length
colon • Retroperitoneal
• Proximal 2/3 from • ~38 cm in length
Transverse • Hepatic flexure to splenic flexure
midgut • Relatively mobile
colon • Intraperitoneal
• Distal 1/3 from hindgut • Left colic flexure is higher than the right colic flexure
Descending • Splenic flexure to pelvic brim
• Hindgut • ~25 cm in length
colon • Retroperitoneal
• ~ 25 to 38 cm in length, 2.5cm in diameter
• Pelvic brim to rectosigmoid
• Narrowest portion, most common site of obstruction
Sigmoid junction at the level of the sacral
• Hindgut • May be redundant and extremely mobile
colon promontory
• Most common site of volvulus
• Intraperitoneal
• Most common site of diverticulosis
Surgery Platinum. 1st ed. 2018. p. 313
NORMAL ENDOSCOPIC ANATOMY OF THE COLON Surgery Platinum. 1st ed. 2018. p. 314

Descending colon • Long straight “tunnel view”


• External bulging bluish mass indenting the
Splenic flexure
colon, descending with respiration
Transverse colon • “Cathedral ceiling” appearance
• Spiral configuration which can cause the
Hepatic flexure taenia to so approximate each other “the
fool’s cecum”
Ascending colon • Lower interhaustral folds
Cecum • “Mercedes-Benz” sign
Ileocecal valve • “pouting lips” sign
Appendiceal orifice • “bow and arrow” sign

BLOOD SUPPLY AND VENOUS DRAINAGE


ARTERIAL SUPPLY VENOUS DRAINAGE
• Superior mesenteric artery (SMA) • Superior mesenteric
branches: vein (SMW): drains
o Ileocolic artery: terminal ileum and to the portal vein
proximal ascending colon
o Right colic artery: ascending colon
o Middle colic artery: transverse colon • Inferior mesenteric
• Inferior mesenteric artery (IMA) vein (IMV): joins the
branches: splenic vein to drain
o Left colic artery: descending colon to the portal vein
o Sigmoidal branches: sigmoid colon
o Superior rectal artery: proximal
rectum
• Anastomoses:
o Marginal artery (of Drummond):
anastomoses between the terminal
branches of the SMA and IMA
o Arc of Riolan or meandering
mesenteric artery (of Moskowitz):
anastomosis between the middle colic
artery of the SMA and the ascending
branch of the left colic artery of the
IMA
TOPNOTCH MEDICAL BOARD PREP SURGERY MAIN DIGITAL HANDOUT BY LOUBOMIR ANTONIO, MD Page 90 of 101
For inquiries visit www.topnotchboardprep.com.ph or https://www.facebook.com/topnotchmedicalboardprep/
This handout is only valid for the September 2021 PLE batch. This will be rendered obsolete for the next batch since we update our handouts regularly.
TOPNOTCH MEDICAL BOARD PREP SURGERY MAIN DIGITAL HANDOUT BY LOUBOMIR ANTONIO, MD
For inquiries visit www.topnotchboardprep.com.ph or https://www.facebook.com/topnotchmedicalboardprep/
This handout is only valid for the September 2021 PLE batch. This will be rendered obsolete for the next batch since we update our handouts regularly.
LYMPHATIC DRAINAGE OF THE COLON (A) Small-bowel obstruction (D) Mesenteric vascular occlusion
• Follows the regional arteries from bowel wall to the root of (B) Large-bowel obstruction (E) Sigmoid volvulus
superior and inferior mesenteric arteries (C) Gallstone ileus
Sigmoid volvulus radiographically shows a loop extending from the left
Epicolic lymph nodes • On the bowel
lower quadrant to right upper quadrant (cecal volvulus is RLQ to LUQ).
• Adjacent to the anastomosing The presence of a redundant and mobile sigmoid colon, with a narrow
Paracolic lymph nodes
arteries base at the mesenteric root, is one of the major predisposing factors for
Intermediate lymph nodes • Located around the SMA and IMA sigmoid volvulus. Other predisposing factors, such as a high-fiber diet,
• Located at the origin of the SMA constipation, previous abdominal surgery, pregnancy, diabetes, or
Main lymph nodes
and IMA neurological and psychiatric diseases such as dementia or schizophrenia
INNERVATION OF THE COLON have been described in the literature. In this case, the plain abdominal
x-ray is diagnostic. Small bowel obstruction x-ray will show generalized
• Sympathetic nerve supply (inhibitory) – from T6-T12 and L1-L3
dilated small bowels, air-fluid levels and paucity of rectal gas. Large-
• Parasympathetic nerve supply (stimulatory) – from the vagus and bowel obstruction will show dilated large bowels not mainly in the right
the nervi erigentes (S2-S4) upper quadrant. Gallstone ileus typically will show small bowel
Sympathetic nerve • T6-T12 obstruction, pneumobilia and gallstone at the right iliac fossa. Thumb-
(inhibitory effect) • L1-L3 printing' is a radiologic sign indicating bowel-wall thickening due to
Parasympathetic nerve • From vagus nerve edema that is often a finding in patients with mesenteric ischemia
Dr. Cocos
(stimulatory effect) • From nervi erigentes (S2-S4)
A 65-year-old woman with a history of chronic constipation is transferred
from a nursing home because of abdominal pain and marked abdominal
SUPPLEMENT: EMBRYOLOGY distention. On examination, her abdomen is found to be distended and
• Embryonic GI tract begins developing during 4th week of gestation tender in the LLQ. What is the most likely diagnosis?
FOREGUT MIDGUT HINDGUT (A) Appendicitis
(B) Carcinoma of the colon
Esophagus, Small intestine, Distal transverse
(C) Volvulus of the sigmoid colon
stomach, ascending colon, and colon, descending
(D) Volvulus of the cecum
pancreas, liver, proximal transverse colon, rectum, and
(E) Small-bowel obstruction
duodenum colon proximal anus
Celiac artery SMA IMA Volvulus of the sigmoid (secondary type) is common in elderly patients who
***distal anus is derived from the ectoderm; supplied by internal pudendal artery are chronically constipated. Appendicitis presents with RLQ tenderness and
usually without distention. Small bowel obstruction usually presents with
bilious vomiting but can also have symptoms of abdominal pain and
COLONIC VOLVULUS distention, as well as cecal volvulus (RLQ). No enough clues point to a left-
• Twisting of an air-filled segment of bowel about its narrow sided carcinoma of the colon (i.e. weight loss, hematochezia etc.) but at least
mesentery a sigmoidoscopy can be used to rule this out.
Dr. Cocos
The standard initial therapy for acute sigmoid volvulus is:
SIGMOID VOLVULUS (A) Laparotomy to reduce the volvulus and replace the sigmoid colon
• 90% of all volvulus to its normal position
(B) IV neostigmine
• Redundant sigmoid colon with a narrow-based mesocolon
(C) Colonoscopy
• Diagnosis: (D) Ileostomy
o Abdominal pain, cramping, distention and obstipation (E) Rigid proctosigmoidoscopy
o Plain abdomen radiographs – inverted U-shaped, sausage like
Endoscopic detorsion is the initial treatment of choice for acute sigmoid
loop, Omega sign, coffee bean sign, bent inner tube sign volvulus with interval resection and anastomosis once the patient is
o Water soluble contrast enema – bird’s beak deformity adequately prepared for surgery. This can be done using a rigid
o Abdominal CT scan: whirl pattern, bird's beak appearance proctosigmoidoscopy (not colonoscopy unless the point of rotation is
• Treatment: beyond the 25-cm rigid scope). Emergency laparotomy is reserved for
o Rigid proctosigmoidoscopy/flexible sigmoidoscopy failed decompression and patients with peritonitis. IV neostigmine is
(endoscopic detorsion) usually effective for patients with pseudocolonic obstruction or Ogilvie’s
o Insert soft rectal tube for decompression/stent syndrome. There is usually no indication for doing an ileostomy in cases
of sigmoid volvulus.
o Resection with primary anastomosis – viable bowel/ Dr. Cocos
adequately prepared patients
o Resection of the sigmoid colon with construction of a colostomy
DIVERTICULAR DISEASE
Hartmann’s pouch – emergent operation in • Diverticulum – abnormal sac or pouch protruding from the wall
strangulation/unprepared bowel of a hollow organ such as the colon
• Colostomy options: • True diverticulum – all layers of the intestinal wall ; rare
o End colostomy (Hartmann) • False diverticulum – lacks a portion of the normal bowel wall
o Loop colostomy
o Double barrel DIVERTICULOSIS
o Devine colostomy • Indicates the presence of diverticula
• Etiology (largely unknown)
CECAL VOLVULUS o Low fiber diet associated with narrow colon → segmental
• Younger patients contractions to isolate small regions into compartments
• 20% of colonic volvulus subjected to high pressure contractions →herniation of
• Diagnosis mucosa through the colonic wall → diverticula
o Abdominal pain
o Appears to have small bowel obstruction
o Plain abdominal radiograph – kidney shaped, air-filled structure
in the LUQ
• Treatment: right hemicolectomy with end ileostomy or primary
ileotransverse anastomosis

Transverse Colon Volvulus


• Rare, clinical picture similar to sigmoid volvulus
• Treatment: transverse colectomy

✔ GUIDE QUESTIONS
An elderly nursing home patient is brought to the hospital with recent onset of DIVERTICULITIS
colicky abdominal pain, distension and obstipation on examination, the
• Infection associated with diverticula
abdomen is markedly distended and tympanitic. There is no marked
tenderness. Plain abdominal x-ray shows a markedly distended loop located • Clinical presentation: pain at the LLQ, alteration in bowel habit,
mainly in the right upper quadrant. The likely diagnosis is: tenderness in palpation

TOPNOTCH MEDICAL BOARD PREP SURGERY MAIN DIGITAL HANDOUT BY LOUBOMIR ANTONIO, MD Page 91 of 101
For inquiries visit www.topnotchboardprep.com.ph or https://www.facebook.com/topnotchmedicalboardprep/
This handout is only valid for the September 2021 PLE batch. This will be rendered obsolete for the next batch since we update our handouts regularly.
TOPNOTCH MEDICAL BOARD PREP SURGERY MAIN DIGITAL HANDOUT BY LOUBOMIR ANTONIO, MD
For inquiries visit www.topnotchboardprep.com.ph or https://www.facebook.com/topnotchmedicalboardprep/
This handout is only valid for the September 2021 PLE batch. This will be rendered obsolete for the next batch since we update our handouts regularly.
• Diagnostic tests ruptures into an adjacent organ such as the bladder, vagina, or small
TEST FINDINGS bowel. It is not a risk factor to developing malignancy but this should be
• Ileus may be seen ruled out via colonoscopy 6 weeks after acute diverticulitis. It does not
Abdominal have extraintestinal manifestations (inflammatory bowel diseases), and
• If with obstruction: colon with air fluid levels
radiograph is not associated with arteriovenous fistulae of the intestine and
• If with perforation: pneumoperitoneum (free air)
• Hypoechoic mass suggestive of inflammatory sclerosing cholangitis.
Dr. Cocos
reaction A patients CT scan reveals diverticulitis confined to the sigmoid colon.
Abdominal
• Diverticular abscess formation There is no associated pericolic abscess. What is best course of treatment?
ultrasound
• Bowel wall thickening >4 mm at the most tender (A) Bowel rest, nasogastric suction, IV fluids, and broad-spectrum
area antibiotics
• Diagnosis is best made on CT with the following (B) Urgent surgical resection
findings: (C) Steroids
Abdominal o Sigmoid diverticula (D) Diverting colostomy
CT scan o Thickened colonic wall >4 mm (E) Ileostomy
o Inflammation within pericolic fat ± collection of
Mild diverticulitis is managed conservatively with bowel rest, hydration and IV
contrast material or fluid
antibiotics (choices B, D, and E are wrong). Elective surgery can be done at least
• Barium enema or colonoscopy should not be
6 months once symptoms have resolved. Hinchey I and II needs percutaneous
performed in the acute setting because of the risk of
Contrast drainage of abscess. Hinchey III and IV are emergency operative cases. There is
perforation associated with insufflation or insertion
(barium) no role for the use of steroids in diverticulitis (choice C is wrong).
of barium-based contrast material under pressure Dr. Cocos
enema or
• Should be performed ̴ 6 weeks after an attack to
colonoscopy
rule out malignancy (because a sigmoid malignancy LOWER GASTROINTESTINAL BLEEDING
can masquerade as diverticular disease) PATHOPHYSIOLOGY
Surgery Platinum. 1st ed. 2018. p. 319

• Treatment: • LGIB: refers to bleeding distal to the ligament of Treitz


o Uncomplicated diverticulitis – clear liquid diet and broad (majority occurs in the colon)
spectrum antibiotics for 7-10 days and high fiber diet • Diverticular disease: arterial disruption where colonic arterioles
o Recurrent attacks – elective resection of diseased colon penetrates muscular wall of bowel
o Generalized peritonitis – urgent laparotomy to control • Angiodysplasia: acquired lesion due to chronic intermittent
infection obstruction of the submucosal veins
o Diverticulitis with abscess – drainage • Others: vascular ectasia, cancer, hemorrhoids, colitis, volvulus,
o Diverticulitis with fistula – seldom a cause for emergency strangulated hernia, anal fissure
surgical treatment CLINICAL MANIFESTATIONS
o Antibiotics, total parenteral nutrition, bowel rest LOCATION OF LGIB MANIFESTATIONS
o Excision of the diseased segment of the colon containing the site • Hematochezia (bright red blood
of perforation LGIB from left side of colon
per rectum)
• Management • Usually maroon-colored stools
o Hospital admission is recommended for complicated diverticulitis LGIB from the right side of
• If bleeding is brisk and massive,
colon
o For uncomplicated diverticulitis, admission should be stools may also be bright red
recommended for those who have: • May present with melena (dark
Cecal bleeding
§ High documented fever stools), typically seen in UGIB
Surgery Platinum. 1st ed. 2018. p. 321
§ Immunocompromised status COMMON DIFFERENTIALS AND MANIFESTATIONS
§ Severe abdominal pain
DIFFERENTIAL TYPICAL FINDINGS
§ Significant or unstable comorbid conditions
• Infectious colitis: young patient with fever,
§ Inability to tolerate oral intake abdominal cramps, hematochezia
o Management of Complicated Diverticular Disease Colitis • Ischemic colitis: elderly patient (multiple
§ Surgery is warranted for complicated cases: obstruction, fistula, cardiovascular comorbidities) with severe
free perforation, abscess; usually for Hinchey stage III and IV abdominal pain, hematochezia, diarrhea
HINCHEY • Elderly patient with painless bleeding & bleeding
DESCRIPTION OPERATIVE PROCEDURE
STAGE Diverticular & paucity of other symptoms
• Percutaneous drainage of abscess bleeding • Usually acute in onset
Diverticulitis with
• Resection with primary • Self-limited in the majority
Stage I a pericolic
anastomosis without diverting • painless, self-limited hematochezia (usually
abscess
stoma venocapillary)
Angiodysplasia
Diverticulitis with • Tends to cause slow but recurrent episodes of
• Percutaneous drainage of abscess
distant abscess bleeding (in contrast to diverticular bleeding)
Stage II • Resection with primary
(retroperitoneal, • Young patient with abdominal pain,
anastomosis ± diversion Inflammatory
pelvic) hematochezia, diarrhea, mucoid discharge per
bowel
• Hartmann procedure rectum
Purulent • Diverting colostomy plus Anal fissure • Stools streaked with blood, perianal pain, and
Stage III
peritonitis percutaneous drainage (high hemorrhoids blood on toilet paper
unstable patients) • Right-sided cancer: present with maroon-colored
• Hartmann procedure stools of melena
• Diverting colonoscopy plus Colon • Left-sided cancer: present with bright red blood
Stage IV Fecal peritonitis
percutaneous drainage (highly carcinoma per rectum
unstable patients) • May be insidious, presenting as iron-deficiency
Surgery Platinum. 1st ed. 2018. p. 320

RIGHT-SIDED DIVERTICULITIS anemia, pallor, or syncope


Surgery Platinum. 1st ed. 2018. p. 321
• Rare location DIAGNOSTICS
• Presentation: younger age group, Asian descent, mimics appendicitis DIAGNOSTIC FINDINGS
• Treatment: segmental ileocecal resection (preferred) • to rule out upper gastrointestinal bleeding
(UGIB) – this is because brisk UGIB can mimic
Insertion of
✔ GUIDE QUESTIONS LGIB
nasogastric
Complications of diverticulitis include: • NGT aspirate or lavage is done to determine
(A) Carcinoma of the colon tube (NGT)
the presence or absence of blood proximal to
(B) Extraintestinal manifestations such as arthritis, iritis, and skin rashes the ligament of Treitz (indicates UGIB)
(C) Fistulation to adjacent organs such as the bladder, with • Normocytic normochromic anemia (acute
ensuing colovesical fistula Complete
bleeding)
(D) Arteriovenous fistulae of the intestine blood count
• Microcytic hypochromic anemia (chronic
(E) Sclerosing cholangitis (CBC)
bleeding)
Diverticulitis of the colon may result in complications such as abscess,
bleeding, obstruction, and fistula formation. Fistula formation by
colonic diverticulitis occurs when a diverticular abscess extends or

TOPNOTCH MEDICAL BOARD PREP SURGERY MAIN DIGITAL HANDOUT BY LOUBOMIR ANTONIO, MD Page 92 of 101
For inquiries visit www.topnotchboardprep.com.ph or https://www.facebook.com/topnotchmedicalboardprep/
This handout is only valid for the September 2021 PLE batch. This will be rendered obsolete for the next batch since we update our handouts regularly.
TOPNOTCH MEDICAL BOARD PREP SURGERY MAIN DIGITAL HANDOUT BY LOUBOMIR ANTONIO, MD
For inquiries visit www.topnotchboardprep.com.ph or https://www.facebook.com/topnotchmedicalboardprep/
This handout is only valid for the September 2021 PLE batch. This will be rendered obsolete for the next batch since we update our handouts regularly.
DIAGNOSTIC FINDINGS ✔ GUIDE QUESTIONS
• use either technetium sulfur colloid scan or Proctoscopy reveals nonbleeding grade I hemorrhoids and maroon stool
99mTc pertechnetate-tagged red blood cells; and clots coming from the proximal colon. Which of the following is TRUE
then the body is scanned to see the tagged RBC in the management of lower gastrointestinal (GI) bleeding?
99mTc-labeled
and its movement in the body (A) Barium enema is a good tool in the early evaluation of massive GI
RBC bleeding.
• Cannot reliably localize the site of hemorrhage,
scintigraphy (B) Technetium sulfur colloid has excellent sensitivity in localizing
but is highly sensitive if with active bleeding
lower GI bleeding.
• Has a role in localization of obscure GI bleeding
(C) Technetium sulfur colloid scan is useful because it may be
(detects bleeding rate as little as 0.1 mL/min) repeated 24 hours later with single injection.
• Test of choice to identify site of lower GI (D) Colonoscopy should be avoided in the evaluation of acute lower
Colonoscopy bleeding GI bleeding.
• May also be therapeutic (E) Sensitivity specificity and accuracy rates vary widely and the
• to rule out UGIB (massive UGIB can present as exact role of red blood cell (RBC) scanning is controversial.
hematochezia because of rapid transit of blood) Barium enema is not a good screening tool for massive LGIB. Tc sulfur
Upper • Clue to UGIB: blood in nasogastric tube (NGT) colloid has poor sensitivity especially for an intermittent bleed due to
endoscopy • Hematochezia associated with hemodynamic background activity in the reticuloendothelial system and short imaging
(EGD) instability may be indicative of UGIB as a time. Furthermore it requires multiple injections. For the management
source of bleeding, which warrants an upper of lower GI bleeding, the most important diagnostic tool (can be
endoscopy (EGD) therapeutic too) is endoscopy: colonoscopy +/- EGD.
Dr. Cocos
Computed • Alternative to catheter-directed angiography The patient responds to resuscitation with normalization of vital signs but
tomography • Able to detect sites of bleeding continues to bleed. He is taken to the angiography suite for further
angiography • Lack therapeutic capability evaluation. Which of the following is TRUE?
(CTA) (A) The inferior mesenteric artery should be injected first because
• Proves highly accurate localization of the site most diverticula are in the sigmoid colon.
Catheter- of bleeding (B) Vasopressin be selectively infused into a bleeding mesenteric
directed • Identifies site of bleeding vessel with virtually no risk to the patient.
angiography • Can do therapeutic intervention (e.g., (C) Embolization with gel foam or autologous clots may be used to
embolization) stop bleeding.
(D) Since angiography is both diagnostic and therapeutic surgery will
Surgery Platinum. 1st ed. 2018. p. 322
not be necessary.
MANAGEMENT (E) A bleeding rate of 0.1 mL/min is necessary for a positive scan.
• Supportive measures: oxygen, establishment of In an ideal setting, angiographic embolization is done in bleeding
intravenous access patients when endoscopic approach is not possible or has failed.
• Appropriate fluid and blood product resuscitation Otherwise, surgical intervention is warranted.
• Management of coagulopathies Vasopressin can be selectively infused into a bleeding mesentric vessel
Non- but has its own risks (e.g. bowel ischemia). A bleeding rate of .5 per
surgical • Interventions during colonoscopy:
o Vasoconstricting agents: vasopressin, alcohol, minute is necessary for a positive angiogram. Temporary success in
stopping the bleeding will not obviate the need for surgery. The
morrhuate sodium or sodium tetradecyl sulfate
angiodysplasia of the colon is one of the most common causes of lower
o Thermal modalities: laser photocoagulation,
GI bleeding in elderly patients. With diverticular disease, 75% of the
electrocoagulation, or heater probe coagulation patients will have only a single episode of hemorrhage, whereas
• Surgical management is reserved for those who angiodysplasia patients are very likely to have recurrent episodes of
are not responsive to medical management variable severity.
• Interventions include: Dr. Cocos
Surgical
o Segment colectomy (bleeding site identified)
o Blind total or sub-total colectomy (patient in shock
with bleeding site unidentified)
Surgery Platinum. 1st ed. 2018. p. 322

INFLAMMATORY BOWEL DISEASE


ULCERATIVE COLITIS (UC) CROHN DISEASE (CD)
Age of onset 15-30 & 60-80 years 15-30 & 60-80 years
Male-female
1:1 1.1-1.8:1
ratio
Smoking May prevent disease May cause disease
Appendectomy Protective Not protective
• Affects any part of the GIT from the mouth to the anus
• Involves rectum & extends proximally to
• Rectum is often spared or normal
involve all of part of colon
• Segmental with skip areas in the midst of diseased intestine
• Distribution is continuous
Pathology • Transmural process
• Limited to the mucosa and superficial
• “Cobblestone” appearance: characteristic of CD, both endoscopically &
submucosa, with deeper layers unaffected
radiography
except in fulminant disease
• Granulomas: pathognomonic feature
• With gross blood and mucus in stool • Occasional blood and mucus in stool
• Occasionally painful • Frequently painful
Clinical
• Infrequent significant perineal disease • Frequently significant perineal disease
manifestations
• Not associated with fistulas • May be associated with fistulas
• Megacolon is frequent • Megacolon is less common
• Total proctocolectomy can be curative • Only for management of complications (not for cure of disease since
Indications for • Active disease unresponsive to medical therapy recurrence is common)
surgery • Risk of cancer • Resection encompasses with the intestine that is grossly involved with the
• Severe bleeding disease
Surgery Platinum. 1st ed. 2018. p. 320
✔ GUIDE QUESTIONS
A 35-year-old man has known ulcerative colitis. Which of the following is The hallmarks of toxic megacolon (toxic colitis), a potentially lethal
an indication for total proctocolectomy? complication of ulcerative colitis, are nonobstructive colonic dilatation
(A) Occasional bouts of colic and diarrhea larger than 6 cm and signs of systemic toxicity. Treatment has 3 main goals:
(B) Sclerosing cholangitis (1) reduce colonic distention to prevent perforation, (2) correct fluid and
(C) Toxic megacolon electrolyte disturbances, and (3) treat toxemia and precipitating
(D) Arthritides factors. Indications for colectomy include free perforation, massive
(E) Iron deficiency anemia hemorrhage (6-8U packed red blood cells), increasing toxicity, and
progression of colonic dilatation. Emergency total colectomy is indicated.
Dr. Cocos

TOPNOTCH MEDICAL BOARD PREP SURGERY MAIN DIGITAL HANDOUT BY LOUBOMIR ANTONIO, MD Page 93 of 101
For inquiries visit www.topnotchboardprep.com.ph or https://www.facebook.com/topnotchmedicalboardprep/
This handout is only valid for the September 2021 PLE batch. This will be rendered obsolete for the next batch since we update our handouts regularly.
TOPNOTCH MEDICAL BOARD PREP SURGERY MAIN DIGITAL HANDOUT BY LOUBOMIR ANTONIO, MD
For inquiries visit www.topnotchboardprep.com.ph or https://www.facebook.com/topnotchmedicalboardprep/
This handout is only valid for the September 2021 PLE batch. This will be rendered obsolete for the next batch since we update our handouts regularly.

NEOPLASTIC DISEASE
• Risk Factors
Aging Cigarette smoking
Inherited syndromes Ureterosigmoidostomy
Family history of colorectal CA Acromegaly
Personal history of colorectal CA Pelvic irradiation
Inflammatory bowel disease Obesity
Diabetes mellitus and insulin resistance Saturated fat (animal fat)
Lack of physical activity Red meat COLORECTAL CANCER
Alcohol consumption Low in fiber and vegetable https://qrs.ly/y5cl1aj

COLORECTAL CANCER PATHOGENESIS


• Arises from adenomatous polyps by accumulation of mutations

Figure 29-22. Schwartz’s Principles of Surgery. 11th ed. 2019

COLORECTAL POLYPS
• Any mass projecting into the lumen of the bowel above the surface HAGGITT CLASSIFICATION
of the intestinal epithelium • Histological classification of the extent of invasion of
• Grossly classified as either pedunculated (with a stalk) or sessile pedunculated malignant colonic polyps (based on the level of
(flat) invasion)
• Slow growing; but some may carry a small risk of becoming • Sessile lesions are classified separately (Kikuchi classification)
malignant RISK OF
• Pathophysiology LEVEL OF LYMPH
DESCRIPTION
o Colonic polyps are benign epithelial neoplasms that arise from INVASION NODE
the epithelial lining of the colon METASTASIS
o Most are found in the rectosigmoid area • Noninvasive (severe dysplasia,
• Histologic Classification of Polyps Level 0 carcinoma-in-situ or intramucosal 0%
HISTOLOGY REMARKS carcinoma)
• Most common (90%) • Cancer invading through the
Level 1 muscularis mucosa (limited to the <1%
• Most commonly occur in the rectosigmoid
Hyperplastic head of polyp)
region in adulthood
polyps • Cancer invading the neck of a
• Not considered pre-malignant, unless large Level 2 <1%
(>2cm) or multiple pedunculated polyp
Hamartomatous • Occurs at any age • Cancer invading the stalk of a
Level 3 <1%
or juvenile pedunculated polyp
• Usually not premalignant
polyps • Cancer invading into the
• Not premalignant submucosa of the bowel wall
Inflammatory or Level 4 below the stalk of a pedunculated 12-25%
• Associated with ulcerative colitis and Crohn
pseudopolyps polyp. All sessile polyps with
disease
invasive cancer are level 4
• Hyperplastic polyps with adenomatous Surgery Platinum. 1st ed. 2018. p. 329
features
Serrated polyps KIKUCHI CLASSIFICATION (SESSILE POLYPS)
• Premalignant
• Treated like adenomatous polyps
Lymphoid • Benign enlargements of lymphoid follicles
polyps usually located in the rectum
• Polyps with neoplastic cells penetration the
muscularis mucosa
Malignant
• Haggitt classification: clinical tool used to
polyps
describe the degree of invasion into a
pedunculated or sessile polyp adenoma
Surgery Platinum. 1st ed. 2018. p. 328
NEOPLASTIC POLYPS
ADENOMA CANCER
LOCATION AND DESCRIPTION
TYPE RISK DISTANCE OF
• Most common NODAL
Tubular CATEGORY SUBMUCOSA
• Can be found throughout the METASTASES
adenoma • < 5% INVASION
large intestine SM1 Superficial 1/3 2%
(75%)
• Usually pedunculated SM2 Superficial 2/3 8%
Tubulovillous SM3 Deep 1/3 23%
adenoma • Throughout large intestine • 25%
Note: formal oncologic resection is warranted for high-risk lesions (Kikuchi
(8-15%)
SM3 or Haggitt 4 or if poorly differentiated or with lymphovascular
• Predominantly in the rectum invasion or positive resection margin)
• Usually sessile, velvety, or Dr. Cocos
Villous cauliflowerlike grossly
adenoma • Larger than tubular or • 40%
(5-10%) tubulovillous adenoma
• Highest morbidity or mortality
rates
Surgery Platinum. 1st ed. 2018. p. 328

TOPNOTCH MEDICAL BOARD PREP SURGERY MAIN DIGITAL HANDOUT BY LOUBOMIR ANTONIO, MD Page 94 of 101
For inquiries visit www.topnotchboardprep.com.ph or https://www.facebook.com/topnotchmedicalboardprep/
This handout is only valid for the September 2021 PLE batch. This will be rendered obsolete for the next batch since we update our handouts regularly.
TOPNOTCH MEDICAL BOARD PREP SURGERY MAIN DIGITAL HANDOUT BY LOUBOMIR ANTONIO, MD
For inquiries visit www.topnotchboardprep.com.ph or https://www.facebook.com/topnotchmedicalboardprep/
This handout is only valid for the September 2021 PLE batch. This will be rendered obsolete for the next batch since we update our handouts regularly.

POLYPOSIS SYNDROMES Muir-Torre • Prone to develop cancers of colon, breast, and


Syndrome genitourinary tract, and skin lesions, such as
HAMARTOMATOUS POLYPS (Autosomal keratoacanthomas and sebaceous tumors
REMARKS dominant) • Genes affected: MLH1, MSH2, MSH6
Juvenile • Most common hamartomatous syndrome Surgery Platinum. 1st ed. 2018. p. 330

polyposis • Associated with other anomalies: macrocephaly, ✔ GUIDE QUESTION


syndrome mental retardation, cleft lip of palate, congenital A 30-year-old male is diagnosed with Peutz-Jeghers syndrome. What
(Autosomal heart disease, genitourinary malformations, and findings is consistent with the diagnosis?
dominant) malrotations (A) Adenomas
• Second most common hamartomatous (B) Hamartomas
(C) Adenomatous polyps
syndrome
Peutz- (D) Villoglandular polyps
• Consists of perioral, buccal, and occasionally
Jeghers (E) Villotubular polyps
genital melanin pigmentation together with
syndrome Peutz–Jeghers syndrome is an autosomal dominant
gastrointestinal hamartomatous polyposis
(Autosomal genetic disorder characterized by the development of benign
• Patients have moderately increased risk to
dominant) hamartomatous polyps in the gastrointestinal tract and
develop gastrointestinal malignancies as well as
hyperpigmented macules on the lips and oral mucosa (melanosis).
extraintestinal malignancies Dr. Cocos
• Caused by a germ-line mutation in the PTEN
Cowden tumor suppressor gene located at 10q22
syndrome • Characterized by macrocephaly, (+)
COLORECTAL CARCINOMA
(Autosomal trichilemmomas, (+) other benign and • Clinical Manifestations
dominant) malignant neoplasms of thyroid, breast, o Common clinical presentations include: iron deficiency anemia,
uterus, skin rectal bleeding, abdominal pain, change in bowel habits,
• Characterized by diffuse polyposis and intestinal obstruction
Cronkite- ectodermal abnormalities (alopecia, o Physical examination findings may be nonspecific; other signs
Canada onychodystrophy, skin hyperpigmentation) include fatigue, weight loss, tenderness, rectal bleeding, palpable
syndrome • Entire GI tract is involved except the mass, hepatomegaly, ascites
esophagus
RIGHT-SIDED LESIONS LEFT-SIDED LESIONS
• Characterized by hamartomatous polyps of the
• Has a large lumen, therefore • Has a smaller lumen (cause
gastrointestinal tract, macrocephaly, mental
Bannayan- retardation, delayed psychomotor development, a tumor must attain a large symptoms earlier)
Riley- lipid storage myopathy, Hashimoto thyroiditis, size before causing • Changes in bowel habits:
Ruvalcaba and hyperpigmentation of the skin o the penis symptoms (usually poorer small (goat-stool-like)
syndrome • No increased risk of colorectal carcinoma, prognosis) stools, colicky pain,
other gastrointestinal malignancies, or • Postprandial discomfort, alternating diarrhea and
extraintestinal malignancy weakness constipation, bloody stools,
Surgery Platinum. 1st ed. 2018. p. 331
• Occult bleeding or anemia tenesmus
ADENOMATOUS POLYPS
• Melena more common • Hematochezia more
ADENOMATOUS common
REMARKS
POLYPS
• Diagnosis: Screening for Colorectal Cancer
• Genetic defect: adenomatous polyposis coli
o Goals of screening are detection of early cancers and prevention
(APC) gene on chromosome 5q21
Familial of cancer by finding & removing adenomas
• Characterized by >100 (often several
adenomatous
thousands) of adenomatous intestinal polyps
polyposis (FAP) RISK ASSESSMENT FOR COLORECTAL CANCER
that start to develop in the late teens and early
(Autosomal • ≥ 50 years
twenties and turn into cancer by age 40-45
dominant) • No history of adenoma or sessile serrated polyp
• (+) duodenal adenomas, (+) desmoid Average
tumors or colorectal cancer (CRC)
risk
• Neoplastic polyps of small bowel and colon • No history of inflammatory bowel disease
• Cancer by age 40 years in 100% undiagnosed • Negative family history of colorectal cancer
Gardner
patients (same as FAP) • Personal history of adenoma, sessile serrated
Syndrome Increased
• (+) epidermoid cysts, (+) osteomas of the polyp, inflammatory bowel disease or CRC
skull, (+) desmoid tumors of abdominal wall risk
• Positive family history
• Association between colorectal adenomatous • Lynch syndrome
Turcot
polyposis and (+) CNS tumors (cerebellar
Syndrome • Polyposis syndromes
medulloblastoma or glioblastoma)
Surgery Platinum. 1st ed. 2018. p. 330 o Classical familial adenomatous polyposis (FAP)
o Attenuated FAP
NONPOLYPOSIS SYNDROME o MUTYH- associated polyposis
High risk
INHERITED o Peutz-Jeghers syndrome
NONPOLYPOSIS REMARKS o Juvenile polyposis syndrome
SYNDROMES o Serrated polyposis syndrome
• Increased risk on developing extracolonic o Cowden syndrome
malignancies (small bowel, stomach, o Li-Fraumeni syndrome
hepatobiliary tract, ovary, brain) Surgery Platinum. 1st ed. 2018. p. 333

• DNA mismatch repair (MMR) genes have


been linked to HNPCC (hMSH2, hMLH1, American Cancer Society (ACS) Recommendations for
Hereditary hMSH6, hPMS1, and hPMS2) Colorectal Cancer Early Detection
nonpolyposis • A meticulous family history should be • Beginning at age 50, men and women who are at average risk for
colorectal obtained to identify potentially affected developing colorectal cancer should have 1 of the 5 screening
cancer individuals using Amsterdam criteria options below:
(HNPCC) or • Amsterdam criteria for diagnosis of HNPCC: FOBT or fecal immunochemical test Every Year
Lynch o At least 3 relatives must have histologically (FIT) every year
syndrome verified colorectal cancer Flexible sigmoidoscopy Every 5 Years
(Autosomal o One must be first-degree relative of the other 2 FOBT or FIT + flexible Every Year + Every 5 Years
dominant) o At least 2 successive generations must be sigmoidoscopy
affected Double-contrast barium enema Every 5 Years
o + at least 1 of relatives must have received Colonoscopy Every 10 Years
the diagnosis before age 50
• Of the first three options, the combination of FOBT or FIT every
o Familial adenomatous polyposis (FAP)
year plus flexible sigmoidoscopy every 5 years is preferable
should be excluded
Note: colonoscopy is the best screening tool for colorectal carcinoma!
Dr. Cocos

TOPNOTCH MEDICAL BOARD PREP SURGERY MAIN DIGITAL HANDOUT BY LOUBOMIR ANTONIO, MD Page 95 of 101
For inquiries visit www.topnotchboardprep.com.ph or https://www.facebook.com/topnotchmedicalboardprep/
This handout is only valid for the September 2021 PLE batch. This will be rendered obsolete for the next batch since we update our handouts regularly.
TOPNOTCH MEDICAL BOARD PREP SURGERY MAIN DIGITAL HANDOUT BY LOUBOMIR ANTONIO, MD
For inquiries visit www.topnotchboardprep.com.ph or https://www.facebook.com/topnotchmedicalboardprep/
This handout is only valid for the September 2021 PLE batch. This will be rendered obsolete for the next batch since we update our handouts regularly.
COMPARISONS OF SCREENING TESTS FOR COLORECTAL CANCER
SCREENING
ADVANTAGES DISADVANTAGES
TEST
• Easy to perform
• Low specificity and
Fecal occult on annual basis
sensitivity
blood test • Strong level of
• Displeasing to some
(FOBT) evidence for its
patients
effectiveness
• Easy to perform • Does not examine
• Does not require entire colon
Flexible
sedation or dull • Follow-up
sigmoidoscopy
bowel colonoscopy needed
preparation for abnormalities
• Bowel preparation
Double- required
• Does not require
contrast • Follow-up
sedation
barium enema colonoscopy needed
for abnormalities
• Examination the • Bowel preparation &
entire colon IV sedation is needed
Colonoscopy
• Both diagnostic • Risk of perforation
and therapeutic • Limited accessibility
Surgery Platinum. 1st ed. 2018. p. 334

EXAMINATION OR
DIAGNOSTIC REMARKS
TOOL
Digital rectal • 10% of colorectal cancers are potentially
examination and within reach of the examiner’s finger
anoscopy (only tumors of the mid and distal rectum
UT1 Confined to submucosa
(8 cm in length) can be assessed)
UT2 Invades muscularis propria
• Detects 20-25% of colorectal tumors
Rigid proctoscopy • Does not provide complete information
UT3 Invades perirectal fat
(25 cm in length) • Rectum is evaluated better by the rigid UT4 Invades adjacent organ
proctoscope than by the flexible instrument
• Advantage: reaches the proximal left colon COLORECTAL CARCINOMA STAGING
Flexible T1 Invades submucosa
or even the splenic flexure
sigmoidoscopy
• Gives inaccurate measurements if used at T2 Invades muscularis propria
(60 cm in length)
the rectum Invades subserosa or nonperitonealized pericolic/
T3
• Method of choice because of its high perirectal tissues
Colonoscopy sensitivity in detecting tumors and its T4 Invades other organs or structures
(100 to 160 cm in ability to take biopsies N1 1 to 3 nodes
length) • Visualized mucosa of entire colon, rectum, N2 4 or more nodes
and usually terminal ileum N3 Any node along a major named vascular trunk
• Air contrast barium enema
Contrast studies M0 No metastasis
• 90% sensitive in detecting polyps >1 cm
• Glycoprotein absent from normal adult M1 Distant metastasis
intestinal mucosa but present in
primitive endoderm STAGE TNM
• CEA 2.5 to 5.0 ng/mL: cut-off points for I T1-2, N0, M0
distinguishing normal from abnormal levels II T3-4, N0, M0
Carcinoembryonic III Tany, N+, M0
• Cannot be used as screening tests for
antigen (CEA)
colorectal cancer because of low sensitivity IV Tany, Nany, M1
and specificity
• Useful in treatment monitoring (a rise in
CEA after apparently successful surgical
treatment → signal a recurrence of the tumor)
Surgery Platinum. 1st ed. 2018. p. 335
STANDARD RESECTIONS OF THE COLON
TUMOR LOCATION RESECTION EXTENT MAJOR BLOOD VESSEL
• Ileocolic artery
• Right colic artery
Cecum • Right hemicolectomy • Terminal ileum to mid-transverse colon
• Right branch middle colic
artery
• Ileocolic artery
• Right colic artery
Ascending colon • Right hemicolectomy • Terminal ileum to mid-transverse colon
• Right branch middle colic
artery
• Ileocolic artery
• Extended right
Hepatic flexure • Terminal ileum to distal transverse colon • Right colic artery
hemicolectomy
• Root of middle colic artery
• Left branch middle colic artery
Splenic flexure • Extended Left hemicolectomy • Mid-transverse colon to rectosigmoid junction • Left colic artery
• Inferior mesenteric artery
• Inferior mesenteric artery
Descending colon • Left hemicolectomy • Splenic flexure to rectosigmoid junction
• Left colic artery
• Digital descending colon to rectosigmoid • Inferior mesenteric artery
Sigmoid colon • Rectosigmoid resection
junction • Superior rectal artery
Total colectomy or subtotal colectomy with ileorectal anastomosis may be required for circumstances in which the patient has been diagnosed with:
HNPCC, Attenuated familial adenomatous polyposis, Synchronous cancers in separate colon segment, Frequently in acute malignant distal colon obstruction with
unknown status of proximal bowel
Surgery Platinum. 1st ed. 2018. p. 337

TOPNOTCH MEDICAL BOARD PREP SURGERY MAIN DIGITAL HANDOUT BY LOUBOMIR ANTONIO, MD Page 96 of 101
For inquiries visit www.topnotchboardprep.com.ph or https://www.facebook.com/topnotchmedicalboardprep/
This handout is only valid for the September 2021 PLE batch. This will be rendered obsolete for the next batch since we update our handouts regularly.
TOPNOTCH MEDICAL BOARD PREP SURGERY MAIN DIGITAL HANDOUT BY LOUBOMIR ANTONIO, MD
For inquiries visit www.topnotchboardprep.com.ph or https://www.facebook.com/topnotchmedicalboardprep/
This handout is only valid for the September 2021 PLE batch. This will be rendered obsolete for the next batch since we update our handouts regularly.
CHEMOTHERAPY OR RADIOTHERAPY ANATOMY OF THE ANAL CANAL
• Stage II • It is the caudal part of the large bowel extending from the
o T4 tumor anorectal ring to the anal verge
o Tumor perforation, obstruction • Anorectal ring: located at the level of the distal end of the
Indications for
o Lymphovascular and perineural invasion ampullary part of the rectum and forms the anorectal angle
adjuvant
o Poorly differentiated or signet ring types • Anal verge: part of the anal canal remaining closed when the
chemotherapy
o Inadequate margin <5 cm buttocks are gently retracted
for colonic CA
o Inadequate lymph node harvest <12
• Anal margin (perianal skin): lies outside the anal verge (5cm
nodes
perimeter)
• Stages III and IV
Indications for • Anus has 8-14 longitudinal mucosal folds called the columns of
• Stage II Morgagni
adjuvant
• Stage III
chemotherapy
• Stage IV
for rectal CA
Indications for
• For locally advanced rectal CA
neoadjuvant
o All T3 lesions and/or
radiation for
o N1
rectal CA

✔ GUIDE QUESTION
A 55-year-old man has had previous hemicolectomy for a carcinoma of
the right colon. At this time, 3 years after the primary resection, a CT scan
shows a solitary lesion in the right lobe of the liver. What is the next step
in management?
(A) Laser cauterization
(B) Radiotherapy
(C) Hepatic artery catheterization and local chemotherapy
(D) Symptomatic treatment with analgesics, because the colon
disease is now stage IV
(E) Exploratory laparotomy and resection of the tumor
Regional treatments for hepatic metastasis from CRC include: surgical
resection, local tumor ablation (microwave or radiofrequency ablation),
chemoembolization etc. However, only resection is associated with a
survival advantage and is the best option if there is no contraindication
to surgery.
Dr. Cocos
A 65-year-old man presents with chronic constipation and abdominal
distention of 5-day duration. He complains of lack of appetite and general
malaise. Findings on physical examination are positive for a large
distended abdomen with hyperactive bowel sounds. Rectal examination
shows minimal stool that is guaiac-positive. Sigmoidoscopy does not
reveal any further findings.
Abdominal x-rays show a large 10-cm cecum and dilated, fluid-filled
transverse and descending colon with very little gas in the rectum. What
is the most probable cause of this condition?
(A) Volvulus of the sigmoid colon
(B) Pseudo-obstruction of the colon
(C) Ischemic colitis
(D) Carcinoma of the colon
(E) Diverticulitis of the colon MUSCLES OF THE ANAL CANAL
• External sphincter (subcutaneous, superficial and deep):
The most likely cause is an obstructing carcinoma given that this
patient has constitutional signs of malignancy like anorexia, malaise
continuation of the puborectalis muscle and is voluntary
and a guaiac-positive stool. The site of obstruction is possibly in the • Internal sphincter: represents the distal condensation of the
sigmoid colon above the level of sigmoidoscopy. Sigmoid volvulus, circular muscle layer and is involuntary
ischemic colitis, and diverticulitis will present some findings on • Conjoined longitudinal muscle: combination of the outer
sigmoidoscopy. Pseudo-obstruction of the colon will manifest as colonic longitudinal layer of the rectum and fibers of the levator ani
distention down to the rectum (adult acute megacolon or Ogilvie’s muscle descending between the internal and external anal
syndrome) in the absence of mechanical obstruction.
Dr. Cocos
sphincter, inserting into the perianal skin
• Functions of the conjoined longitudinal muscle:
o Attaches the anorectum to the pelvis
RECTUM & ANUS o Supports and binds the internal and external sphincter complex
ANATOMY OF THE RECTUM o Act as a support to prevent hemorrhoids and rectal prolapse
• At mid sacral level, the sigmoid loses its mesentery and gradually o Maintenance of an anal seal
becomes the rectum
• Approximately 12-15 cm in length
• Three lateral curves (valves of Houston)
• Convex to the
Upper and lower lateral curves
right
Middle lateral curve • Convex to the left
• The most consistent valve is the middle valve (Kohlrausch
valve) - corresponds to the level of the anterior peritoneal
reflection
• Only the upper 2/3 of the rectum is covered by peritoneum (upper
third of the rectum both anteriorly and laterally; the middle third
on its anterior aspect only, while the distal third is completely
devoid of peritoneum)

ANATOMIC AND SURGICAL RECTUM


EXTENT ANATOMIC SURGICAL
Proximal • 3rd sacral vertebra (S3) • Sacral promontory
Distal • Dentate line • Anorectal ring
Surgery Platinum. 1st ed. 2018. p. 315

TOPNOTCH MEDICAL BOARD PREP SURGERY MAIN DIGITAL HANDOUT BY LOUBOMIR ANTONIO, MD Page 97 of 101
For inquiries visit www.topnotchboardprep.com.ph or https://www.facebook.com/topnotchmedicalboardprep/
This handout is only valid for the September 2021 PLE batch. This will be rendered obsolete for the next batch since we update our handouts regularly.
TOPNOTCH MEDICAL BOARD PREP SURGERY MAIN DIGITAL HANDOUT BY LOUBOMIR ANTONIO, MD
For inquiries visit www.topnotchboardprep.com.ph or https://www.facebook.com/topnotchmedicalboardprep/
This handout is only valid for the September 2021 PLE batch. This will be rendered obsolete for the next batch since we update our handouts regularly.
PELVIC ANATOMY
• Levator ani muscle supports the pelvic viscera - consists of the
pubococcygeus, Iliococcygeus, puborectalis muscles

INNERVATION OF THE RECTUM AND ANUS


• Both sympathetic and parasympathetic nerves innervate the
FASCIAL LAYERS AND LIGAMENTS OF THE ANORECTAL anoderm
REGION • Derived from L1-L3 and joins the preaortic
• Lines the rectum, and part of the visceral plexus
Fascia propria • Preaortic fibers extends below the aorta to
layer of the endopelvic fascia Sympathetic
• Separates the rectum from the presacral form the hypogastric plexus
Presacral fascia fibers
venous plexus and pelvic nerves • Subsequently, the hypogastric plexus joins
• Extends anteriorly and caudally the parasympathetic fibers to form the
Rectosacral fascia pelvic plexus
• Attaches to the fascia propria at the
(Waldeyer’s fascia) • Known as nervi erigentes
anorectal junction
• Separates the rectum from the prostate Parasympathetic • Originates from S2-S4
Denonvilliers’ fibers • These fibers join the sympathetic fibers to
gland and seminal vesicles in men and
fascia form the pelvic plexus
from vagina in women
Lateral ligaments • Support the lower rectum
• Layer of fatty connective tissue surround MUSCLE INNERVATION
the rectum and it contains its vessels and Internal anal • Innervated by parasympathetic and
lymph nodes sphincter sympathetic nerve fibers
• Can be metastatic site for rectal cancer External anal
Mesorectum • Innervated by the inferior rectal
– total mesorectal excision (TME) sphincter,
branch of the internal pudendal nerve
correlates to a higher survival rate and puborectalis
lower recurrence among patients with • Innervated by both the internal pudendal
Levator ani
rectal cancer nerve, and direct branches of S3 to S5
Sensory innervation • Innervated by the inferior rectal
to the anal canal branch of the pudendal nerve
Sensory innervation
• Insensate
to the rectum
Anal canal BELOW
• Receives somatic innervation
the dentate line

ARTERIAL SUPPLY OF THE ANORECTAL REGION


Upper rectum • Superior rectal artery ← IMA
Middle rectum • Middle rectal artery ← internal iliac artery
• Inferior rectal artery ← internal pudendal
Inferior rectum
artery ← internal iliac artery

VENOUS DRAINAGE OF THE ANORECTAL REGION


• Parallels the arterial supply of the anorectal region
Upper rectum • Superior rectal vein → IMV
Middle rectum • Middle rectal vein → internal iliac vein
• Inferior rectal vein → internal pudendal
Inferior rectum
vein → internal iliac vein

LYMPHATIC DRAINAGE OF THE RECTUM AND ANUS


• Parallels the vascular supply of the anorectal region
Upper rectum,
• Inferior mesenteric lymph nodes
Middle rectum
Lower rectum,
• Inferior mesenteric lymph nodes
Anal region proximal
• Internal iliac lymph nodes
to the dentate line
Low anterior resection (LAR)
• Inguinal lymph nodes (primary) • generally performed for lesions in the middle and upper third of the
Distal to the dentate
• Inferior mesenteric lymph nodes rectum and, occasionally, for lesions in the lower third
line
• Internal iliac lymph nodes • anal-sparing procedure
• temporary colostomy or ileostomy might be necessary to protect
anastomosis
TOPNOTCH MEDICAL BOARD PREP SURGERY MAIN DIGITAL HANDOUT BY LOUBOMIR ANTONIO, MD Page 98 of 101
For inquiries visit www.topnotchboardprep.com.ph or https://www.facebook.com/topnotchmedicalboardprep/
This handout is only valid for the September 2021 PLE batch. This will be rendered obsolete for the next batch since we update our handouts regularly.
TOPNOTCH MEDICAL BOARD PREP SURGERY MAIN DIGITAL HANDOUT BY LOUBOMIR ANTONIO, MD
For inquiries visit www.topnotchboardprep.com.ph or https://www.facebook.com/topnotchmedicalboardprep/
This handout is only valid for the September 2021 PLE batch. This will be rendered obsolete for the next batch since we update our handouts regularly.
Abdominoperineal resection (APR) • Pathophysiology
• includes the resection of part of the sigmoid colon, rectum, and anus o Sliding anal cushion theory: repeated stretching causes
• construction of a permanent end colostomy fragmentation of the submucosal muscle of Treitz which anchors
• indications: perianal skin involvement, puborectalis and sphincter and suspends the anal lining
involvement, fecal incontinence
Dr. Cocos
o Proposed factors for the development of hemorrhoids:
§ Deterioration of connective tissue anchors
§ Increased tone of the internal anal sphincter
DISORDERS OF THE RECTUM AND ANUS § Abnormal distension of the veins of the internal hemorrhoids
RECTAL PROLAPSE • Clinical Manifestations
• Pathophysiology TYPE DESCRIPTION AND MANIFESTATIONS
o Circumferential, full-thickness protrusion of the rectal wall • Located above the dentate line
through the anal orifice • Covered with mucosa
o More common in women than in men; peaks in women >60 years Internal • Usually presents as painless bright red bleeding
• Common causes: internal intussusception of the rectum, long hemorrhoids during defecation
term result of an outlet obstruction, long-term laxative use • Can progress into four different stages (see
below)
RECTAL PROLAPSE HEMORRHOIDS • Located below or distal to the dentate line
Tissue folds Circumferential Radial • Covered by anoderm
Sulcus between External • Usually with itching and pain around the anus
Circumferential None hemorrhoids
prolapse & rectum (because the anoderm is richly innervated),
Abnormality on Hemorrhoidal especially when thrombosed
Double rectal wall • A skin tag may remain after healing
palpation plexus
Resting and • Combination of external and internal
Decreased Normal
squeeze pressure Mixed hemorrhoids
Surgery Platinum. 1st ed. 2018. p. 323 hemorrhoids • Hemorrhoidectomy; often required for large,
symptomatic, mixed hemorrhoids
Surgery Platinum. 1st ed. 2018. p. 324
• Diagnostics
o Patients with anorectal complaints must undergo anoscopy,
rigid proctosigmoidoscopy, and/or flexible sigmoidoscopy
(further work-up depends on physical examination, patient age,
if more than 50 years old & history)
o American Society for Gastrointestinal Endoscopy and the Society
for Surgery of the Alimentary Tract guidelines suggest anoscopy
and flexible sigmoidoscopy for bright-red rectal bleeding
• Diagnosis
• Principles of Management
o Concentric rings of mucosa on examination
• Main goal of this treatment is to minimize
o Proctoscopy, sigmoidoscopy, or colonoscopy:
straining
§ Reveals congestion and edema of the distal rectal mucosa
• Warm sitz bath (40 C): most effective topical
§ Also done to rule out malignancy as a lead point of prolapse treatment for relief of symptoms (soaking time
• Management Nonsurgical
of 15 minutes)
Stool-bulking agents or fiber supplementation to • Supportive: increasing fluid and fiber in the diet,
Nonsurgical
ease process of evacuation recommending exercise, and adding
Surgical correction of rectal prolapse is the mainstay supplemental fiber agents
Surgical of therapy – transperineal or transrectal • Rubber band ligation (RBL): elastic bands are
approach applied onto an internal hemorrhoid at least 1
Note: A transperineal or transrectal procedure is preferred for patients who cm above the dentate line to cut-off the blood
are high-risk (i.e. elderly, multiple comorbidities, etc.) Procedures supply
Dr. Cocos
• Sclerotherapy: injection of a sclerosing agent
HEMORRHOIDS into the hemorrhoid, causing the veins to
• Normal part of the anal canal collapse
• Aid in continence by preventing damage to the sphincter muscle • Excision hemorrhoidectomy: surgical excision
• Engorgement of the venous plexus of rectum or anus with of hemorrhoids, usually recommended for
protrusion of the mucosa & anal margin thrombosed external hemorrhoids
• Three main hemorrhoidal complexes (cushions) traverse the anal • Either closed (Parks-Ferguson) or open
canal: (Milligan-Morgan) technique
o Left lateral (3 o’clock) Surgical • Also indicated for the following:
o Right posterior (7 o’clock) o Failure of conservative management
o Grade III-IV internal hemorrhoids with severe
o Right anterior (11 o’clock)
symptoms
o Concomitant anorectal conditions (e.g., anal
fissure or fistula)
Surgery Platinum. 1st ed. 2018. p. 324

• Management of External Hemorrhoids


o If the pain is intense (onset <72 hours): excision
o If the pain is subsiding: conservative management may suffice
(e.g., warm sitz baths, analgesics, and bulk-producing fiber
supplements)
o Anoscopy and proctoscopy to rule out associated anorectal
disease are postponed to a later date when the patient is not in
acute pain
MANAGEMENT OF INTERNAL HEMORRHOIDS
MANAGEMENT
GRADE DESCRIPTION
MEDICAL SCLEROTHERAPY RBL SURGERY
I • Protrudes through the anal canal, but not beyond the anal verge Yes Yes Yes
II • Protrusion, but with spontaneous reduction Yes Yes Yes
III • Protrusion requiring manual reduction Yes Yes* Yes* Yes
IV • Protrusion that cannot be reduced (at risk for strangulation) Yes Yes
* Selected cases of Grade III hemorrhoids only
Surgery Platinum. 1st ed. 2018. p. 325

TOPNOTCH MEDICAL BOARD PREP SURGERY MAIN DIGITAL HANDOUT BY LOUBOMIR ANTONIO, MD Page 99 of 101
For inquiries visit www.topnotchboardprep.com.ph or https://www.facebook.com/topnotchmedicalboardprep/
This handout is only valid for the September 2021 PLE batch. This will be rendered obsolete for the next batch since we update our handouts regularly.
TOPNOTCH MEDICAL BOARD PREP SURGERY MAIN DIGITAL HANDOUT BY LOUBOMIR ANTONIO, MD
For inquiries visit www.topnotchboardprep.com.ph or https://www.facebook.com/topnotchmedicalboardprep/
This handout is only valid for the September 2021 PLE batch. This will be rendered obsolete for the next batch since we update our handouts regularly.
SUPRALEVATOR ABSCESS
• May result from an upward
extension of an intersphincteric
abscess – should be drained
transrectally
o Drainage through the rectum
may result in an
extrasphincteric fistula
• May result from an upward
extension of an ischiorectal
abscess – should be drained
through the ischiorectal fossa
• May be secondary to an
intraabdominal disease – should
be drained via the most direct
route (transabdominally,
rectally, or through the
ischiorectal fossa)
This figure illustrates how rubber band ligation is done. It is a clinic-based
procedure and does not warrant local anesthesia. ✔ GUIDE QUESTION
Dr. Cocos A 43-year-old man is seen in his physician’s office for severe pain in the
perineum. Examination reveals exquisite tenderness in the area to the
right side of the anal verge due to a perianal abscess. Rectal examination
is refused. What should be the next step in management?
(A) Drainage of the abscess in the office under local anesthesia.
(B) Excision of the vertical fold of Morgagni.
(C) Drainage under general anesthesia and immediate colonoscopy.
(D) CT scan of the abdomen.
(E) Insertion of a rectal tube.
The ducts of the anal glands drain into the anus and are covered by the
vertical columns of Morgagni (but choice B is wrong, there is no need to
excise this). Infection of these glands may account for some cases of
perianal abscess. A fluctuant perianal abscess should be drained to
relieve the symptoms of this patient. It can be done as a bedside
procedure if clinically there is no extension to deeper structures. There
is no need to do the drainage under general anesthesia and colonoscopy
can be done electively. There is no indication to do a CT scan and
inserting a rectal tube will not address the perianal abscess.
Dr. Cocos

FISTULA-IN-ANO
This figure illustrates Parks-Ferguson hemorrhoidectomy wherein you • Chronic form of anorectal abscess
close the defect after hemorrhoidectomy, in contrast to Milligan-Morgan
• Etiology - infected crypt forms a tract extending to an external
method where you do not suture the defect (preferred for thrombosed
hemorrhoids). opening as a site of drainage
Dr. Cocos • Diagnosis: Goodsall’s rule
• Treatment (goal: eradication of sepsis without sacrificing
ANAL FISSURE continence)
• A tear in the anoderm just distal to the dentate line • Lays open the fistula tract connecting the
• Etiology: hard stools or prolonged diarrhea Fistulotomy internal and external opening
o Acute and sharp pain is caused by spasm of the internal • For superficial fistulas
sphincter, causing further tearing • Identification of the intersphincteric tract
LIFT
• Treatment: and ligation close to the internal sphincter
(ligation of
• Medical management (stool softener, • Then, it is divided distal to the point of
intersphincteric
Acute anal fissure bulk diet, topical nitrates, botulinum ligation
fistula tract)
toxin A) • For transsphincteric fistula
Chronic anal fissure • Lateral internal sphincterotomy

ANORECTAL ABSCESS
• Infection of the anal glands found in the intersphincteric space
• Etiology – glands open into the anal crypts at the dentate line
• Diagnosis
o Severe anal pain precludes digital rectal examination (DRE)
o Rectal exam under anesthesia
• Treatment – drainage of anorectal abscess

Goodsall’s rule predict the course of a fistula tract. Any fistula that originates
anterior to the transverse line will course anteriorly in a direct or radial route.
Fistulae that originate posterior to the line will have a curved path and the
internal opening is usually at the posterior midline. An exception to the rule
are anterior fistulae lying more than 3 cm from the anus, which usually has a
curvilinear tract draining into the posterior midline as well.
Dr. Cocos

ANAL FISTULA
https://qrs.ly/83bjes2

TOPNOTCH MEDICAL BOARD PREP SURGERY MAIN DIGITAL HANDOUT BY LOUBOMIR ANTONIO, MD Page 100 of 101
For inquiries visit www.topnotchboardprep.com.ph or https://www.facebook.com/topnotchmedicalboardprep/
This handout is only valid for the September 2021 PLE batch. This will be rendered obsolete for the next batch since we update our handouts regularly.
TOPNOTCH MEDICAL BOARD PREP SURGERY MAIN DIGITAL HANDOUT BY LOUBOMIR ANTONIO, MD
For inquiries visit www.topnotchboardprep.com.ph or https://www.facebook.com/topnotchmedicalboardprep/
This handout is only valid for the September 2021 PLE batch. This will be rendered obsolete for the next batch since we update our handouts regularly.
✔ GUIDE QUESTION • Small and not invading sphincters: local excision
A 25-year-old man has recurrent, indolent fistula in ano. He also • Large, invading sphincters or fungating: palliative
complains of weight loss, recurrent attacks of diarrhea with blood mixed
Melanoma
abdominoperineal resection (APR)
in the stool, and tenesmus. Proctoscopy revealed a healthy, normal- • Associated with very poor prognosis
appearing rectum. What is the most likely diagnosis? Surgery Platinum. 1st ed. 2018. p. 340
(A) Crohn’s disease HIRSCHSPRUNG’S DISEASE
(B) Ulcerative colitis • Developmental disorder
(C) Amoebic colitis
characterized by the
(D) Ischemic colitis
(E) Colitis associated with acquired immunodeficiency syndrome (AIDS) absence of ganglia in the
With Crohn's disease (unlike ulcerative colitis), a deep ulcer or a sore in distal colon, resulting in a
the inflamed intestinal tract can spread and tunnel into deeper tissues functional obstruction –
— most commonly in and around the anus — resulting in a due to absence of the
perianal fistula. It is a rectal-sparing disease (unlike ulcerative colitis, so Auerbach plexus
choice B is wrong). Symptoms of weight loss, recurrent attacks of (Schwartz)
diarrhea, hematochezia and tenesmus can be present in both ulcerative • Manifestations:
colitis and Crohn’s. Other colitis (amoebic, ischemic, AIDS-associated)
o Delayed passage of
may present with recurrent episodes of diarrhea and bleeding however
a concomitant fistula-in-ano is unlikely. meconium more than 24
Dr. Cocos hours after birth
ANAL REGION NEOPLASMS o Collapsed rectal ampulla
with explosive discharge
• Anal canal cancers are more common in women
of liquid feces on rectal
• Anal margin cancers are more common in males
Epidemiology examination
• Carcinomas of the anal margin have a better
prognosis than that of tumors of the anal canal • Diagnosis
• Cigarette smoking Abdominal
• HPV infection • Shows pattern of distal obstruction
radiograph
• HIV infection • HSV infection
Risk factors • Demonstrates a transition zone
• Promiscuity and male • Chronic anal
for anal Barium enema between distal nondilated bowel and
homosexuality fistula
cancer proximal dilated bowel
• Immunosuppression • Crohn disease
and transplant patients • Prior irradiation • Gold standard for diagnosis:
(+) absence of ganglion cells in the
CLASSIFICATION REMARKS myenteric and submucosal plexus
Suction rectal biopsy
• Above the dentate line (+) increased acetylcholinesterase
• Extends from the anorectal junction to the positive nerve fibers
Anal canal anal margin (+) hypertrophied nerve bundles
tumors • Lesions that cannot be visualized at all (or are • Surgical Options
incompletely visualized) while gentle
traction is placed on the buttocks
• Below the dentate line
• Pigmented skin immediately surrounding the
Anal margin anal orifice, extending laterally to a radius
tumors of 5 cm
• Completely visible when gentle traction is
placed on the buttocks
• Fall outside of the 5 cm radius of the anal
Skin
opening
Surgery Platinum. 1st ed. 2018. p. 339
Duhamel – side-to-side anastomosis; residual pouch of aganglionic bowel
• Clinical Manifestations left intact with the ganglionic bowel attached behind
o Small, early cancers are sometimes diagnosed incidentally Swenson - resection of aganglionic segment with end to end anastomosis
following simple anal procedures Soave - mucosa of aganglionic segment stripped but the outer muscular
o Most common symptom of anal carcinoma: anal bleeding cuff is left, and anastomosis done
o More advanced lesions are usually encountered in the distal anal Dr. Cocos

canal (present with mass, non-healing ulcer, pain, bleeding, IMPERFORATE ANUS
itching, discharge and fecal incontinence) • Results from failure of the rectum to descend through the
• Diagnosis external sphincter complex, can be a part of a spectrum of
o Digital rectal examination (DRE) & vaginal examination – anorectal malformations
should determine site and size of the primary tumor and nodal
involvement
o Careful clinical assessment of the inguinal nodes is important IMPERFORATE ANUS
o Proctoscopy – facilitates biopsy and clarification of anatomical https://qrs.ly/wdcl1bn
relations to surrounding structures.
o Histological confirmation is mandatory
• Staging process is completed by: • Types of imperforate anus
o CT of the chest, abdomen, and pelvis High imperforate anus • Above the levator ani muscle
o Endoanal ultrasound – to assess depth of invasion and aid in Low imperforate anus • Below the levator ani muscle
establishing the size of the tumor • Management
• Management Diagnosis • Invertogram
NATURE • Low imperforate anus: perineal approach without
OF THE MANAGEMENT Treatment colostomy
MASS • High imperforate anus: colostomy then pull-through
• Nigro protocol (S-FU, mitomycin C, and
radiotherapy): standard of treatment associated
Anal canal with 80% success rate
(SCC) • For <1 cm size & well differentiated: wide local
excision
• Sessile lesions removed in piecemeal technique
• Primarily treated by surgical excision
Anal o No sphincter invasion: wide local excision to
margin negative margins
(SCC) o Sphincter involvement of large mass:
chemotherapy + radiotherapy

TOPNOTCH MEDICAL BOARD PREP SURGERY MAIN DIGITAL HANDOUT BY LOUBOMIR ANTONIO, MD Page 101 of 101
For inquiries visit www.topnotchboardprep.com.ph or https://www.facebook.com/topnotchmedicalboardprep/
This handout is only valid for the September 2021 PLE batch. This will be rendered obsolete for the next batch since we update our handouts regularly.
TOPNOTCH MEDICAL BOARD PREP SURGERY MAIN DIGITAL HANDOUT BY LOUBOMIR ANTONIO, MD
For inquiries visit www.topnotchboardprep.com.ph or https://www.facebook.com/topnotchmedicalboardprep/
This handout is only valid for the September 2021 PLE batch. This will be rendered obsolete for the next batch since we update our handouts regularly.

END OF SURGERY – PHASE 0

a bit more. I did the right thing during the interview which was stand up for
my beliefs so I could look strong and determined, even when faced with a
Important Legal Information consultant who aggressively insulted Opus Dei, the Catholic Church,
The handouts, videos and other review materials, provided by Topnotch Medical Board
Preparation Incorporated are duly protected by RA 8293 otherwise known as the Intellectual
opponents of the RH bill and the local bishop all in the same breath. Soon
Property Code of the Philippines, and shall only be for the sole use of the person: a) whose however it became clear that none of those things prepared me for the sheer
name appear on the handout or review material, b) person subscribed to Topnotch Medical intellectual load of med school. So many diseases. So many histopathologic
Board Preparation Incorporated Program or c) is the recipient of this electronic correlations. So many bones and insertions and actions and so many
communication. No part of the handout, video or other review material may be reproduced,
pathways and chemicals and so many drugs and mechanisms of action. So
shared, sold and distributed through any printed form, audio or video recording, electronic
medium or machine-readable form, in whole or in part without the written consent of many congenital anomalies, so many degenerative diseases, and the
Topnotch Medical Board Preparation Incorporated. Any violation and or infringement, overwhelming feeling that we’re expected to know the S-O-A-P of each one—
whether intended or otherwise shall be subject to legal action and prosecution to the full extent that’s maybe the one thing that has never changed in all five years of med
guaranteed by law. school. So many nights studying.

DISCLOSURE I don’t regret a single one.


The handouts/review materials must be treated with utmost confidentiality. It shall be the
responsibility of the person, whose name appears therein, that the handouts/review materials It doesn’t matter that my intellectual retentive capacity is the size of a
are not photocopied or in any way reproduced, shared or lent to any person or disposed in any dessert spoon and I don’t remember a single thing I studied all those
manner. Any handout/review material found in the possession of another person whose name
does not appear therein shall be prima facie evidence of violation of RA 8293. Topnotch review nights at the Gloria Jean’s in Pan Pacific with my roommate Dane. It also
materials are updated every six (6) months based on the current trends and feedback. Please doesn’t even matter that I didn’t make the best use of those nights thanks to
buy all recommended review books and other materials listed below. Facebook and this wonderful thing called fanfiction. What matters is that,
THIS HANDOUT IS NOT FOR SALE! surrounded by those better than me, smarter than me, more hardworking
than me, I found myself having to try harder. Maybe I did luck out. But I would
Dr. Banzuela: The following article is one of the best written pieces about Medical fight to deserve the slot that my country gave me.
Internship in the Philippines. All the fear, joy, angst, and the lessons have been Thankfully, studying became less of a chore by the time clerkship rolled
captured. You can substitute the phrase “PGH” with any other Government Hospital; around. Unfortunately, the battleground was no longer the classroom or the
and substitute the phrase “UPCM” with any other Philippine Med School – and the histology lab but had become something far worse: the wards and the ER. I
article would still hold true - the experiences by Dr. Rivera are surprisingly universal could write a person-sized scroll of tips for those who are starting clerkship
among Filipino Physicians – and the emotions even more so. Internship experiences or internship in PGH. Where do you find a first-year to sign your SAPOD
binds us all, the Med Boards binds us all, and Topnotch binds us all. referral? Where can you find heparin? What is the easiest route from the
Enjoy. =)
NICU to the main labs? (The answer involves trekking down a dark stairwell
from the anesth offices. Not a recommended route for 3 in the morning.)
UP Medicine: the (un)easy life
Clerkship and internship, apart from being intellectually demanding and
By Topnotch MD Kay Rivera being lived with the prospect of perpetual embarrassment, had the added
“Some things are hard to write about. After something happens to you, you go to write it down,
component of a Need for Speed. Being “mabagal” earned you whispers
and either you over dramatize it, or underplay it, exaggerate the wrong parts or ignore the from your residents and, on one memorable occasion, a loud “FUUUCK!
important ones. At any rate, you never write it quite the way you want to.” — Sylvia Plath, The PUTAAAANG INAAAAAA!” from a resident to me. It was trying and horrible
Unabridged Journals of Sylvia Plath and often made me feel like I had been sent out into battle with no weapons.
If this was Survivor, I was the fat loser who got voted off the island on the
Recently my blockmate Fred Ting’s post on why an intern should do his or first week for being so lampa and not offering anyone a survival
her post graduate internship in PGH has been getting a lot of attention and advantage. Being surrounded by residents who interrupted your work to
my snide supportive blockmates suggested I write a version of my own. I ask for cotton and leukoplast, and who still somehow managed to wonder
didn’t give it much thought because I’ve never been a popular blogger and I what was taking you so long to do other things… being left to figure things
don’t think my thoughts are that important. I’m also not the sort of person out on your own and being faced with annoyance when you had to ask a
whose thoughts will uplift or inspire, or whose writing style will inspire rave nurse or resident for instructions or directions… being expected to magically
reviews since my writing influences now mostly consist of badly written come up with tops and syringes and fluids in the OBAS and in the ER… it has
fanfic and Harrison’s Principles of Internal Medicine. But on my walk home so far been taxing, frustrating and, let’s face it, a little soul-destroying. It has
today, after a 12 hour duty at the Pedia ER, I was thinking of how I would also been joyful, exhilarating and probably the best experience of my life.
forget all the little things that made life in PGH so happy and so terrible and I won’t pretend that I’ll ever earn any awards as Most Outstanding Intern of
how I wanted to preserve them, like insects and their spindly legs in amber, anything. I won’t pretend that I didn’t complain, didn’t sleep when I wasn’t
to mull over later, to look at the lessons I might be missing. Hence this entry. supposed to, didn’t fall asleep during lectures or conferences, didn’t fudge
Which I may never write the way I truly want to but will at least help me any monitoring. I have so far been the average PGH intern—maybe less than
remember. average because for everyone thing I did right, I got maybe three things
wrong. But I want to go on record as saying that every time I did get
something right, I was truly happy. Being praised during rounds with an
Do I think that med school was a good decision? Yes. Do I think that med IM consultant I deeply respected; being told my stitches were perfect by a
school in the UP College of Medicine, and subsequently clerkship in the uro surgeon that I would later idolize; every single time I read “highly
Philippine General Hospital, might be the best decision I could have made? appreciate SIC efforts!” or “please carry out Dr Rivera’s orders”, God. You
Yes, and yes, with qualifications. can’t buy that happiness. And in PGH it felt so hard-earned. Maybe it’s the
juxtaposition with so much sadness that makes every happiness seem all the
I was miserable throughout the first years of med school; I won’t hesitate brighter. Maybe this was just me growing up to realize that truly, nothing
to say. I felt alone and bullied and unintelligent. There was a lot of gazing worthwhile is ever easy.
into coffee cups and staring over landscapes regretting the fact that I hadn’t
taken some course I would be naturally good at, like maybe foreign Also—and I’m not endorsing PGH clerkship/internship as the best way to
languages, instead surrounding myself with these people who were not only learn resourcefulness or anything—PGH truly made me realize how
ridiculously smart but also ridiculously talented at other things I could never necessity is the mother of invention. We are not unique among Filipino
hope to be good at. I’ll say this for med school: it brings you closer to people hospitals in learning how to make do with Makeshift Anything ™.
who are highly intelligent in so many different areas. In LU3 we were
constantly told we were the cream of the crop. I still believe my batchmates How many IV bottles and drinking water bottles have we cut in half for ABG
to be so. UP Medicine allows so many opportunities for people to thrive in so transport purposes? What would we do without splints made out of pieces
many different fields and the best parts of those first three years were of cardboard? Who knew that we could save lives using a Frixxion Pen? We
actually the extracurricular activities—the Tao Rin Pala in December that I are given so many tasks and so few resources to use. I know that it gets worse
had been waiting for years to be a part of; Mediscene and Medisings and an in residency when doctors shell out tens of thousands of pesos just to get a
infinite number of activities made with med school puns added color to those patient in the OR or just so a patient can have some blood from the Bank. But
gray days when we sat and stewed in the BSLR, growing fat and indolent, as an intern who’s feeling the burden of nine years of parasitizing off my
while consultants stared at us over this chasm of experience and parents, I felt like I was punched in the gut every time I had to shell out money
knowledge and tried their best to motivate us into learning more, for stash. And yet our patients live, and so do I, more resourceful than ever.
integrating better, seeing the bigger picture. A word on bedside manner is also probably indicated here. When I started
Speaking of intelligence: did I sometimes wish, for the sake of my battered out in PGH I heard from a friend of my mother’s that PGH doctors were
self-esteem, that I’d taken up medicine elsewhere so I could shine brighter in famous for being great at their fields and terrible with patients. I can name
my studies? Never. Within a month of med school, I came to accept that maybe one or two exceptions, but I don’t think I’m wrong when I say that
maybe, just maybe, I’d lucked out when I made it here. I did the right thing EVERYONE, every intern who has passed through the doors of PGH, has
by shifting from a harder course—BS Biochemistry—to the still-difficult-but- snapped at a patient. It doesn’t have to be a shout or a snide comment like
rumored-to-get-higher-acceptance-rates BS Biology. I also did the right thing the type I excel at, but everyone has felt the pressure of getting something
by enrolling at a review center before taking my NMAT so I could get the done and the frustration of being interrupted by a seemingly stupid question
coveted 99 and above percentile marks to make my application stand out just or being held up by a slow bantay. It should probably go without saying that
TOPNOTCH MEDICAL BOARD PREP SURGERY MAIN DIGITAL HANDOUT BY LOUBOMIR ANTONIO, MD Appendix
For inquiries visit www.topnotchboardprep.com.ph or https://www.facebook.com/topnotchmedicalboardprep/
This handout is only valid for the September 2021 PLE batch. This will be rendered obsolete for the next batch since we update our handouts regularly.
TOPNOTCH MEDICAL BOARD PREP SURGERY MAIN DIGITAL HANDOUT BY LOUBOMIR ANTONIO, MD
For inquiries visit www.topnotchboardprep.com.ph or https://www.facebook.com/topnotchmedicalboardprep/
This handout is only valid for the September 2021 PLE batch. This will be rendered obsolete for the next batch since we update our handouts regularly.
in the ideal world, a bantay is not even supposed to be part of the health team.
Watchers are supposed to sit back and relax as best as they can while
worrying for their patients. Interns and clerks should do the scutwork. It
should also be said that if such a system were to be upheld in PGH without a
change in the number of students, our patients would truly suffer. You’d
think that the best we can do would be to at least stay polite and caring to
our patients’ families while they do jobs that we were supposed to do. But
that’s not what happens. Am I ashamed of that? Yes, more than I can say. Do
I think I could have helped it? Maybe sometimes, but not always. The human
body can only take so much tiredness and academic pressure before
succumbing to brisk manner and brusque words. So how will these
anecdotes ever help someone deciding if they want to go to PGH for clerkship
and internship? The one thing I can say is that, while PGH makes monsters of
us all (well, nearly all), we can be consoled by the fact that this is the meanest
we might ever be. It’s all uphill from here. After internship I think we know
our limits better. I think we’re more self-aware. I think we know our
triggers and, once the doors of PGH close behind us, we’ll be better able
to realize that we had no right to push our authority on these patients,
and to take advantage of their inexperience and possible ignorance to
make ourselves feel superior. It’s a humbling experience to know that, far
from the idealistic people we once were, we are actually just human. Tao
rin pala. And the struggle that goes into being kind may be a lifelong one, but
the fact that we continue struggling is all that matters.
This brings me to the one thing I think truly defines why someone should
choose the UP College of Medicine. Many others will give you a multitude of
reasons—the excellent academic environment where students are allowed
to challenge their superiors; the wealth of experience in seeing cases every
day that some doctors can go their whole lives without ever seeing. (This part
is true and even on its own might be worth those five frustrating years.) But
I am convinced that PGH’s greatest gift to its students is that it tells us where
we’re needed. I went into med school with idealistic dreams of putting up a
free clinic to make my parents proud, to serve in a different way. Growing up
here, we have seen firsthand the pitfalls of the health system, where money
makes the difference between life and death, between ability and disability.
At the same time, we have been educated about the bigger picture of health
policy and administration, with the attendant corruption and the
discrepancy between the ideal and the actual. We are not, we are NEVER
sheltered from the reality of poverty. When we are weak, PGH produces
doctors like this: doctors who grow tired of the demand of public service and
who dream of practicing abroad or in private hospitals, where money is
never an issue and you never have to think twice about ordering a CT scan or
a CK panel. Doctors who titter over their patients “na walang pera” who are
“chaka” and “dugyot.” But more often than not PGH, somehow, some way,
manages to bring out this kernel of goodness in most of us that allows us to
think of more important things than comfort or money. Some of us will go
into research and academe; some into health policy. I know one excellent
classmate of mine who has shaped the outline of her future to conform with
what is most needed in her home town. A lot of us will go into residency. But
none of us will ever be able to unsee what we have seen and all of us will go
off into the future knowing our place in the health system. We know how we
can contribute. We know what the poor endure. I used to remark to my
parents how amazing it was that leaders of the DOH, the WHO, and a number
of movers and shakers in the health sciences and industries are plucked from
the graduates of PGH. Now I understand why. I think it has less to do with
intellectual ability than with feeling, with attitude, with experience, with
exposure, with desire and passion for serving the underserved. Big
words; real truths.

PGH is not perfect. Going on triage makes me feel like an accessory to a crime.
Scrubbing into an OR with male surgeons is like arming yourself in
preparation for sexist barbs and blatant harassment. Asking most nurses for
help (with exceptions of course), thanks to the history of backstabbing and
the unfortunate tradition that is the Sunog-Puri list, can be the most
challenging part of a duty. If you are an incoming clerk or intern, I am telling
you right now that there will be weeks when you spend more on stash than
on your food. Your CM classmates will have their flaws and so will your PGIs.
The work is moderately difficult, but the posts are many. It will be difficult to
deck duties. Nobody said it was easy. But who wants an easy life? Theodore
Roosevelt wrote, “Nothing in the world is worth having or worth doing
unless it means effort, pain, difficulty… I have never in my life envied a
human being who led an easy life. I have envied a great many people who
led difficult lives and led them well.” Truer words, never spoken.

They say you don’t know joy until you have known hardship. Thank you,
PGH, for helping me make my acquaintance with both. No number of blog
entries will ever do the experience justice. Thank you for the amazing five
years; I’m glad I lucked out.

Doctor Rivera, public servant, monitoring machine, NICU manong, labs master,
JWAPOD, signing out.

TOPNOTCH MEDICAL BOARD PREP SURGERY MAIN DIGITAL HANDOUT BY LOUBOMIR ANTONIO, MD Page Appendix
For inquiries visit www.topnotchboardprep.com.ph or https://www.facebook.com/topnotchmedicalboardprep/
This handout is only valid for the Sept 2020 PLE batch. This will be rendered obsolete for the next batch since we update our handouts regularly.
TOPNOTCH MEDICAL BOARD PREP SURGERY PHASE 0 CLARIFICATIONS HANDOUT
For inquiries visit www.topnotchboardprep.com.ph or https://www.facebook.com/topnotchmedicalboardprep/
This handout is only valid for September 2021 PLE batch. This will be rendered obsolete for the next batch since we update our handouts regularly.

INSTRUCTIONS
For those who have printed the initial handout:
Please use this handout as a guide to correct the initial handout.
Page guides are available to assist you in doing so.

CORRECTIONS
• Page 3 RLQ, on Hypocalcemia

6. HYPOcalcemia
• Serum calcium level below 8.5 mEq/l or a decrease in the
ionized calcium level below 4.2 mg/dl
• Causes include pancreatitis, malignancies associated with
increased osteoblastic activity (breast and prostate cancer),
massive soft tissue infections such as necrotizing fasciitis, renal
failure, pancreatic and small bowel fistulas, hypoparathyroidism,
toxic shock syndrome, and tumor lysis syndrome

• Page 12 RUQ, on trauma

✔ GUIDE QUESTION
A 70-year-old man is brought into the emergency department
following his injury as a passenger in a car crash. He complains of
right-side chest pain. Physical examination reveals a respiratory rate
of 42 breaths per minute and multiple broken ribs of a segment of the
chest wall that moves paradoxically with respiration. What should the
next step be?
(A) Tube thoracostomy (D) Endotracheal intubation
(B) Tracheostomy + PEEP (E) Intercostal nerve blocks
(C) Needle Thoracostomy
This is a sample case of a flail chest injury. Prompt intubation is
needed to secure the airway.

END OF SURGERY PHASE 0 CLARIFICATIONS

TOPNOTCH MEDICAL BOARD PREP PEDIATRICS PHASE 0 CLARIFICATIONS HANDOUT Page 1 of 1


For inquiries visit www.topnotchboardprep.com.ph or email us at topnotchmedicalboardprep@gmail.com
This handout is only valid for the September 2021 PLE batch. This will be rendered obsolete for the next batch since we update our handouts regularly.
TOPNOTCH MEDICAL BOARD PREP SURGERY PHASE 2 HANDOUT BY DRS. COCOS-ALCANTARA, ASPERAS, RUBIO
For inquiries visit www.topnotchboardprep.com.ph or https://www.facebook.com/topnotchmedicalboardprep/
This handout is only valid for the September 2021 PLE batch. This will be rendered obsolete for the next batch since we update our handouts regularly.
During resuscitation of a patient in shock, which of
Important Legal Information
The handouts, videos and other review materials, provided by Topnotch Medical Board the following statements is false?
Preparation Incorporated are duly protected by RA 8293 otherwise known as the A. Administration of inotropic agents to an empty
Intellectual Property Code of the Philippines, and shall only be for the sole use of the person: heart will help to increase diastolic filling and
a) whose name appear on the handout or review material, b) person subscribed to Topnotch
Medical Board Preparation Incorporated Program or c) is the recipient of this electronic coronary perfusion.
communication. No part of the handout, video or other review material may be reproduced, B. In all cases, regardless of classification,
shared, sold and distributed through any printed form, audio or video recording, electronic
medium or machine-readable form, in whole or in part without the written consent of 5. hypovolemia and preload must be addressed first.
Topnotch Medical Board Preparation Incorporated. Any violation and or infringement, C. The oxygen-carrying capacity of both colloids
whether intended or otherwise shall be subject to legal action and prosecution to the full and crystalloids is zero.
extent guaranteed by law.
D. Hypotonic solutions are poor volume
expanders and should not be used in shock
DISCLOSURE except in conditions of free water loss or
The handouts/review materials must be treated with utmost confidentiality. It shall be the
responsibility of the person, whose name appears therein, that the handouts/review sodium overload.
materials are not photocopied or in any way reproduced, shared or lent to any person or The effectiveness of prophylactic antibiotics in
disposed in any manner. Any handout/review material found in the possession of another
person whose name does not appear therein shall be prima facie evidence of violation of RA
surgery is mostly related to the:
8293. Topnotch review materials are updated every six (6) months based on the current A. use of broad-spectrum agents
trends and feedback. Please buy all recommended review books and other materials listed B. continuation of antibiotics for 24 hours after
below. 6.
THIS HANDOUT IS NOT FOR SALE! surgery
C. timing of initial administration
D. use of two synergistic agents
REMINDERS E. use of bactericidal agents
For Phase 2:
1. Finish the Phase 0 handout and Phase 1 video before proceeding to the
Which of the following statements about types of
Phase 2 handout and video. wounds is true?
2. Phase 2 handouts are based on commonly used review books and A. The infection rate in a ‘clean wound’ is between 1-2%
previous question feedback from students. B. The wound after a biliary surgery is classified
2. Answer the Pre-Test (Guide Questions) first prior to watching the video 7. as ‘contaminated’.
lectures. C. A ‘contaminated wound’ has an infection rate
3. The guided content of the video lectures are in the 2nd part of the Phase of less than 3 per cent.
2 handouts and are meant to complement the video lecture. D. Antibiotic prophylaxis would be mandatory in
‘dirty wounds’.
This handout is only valid for the September 2021 PLE batch. Which of the following is false regarding the
This will be rendered obsolete for the next batch sequence of events in wound healing?
since we update our handouts regularly. A. The initial vascular response is
vasoconstriction, followed by vasodilation.
B. The phases of wound healing are
SURGERY – PHASE 2 8.
inflammatory, proliferative, and remodeling.
C. The dominant cells of the inflammatory phase
By Angeli Andrea Cocos-Alcantara, MD are macrophages.
Kurt Roland A. Asperas, MD D. The proliferative phase is mediated by fibroblasts.
Frinz Moey C. Rubio, MD, MD E. The content of collagen in the wound
continues to increase throughout the
remodeling phase.
Sequence of return of gastrointestinal motility after
“Acute abdomen” commonly refers to possible
surgery is:
surgical problem occurring abdominally and
A. intestine, stomach, colon
necessitating immediate action. The most constant
1. B. stomach, intestine, colon
component in this symptoms complex is:
C. colon,intestine,stomach 9.
A. Pain
D. colon, stomach, intestine
B. Tenderness
E. stomach,colon,intestine
C. Rigidity
A 42-year old female underwent cholecystectomy
D. Distention
and 16 hours after the operation she developed
The most common fatal infection in burn victims is:
fever. This is most probably due to:
A. pneumonia
2. A. Pneumonia
10. B. venous line–related sepsis
B. UTI
C. burn wounds sepsis
C. Atelectasis
D. urinary tract infection
D. Wound infection
A 25-year-old motorcyclist comes in on a spine
Which if the following can be seen in SIRS?
board with cervical spine control. He was found at
A. Hypothermia <37oC
the roadside unconscious following a collision with
B. White cell count (WBC) = 4.0
3. an oncoming car. The airway is clear but his
C. No documented infection
breathing is clearly labored with a respiratory rate
D. Tachycardia >80/min
of 40/min. His pulse is 110/min and his blood
E. Tachypnea >24/min
pressure is 90/60. It appears that the wing mirror of
With regard to von Willebrand disease, which of the
the car has penetrated his chest and there is a
following statements is true? 11.
wound on the right side which is bubbling. He is not
A. It is more common than hemophilia.
responding to verbal command but is muttering
4. B. It is best treated with cryoprecipitate plasma.
incoherently. He will not open his eyes. What is the
C. Factor VIII levels are constant over time in a
next best step?
given patient.
A. Insert a chest drain.
D. Bleeding after elective surgery is rare.
B. Put a flap valve dressing over the bubbling wound
C. Start checking for other injuries.
D. Start a blood transfusion.
The most common complication of blood
transfusion is
A. hemolytic reaction
12. B. human immunodeficiency virus transmission
C. allergic reaction
D. volume overload
E. coagulopathy

TOPNOTCH MEDICAL BOARD PREP SURGERY PHASE 2 HANDOUT BY DRS. COCOS-ALCANTARA, ASPERAS, RUBIO Page 1 of 13
For inquiries visit www.topnotchboardprep.com.ph or email us at topnotchmedicalboardprep@gmail.com
This handout is only valid for the September 2021 PLE batch. This will be rendered obsolete for the next batch since we update our handouts regularly.
TOPNOTCH MEDICAL BOARD PREP SURGERY PHASE 2 HANDOUT BY DRS. COCOS-ALCANTARA, ASPERAS, RUBIO
For inquiries visit www.topnotchboardprep.com.ph or https://www.facebook.com/topnotchmedicalboardprep/
This handout is only valid for the September 2021 PLE batch. This will be rendered obsolete for the next batch since we update our handouts regularly.
The most important initial management of Metastasis of gastric malignancy may be noted in the
suspected blunt myocardial injury is: following sites:
A. ECG monitoring A. Krukenberg’s tumor – involvement of the
13. B. chest computed tomography scan pelvic cul-de-sac
23.
C. assessment of cardiac enzymes B. Blumer’s shelf – involvement of ovaries
D. insertion of pulmonary artery catheter C. Virchow’s node – ipsilateral axillary node
E. echocardiogram D. Sister Joseph’s nodule – deposit to the
A patient with blunt abdominal injury underwent a umbilicus
Focused Abdominal Sonogram for Trauma (FAST) Aside from hypotension, the additional criteria for
and the resident says it is positive. What does this classifying a patient with Charcot’s triad to be in
mean? Reynold’s pentad is:
A. The patient must undergo laparotomy 24. A. Tachycardia
14. B. There is fluid noted in the dependent portions B. Severe RUQ pain
of the peritoneal cavity C. Temperature > 39oC
C. There is intraperitoneal bleeding, most D. Sensorium Change
probably from the liver On digital rectal exam, a strong 63 year old male
D. The patient must undergo a triple-contrast sexually active male was found to have a 1.0cm
abdominal CT scan nodule on the right lobe of the prostate gland.
A patient with head injury opens his eyes and However, he was asymptomatic, what will be your
withdraws his arm to pain. He is making 25. next plan of management?
incomprehensible sounds. His Glasgow Coma Scale A. Transrectal prostate ultrasound
score is: B. Transabdominal KUB prostate ultrasound
15.
A. 12 C. Acid phosphatase determination
B. 10 D. Prostate specific antigen test
C. 8 A 28-year-old female volleyball player presents
D. 6 several weeks after having sustained an injury to
Gastroschisis: her left breast. She has a painful mass in the upper
A. is usually associated with other anomalies outer quadrant. Skin retraction is noticed, and a
B. is usually associated with chromosomal hard mass, 3–4 cm in diameter, can easily be
16. 26.
disorders palpated. What is the most likely diagnosis?
C. is located on the left of the umbilical cord A. Infiltrating carcinoma
D. repair is followed by prolonged ileus B. Breast abscess
The initial step in management of a 25-year-old C. Hematoma
male with a painless scrotal mass is: D. Fat necrosis
A. fine-needle aspiration and cytology After undergoing modified radical mastectomy for
B. abdominal computed tomography scan cancer of the right breast, a 52-year-old female
17.
C. observation and repeat examination in 2 teacher becomes aware that the medial end of her
weeks scapula becomes prominent in protraction
D. scrotal ultrasound movements at the shoulder. She also complains of
E. orchiectomy 27. some weakness in complete abduction of the same
What anatomic abnormality is present in an shoulder. What nerve was injured?
inguinal hernia in an infant? A. Long thoracic
A. Patent processus vaginalis B. Thoracodorsal
18. B. Weakness in the inguinal floor C. Medial pectoral nerve
C. Obliteration of the inguinal floor D. Lateral pectoral nerve
D. Congenital absence of both the external and A 62-year-old alcoholic presents with an indurated
internal rings ulcer, 1.5 cm in length, in the left lateral aspect of her
The classic triad of symptoms for renal cell cancer tongue. Biopsy reveals squamous cell carcinoma.
are the following EXCEPT: There is a palpable neck node at the left upper
A. Weight loss jugular chain of node (Level II) and FNAB showed
19.
B. Flank pain positive for malignant cells. What should she
28.
C. Gross hematuria undergo?
D. Palpable Flank mass A. Wide excision of tongue ulcer and neck node
Which of the following is true of thoracic anatomy? B. Wide excision and selective node dissection
A. The left lung has three lobes. C. Wide excision and modified radical neck
B. The azygous vein runs along the left side dissection
draining into the subclavian vein. D. Wide excision and radiotherapy
C. The vagus nerve runs anterior to the lung Following surgical resection of a large thyroid
20.
hilum. mass, a patient complains of persistent hoarseness
D. The sternocleidomastoid muscle is not and a weak voice. What is the most likely cause of
considered an accessory muscle to breathing. these symptoms?
29.
E. The phrenic nerve runs superior to the lung A. Traumatic intubation
hilum. B. Prolonged intubation
The following pertains to Gustillo and Anderson C. Injury to the recurrent laryngeal nerve
classification of fractures EXCEPT? D. Injury to the superior laryngeal nerve
A. It relies on the length of any laceration. Which of the following is used for surveillance of
21. B. It is influenced primarily by the energy involved. patients who underwent total thyroidectomy for
C. It takes account of whether or not there is soft- thyroid cancer?
tissue cover of fractured bone. 30. A. Anti-thyroglobulin
D. It takes account of contamination. B. TSH
The following statements are true regarding acute C. Thyroid Peroxidase
subdural hematoma EXCEPT: D. None of the above
A. It is a collection of blood between the dura and
arachnoid membranes
22.
B. It can be caused by laceration of the brain.
C. It can be due to disruption of a cortical blood
vessel.
D. It has a biconvex shape on CT scans.

TOPNOTCH MEDICAL BOARD PREP SURGERY PHASE 2 HANDOUT BY DRS. COCOS-ALCANTARA, ASPERAS, RUBIO Page 2 of 13
For inquiries visit www.topnotchboardprep.com.ph or email us at topnotchmedicalboardprep@gmail.com
This handout is only valid for the September 2021 PLE batch. This will be rendered obsolete for the next batch since we update our handouts regularly.
TOPNOTCH MEDICAL BOARD PREP SURGERY PHASE 2 HANDOUT BY DRS. COCOS-ALCANTARA, ASPERAS, RUBIO
For inquiries visit www.topnotchboardprep.com.ph or https://www.facebook.com/topnotchmedicalboardprep/
This handout is only valid for the September 2021 PLE batch. This will be rendered obsolete for the next batch since we update our handouts regularly.
A 45-year-old man presents with an upper A 68-year-old male musician with a sudden onset of
gastrointestinal bleed. An upper endoscopy reveals colicky abdominal pain and massive vomiting of 4-
multiple duodenal ulcers and an enlarged stomach. hour duration. Examination shows an elevated WBC
Further work-up showed that he has Zollinger of 13,200 with a HCT of 45%. An erect film of the
Ellison Syndrome occurring as part of MEN-1 abdomen reveals dilatation of the stomach with
31.
syndrome. The following is also expected: 39. minimal distended loops of bowel. What is his
A. Parathyroid hyperplasia clinical diagnosis?
B. Medullary thyroid cancer A. Complete jejunal obstruction
C. Pheochromocytoma B. Incomplete jejunal obstruction
D. Marfanoid habitus C. Complete ileal obstruction
A 64-year-old man develops increasing dysphagia D. Incomplete ileal obstruction
over many months. A barium swallow is performed. A 64-year-old female underwent screening
What is the most likely cause of his clinical colonoscopy. Nodularities were seen at the cecum
presentation? and biopsy results showed tuberculosis. There is no
32.
A. Carcinoma of the esophagus evidence of intestinal obstruction. What should the
B. Achalasia 40. next step in treatment involve?
C. Esophageal diverticulum A. Diagnostic laparoscopy
D. Paraesophageal hernia B. Laparotomy and ileal resection
A 64-year-old man has symptoms of reflux C. A full course of anti-tuberculous drugs
esophagitis for 20 years. The barium study D. Steroids
shown demonstrates a sliding hiatal A 32-year-old man presents to the ED with a 2-
hernia. Which is TRUE in sliding hiatal hernia? month history of alternating diarrhea and
33. A. A hernia sac is absent. constipation, rectal bleeding, a 20-lb weight loss,
B. The cardia is displaced into the posterior and worsening fatigue. What are the most common
mediastinum. genetic mutations that could have led to the
41.
C. Reflux esophagitis always occur. development of this patient’s colon cancer?
D. A stricture does not develop. A. APC, DCC, p53
A 55-year-old man complains of difficulty B. APC, BRCA1, K-ras
swallowing and underwent an EGD. How many C. DCC, p53, and MYH gene on chromosome 6p
centimeters from the incisors is the lower D. MYH gene on chromosome 1p, APC, K-ras
esophageal sphincter located? The most common type of fistula-in-ano is:
34.
A. 15-18 cm A. Intersphincteric
B. 24-26 cm 42. B. Transphincteric
C. 38-40 cm C. Suprasphincteric
D. Cannot be determined D. Extrasphincteric
Which sign/symptom of appendicitis scores 2 points Which of the following statements about hernia
in the Alvarado Scoring System? repair is FALSE?
A. Anorexia A. McVay repair addresses both inguinal and
35.
B. Nausea/Vomiting femoral ring defects.
C. Fever B. In experienced hands, the overall recurrence
D. Right lower quadrant tenderness rate for the Shouldice repair is about 1%.
43.
A 33-year-old woman is noted to have a Meckel’s C. The Lichtenstein technique has an overall
diverticulum when she undergoes an emergency recurrence rate of 0.2%
appendectomy. The diverticulum is approximately D. The Stoppa technique results in longer
60 cm from the ileocecal valve and measures 2–3 cm operative duration, increased postoperative
in length. What is the most common complication of acute pain and increased recurrent rates
36.
Meckel’s diverticulum among adults? compared to the Lichtenstein technique.
A. Bleeding Which of the following statements about pancreatic
B. Perforation collections after acute pancreatitis is TRUE?
C. Intestinal obstruction A. An acute pancreatic fluid collection (APFC)
D. Ulceration conforms to retroperitoneal structures and
A 40-year-old man with a long history of bloody has a defined wall
diarrhea presents with increased abdominal pain, B. A pseudocyst occurs within the first 4 weeks of
vomiting, and fever. On examination, he is found to acute pancreatitis
44.
be dehydrated and shows tachycardia and C. Walled-off Necrosis (WON) can involve both
hypotension. The abdomen is markedly tender with the pancreatic parenchyma &/or the
37.
guarding and rigidity. What is the most likely cause? peripancreatic tissues.
A. Toxic megacolon in ulcerative colitis D. Acute necrotic collection (ANC) is a collection
B. Perforated carcinoma of the sigmoid colon of pancreatic &/or peripancreatic necrosis
C. Volvulus of the sigmoid colon that has developed a well-defined
D. Acute perforated diverticulitis inflammatory wall.
A 65-year-old woman presents with pain in the LLQ A 60-year-old woman is recovering from a major
of the abdomen. On examination, she is febrile and operation and develops severe abdominal pain and
found to be dehydrated due to excessive bilious sepsis. HBT ultrasound showed inflammation of the
vomiting. A CT scan shows a mass in the LLQ gallbladder but there is no evidence of gallbladder
involving the sigmoid colon. There is a minimal stones. Cholecystectomy is performed. Which is
38. amount of free fluid and no free air. What should the TRUE of acalculous cholecystitis?
initial treatment of this patient include? 45. A. It is characterized on HIDA scan by
A. IV fluids, cefoxitin, and nasogastric drainage filling of the gallbladder.
B. IV fluids, cefoxitin and percutaneous drainage B. It occurs in 10–20% of cases of cholecystitis.
C. IV fluids, cefoxitin, and laparotomy C. It has a more favorable prognosis than
D. immediate laparotomy calculous cholecystitis.
D. It is increased in frequency after trauma
or major operation.

TOPNOTCH MEDICAL BOARD PREP SURGERY PHASE 2 HANDOUT BY DRS. COCOS-ALCANTARA, ASPERAS, RUBIO Page 3 of 13
For inquiries visit www.topnotchboardprep.com.ph or email us at topnotchmedicalboardprep@gmail.com
This handout is only valid for the September 2021 PLE batch. This will be rendered obsolete for the next batch since we update our handouts regularly.
TOPNOTCH MEDICAL BOARD PREP SURGERY PHASE 2 HANDOUT BY DRS. COCOS-ALCANTARA, ASPERAS, RUBIO
For inquiries visit www.topnotchboardprep.com.ph or https://www.facebook.com/topnotchmedicalboardprep/
This handout is only valid for the September 2021 PLE batch. This will be rendered obsolete for the next batch since we update our handouts regularly.
A 26 year-old was brought to the emergency room 5W: COMMON CAUSES OF POST-OPERATIVE FEVER
for blunt abdominal trauma from a vehicular crash. TIMING ETIOLOGY PREVENTION MNEMONIC
Abdominal Ct scan revealed that he sustained a Drug
grade III splenic injury. This means that: reactions, Wonder
A. there is a splenic laceration involving Anytime N/A
malignant drugs
46. segmental or hilar vessels producing major hyperthermia
devascularization (>25% of spleen) Incentive
B. there is >4cm parenchymal splenic laceration Atelectasis
spirometry, early
C. he should be observed at the wards and POD 1-3 Pneumonia Wind
mobilization
monitored q1 (day 3)
Antibiotics
D. he should undergo emergent splenectomy Short-term Foley
The hepatoduodenal ligament contains the POD 3-4 UTI Water
use
following EXCEPT: Early mobilization,
A. portal vein Deep venous
47. heparin,
B. hepatic artery thrombosis/
POD 4-5 sequential Walking
C. hepatic vein pulmonary
compression socks
D. bile duct embolism
(SCDs)
The following are risk factors for the development Dressing changes,
of hepatoma, EXCEPT: Surgical site
POD 7+ preoperative Wound
A. fatty liver infection
48. antibiotics
B. oral contraceptive use
C. viral hepatitis Which if the following can be seen in SIRS?
D. hemochromatosis A. Hypothermia <37oC
The procedure of choice for patients with suspected B. White cell count (WBC) = 4.0
common bile duct stones to confirm the diagnosis is: 3.
C. No documented infection
A. MRCP D. Tachycardia >80/min
49.
B. Ultrasound E. Tachypnea >24/min
C. Abdominal CT Scan
D. ERCP
Which of the following statements on pancreatic
neuroendocrine tumors is TRUE?
A. Gastrinomas are the most common pancreatic
endocrine neoplasm.
50. B. Majority of insulinomas are malignant.
C. VIPoma syndrome is also known as WDHA
syndrome.
D. Glucagonomas are more often found in the
head and neck of the pancreas. Temperature <36°C or >38°C
Heart Rate >90 beats per minute
>20 breathes per minute or
Tachypnea
RATIONALE PaCO2 <32 mm Hg
White Blood WBC <4,000/mm³ or WBC >12,000/mm³ or
Cell Count > 10%immature (band)forms
Sequence of return of gastrointestinal motility after
• Systemic inflammatory response syndrome
surgery is:
o ≥2 criteria
A. intestine, stomach, colon
1. B. stomach, intestine, colon • Sepsis
C. colon,intestine,stomach o SIRS plus confirmed or presumed infection
D. colon, stomach, intestine • Severe sepsis
E. stomach,colon,intestine o Sepsis plus organ dysfunction
A 42-year old female underwent cholecystectomy • Septic shock
and 16 hours after the operation she developed o Severe sepsis plus refractory hypotension
fever. This is most probably due to: • Multiple organ dysfunction syndrome
2. A. Pneumonia o Evidence of ≥2 organs failing
B. UTI
C. Atelectasis With regard to von Willebrand disease, which of the
D. Wound infection following statements is true?
A. It is more common than hemophilia.
4. B. It is best treated with cryoprecipitate plasma.
C. Factor VIII levels are constant over time in a
given patient.
D. Bleeding after elective surgery is rare.

TOPNOTCH MEDICAL BOARD PREP SURGERY PHASE 2 HANDOUT BY DRS. COCOS-ALCANTARA, ASPERAS, RUBIO Page 4 of 13
For inquiries visit www.topnotchboardprep.com.ph or email us at topnotchmedicalboardprep@gmail.com
This handout is only valid for the September 2021 PLE batch. This will be rendered obsolete for the next batch since we update our handouts regularly.
TOPNOTCH MEDICAL BOARD PREP SURGERY PHASE 2 HANDOUT BY DRS. COCOS-ALCANTARA, ASPERAS, RUBIO
For inquiries visit www.topnotchboardprep.com.ph or https://www.facebook.com/topnotchmedicalboardprep/
This handout is only valid for the September 2021 PLE batch. This will be rendered obsolete for the next batch since we update our handouts regularly.

PT aPTT The effectiveness of prophylactic antibiotics in


VII XII surgery is mostly related to the:
X High molecular weight kininogen A. use of broad-spectrum agents
V Prekallikrein B. continuation of antibiotics for 24 hours after
6.
II (prothrombin) XI surgery
Fibrinogen IX C. timing of initial administration
VIII D. use of two synergistic agents
X E. use of bactericidal agents
V Which of the following statements about types of
II wounds is true?
Fibrinogen A. The infection rate in a ‘clean wound’ is
between 1-2%
B. The wound after a biliary surgery is classified
During resuscitation of a patient in shock, which of 7.
as ‘contaminated’.
the following statements is false?
C. A ‘contaminated wound’ has an infection rate
A. Administration of inotropic agents to an
of less than 3 per cent.
empty heart will help to increase diastolic
D. Antibiotic prophylaxis would be mandatory in
filling and coronary perfusion.
‘dirty wounds’.
B. In all cases, regardless of classification,
hypovolemia and preload must be addressed
5. Wound class, representative procedure, and expected
first.
infection rates
C. The oxygen-carrying capacity of both colloids
WOUND EXAMPLES OF CASES EXPECTED
and crystalloids is zero.
D. Hypotonic solutions are poor volume CLASS INFECTION
expanders and should not be used in shock RATES
except in conditions of free water loss or Clean (class I) Hernia repair, breast biopsy 1-2%
sodium overload. Clean/
Cholecystectomy, elective GI
contaminated 2.1-9.5%
surgery (not colon)
(class II)

TOPNOTCH MEDICAL BOARD PREP SURGERY PHASE 2 HANDOUT BY DRS. COCOS-ALCANTARA, ASPERAS, RUBIO Page 5 of 13
For inquiries visit www.topnotchboardprep.com.ph or email us at topnotchmedicalboardprep@gmail.com
This handout is only valid for the September 2021 PLE batch. This will be rendered obsolete for the next batch since we update our handouts regularly.
TOPNOTCH MEDICAL BOARD PREP SURGERY PHASE 2 HANDOUT BY DRS. COCOS-ALCANTARA, ASPERAS, RUBIO
For inquiries visit www.topnotchboardprep.com.ph or https://www.facebook.com/topnotchmedicalboardprep/
This handout is only valid for the September 2021 PLE batch. This will be rendered obsolete for the next batch since we update our handouts regularly.
WOUND EXAMPLES OF CASES EXPECTED
CLASS INFECTION
RATES
Clean/
contaminated Colorectal surgery 4-14%
(class II)
Contaminated Penetrating abdominal
(class III) trauma, large tissue injury,
3.4-13.2%
enterotomy during bowel
obstruction
Dirty (class Perforated diverticulitis,
IV) necrotizing soft tissue 3.1-12.8%
infections

Which of the following is false regarding the


sequence of events in wound healing?
A. The initial vascular response is
vasoconstriction, followed by vasodilation.
B. The phases of wound healing are
inflammatory, proliferative, and remodeling.
8. C. The dominant cells of the inflammatory phase The most common fatal infection in burn victims is:
are macrophages. A. pneumonia
D. The proliferative phase is mediated by 10. B. venous line–related sepsis
fibroblasts. C. burn wounds sepsis
E. The content of collagen in the wound D. urinary tract infection
continues to increase throughout the
A 25-year-old motorcyclist comes in on a spine
remodeling phase.
board with cervical spine control. He was found at
Cutaneous wound healing is generally divided into three the roadside unconscious following a collision with
phases: an oncoming car. The airway is clear but his
1. Hemostasis and Inflammation (early and late) breathing is clearly labored with a respiratory rate
2. Granulation tissue formation and reepithelialization of 40/min. His pulse is 110/min and his blood
3. Wound contraction, ECM deposition, and remodeling pressure is 90/60. It appears that the wing mirror of
the car has penetrated his chest and there is a
11. wound on the right side which is bubbling. He is not
responding to verbal command but is muttering
incoherently. He will not open his eyes. What is the
next best step?
A. Insert a chest drain.
B. Put a flap valve dressing over the bubbling
wound
C. Start checking for other injuries.
D. Start a blood transfusion.

ADVANCED TRAUMA LIFE SUPPORT (ATLS):


• Primary survey (ABCDE) / Concurrent resuscitation
• Secondary survey (AMPLE) / diagnostic evaluation
• Definitive care
• Tertiary survey
AIRWAY BREATHING
• Tension pneumothorax
• Airway obstruction • Open pneumothorax
• Airway injury • Flail chest with underlying pulmonary
contusion
CIRCULATION
• Massive hemothorax or hemoperitoneum
Hemorrhagic shock • Mechanically unstable pelvis fracture
• Extremity blood loss
• T. pneumothorax
• Cardiac tamponade
Cardiogenic shock
• Blunt cardiac injury, MI
• Bronchovenous air embolism
Neurogenic shock
Intracranial hemorrhage/mass
DISABILITY
lesion/Cervical spine injury
completely undress the patient, usually by
“Acute abdomen” commonly refers to possible EXPOSURE/
cutting off his or her garments to facilitate a
surgical problem occurring abdominally and ENVIRONMENTAL
thorough examination and assessment.
necessitating immediate action. The most constant
component in this symptoms complex is: WOUND DRESSING FOR AN OPEN PNEUMOTHORAX
9.
A. Pain
B. Tenderness
C. Rigidity
D. Distention

TOPNOTCH MEDICAL BOARD PREP SURGERY PHASE 2 HANDOUT BY DRS. COCOS-ALCANTARA, ASPERAS, RUBIO Page 6 of 13
For inquiries visit www.topnotchboardprep.com.ph or email us at topnotchmedicalboardprep@gmail.com
This handout is only valid for the September 2021 PLE batch. This will be rendered obsolete for the next batch since we update our handouts regularly.
TOPNOTCH MEDICAL BOARD PREP SURGERY PHASE 2 HANDOUT BY DRS. COCOS-ALCANTARA, ASPERAS, RUBIO
For inquiries visit www.topnotchboardprep.com.ph or https://www.facebook.com/topnotchmedicalboardprep/
This handout is only valid for the September 2021 PLE batch. This will be rendered obsolete for the next batch since we update our handouts regularly.
The most common complication of blood
transfusion is
A. hemolytic reaction
12. B. human immunodeficiency virus transmission
C. allergic reaction
D. volume overload
E. coagulopathy
CLASS I CLASS II CLASS III CLASS IV
Blood loss (mL) Up to 750 750-1500 1500-2000 >2000
Blood loss (% BV) Up to 15% 15%-30% 30%-40% >40%
Pulse rate <100 >100 >120 >140
Blood pressure Normal Normal Decreased Decreased
Pulse pressure (mmHg) Normal or increased Decreased Decreased Decreased
Respiratory rate 14-20 >20-30 30-40 >35
Urine output (mL/h) >30 >20-30 5-15 Negligible
CNS/mental status Slightly anxious Mildly anxious Anxious and confused Confused and lethargic

• Average man has ~5L blood (B-L-O-O-D has 5 Letters!) A patient with blunt abdominal injury underwent a
• AT LEAST CLASS 2! Focused Abdominal Sonogram for Trauma (FAST)
o Blood loss >1.5L: (Imagine 1.5L coke bottle) and the resident says it is positive. What does this
o Tachycardiac (>100bpm) mean?
o Dyspneic (>20/min) A. The patient must undergo laparotomy
• AT LEAST CLASS 3! 14. B. There is fluid noted in the dependent
o Hypotension portions of the peritoneal cavity
o Decreased UO C. There is intraperitoneal bleeding, most
• ”R-B-C” – three letters: Transfuse pRBC if at least CLASS III! probably from the liver
D. The patient must undergo a triple-contrast
The most important initial management of abdominal CT scan
suspected blunt myocardial injury is:
A. ECG monitoring FAST
13. B. chest computed tomography scan FOCUSED ABDOMINAL SONOGRAM FOR TRAUMA
C. assessment of cardiac enzymes Dependent portions of the peritoneal cavity:
D. insertion of pulmonary artery catheter o Hepatorenal
E. echocardiogram o Splenorenal
o Pelvic, and
o Pericardial

A patient with head injury opens his eyes and OMPHALOCELE GASTROSCHISIS
withdraws his arm to pain. He is making Herniated abdominal Herniatied INTESTINES ONLY
incomprehensible sounds. His Glasgow Coma Scale VISCERA!
score is: WITH SAC! NO SAC!
15.
A. 12 if the sac is intact, surgical Silo bag can be placed to
B. 10 correction until the patient is reduce bowel contents until
C. 8 optimized surgery can be performed
D. 6
Gastroschisis: The initial step in management of a 25-year-old
A. is usually associated with other anomalies male with a painless scrotal mass is:
B. is usually associated with chromosomal A. fine-needle aspiration and cytology
16.
disorders B. abdominal computed tomography scan
C. is located on the left of the umbilical cord 17.
C. observation and repeat examination in 2
D. repair is followed by prolonged ileus weeks
D. scrotal ultrasound
E. orchiectomy
What anatomic abnormality is present in an
inguinal hernia in an infant?
A. Patent processus vaginalis
18. B. Weakness in the inguinal floor
C. Obliteration of the inguinal floor
D. Congenital absence of both the external and
internal rings
TOPNOTCH MEDICAL BOARD PREP SURGERY PHASE 2 HANDOUT BY DRS. COCOS-ALCANTARA, ASPERAS, RUBIO Page 7 of 13
For inquiries visit www.topnotchboardprep.com.ph or email us at topnotchmedicalboardprep@gmail.com
This handout is only valid for the September 2021 PLE batch. This will be rendered obsolete for the next batch since we update our handouts regularly.
TOPNOTCH MEDICAL BOARD PREP SURGERY PHASE 2 HANDOUT BY DRS. COCOS-ALCANTARA, ASPERAS, RUBIO
For inquiries visit www.topnotchboardprep.com.ph or https://www.facebook.com/topnotchmedicalboardprep/
This handout is only valid for the September 2021 PLE batch. This will be rendered obsolete for the next batch since we update our handouts regularly.
The classic triad of symptoms for renal cell cancer
are the following EXCEPT:
A. Weight loss
19.
B. Flank pain
C. Gross hematuria
D. Palpable Flank mass

RENAL CELL CARCINOMA


• Most common 1° renal
malignancy
• Originates from PCT →
invades renal vein
• may develop varicocele if left
sided) IVC → hematogenous
spread → metastasis to lung
and bone.
• Treatment: surgery/ablation
for localized disease.
Immunotherapy or targeted
therapy for metastatic
disease

Which of the following is true of thoracic anatomy?


A. The left lung has three lobes.
B. The azygous vein runs along the left side
draining into the subclavian vein.
C. The vagus nerve runs anterior to the lung
20. hilum.
D. The sternocleidomastoid muscle is not
considered an accessory muscle to
breathing.
E. The phrenic nerve runs superior to the lung
hilum.
The following pertains to Gustillo and Anderson
classification of fractures EXCEPT?
A. It relies on the length of any laceration.
B. It is influenced primarily by the energy
21.
involved.
C. It takes account of whether or not there is soft-
tissue cover of fractured bone.
D. It takes account of contamination.

OPEN BONE FRACTURES: GUSTILO-ANDERSON CLASSIFICATION


Wound length Minimal soft tissue damage, Adequate soft-
Type 1 Periosteum intact Vasculature intact
<1cm contamination, and comminution tissue coverage
Wound length Moderate soft tissue damage, Adequate soft-
Type 2 Periosteum intact Vasculature intact
≥1cm contamination, or comminution tissue coverage
Extensive soft tissue damage,
Extensive Adequate soft-
Type 3a contamination, or comminution; Periosteal stripping Vasculature intact
wound tissue coverage
segmentalfracture
Extensive soft tissue damage,
Extensive Inadequate soft-
Type 3b contamination, or comminution; Periosteal stripping Vasculature intact
wound tissue coverage
segmental fracture
Extensive soft tissue damage,
Extensive Inadequate soft-
Type 3c contamination, or comminution; Periosteal stripping Arterial Damage
wound tissue coverage
segmental fracture

The following statements are true regarding acute


subdural hematoma EXCEPT:
A. It is a collection of blood between the dura and
arachnoid membranes
22.
B. It can be caused by laceration of the brain.
C. It can be due to disruption of a cortical blood
vessel.
D. It has a biconvex shape on CT scans.

EPIDURAL HEMATOMA SUBDURAL HEMATOMA


“Lenticular or Binconvex” “Crescent”
Bleeding from Middle Bleeding from Dural Bridging Veins
Meningeal Artery
Between Dura and Skull Between Dura and Arachnoid Mater
Cannot cross suture lines CAN cross suture lines
(Dura is anchored)
LUCID interval acute (immediate), subacute
(days), or chronic (weeks)
TOPNOTCH MEDICAL BOARD PREP SURGERY PHASE 2 HANDOUT BY DRS. COCOS-ALCANTARA, ASPERAS, RUBIO Page 8 of 13
For inquiries visit www.topnotchboardprep.com.ph or email us at topnotchmedicalboardprep@gmail.com
This handout is only valid for the September 2021 PLE batch. This will be rendered obsolete for the next batch since we update our handouts regularly.
TOPNOTCH MEDICAL BOARD PREP SURGERY PHASE 2 HANDOUT BY DRS. COCOS-ALCANTARA, ASPERAS, RUBIO
For inquiries visit www.topnotchboardprep.com.ph or https://www.facebook.com/topnotchmedicalboardprep/
This handout is only valid for the September 2021 PLE batch. This will be rendered obsolete for the next batch since we update our handouts regularly.
Metastasis of gastric malignancy may be noted in the PECTORAL REGION - MUSCLES
following sites:
A. Krukenberg’s tumor – involvement of the
pelvic cul-de-sac
23.
B. Blumer’s shelf – involvement of ovaries
C. Virchow’s node – ipsilateral axillary node
D. Sister Joseph’s nodule – deposit to the
umbilicus
GASTRIC CANCER

A 62-year-old alcoholic presents with an indurated


VIRCHOW NODE KRUKENBERG TUMOR
ulcer, 1.5 cm in length, in the left lateral aspect of her
involvement of left bilateral metastases to tongue. Biopsy reveals squamous cell carcinoma.
supraclavicular node ovaries There is a palpable neck node at the left upper
jugular chain of node (Level II) and FNAB showed
positive for malignant cells. What should she
28.
undergo?
A. Wide excision of tongue ulcer and neck node
B. Wide excision and selective node dissection
C. Wide excision and modified radical neck
dissection
D. Wide excision and radiotherapy
SISTER MARY JOSEPH BLUMER SHELF
NODULE
subcutaneous periumbilical palpable mass on digital rectal
metastasis exam

Aside from hypotension, the additional criteria for


classifying a patient with Charcot’s triad to be in
Reynold’s pentad is:
24. A. Tachycardia
B. Severe RUQ pain
C. Temperature > 39oC
D. Sensorium Change
On digital rectal exam, a strong 63 year old male
sexually active male was found to have a 1.0cm
nodule on the right lobe of the prostate gland.
However, he was asymptomatic, what will be your
25. next plan of management?
A. Transrectal prostate ultrasound
B. Transabdominal KUB prostate ultrasound
C. Acid phosphatase determination
D. Prostate specific antigen test Following surgical resection of a large thyroid
TUMOR MARKERS mass, a patient complains of persistent hoarseness
COLON CANCER CEA and a weak voice. What is the most likely cause of
HEPATOMA AFP these symptoms?
29.
A. Traumatic intubation
PANCREATIC CARCINOMA CA 19-9
B. Prolonged intubation
OVARIAN CANCER CA-125
C. Injury to the recurrent laryngeal nerve
PROSTATE CANCER PSA
D. Injury to the superior laryngeal nerve
CEA: for monitoring recurrence, should not be used for screening.
Which of the following is used for surveillance of
patients who underwent total thyroidectomy for
A 28-year-old female volleyball player presents
thyroid cancer?
several weeks after having sustained an injury to 30. A. Anti-thyroglobulin
her left breast. She has a painful mass in the upper B. TSH
outer quadrant. Skin retraction is noticed, and a
C. Thyroid Peroxidase
hard mass, 3–4 cm in diameter, can easily be
26. D. None of the above
palpated. What is the most likely diagnosis?
A 45-year-old man presents with an upper
A. Infiltrating carcinoma
gastrointestinal bleed. An upper endoscopy reveals
B. Breast abscess
multiple duodenal ulcers and an enlarged stomach.
C. Hematoma
Further work-up showed that he has Zollinger
D. Fat necrosis
Ellison Syndrome occurring as part of MEN-1
After undergoing modified radical mastectomy for 31.
syndrome. The following is also expected:
cancer of the right breast, a 52-year-old female
A. Parathyroid hyperplasia
teacher becomes aware that the medial end of her B. Medullary thyroid cancer
scapula becomes prominent in protraction
C. Pheochromocytoma
movements at the shoulder. She also complains of D. Marfanoid habitus
27. some weakness in complete abduction of the same
shoulder. What nerve was injured?
A. Long thoracic
B. Thoracodorsal
C. Medial pectoral nerve
D. Lateral pectoral nerve
TOPNOTCH MEDICAL BOARD PREP SURGERY PHASE 2 HANDOUT BY DRS. COCOS-ALCANTARA, ASPERAS, RUBIO Page 9 of 13
For inquiries visit www.topnotchboardprep.com.ph or email us at topnotchmedicalboardprep@gmail.com
This handout is only valid for the September 2021 PLE batch. This will be rendered obsolete for the next batch since we update our handouts regularly.
TOPNOTCH MEDICAL BOARD PREP SURGERY PHASE 2 HANDOUT BY DRS. COCOS-ALCANTARA, ASPERAS, RUBIO
For inquiries visit www.topnotchboardprep.com.ph or https://www.facebook.com/topnotchmedicalboardprep/
This handout is only valid for the September 2021 PLE batch. This will be rendered obsolete for the next batch since we update our handouts regularly.
MEN 1 (3 PS) MEN 2A (1M, 2PS) MEN 2B (2MS, 1P)
3P 2P 1M 1P 2M • Pituitary • MTC • MTC
• Parathyroid • Pheochromocytoma • Marfanoid
• Pancreatic • Parathyroid habitus/mucosal
neuroma
• Pheochromocytoma

A 64-year-old man develops increasing dysphagia


over many months. A barium swallow is performed.
What is the most likely cause of his clinical
presentation?
32.
A. Carcinoma of the esophagus
B. Achalasia
C. Esophageal diverticulum
D. Paraesophageal hernia
A 64-year-old man has symptoms of reflux
esophagitis for 20 years. The barium study
shown demonstrates a sliding hiatal
hernia. Which is TRUE in sliding hiatal hernia?
33. A. A hernia sac is absent.
B. The cardia is displaced into the posterior
mediastinum.
C. Reflux esophagitis always occur.
D. A stricture does not develop.

A 55-year-old man complains of difficulty VERTEBRAL


SITE
swallowing and underwent an EGD. How many LEVEL
centimeters from the incisors is the lower CRICOPHARYNX C6
esophageal sphincter located? AORTIC ARCH T4
34.
A. 15-18 cm LEFT MAIN BRONCHUS T6
B. 24-26 cm ESOPHAGEAL HIATUS T10
C. 38-40 cm
D. Cannot be determined

TOPNOTCH MEDICAL BOARD PREP SURGERY PHASE 2 HANDOUT BY DRS. COCOS-ALCANTARA, ASPERAS, RUBIO Page 10 of 13
For inquiries visit www.topnotchboardprep.com.ph or email us at topnotchmedicalboardprep@gmail.com
This handout is only valid for the September 2021 PLE batch. This will be rendered obsolete for the next batch since we update our handouts regularly.
TOPNOTCH MEDICAL BOARD PREP SURGERY PHASE 2 HANDOUT BY DRS. COCOS-ALCANTARA, ASPERAS, RUBIO
For inquiries visit www.topnotchboardprep.com.ph or https://www.facebook.com/topnotchmedicalboardprep/
This handout is only valid for the September 2021 PLE batch. This will be rendered obsolete for the next batch since we update our handouts regularly.
A 33-year-old woman is noted to have a Meckel’s
diverticulum when she undergoes an emergency
appendectomy. The diverticulum is approximately
60 cm from the ileocecal valve and measures 2–3 cm
in length. What is the most common complication of
36.
Meckel’s diverticulum among adults?
A. Bleeding
B. Perforation
C. Intestinal obstruction
D. Ulceration
A 40-year-old man with a long history of bloody
diarrhea presents with increased abdominal pain,
vomiting, and fever. On examination, he is found to
be dehydrated and shows tachycardia and
hypotension. The abdomen is markedly tender with
37.
guarding and rigidity. What is the most likely cause?
A. Toxic megacolon in ulcerative colitis
B. Perforated carcinoma of the sigmoid colon
C. Volvulus of the sigmoid colon
D. Acute perforated diverticulitis
A 65-year-old woman presents with pain in the LLQ
of the abdomen. On examination, she is febrile and
found to be dehydrated due to excessive bilious
vomiting. A CT scan shows a mass in the LLQ
involving the sigmoid colon. There is a minimal
amount of free fluid and no free air. What should the
38.
initial treatment of this patient include?
A. IV fluids, cefoxitin, and nasogastric drainage
B. IV fluids, cefoxitin and percutaneous
drainage
C. IV fluids, cefoxitin, and laparotomy
D. immediate laparotomy
HINCHEY CLASSIFICATION

Which sign/symptom of appendicitis scores 2 points


in the Alvarado Scoring System?
A. Anorexia
35.
B. Nausea/Vomiting
C. Fever
D. Right lower quadrant tenderness

A 68-year-old male musician with a sudden onset of


colicky abdominal pain and massive vomiting of 4-
hour duration. Examination shows an elevated WBC
of 13,200 with a HCT of 45%. An erect film of the
abdomen reveals dilatation of the stomach with
39. minimal distended loops of bowel. What is his
clinical diagnosis?
A. Complete jejunal obstruction
B. Incomplete jejunal obstruction
C. Complete ileal obstruction
D. Incomplete ileal obstruction
A 64-year-old female underwent screening
PREDICTED NUMBER OF PATIENTS WITH APPENDICITIS colonoscopy. Nodularities were seen at the cecum
• Alvarado score 1-4 30% and biopsy results showed tuberculosis. There is no
• Alvarado score 5-6 66% evidence of intestinal obstruction. What should the
40. next step in treatment involve?
• Alvarado score 7-10 93%
A. Diagnostic laparoscopy
B. Laparotomy and ileal resection
C. A full course of anti-tuberculous drugs
D. Steroids

TOPNOTCH MEDICAL BOARD PREP SURGERY PHASE 2 HANDOUT BY DRS. COCOS-ALCANTARA, ASPERAS, RUBIO Page 11 of 13
For inquiries visit www.topnotchboardprep.com.ph or email us at topnotchmedicalboardprep@gmail.com
This handout is only valid for the September 2021 PLE batch. This will be rendered obsolete for the next batch since we update our handouts regularly.
TOPNOTCH MEDICAL BOARD PREP SURGERY PHASE 2 HANDOUT BY DRS. COCOS-ALCANTARA, ASPERAS, RUBIO
For inquiries visit www.topnotchboardprep.com.ph or https://www.facebook.com/topnotchmedicalboardprep/
This handout is only valid for the September 2021 PLE batch. This will be rendered obsolete for the next batch since we update our handouts regularly.
A 32-year-old man presents to the ED with a 2- A 60-year-old woman is recovering from a major
month history of alternating diarrhea and operation and develops severe abdominal pain and
constipation, rectal bleeding, a 20-lb weight loss, sepsis. HBT ultrasound showed inflammation of the
and worsening fatigue. What are the most common gallbladder but there is no evidence of gallbladder
genetic mutations that could have led to the stones. Cholecystectomy is performed. Which is
41.
development of this patient’s colon cancer? TRUE of acalculous cholecystitis?
A. APC, DCC, p53 45. A. It is characterized on HIDA scan by
B. APC, BRCA1, K-ras filling of the gallbladder.
C. DCC, p53, and MYH gene on chromosome 6p B. It occurs in 10–20% of cases of cholecystitis.
D. MYH gene on chromosome 1p, APC, K-ras C. It has a more favorable prognosis than
The most common type of fistula-in-ano is: calculous cholecystitis.
A. Intersphincteric D. It is increased in frequency after trauma
42. B. Transphincteric or major operation.
C. Suprasphincteric A 26 year-old was brought to the emergency room
D. Extrasphincteric for blunt abdominal trauma from a vehicular crash.
Abdominal Ct scan revealed that he sustained a
grade III splenic injury. This means that:
A. there is a splenic laceration involving
segmental or hilar vessels producing major
46.
devascularization (>25% of spleen)
B. there is >4cm parenchymal splenic
laceration
C. he should be observed at the wards and
monitored q1
D. he should undergo emergent splenectomy

Which of the following statements about hernia


repair is FALSE?
A. McVay repair addresses both inguinal and
femoral ring defects.
B. In experienced hands, the overall recurrence
rate for the Shouldice repair is about 1%. Grade Type Description of Injury
43.
C. The Lichtenstein technique has an overall Hematoma Subcapsular, <10% surface area
recurrence rate of 0.2% 1 Capsular tear, <1 cm parenchymal
D. The Stoppa technique results in longer Laceration
depth
operative duration, increased postoperative
Hematoma Subcapsular, 10-50% surface area
acute pain and increased recurrent rates
2 Intraparenchymal, <5 cm in
compared to the Lichtenstein technique. Laceration
diameter
Which of the following statements about pancreatic
collections after acute pancreatitis is TRUE?
A. An acute pancreatic fluid collection (APFC)
conforms to retroperitoneal structures and
has a defined wall
B. A pseudocyst occurs within the first 4 weeks of
acute pancreatitis
44.
C. Walled-off Necrosis (WON) can involve both
the pancreatic parenchyma &/or the
peripancreatic tissues.
D. Acute necrotic collection (ANC) is a collection
of pancreatic &/or peripancreatic necrosis
that has developed a well-defined
inflammatory wall.
Grade Type Description of Injury
Subcapsular, >50% surface area or
Hematoma expanding; ruptured subcapsular
3 or parenchymal hematoma
>3 cm parenchymal depth or
Laceration
involved trabecular vessels

TOPNOTCH MEDICAL BOARD PREP SURGERY PHASE 2 HANDOUT BY DRS. COCOS-ALCANTARA, ASPERAS, RUBIO Page 12 of 13
For inquiries visit www.topnotchboardprep.com.ph or email us at topnotchmedicalboardprep@gmail.com
This handout is only valid for the September 2021 PLE batch. This will be rendered obsolete for the next batch since we update our handouts regularly.
TOPNOTCH MEDICAL BOARD PREP SURGERY PHASE 2 HANDOUT BY DRS. COCOS-ALCANTARA, ASPERAS, RUBIO
For inquiries visit www.topnotchboardprep.com.ph or https://www.facebook.com/topnotchmedicalboardprep/
This handout is only valid for the September 2021 PLE batch. This will be rendered obsolete for the next batch since we update our handouts regularly.

Grade Type Description of Injury


Laceration involving segmental or
hilar vessels and producing major
4 Laceration
devascularization
(>25% of spleen)
Laceration Completely shattered spleen
5 Hilar vascular injury that
Vascular
devascularizes spleen
Advance one grade for multiple injuries up to grade 3.
The following are risk factors for the development
The hepatoduodenal ligament contains the of hepatoma, EXCEPT:
following EXCEPT: A. fatty liver
48.
A. portal vein B. oral contraceptive use
47. C. viral hepatitis
B. hepatic artery
C. hepatic vein D. hemochromatosis
D. bile duct The procedure of choice for patients with suspected
common bile duct stones to confirm the diagnosis is:
A. MRCP
49.
B. Ultrasound
C. Abdominal CT Scan
D. ERCP
Which of the following statements on pancreatic
neuroendocrine tumors is TRUE?
A. Gastrinomas are the most common pancreatic
endocrine neoplasm.
50. B. Majority of insulinomas are malignant.
C. VIPoma syndrome is also known as WDHA
syndrome.
D. Glucagonomas are more often found in the
head and neck of the pancreas.

Secreted Malignant
Tumor Incidence Location Clinical features Biochemical diagnosis
hormone (%)
Insulin ≥ 5 mlU/l
Glucose 40mg/dl
Pancreas Hypoglycemic syndromes C-peptide 0.6 ng/ml
Insulinoma 40-55% Insulin <10
(>99%) (Whipple's triad) Proinsulin ≥ 20 pmol/l
(or > 25% of
immunoreactive insulin)
Duodenum Zollinger-Ellison syndrome
(70%) (abdominal pain, Serum gastrin level ≥10
Gastrinoma 25-50% Pancreas Gastrin 60-90 gastroesophageal reflux, times normal range +
(25%) Others diarrhea, duodenal ulcers, gastric pH < 2
(5%) PUD/GERD)
Rash, glucose intolerance,
Pancreas
Glucagonoma Rare Glucagon 50-80 necrolytic migratory erythema, Glucagon > 500 pg/ml
(100%)
weight loss
Pancreas
(55%)
Diabetes mellitus, cholelithiasis, Somatostatin-fasting
Somatostatinoma Rare Duodenum- Somatostatin >70
diarrhea serum level
jejunum
(45%)
Pancreas Vasoactive
VIPoma (Verner-
Rare (90%) intestinal 40-70 WHDA VIP fasting serum level
Morrison)
Other (10%) peptide
Adreno
Pancreas
Cortico
(4-16% all
ACTHoma Rare Tropic 95 Cushing's syndrome ---
ectopic
Hormone
Cushing's)
(ACTH)
pNET-causing
Pancreas Serotonin, Urinary 5-HIAA in a 24-h
carcinoid Rare 60-90 Carcinoid syndrome
(100%) tachynins urine collection
syndrome

END OF SURGERY PHASE 2

TOPNOTCH MEDICAL BOARD PREP SURGERY PHASE 2 HANDOUT BY DRS. COCOS-ALCANTARA, ASPERAS, RUBIO Page 13 of 13
For inquiries visit www.topnotchboardprep.com.ph or email us at topnotchmedicalboardprep@gmail.com
This handout is only valid for the September 2021 PLE batch. This will be rendered obsolete for the next batch since we update our handouts regularly.
TOPNOTCH MEDICAL BOARD PREP SURGERY PHASE 3 HANDOUT BY DR LOUBOMIR ANTONIO
For inquiries visit www.topnotchboardprep.com.ph or https://www.facebook.com/topnotchmedicalboardprep/
This handout is only valid for the September 2021 PLE batch. This will be rendered obsolete for the next batch since we update our handouts regularly.
Excessive administration of normal saline for fluid
Important Legal Information
The handouts, videos and other review materials, provided by Topnotch Medical Board resuscitation can lead to what metabolic derangement?
Preparation Incorporated are duly protected by RA 8293 otherwise known as the A. Metabolic alkalosis
Intellectual Property Code of the Philippines, and shall only be for the sole use of the person: B. Metabolic acidosis
a) whose name appear on the handout or review material, b) person subscribed to Topnotch
Medical Board Preparation Incorporated Program or c) is the recipient of this electronic C. Respiratory alkalosis
communication. No part of the handout, video or other review material may be reproduced, D. Respiratory acidosis
shared, sold and distributed through any printed form, audio or video recording, electronic
medium or machine-readable form, in whole or in part without the written consent of In a previously unexposed patient, when does the platelet
Topnotch Medical Board Preparation Incorporated. Any violation and or infringement, count fall in heparin-induced thrombocytopenia(HI )?
whether intended or otherwise shall be subject to legal action and prosecution to the full A. <24 hours
extent guaranteed by law.
B. 24–28 hours
C. 3–4 days
DISCLOSURE D. 5–7 days
The handouts/review materials must be treated with utmost confidentiality. It shall be the
responsibility of the person, whose name appears therein, that the handouts/review Which finding is not consistent with TTP?
materials are not photocopied or in any way reproduced, shared or lent to any person or A. Microangiopathic hemolytic anemia
disposed in any manner. Any handout/review material found in the possession of another
person whose name does not appear therein shall be prima facie evidence of violation of RA B. Schistocytes on peripheral blood smear
8293. Topnotch review materials are updated every six (6) months based on the current C. Fever
trends and feedback. Please buy all recommended review books and other materials listed D. Splenomegaly
below.
THIS HANDOUT IS NOT FOR SALE! Less than 0.5% of transfusions result in a serious
transfusion-related complication. What is the leading
cause of transfusion-related deaths?
REMINDERS A. Transfusion-related acute lung injury (TRALI)
1. Phase 3 serves as the final coaching. It is expected that you have finished
at least the Phase 1 videos prior to watching the Phase 3 videos B. ABO hemolytic transfusion reactions
2. The guided content of the video lectures are seen within the handout. C. Bacterial contamination of platelets
Answers to questions / blanks will be seen in the Phase 3 video. D. Iatrogenic hepatitis C infection
What is the risk of Hepatitis C and HIV-1 transmission with
blood transfusion?
This handout is only valid for the September 2021 PLE batch. A. 1:10,000,000
This will be rendered obsolete for the next batch B. 1:1,000,000
since we update our handouts regularly. C. 1:300,000
D. 1:100,000
SURGERY – PHASE 3 Shock caused by a large tension pneumothorax is
categorized as?
By Loubomir E. Antonio, MD, FSOSP, FPCS, FPSGS, A. Traumatic
B. Vasodilatory
Which of the following is an antifibrinolytic agent used in C. Cardiogenic
regular clinical practice? D. Obstructive
A. Unfractionated heparin (UFH) A 70-kg male patient presents to ED following a stab wound
B. Tranexamic acid (TXA) to the abdomen. He is hypotensive, markedly tachycardic,
C. Protamine sulfate and appears confused. What percent of blood volume has
D. Tissue plasminogen activator (tPA) he lost?
E. Plasminogen activator inhibitor-1 (PAI-1) A. 5%
A unit of blood could be transfused most rapidly through B. 15%
which of the following vascular access catheters? C. 35%
A. An 18-gauge, 16-cm, triple-lumen catheter in the D. 55%
right IJV Vasodilatory shock
B. A 16-gauge, 30-cm, triple-lumen catheter in the right A. Is characterized by failure of vascular smooth muscle
SCV to constrict due to low levels of catecholamines
C. An 18-gauge catheter inserted into the cephalic vein B. Leads to suppression of the renin-angiotensin system
at the wrist C. Can also be caused by carbon monoxide poisoning
D. A 16-gauge, 3-cm peripheral IV in the right D. Is similar to early cardiogenic shock
antecubital fossa A patient with necrotizing pancreatitis undergoes
E. An 18-gauge, 3-cm peripheral IV in the right computed tomography (CT)-guided aspiration, which
antecubital fossa results in growth of Escherichia coli on culture. The most
In cirrhotic patients who are actively bleeding, the appropriate treatment is ?
coagulopathy of end-stage liver disease can be A. Culture-appropriate antibiotic therapy
differentiated from DIC most readily by estimation of B. ERCP with sphincterotomy
which of the following factors? C. CT-guided placement of drains
A. Factor II D. Exploratory laparotomy
B. Factor IX Which factor does not influence the development of
C. Factor VII Surgical Site Infections (SSIs)?
D. Factor VIII:C A. Degree of microbial contamination
E. Factor X B. Duration of the procedure
A 26-year-old man is found to be brain-dead after a C. Malnutrition
gunshot wound to the head. His family consents to organ D. General anesthesia
donation. Which of the following is a contraindication to What is the estimated risk of transmission of human
organ donation? immune deficiency virus (HIV) from a needle stick from a
A. Positive hepatitis B core antibody source with HIV-infected blood?
B. Active hepatitis C A. <0.5%
C. History of basal cell carcinoma 5 years ago B. 1.8%
D. Donor liver biopsy with 10% steatosis C. 5%
E. Creutzfeldt–Jakob disease D. 10%
Nutritional formulas used to treat pulmonary failure
typically increase the fat intake of a patient’s total caloric
intake to?
A. 50%
B. 20%
C. 80%
D. 10%

TOPNOTCH MEDICAL BOARD PREP SURGERY PHASE 3 HANDOUT BY DR LOUBOMIR ANTONIO Page 1 of 15
For inquiries visit www.topnotchboardprep.com.ph or email us at topnotchmedicalboardprep@gmail.com
This handout is only valid for the September 2021 PLE batch. This will be rendered obsolete for the next batch since we update our handouts regularly.
TOPNOTCH MEDICAL BOARD PREP SURGERY PHASE 3 HANDOUT BY DR LOUBOMIR ANTONIO
For inquiries visit www.topnotchboardprep.com.ph or https://www.facebook.com/topnotchmedicalboardprep/
This handout is only valid for the September 2021 PLE batch. This will be rendered obsolete for the next batch since we update our handouts regularly.
Closure of an appendectomy wound in a patient with Which of the following is the only thing that has been
perforated appendicitis who is receiving appropriate shown to decrease wound infections in surgical patients
antibiotics will result in a wound infection in what with contaminated wounds?
percentage of patients? A. Use of iodophor-impregnated polyvinyl drapes.
A. 3–4% B. Saline irrigation of the peritoneum and wound.
B. 8–12% C. Antibiotic irrigation of the peritoneum and wound.
C. 15–18% D. 24 hours of appropriate antibiotics postoperatively
D. 22–25%- (in addition to preoperative dose).
A chronic carrier state occurs with hepatitis C infection in Following caustic injury to the skin with an alkaline agent
what percentage of patients? the affected area should initially be?
A. 90–99% A. Treated with running water or saline for 30 minutes
B. 75–80% B. Treated with running water or saline for 2 hours
C. 50–60% C. Treated with a neutralizing agent
D. 10–30% D. Treated with topical emollients and oral analgesics
The most effective post exposure prophylaxis for a surgeon In the ABCDE of melanoma, the D stands for diameter
stuck with a needle while operating on an HIV-positive greater than?
patient is? A. 2 mm
A. None B. 4 mm
B. Two or three drug therapy started within hours of C. 6 mm
injury D. 8 mm
C. Single drug therapy started within 24 hours Ocular melanoma
D. Triple drug therapy started within 24 hours A. Exclusively metastasizes to the lungs
At what pressure is operative decompression of a B. Exclusively metastasizes to the brain
compartment mandatory? C. Exclusively metastasizes to regional lymph nodes
A. 15 mmHg D. Exclusively metastasizes to the liver
B. 25 mmHg The most common pattern of benign calcification in lung
C. 35 mmHg hamartomas is?
D. 45 mmHg A. Solid
The proliferative phase of wound healing occurs how long B. Diffuse
after the injury? C. Central
A. 1 day D. Popcorn
B. 2 days An “onion-peel” appearance of a rib on CT is suggestive of?
C. 7 days A. Chondroma
D. 14 days B. Ewing sarcoma
The annual age-adjusted cancer incidence rates among C. Plasmacytoma
men and women are decreasing for all of the following D. Osteosarcoma
EXCEPT: A chylothorax is likely to be present in a patient whose
A. Colorectal pleural fluid analysis results show a triglyceride level of?
B. Oropharynx A. 80mg/100mL
C. Lung B. 100mg/100mL
D. Thyroid C. 45mg/100mL
After completion of vascular anastomoses, drainage of a D. 130mg/100mL
transplanted pancreas is accomplished by anastomosis to? The most common age to close asymptomatic ASDs is?
A. Right colon A. In the immediate newborn period
B. Left colon B. After the child reaches 10kg in weight
C. Duodenum C. Age 4–5 years
D. Bladder or small bowel D. During puberty
The Donabedian model of measuring quality identifies all Which of the following is a true surgical emergency in a
of the following as main types of improvements EXCEPT? newborn?
A. Changes to structure A. Tetralogy of Fallot (TOF)
B. Changes to process B. Truncus arteriosus (TA)
C. Changes to culture C. Total anomalous pulmonary venous connection (TAPVC)
D. Changes to outcomes D. Coarctation of the Aorta (COA)
Prophylaxis using low-dose UFH reduces the incidence of The arterial switch operation for transposition of the great
fatal pulmonary embolisms by? vessels is best performed?
A. 45% A. Within 2 weeks of birth
B. 50% B. At 1year of age
C. 60% C. At 10kg of weight
D. 35% D. In adolescence
Which of the following is the best test to predict successful The most commonly recommended age for correction of a
extubation of a patient? TOF is ?
A. Respiratory rate A. Neonate younger than 3 months
B. Negative inspiratory pressure B. 6 months of age
C. Tobin index C. 1year of age
D. Minute ventilation D. 4–5years of age
The root cause of the majority of wrong-site surgeries What is the best predictor of spontaneous closure of a VSD?
result from? A. Size
A. Communication errors B. Age at diagnosis
B. Emergency surgery C. Gestational age
C. Multiple procedures D. Lack of electrocardiogram changes
D. Multiple surgeons During left thoracotomy or repair of patent ductus
Which of the following have been shown to decrease the arteriosus the blood pressure is 70/22. Immediately after
time of post-operative ileus? placement of a clip across the duct the blood pressure is?
A. Cyclooxygenase-1inhibitors A. 70/22
B. Morphine patient-controlled analgesia B. 70/40
C. Nasogastric drainage until full return of bowel function C. 90/ 22
D. Erythromycin D. 90/ 40

TOPNOTCH MEDICAL BOARD PREP SURGERY PHASE 3 HANDOUT BY DR LOUBOMIR ANTONIO Page 2 of 15
For inquiries visit www.topnotchboardprep.com.ph or email us at topnotchmedicalboardprep@gmail.com
This handout is only valid for the September 2021 PLE batch. This will be rendered obsolete for the next batch since we update our handouts regularly.
TOPNOTCH MEDICAL BOARD PREP SURGERY PHASE 3 HANDOUT BY DR LOUBOMIR ANTONIO
For inquiries visit www.topnotchboardprep.com.ph or https://www.facebook.com/topnotchmedicalboardprep/
This handout is only valid for the September 2021 PLE batch. This will be rendered obsolete for the next batch since we update our handouts regularly.
The bypass conduit with the highest patency rate is the? Which of the following statements about the use of latex
A. Radial artery gloves during an operation is correct?
B. Internal thoracic artery A. The incidence of puncture holes is not related to the
C. Greater saphenous vein duration of the surgery
D. Radial artery B. The right index finger is the most common site of
Each of the following effects is anticipated after insertion perforation
of an intra-aortic balloon pump EXCEPT? C. About 40,000 organisms may pass through a glove
A. Preload decrease pinhole in a 20-minute period
B. Increased total myocardial oxygen consumption D. Gloves made of latex maybe re-used after re-
C. Improvement in cardiac index sterilization
D. Coronary blood flow increase Anesthesia dolorosa is pain felt in the face which is
During cardiopulmonary bypass (CPB) anticoagulation, completely numb to touch. The pain is characterized as
the range of heparin needed to increase the activated constant, burning, or aching. It is usually a side effect of
clotting time to greater than 450 seconds is? surgery of ____?
A. 200 to 300 A. Chemodectoma
B. 300 to 400 B. Bell’s palsy
C. 400 to 500 C. Vestibular schwannoma
D. 450 to 550 D. Trigeminal neuralgia
Which of the following is the most common cause of Tic douloureux is a chronic pain that involves cranial nerve
thoracic aortic aneurysms? V. Which of the following does not explain this pain?
A. Atherosclerosis A. Trigeminal nerve myelin sheath loss
B. Marfan syndrome B. Nerve compression of a tumor
C. Takayasu arteritis C. Injury during parotid surgery
D. Nonspecific medial degeneration D. Blood vessel pressing on the nerve at it exits the
The most common complication of extensive repair or brainstem
distal aortic aneurysms is? Which of the following is FALSE regarding the immediate
A. Spinal cord ischemia post-operative period?
B. Renal failure A. Detailed treatment orders should be written upon
C. Pulmonary dysfunction arrival at the recovery room
D. Left recurrent laryngeal nerve injury B. While the patient is en-route to the recovery room, the
According to the Crawford classification scheme, surgical patient should be accompanied by a physician and
repair of thoracoabdominal aortic aneurysms with repairs qualified attendants
beginning near the left subclavian artery but extending C. The post-anesthetic recovery unit should be staffed by
distally into the infrarenal abdominal aorta, often reaching specially trained personnel
the aortic bifurcation is classified as D. Written orders are sufficient for unusual or
A. Extent I particularly important post-operative orders.
B. Extent II Discharge criteria following ambulatory surgery includes
C. Extent III all of the following except;
D. Extent IV A. Ability to eat solid food
The compartment most commonly affected in a lower leg B. Stable vital signs
compartment syndrome is the? C. Ability to ambulate
A. Anterior compartment D. Adequate pain control
B. Lateral compartment Which of the following cell types is not crucial for healing a
C. Deep posterior compartment clean incisional wound?
D. Superficial posterior compartment A. Macrophage
The correct classification for the degree of stenosis in the B. Polymorphonuclear leukocyte
internal carotid artery of a patient with a luminal diameter C. Platelet
of 69% is? D. Fibroblast
A. Mild All of the following situations are known to increase the
B. Moderate risk of infection of the surgical site following elective
C. Severe hernia repair except:
D. No stenosis A. Failing to continue prophylactic antibiotics for 24
Hollenhorst plaque is found within the? hours after the end of the operation
A. Internal carotid artery B. Shaving the operative site before taking the patient to
B. Retinal vessels the operating room
C. Peripheral arteries C. Placing a drain in the wound
D. Renal arteries D. Tying the skin sutures very tightly
Which of the following statements concerning carotid body The most common source of bacteria that causes SSI’s
tumors is true? following procedures other than elective clean operation
A. Over 50% are hereditary. is?
B. Require resection of the underlying carotid artery A. The hands of the operating team
with reconstruction or cure. B. The nasopharynges of the operating room personnel
C. Are associated with catecholamine release. C. The patient’s endogenous bacteria
D. Are usually benign. D. Airborne bacteria from the operating room
The most common location for the development of environment
atherosclerotic disease is? Which of the following is the most common form of
A. The renal artery. entrapment neuropathy?
B. The coronary arteries. A. Cubital tunnel syndrome
C. The abdominal aorta B. Tunnel of Guyon syndrome
D. The arteries of the circle of Willis C. Tarsal tunnel syndrome
D. D. Carpal tunnel syndrome

TOPNOTCH MEDICAL BOARD PREP SURGERY PHASE 3 HANDOUT BY DR LOUBOMIR ANTONIO Page 3 of 15
For inquiries visit www.topnotchboardprep.com.ph or email us at topnotchmedicalboardprep@gmail.com
This handout is only valid for the September 2021 PLE batch. This will be rendered obsolete for the next batch since we update our handouts regularly.
TOPNOTCH MEDICAL BOARD PREP SURGERY PHASE 3 HANDOUT BY DR LOUBOMIR ANTONIO
For inquiries visit www.topnotchboardprep.com.ph or https://www.facebook.com/topnotchmedicalboardprep/
This handout is only valid for the September 2021 PLE batch. This will be rendered obsolete for the next batch since we update our handouts regularly.
Superior vena cava (SVC) syndrome is a group of symptoms On closer examination, the body parts with surface area
caused by compression of the superior vena cava most burn are front of the torso, front and back of her right arm
often caused by bronchogenic carcinoma. Which of the as well as front and back areas of her right leg. What is the
following treatment options should extreme care be taken appropriate percentage of burn injury sustained by the
for it can further worsen SVC compression? patient?
A. Glucocorticoid A. 75%
B. Continuous positive pressure B. 45%
C. Endovascular stenting C. 15%
D. Furosemide D. 80%
Following laparotomy, gastrointestinal peristalsis The following statements about early post-operative fever
temporarily decreases. Which of the following statements are correct except:
is INCORRECT? A. Close to 90% of patients with fever on the first post-
A. Opioid can interfere with gastric motility operative day have a radiographic evidence of
B. NGT insertion is performed on all patients undergoing atelectasis
laparotomy. B. About 75% of patients with atelectasis doesn’t have
C. Peristalsis returns within 24 hours in the small fever
intestines followed by the stomach and then the colon. C. Atelectasis is a common cause of post-operative fever
D. In operations not involving the bowels, diet is D. Most fevers that appear in the first 24 hours after
resumed as soon as the effect of the anesthesia has surgery are the result of tissue injury sustained during
worn – off. the procedure
Which of the following statements correctly describes a Components of the Whipple’s triad except?
primary microbial peritonitis? A. Reproduction of symptoms with insulin
A. Microbes invade the normally sterile confines of the administration
peritoneal cavity through hematogenous route. B. Symptomatic fasting hypoglycemia
B. Treatment aimed at identifying the source of the C. Serum glucose <50mg/dL
contamination and possible resection of the diseased D. Relief of symptoms with glucose administration
organ A 64-year-old woman undergoing radical hysterectomy
C. Contamination of the peritoneal cavity from under general anesthesia is transfused with 2 U of packed
perforation or infection of any intra- abdominal organ RBCs. A hemolytic transfusion reaction during anesthesia
D. Antibiotic regimen should include coverage for will be characterized by which of the
aerobic and anaerobic organisms following?
Which of the following is an adjuvant treatment for breast A. Shaking chills and muscle spasms
cancer patients with estrogen receptor positive tumor? B. Fever and oliguria
A. Anakinra C. Hyperpyrexia and hypotension
B. Anastrozole D. Bleeding and hypotension
C. Trastuzumab A 35 year old female patient presents with fatigue,
D. Bicalutamide shortness of breath and increasing abdominal girth. She
Which drug is a kinase inhibitor, a type of targeted drug, also reports recent weight gain around 10 lbs the past 2
given to treat advanced breast cancer? months. PE: reveals dullness to percussion over the right
A. Capecitabine side, decreased breath sounds on the right, (+) shifting
B. Letrozole dullness and palpable left adnexal mass, what is the most
C. Lapatinib likely diagnosis?
D. Cisplatin A. Meig’s syndrome
Which of the following is NOT typically seen in patients B. Ruptured ectopic pregnancy
with benign prostatic hyperplasia (BPH)? C. Ovarian torsion
A. Poor correlation between size of prostate gland and D. Subserosal uterine myoma
degree of symptoms According to the transfusion guidelines, which among the
B. Higher intravesical pressure required to accomplish ff. is an inaccurate statement?
voiding A. A patient scheduled to undergo a major surgery with
C. Vesical muscle hypertrophy Hgb of 9.5mg/dl should be given 1 unit of pRBC
D. Decrease urinary outflow resistance. B. In a patient receiving massive transfusion, platelets
Patients undergoing resection of pheochromocytoma can and FFP should be transfused together with pRBCs in
be affected by the sudden release of catecholamines. Which a 1:1:1 ratio
of the following need NOT be available at the time of C. Most surgeries can be carried out safely with a platelet
surgery? count above 50x109 or INR below 2.0
A. Arterial line for monitoring D. Massive transfusion should be terminated once the
B. Nitroprusside patient is no longer actively bleeding
C. Beta Blocker Felty’s syndrome is composed of all of the ff. except?
D. Corticosteroid A. Rheumatoid arthritis
A 55 year old was rescued from a fire in Pasay City. She was B. Splenomegaly
rushed to the emergency room with burn injury . She is C. Leukopenia
unconscious. Responding only to pain stimuli with a blood D. Weight loss
pressure of 150/90, heart rate of 110/ minute, respiratory A 23 year old male patient presents with nausea, vomiting
rate of 30/min. 02 saturation at the ER showed 90 mmHg and stabbing abdominal pain most prominent after meals.
with 5L/min oxygen delivered by facemask. (+) abdominal distention, reflux episodes and tenderness
in the abdomen area. CT scan of the abdomen shows
Which of the following does not necessarily indicate that compression of the duodenum between the superior
the patient received pulmonary burn? mesenteric artery and the abdominal aorta, what is the
A. Percentage of burn injury most likely diagnosis?
B. Carbonaceous sputum A. SMA syndrome
C. Patient inhales explosive gases B. Wilkie’s syndrome
D. Upper torso burn C. Cast syndrome
D. Mesenteric root syndrome
E. All of the above

TOPNOTCH MEDICAL BOARD PREP SURGERY PHASE 3 HANDOUT BY DR LOUBOMIR ANTONIO Page 4 of 15
For inquiries visit www.topnotchboardprep.com.ph or email us at topnotchmedicalboardprep@gmail.com
This handout is only valid for the September 2021 PLE batch. This will be rendered obsolete for the next batch since we update our handouts regularly.
TOPNOTCH MEDICAL BOARD PREP SURGERY PHASE 3 HANDOUT BY DR LOUBOMIR ANTONIO
For inquiries visit www.topnotchboardprep.com.ph or https://www.facebook.com/topnotchmedicalboardprep/
This handout is only valid for the September 2021 PLE batch. This will be rendered obsolete for the next batch since we update our handouts regularly.
The classic triad of Plummer-Vinson syndrome is
composed of the ff. except?
A. Dysphagia
B. Esophageal webs
C. Iron deficiency anemia
D. Heartburn
Plummer’s disease is seen in
A. Grave’s disease
B. Toxic multinodular goiter
C. Hashimoto’s
D. Hypothyroidism
All of the ff. statements regarding blood transfusion are
correct except?
A. It is advised to adhere to a restrictive transfusion
strategy as much as possible
B. If the patient is anemic and symptomatic, transfusion
is warranted A simple mastectomy is different from a radical
C. Peripheral blood hematocrit should be the basis for mastectomy in that a simple mastectomy involves all of the
transfusion during acute blood loss ff. structures except?
D. There should be no universal Hgb level transfusion A. Nipple
trigger B. All breast tissue
In a patient with viral conjunctivitis, which among the ff. C. Areola
treatments should you avoid? D. Level I axillary lymph nodes
A. Oral acyclovir Which of the ff. structures is not part of the uveal tract?
B. Acyclovir eye ointment A. Choroid
C. Topical steroids B. Ciliary body
D. Supportive treatment C. Iris
Which of the ff. statements regarding fractures in children D. Retina
are true? A 24 year old male patient sustained a penetrating chest
A. Healing of fractures in adults is faster than in children trauma to the right hemithorax, he was brought to the
B. Immobilization is not tolerated well by children emergency room and was conscious but anxious, BP was
C. Healing is slower in fractures on the same plane of the 90/60, HR 135, RR 35, T>36.5C, PE revealed decreased to
movement of the joint absent breath sounds over the right hemithorax, with
D. Stabilization of epiphyseal fractures is critical to lagging over the right, dull to percussion, Xray reveals
minimize growth disturbances effusion up to half of the right hemithorax, a chest tube was
What knot tie is used when there is increased wound to be placed, where should be the site?
tension? A. 4th ICS posterior axillary line
A. Slip knot B. 2nd ICS parasternal line
B. Surgeon’s knot C. 5th ICS MCL
C. Square knot D. 5th ICS midaxillary line
D. Double square knot In relation to the question above, the patient is in what
SUTURE KNOTS stage of shock?
A. Class I
B. Class II
Square Knot
C. Class III
• Consists of two sequential D. Class IV
throws that lie in opposite
directions
• This is necessary to create a
knot that will not slip

Surgeon’s Knot
• Variation in which a double
throw is followed by a single
throw to increase the
friction on the suture
In relation to the question above, it is classified as a
material and to decrease the
massive hemothorax if
initial slip until a full square
A. >750 mL of blood in the pleural space
knot has been completed
B. >1000 mL of blood in the pleural space
C. >1500 mL of blood in the pleural space
All of the ff. are true regarding disc herniations except?
D. >2000 mL of blood in the pleural space
A. The L4 nerve root passes over the L4-L5 disc space
and may be compressed at this level If there is suspected cardiac injury in the case above,
B. Lumbar herniation can occur with or without history acutely, how many mL of blood can lead to cardiac
of trauma tamponade?
C. Surgery is indicated for progressive motor deficits A. <50 mL
D. Most herniated lumbar discs improve B. <100 mL
symptomatically without surgery C. <150 mL
D. <200 mL
In relation to the above case, if fluid resuscitation is to be
given to this patient, which among the ff. should be given
initially?
A. 1L colloid bolus
B. 20 mL/kg IV saline bolus
C. 1L IV Lactated Ringers bolus, 2 lines
D. 2L IV Lactated Ringers bolus 2 lines

TOPNOTCH MEDICAL BOARD PREP SURGERY PHASE 3 HANDOUT BY DR LOUBOMIR ANTONIO Page 5 of 15
For inquiries visit www.topnotchboardprep.com.ph or email us at topnotchmedicalboardprep@gmail.com
This handout is only valid for the September 2021 PLE batch. This will be rendered obsolete for the next batch since we update our handouts regularly.
TOPNOTCH MEDICAL BOARD PREP SURGERY PHASE 3 HANDOUT BY DR LOUBOMIR ANTONIO
For inquiries visit www.topnotchboardprep.com.ph or https://www.facebook.com/topnotchmedicalboardprep/
This handout is only valid for the September 2021 PLE batch. This will be rendered obsolete for the next batch since we update our handouts regularly.
Which of the following is not an indicated approach for 30 minutes after induction of anesthesia, the patient
pericardial decompression in patients with cardiac develops a fever of 39.5C, increased heart rate, rapid
tamponade? breathing, rigidity and increased CO2 production. This is
A. Right anterior thoracotomy the patient’s first surgery for thyroidectomy. Which among
B. Left anterior thoracotomy the ff. is the best management for this patient?
C. Median sternotomy A. Increase tidal volume and FiO2
D. Pericardiocentesis B. Give epinephrine
What is the sequence in which the body utilizes its stored C. Give dantrolene
energy? D. Give IV methylprednisolone
A. Glycogen -> fats -> protein A 60 year old male patient presents with a lesion on the
B. Protein -> fats -> glycogen face. The lesion is a pinkish erythematous plaque with a
C. Fats -> protein -> glycogen depressed center with raised borders. Hx shows significant
D. Glycogen -> protein - > fats sun exposure because he works as a farmer up to the
How many months will it take for the wound to achieve 70- present. What is the most likely diagnosis in this case?
80% of its tensile strength? A. Basal cell carcinoma
A. 1 month B. Malignant melanoma
B. 2 months C. Squamous cell carcinoma
C. 3 months D. Kaposi’s sarcoma
D. 4 months Which among the ff. skin cancers is most likely to recur
A type of hernia occurring at the lateral border of the locally and invasive?
rectus abdominis muscle? A. Squamous cell carcinoma
A. Morgagni hernia B. Basal cell carcinoma
B. Bankart C. Malignant melanoma
C. Pantaloon D. Dysplastic nevus
D. Spigelian Which among the ff. is the most common type of skin
E. Richter’s cancer?
A hernia through the superior lumbar triangle is called? A. Squamous cell carcinoma
A. Grynfeltt’s B. Basal cell carcinoma
B. Littre’s C. Malignant melanoma
C. Pantaloons D. Dysplastic nevus
D. Richter’s Which of the ff. is considered a risk factor for malignant
E. Spigelian melanoma?
During an abdominal surgery involving an extensive A. Presence of myelodysplastic nevus
hepatic laceration from trauma, the head surgeon says that B. Irregular border
they have to do a Pringle maneuver, which among the ff. C. Change in color
structures is least likely to be compressed? D. Raised surface
A. Common bile duct The type of malignant melanoma common among dark
B. Cystic duct skinned individuals found usually on the palmar, plantar
C. Hepatic artery and subungual areas?
D. Portal vein A. Superficial spreading
A 50 year old female patient underwent a needle guided B. Nodular
excisional biopsy in her right breast. The rest of her history C. Lentigo maligna
is unremarkable. This is classified under which of the ff. D. Acral lentiginous
wound classes? All of the ff. are part of the vertical facial buttress except?
A. Clean A. Nasomaxillary
B. Clean/contaminated B. Zygomaticomaxillary
C. Contaminated C. Pterygomaxillary
D. Dirty D. Hard palate and maxillary alveolus
In relation to the above case, which among the ff. Which among the ff. is a disorder of delayed gastric
statements are true? emptying in the absence of mechanical obstruction after
A. Prophylactic antibiotics should be given 30minutes post gastrectomy?
prior to incision A. Dumping disease
B. Readministration of antibiotics must be done to cover B. Diarrhea
1 to 2 half-lives of the antibiotic C. Gastric stasis
C. Antibiotics should be limited only to up to 72 hours D. Roux syndrome
post-surgery The cause of Zollinger Ellison syndrome is commonly
D. All of the above found at the?
E. None of the above A. Pancreas
Which antibiotic provides adequate coverage for B. Proximal duodenum
prophylaxis for most types of procedures? C. Abdominal lymph nodes
A. Cefuroxime D. Gastrinoma triangle
B. Cefazolin Which among the ff. is the least likely manifestation of
C. Metronidazole Menetrier’s disease?
D. Ampicillin sulbactam A. Large rugal folds
During elective colorectal surgery, considerable spillage of B. Diffuse hyperplasia of mucus cells
colon contents occurs, the surgeon then decides to C. Hypoplasia of parietal cells
administer antibiotic therapy, this is classified under D. Increased gastric acid
A. Prophylaxis A 36 year old male patient presents with recurrent
B. Empiric therapy intermittent hematemesis. Initial Upper GI endoscopy
C. Therapy of established infection showed negative findings. A repeat upper GI endoscopy
D. Culture guided therapy showed a large submucosal artery near the
gastroesophageal junction what is the diagnosis?
A. Gastric vascular ectasia
B. Dieulafoy’s lesion
C. Esophageal varices
D. Menetrier’s disease

TOPNOTCH MEDICAL BOARD PREP SURGERY PHASE 3 HANDOUT BY DR LOUBOMIR ANTONIO Page 6 of 15
For inquiries visit www.topnotchboardprep.com.ph or email us at topnotchmedicalboardprep@gmail.com
This handout is only valid for the September 2021 PLE batch. This will be rendered obsolete for the next batch since we update our handouts regularly.
TOPNOTCH MEDICAL BOARD PREP SURGERY PHASE 3 HANDOUT BY DR LOUBOMIR ANTONIO
For inquiries visit www.topnotchboardprep.com.ph or https://www.facebook.com/topnotchmedicalboardprep/
This handout is only valid for the September 2021 PLE batch. This will be rendered obsolete for the next batch since we update our handouts regularly.
A 30 year old male sheep farmer presents with vague What is the most vulnerable segment of the large intestine
abdominal discomfort. Workup reveals multiple small well to obstruction?
defined hypodense lesions in the liver with distinct wall, A. Ascending colon
some containing calcifications. Which among the ff. is true B. Descending colon
regarding this condition? C. Transverse colon
A. Mebendazole is the best drug for this condition D. Sigmoid colon
B. Metronidazole should be given A 36 year old male patient presents in the emergency
C. Spillage of cyst contents may result in anaphylaxis department with abdominal pain generalized intermittent
D. Large abscess which do not respond to medical characterized as spasmodic accompanied by nausea and
therapy can be aspirated vomiting. There is abdominal distention as well. There is
In relation to the above case, the causative agent of the no rebound tenderness noted. Patient has not passed any
above disease is? stool for the last 2 days. There is also no passage of flatus.
A. Entamoeba histolytica What is the next best step in the management of this
B. Echinococcus granulosus patient?
C. Escherichia coli A. Scout film of the abdomen
D. All of the above B. Abdominal CT scan
Gallbladder stones commonly lodge where? C. Ultrasound of the abdomen
A. Common bile duct D. Colonoscopy
B. Common hepatic duct In relation to the above case, which among the ff. is the
C. Cystic duct most likely finding after workup?
D. Ampulla of Vater A. Appendicolith
Which among the ff. is the least likely manifestation of B. Bent inner tube or coffee bean appearance
choledochal cysts? C. Edematous pancreas with fat stranding
A. Fever D. Multiple colonic diverticula
B. Pain If the above diagnosis turns out to be volvulus, what is the
C. Jaundice next best step in the management?
D. Mass A. Endoscopic detorsion
In relation to the above case, what is the management of B. Emergency laparotomy
choice for a type 3 choledochal cyst? C. IV fluids and observation
A. Segmental resection of the liver D. Surgical debridement
B. Roux-en-Y hepaticojejunostomy What is the most common site of volvulus?
C. Sphincterotomy A. Cecum
D. All of the above B. Sigmoid
What is the most common functional pancreatic endocrine C. Transverse colon
neoplasm? D. Jejunum
A. Gastrinoma All of the ff. are indications for emergency surgery in
B. Insulinoma ulcerative colitis except?
C. VIPoma A. Toxic megacolon
D. Glucagonoma B. Massive hemorrhage
A 60 year old male patient a known diabetic for 20 years C. Fulminant colitis
presents with jaundice. He denies any abdominal pain, D. Intractability despite maximal medical therapy
vomiting or nausea or anorexia but with some documented E. None of the above
10lb weight loss over the past month. There was no All of the ff. are true regarding ulcerative colitis except?
palpable abdominal mass. The patient also reports dark A. The rectum is invariably involved
colored urine and pale stools. What is the next best step in B. Colon may have lead pipe appearance
the management of this patient? C. Strictures are common
A. Abdominal CT scan D. It is associated strongly with primary sclerosing
B. Abdominal ultrasound cholangitis
C. ERCP Which among the ff. tumor markers has utility for
D. HIDA scan pancreatic cancer?
In relation to the above case, if workup reveals dilated bile A. Calcitonin
ducts what is the next best step in the management of this B. CA 19-9
patient? C. AFP
A. Abdominal CT scan D. CD117
B. Abdominal ultrasound A 24 year old patient was brought to the emergency
C. ERCP department after being involved in a scuffle. He sustained
D. HIDA scan numerous deep lacerations over the thigh and abrasions
In relation to the above case, what is the most likely over the extremities. He has unrecalled vaccination
diagnosis? history. Which among the ff. should be given?
A. Choledocholithiasis A. Tetanus immune globulin only
B. Viral hepatitis B. Td vaccine only
C. Liver cirrhosis C. Both A and B
D. Ampullary mass D. Tetanus antitoxin
In relation to the above case, if for example the diagnosis is Medications administered per rectum has less first pass
a pancreatic head mass, what is the best definitive effect because the blood supply bypasses the portal tract
management for this case? via
A. Pancreaticoduodenectomy A. Superior rectal vein
B. Minimally invasive pancreatectomy B. Middle rectal vein
C. Combine chemoradiation C. Inferior rectal vein
D. Endoscopy and biliary stenting D. Both B and C
In a case of an acute abdomen, the examiner places the E. Both A and B
patient on his left side and then proceeds to slowly extend
the right thigh to elicit abdominal pain on the RLQ, what is
this called?
A. Blumberg sign
B. Rovsing’s sign
C. Psoas sign
D. Obturator sign

TOPNOTCH MEDICAL BOARD PREP SURGERY PHASE 3 HANDOUT BY DR LOUBOMIR ANTONIO Page 7 of 15
For inquiries visit www.topnotchboardprep.com.ph or email us at topnotchmedicalboardprep@gmail.com
This handout is only valid for the September 2021 PLE batch. This will be rendered obsolete for the next batch since we update our handouts regularly.
TOPNOTCH MEDICAL BOARD PREP SURGERY PHASE 3 HANDOUT BY DR LOUBOMIR ANTONIO
For inquiries visit www.topnotchboardprep.com.ph or https://www.facebook.com/topnotchmedicalboardprep/
This handout is only valid for the September 2021 PLE batch. This will be rendered obsolete for the next batch since we update our handouts regularly.
A patient with liver cirrhosis was treated with a shunt for In relation to the above case, if initial tissue histology was
the treatment of portal hypertension, which of the ff. blood attempted initially and was an acellular specimen with cyst
vessels are the ones connected? fluid and blood, what is the next best step in the
A. Portal vein and inferior vena cava management?
B. Hepatic vein and portal vein A. Repeat biopsy under ultrasound guidance
C. External iliac and inferior vena cava B. Repeat biopsy after 6 months
D. Inferior vena cava and azygous veins C. For observation
A patient involved in a MVA comes into the ED. The D. For elective surgery
physician knows that the most commonly injured organ In relation to the above case, if initial tissue histology
with blunt abdominal trauma is returned as suspicious for malignancy, what is the next
A. Liver best step in the management?
B. Spleen A. Repeat biopsy under ultrasound guidance
C. Small intestine B. Repeat biopsy after 6 months
D. Colon C. For observation
The most common clinical sign in pheochromocytoma is D. For elective surgery
A. Palpitations In relation to the above case, if tissue biopsy revealed
B. Anxiety polygonal spindle shaped cells with amyloid, what is the
C. Diaphoresis most likely diagnosis?
D. Hypertension A. Metastatic lymphoma
Which medication should be given 1-3 weeks before B. Papillary thyroid cancer
surgery of pheochromocytoma? C. Medullary thyroid cancer
A. Phenoxybenzamine D. Follicular thyroid cancer
B. Clonidine E. Anaplastic cancer
C. Prazosin In relation to the above question, what is the treatment of
D. Metoprolol choice for this patient?
A 1 week old infant presents bilious vomiting and A. RAI ablation
irritability. On examination he has no abdominal B. Total thyroidectomy
distention, no palpable masses, ultrasound reveals 2 C. Total lobectomy
discrete cystic structures in the upper abdomen, what is D. Subtotal thyroidectomy
the most likely diagnosis? E. Both A and D
A. Hypertrophic pyloric stenosis In addition to the above management which among the ff.
B. Midgut volvulus should be included?
C. Duodenal atresia A. Right lateral neck dissection
D. Malrotation B. Bilateral lateral neck dissection
A 28 year old female patient presents with painful C. Bilateral central neck dissection
subcutaneous nodules, draining sinus tracts with foul D. Right central neck dissection
smelling exudates, some previous scars seen in the right E. None of the above
axillary area. No other complaints were noted. This has been In relation to the above case, which among the ff. is the
on and off for past 2 years. What is the most likely diagnosis? more sensitive tumor marker for this disease?
A. Pyoderma gangrenosum A. Calcitonin
B. Hidradenitis suppurativa B. CEA
C. Tuberculous lymphadenitis C. Thyroglobulin
D. Squamous cell carcinoma D. AFP
Which of the ff. cancers can spread and metastasize Each of the following conditions is a primary indication for
through the Batson’s plexus? thyroidectomy EXCEPT:
A. Breast A. 2 cms thyroid nodule with hoarseness
B. Prostate B. Thyroiditis
C. Colorectal C. Recurrent Graves disease in second trimester
D. All of the above pregnancy
E. None of the above D. Dysphagia with submanubrial goiter
24 year old male patient was brought to the emergency Which of the following is not an acceptable indication for
department, he looks cyanotic, gasping for air, dyspneic. He thyroidectomy in a patient with Grave’s disease?
was involved in a local scuffle. Examination reveals A. When the patient complains of difficulty of breathing.
multiple scattered abrasions and lacerations and swelling B. When remission is not achieved after 1 year of
and hematoma over the neck region. Diagnosis was medical therapy.
suspected laryngeal fracture, what is the next best in the C. When patient refuses to take medications
management of this patient? D. When the patient is allergic to the medications
A. Endotracheal intubation Which surgeon was awarded the Nobel Prize in Physiology
B. Emergency tracheostomy for Medicine for his work on the “physiology, pathology,
C. Cardiopulmonary resuscitation and surgery of the thyroid gland?”
D. Rigid bronchoscopy A. Theodore Billroth
A 32 year old female patient presents with a neck mass B. Emil Kocher
which she incidentally discovered 6 months ago. The mass C. John Hunter
has been noted to slowly enlarge. She is otherwise D. Harvey Cushing
asymptomatic and sought consult because she was merely The most common variant of a congenital diaphragmatic
concerned. PE reveals an asymmetrically enlarged thyroid hernia constitutes which of the ff. defects?
with the right lobe larger with an ill defined mass, there A. Anterolateral
was some palpable cervical lymphadenopathy. Initial TSH B. Posterolateral
was within normal limits, what is the next best step in the C. Anteromedial
diagnosis of this patient? D. Posteromedial
A. FNAB In relation to the above case, it is usually called this type of
B. excisional biopsy hernia?
C. neck ultrasound A. Morgagni
D. CT scan of the neck B. Richter’s
C. Bochdalek
D. Spigelian

TOPNOTCH MEDICAL BOARD PREP SURGERY PHASE 3 HANDOUT BY DR LOUBOMIR ANTONIO Page 8 of 15
For inquiries visit www.topnotchboardprep.com.ph or email us at topnotchmedicalboardprep@gmail.com
This handout is only valid for the September 2021 PLE batch. This will be rendered obsolete for the next batch since we update our handouts regularly.
TOPNOTCH MEDICAL BOARD PREP SURGERY PHASE 3 HANDOUT BY DR LOUBOMIR ANTONIO
For inquiries visit www.topnotchboardprep.com.ph or https://www.facebook.com/topnotchmedicalboardprep/
This handout is only valid for the September 2021 PLE batch. This will be rendered obsolete for the next batch since we update our handouts regularly.
What is the management of choice in newborn infants who 30 y/o M victim of a motor vehicular accident while riding
experience respiratory distress and are found to have his motorcycle under alcohol intoxication came to the ER.
congenital diaphragmatic hernia? BP 80/60, HR 121, RR20, T>36.5 with ecchymosis and
A. Emergency surgery abrasions on the abdomen. Patient is anxious and
B. Mechanical ventilation confused, Which of the ff. is the appropriate next step in
C. Keeping the ductus arteriosus patent with PGE1 evaluation of the abdomen?
D. IV antibiotics A. FAST
What is the most common of the primary esophageal B. diagnostic peritoneal lavage
motility disorders? C. CT scan of the abdomen
A. Achalasia D. Laparotomy
B. Nutcracker esophagus The above patient is most likely in which stage or class of
C. Diffuse esophageal spasm shock?
D. Hypertensive Lower esophageal sphincter A. Class 1
In relation to the question above, all of the ff. manometric B. Class 2
findings are consistent with the disease above except? C. Class 3
A. Increased mean peristaltic amplitude in distal D. Class 4
esophagus Evaluation of the victim above showed free intraperitoneal
B. Increased duration of contractions fluid in the left upper quadrant, what is the appropriate
C. Incomplete LES relaxation next step in the management of this patient?
D. Normal peristaltic sequence A. FAST
What is the most common cause of ulcer related death? B. Diagnostic peritoneal lavage
A. Perforation C. CT scan of the abdomen
B. Bleeding D. Laparotomy
C. Malignancy Patient 23 y/o M had a motor vehicular accident, his
D. Penetration helmet was crushed with visible ecchymosis over the right
All of the ff. are tissue-based repairs of inguinal hernia eye area, patient was awake alert with no reported loss of
except? consciousness, Vital signs were stable, PE: revealed some
A. Lichtenstein deficits with extraocular movements of the right eye, which
B. McVay of the ff. is the most common site of orbital blowout
C. Shouldice fractures?
D. Bassini A. Roof
A 26 year old male patient was brought into the emergency B. Medial wall
was involved in a scuffle there was penetrating trauma a C. Lateral wall
knife to the left chest wall, patient was dead on arrival and D. Floor
CPR was performed for 5 minutes already while on the way In the case above, which bone is usually affected?
to the hospital, what is the next best step in the A. Zygomatic bone
management? B. Lamina papyracea
A. Continue CPR C. Maxillary bone
B. Resuscitative thoracotomy D. Palatine bone
C. Immediate chest tube insertion Patient who sustained a motor vehicular accident without
D. Emergency thoracentesis a helmet was brought to the ER, patient sustained multiple
Which of the ff. is not included in a skin sparing lacerations and abrasions and contusions over the facial
mastectomy? area, there was (+) epistaxis, patient was noted to be
A. Skin anxious and was breathing noisily, BP100/60 HR108 RR 30
B. Nipple T>36.5 which of the ff. is the immediate course of action?
C. Areola A. Establish a patent airway
D. Breast tissue B. Insert 2 large bore IV cannulas
26 y/o patient with an anterior abdominal stab wound C. Emergent laparotomy
came to the ER. BP 120/80, HR108 RR20 D. FAST
T-afebrile, the knife wound is near the RUQ and right flank, Upon careful examination, patient was noted to have a step
what is the appropriate next step in the management of deformity near the infraorbital margin, bilateral
this patient? circumorbital ecchymosis, epistaxis, zygomatic area is
A. Diagnostic peritoneal lavage intact, nasal bones are mobile as well as the center midface,
B. Emergent laparotomy some numbness over the maxillary area, which type of
C. Local wound exploration fracture is this?
D. FAST A. Le Fort 1
Upon close examination of the patient above, it was found B. Le Fort 2
out that there is violation of the posterior fascia, what test C. Le Fort 3
will you order to know the trajectory of the knife and be D. Le Fort 4
able to observe the patient after? In a Le Fort fracture, if the midface was involved, you may
A. Diagnostic peritoneal lavage find the ff. symptoms in the patient except?
B. FAST A. Periorbital ecchymosis
C. Local wound exploration B. Anosmia
D. Abdominal CT scan C. Inability to protrude tongue
Just in case a diagnostic peritoneal lavage was done, the D. Epistaxis
surgeon knows that all of the ff. are positive findings for What would decrease the chance of recurrence to a CIS
abdominal trauma except? breast cancer?
A. RBC count >100,000 A. Chemotherapy
B. Lipase >2000 U/L B. Lumpectomy
C. Alkaline Phosphatase >2 IU/L C. MRM
D. Bilirubin >0.01 mg/dL D. Radiation
A modified radical (“Patey”) mastectomy removes all of the
ff. except?
A. All breast tissue
B. Skin
C. Level 1,2,3 axillary lymph nodes
D. Nipple-areola complex
E. Pectoralis major and minor

TOPNOTCH MEDICAL BOARD PREP SURGERY PHASE 3 HANDOUT BY DR LOUBOMIR ANTONIO Page 9 of 15
For inquiries visit www.topnotchboardprep.com.ph or email us at topnotchmedicalboardprep@gmail.com
This handout is only valid for the September 2021 PLE batch. This will be rendered obsolete for the next batch since we update our handouts regularly.
TOPNOTCH MEDICAL BOARD PREP SURGERY PHASE 3 HANDOUT BY DR LOUBOMIR ANTONIO
For inquiries visit www.topnotchboardprep.com.ph or https://www.facebook.com/topnotchmedicalboardprep/
This handout is only valid for the September 2021 PLE batch. This will be rendered obsolete for the next batch since we update our handouts regularly.
A Halsted radical mastectomy removes all of ff. except? A 26 y/o patient after an episode of an alcoholic binge
A. All breast tissue drinking had several episodes of retching and vomiting.
B. Level 1,2,3 axillary lymph nodes Patient started having profuse hematemesis afterward.
C. Nipple-areola complex Endoscopy revealed longitudinal fissures in the mucosa of
D. Pectoralis major and minor the herniated stomach with bleeding, what is the
E. None of the above diagnosis?
After undergoing right breast surgery, patient developed A. Boerhaave’s syndrome
shoulder weakness and winging of the right scapula, which B. Spontaneous esophageal rupture
nerve is implicated? C. Mallory Weiss tear
A. Thoracodorsal nerve D. Bleeding esophageal varices
B. Axillary nerve A 30 y/o M patient was rushed to the ER after being
C. Long thoracic nerve involved in a stabbing incident on the left lateral upper
D. Anterior thoracic pectoral nerve chest wall, patient was noted to be gasping for air, anxious,
All of the following are potential candidates for radiation BP70/50 despite bolus fluid resuscitation it increased to
therapy except? 80/60, HR 130, T>36.5, O2 saturation 80%, PE: noted
A. Those undergoing breast conservation surgery costal retractions, decreased breath sounds over the left
B. With ≥4 involved axillary lymph nodes lung field, hyperresonant to percussion on the left
C. Low risk patients after simple mastectomy with hemithorax, what is the next most appropriate step for this
negative margins patient?
D. Internal mammary node involvement A. CTT
E. Tumor with chest wall and skin involvement B. Needling
All of the ff. are potential candidates for chemotherapy in C. Chest xray
breast cancer except? D. Chest CT scan
A. Hormone receptor negative >1cm in size If a chest xray was requested in the above patient, which of
B. Node positive breast cancer the ff. findings is most compatible with the above case?
C. Tubular/mucinous/medullary breast cancer >3cm A. Consolidation of the left upper lung
D. For stage 3A breast cancer as neoadjuvant B. Hyperlucent right lung field
chemotherapy C. Deviation of trachea towards the right
E. Negative node breast cancer ≤0.5cm D. Absence of vascular markings over the right
Routine screening mammography in women for breast hemithorax
cancer is started at what age? Intervention for the above patient should be done at this
A. 35 location
B. 40 A. Fifth Left ICS posterior axillary line
C. 45 B. 2nd left ICS mid clavicular line
D. 50 C. 2nd right ICS mid clavicular line
The preferred method for diagnosis of palpable or D. 7th left ICS posterior axillary line
nonpalpable breast abnormalities? E. 7th right ICS posterior axillary line
A. Fine needle aspiration biopsy A 56 y/o male with chronic alcoholism presented to the ED
B. Needle wire localization excision biopsy with massive hematemesis, patient has no abdominal pain,
C. Core needle biopsy patient has no history of recent NSAID use or persistent
D. UTZ guided fine needle aspiration abdominal pain or reflux, no previous retching or vomiting
Mechanism of characteristic skin retraction in breast episodes until this present one, BP80/60 HR128 RR 28
cancer? T>36.5, what is the most likely diagnosis?
A. Localized edema from blocked drainage of lymph A. Bleeding peptic ulcer disease
B. Peau d’orange response B. Mallory-Weiss tear
C. Desmoplastic response shortens Cooper’s ligaments C. NSAID gastropathy
D. Nipple inversion D. Bleeding esophageal varices
All of the ff. characteristics point to DCIS except? In relation to the patient above, what is the next best step
A. Usually in the older age group of management?
B. With microcalcifications on mammography A. Upper GI Endoscopy
C. Usually multicentric and bilateral B. Sengstaken
D. With high incidence of synchronous invasive cancer C. Blakemore tube insertion
The type of esophageal hernia where there is an upward D. Epinephrine injection
dislocation of the cardiac portion of the stomach into the E. IV fluid resuscitation
posterior mediastinum In relation to the patient above, which of the ff. can be done
A. Type 1 to help stop the acute bleeding of esophageal varices?
B. Type 2 A. Epinephrine
C. Type 3 B. Octreotide
D. Type 4 C. Vasopressin
Which of the ff. factors will lead to relaxation of the Lower D. Propranolol
esophageal sphincter? Which of the ff. sign or symptom if present makes the
A. Swallowing diagnosis of esophageal perforation almost certain?
B. Decreased gastric luminal pressure A. Subcutaneous emphysema
C. Increased esophageal pressure B. Pain
D. Increased esophageal pH C. retching/vomiting
A tooth abscess which spread to the neck area? D. Fever
A. Bezold’s abscess Tumor affectation of the nasopharyngeal area will most
B. Ludwig’s angina likely metastasize and involve which lymph nodes?
C. Pautrier’s abscess A. Level VI
D. Munro’s abscess B. Level V
Rovsing’s sign is elicited by? C. Level II
A. Pain in the RLQ when the RLQ is palpated D. Level IV
B. Pain in the RLQ when the LLQ is palpated All of the ff. are indications for bariatric surgery except?
C. Pain in the LLQ when the RLQ is palpated A. Patient feels fat
D. Pain in the LLQ when the LLQ is palpated B. Psychologically stable
C. BMI ≥40 with no comorbids
D. BMI ≥35 with comorbids

TOPNOTCH MEDICAL BOARD PREP SURGERY PHASE 3 HANDOUT BY DR LOUBOMIR ANTONIO Page 10 of 15
For inquiries visit www.topnotchboardprep.com.ph or email us at topnotchmedicalboardprep@gmail.com
This handout is only valid for the September 2021 PLE batch. This will be rendered obsolete for the next batch since we update our handouts regularly.
TOPNOTCH MEDICAL BOARD PREP SURGERY PHASE 3 HANDOUT BY DR LOUBOMIR ANTONIO
For inquiries visit www.topnotchboardprep.com.ph or https://www.facebook.com/topnotchmedicalboardprep/
This handout is only valid for the September 2021 PLE batch. This will be rendered obsolete for the next batch since we update our handouts regularly.
A 26 y/o patient who had a motor vehicular accident All of the ff. are possible surgical indications for surgery in
sustained an open fracture of the right femur, all of the ff. Crohn’s disease except?
are appropriate interventions for this patient except? A. Repair or removal of segment with fistulae
A. Immediate casting and immobilization B. Stricturoplasty or resection
B. Irrigation and wound debridement C. Intraabdominal abscess
C. antibiotic treatment D. Prevention of recurrence of the disease
D. assessment of neurovascular status of the limb All of the ff. findings point to fracture of the skull base
All of the ff. are appropriate in the management of burn except?
patients except? A. Otorrhea
A. Give tetanus vaccination if appropriate B. Battle’s sign
B. Early excision and grafting of full and deep partial C. Anosmia
thickness burns is appropriate D. Raccoon eyes
C. Give prophylactic IV antibiotic therapy E. None of the above
D. Electrical, chemical and inhalational burns warrant Most common cause of cataract?
referral to a burn center A. Steroid use
Topical burn wound therapy that is used mostly for B. Trauma
prevention of wound infections, soothing quality, C. Aging
inexpensive, not absorbed systemically, may cause D. Congenital
neutropenia and cannot be used for skin graft sites A 44 y/o female patient presents to the ED with sudden
A. Silver sulfadiazine blurring of vision more prominent on the right eye,
B. Mafenide acetate accompanied by unilateral right sided headache and eye
C. Silver nitrate pain, nausea and vomiting, PE: ciliary injection over the
D. Biobrane right eye, smoky cornea, mid-dilated pupil, what is the
Topical burn wound therapy that is used mostly for both most likely diagnosis?
treatment and prevention of burn wound infections, can be A. Open angle glaucoma
used on skin grafts and can penetrate eschar, associated B. Optic neuritis
with pain on application and absorbed systemically C. Acute angle closure glaucoma
causing metabolic acidosis D. Acute retinal detachment
A. Silver sulfadiazine E. CRAO
B. Mafenide acetate Which of the ff. is an expected fundoscopic finding in this
C. Silver nitrate patient?
D. Biobrane A. Cherry red spot
Topical burn wound therapy that is used mostly as dilute B. Papilledema
solution with broad antimicrobial activity, it is associated C. Absent ROR and cannot visualize the retina
with hyponatremia and methemoglobinemia. Inexpensive D. Increased cup to disc ratio
but causes black stains on garments All of the ff. are fundoscopic findings in glaucoma except?
A. Silver sulfadiazine A. Thin neuroretinal rim
B. Mafenide acetate B. Notching of the optic cup at the inferior rim
C. Silver nitrate C. Cherry red spot in the macula
D. Biobrane D. Displacement of central retinal vessels
The most common symptom or presentation of Which of the ff. diagnostic procedures can visualize the
intussusceptions in the adult? anterior chamber angle?
A. Bleeding A. Gonioscopy
B. Constipation B. Tonometry
C. Weight loss C. Indirect fundoscopy
D. Bowel obstruction D. Goldmann perimetry
Patient is a 7 y/o male who presents with left groin or knee All of the ff. medications used for glaucoma suppress
pain, decreased hip motion and a limp. There was no aqueous production except?
history of injury/fall, xray reveals osteonecrosis of the A. Timolol
proximal femoral epiphysis, what is the most likely B. Latanoprost
diagnosis C. Brimonidine
A. Legg-Calve-Perthes disease D. Dorzolamide
B. Osgood-Schlatter disease All of the ff. are surgical procedures for the treatment of
C. Slipped capital femoral epiphysis glaucoma except?
D. Developmental dysplasia of the hip A. Iridotomy
Patient is an active 14 y/o Male who is part of the schools B. Laser trabeculoplasty
track and field team, he presents with localized pain and C. Panretinal laser photocoagulation
tenderness at the area of the tibial tubercle, xray reveals D. Trabeculectomy
calcification of the tendon near the tibial tubercle, what is A 55 y/o male patient with hypertension and diabetes
the diagnosis? wakes up in the morning only to find out that his right eye
A. Legg-Calve-Perthes disease was bright red near the lower part of the pupil, patient has
B. Osgood-Schlatter disease no eye pain or blurring of vision or discharge, no previous
C. Slipped capital femoral epiphysis history of similar episodes, no other associated symptoms.
D. Developmental dysplasia of the hip Patient on antihypertensives, oral hypoglycaemic agents
Which of the following statements is INCORRECT regarding and aspirin. What is the most likely diagnosis?
Galeazzi’s fracture? A. Anterior uveitis
A. This is fracture of radial shaft with a distal radio-ulnar B. Allergic conjunctivitis
joint injury. C. Iritis
B. If the distal radio-ulnar joint is unstable , wires are D. Subconjunctival hemorrhage
used to pin the distal radius in a reduced position and
immobilized post- operatively for 4-6 weeks
C. In adults Galeazzi fracture is managed non-
operatively.
D. The recommended treatment is open reduction –
internal fixation (ORIF) and operative evaluation
stability of the distal radio-ulnar joint.

TOPNOTCH MEDICAL BOARD PREP SURGERY PHASE 3 HANDOUT BY DR LOUBOMIR ANTONIO Page 11 of 15
For inquiries visit www.topnotchboardprep.com.ph or email us at topnotchmedicalboardprep@gmail.com
This handout is only valid for the September 2021 PLE batch. This will be rendered obsolete for the next batch since we update our handouts regularly.
TOPNOTCH MEDICAL BOARD PREP SURGERY PHASE 3 HANDOUT BY DR LOUBOMIR ANTONIO
For inquiries visit www.topnotchboardprep.com.ph or https://www.facebook.com/topnotchmedicalboardprep/
This handout is only valid for the September 2021 PLE batch. This will be rendered obsolete for the next batch since we update our handouts regularly.
A 24 y/o sexually active male presents to the clinic with red Type of shock where cardiac index is increased, systemic
eye initially over the right eye eventually involving the left. vascular resistance is decreased, decreased CVP and
(+) mild pain and foreign body sensation, PE reveals: decreased mixed venous O2, and no change in venous
conjunctival injection with thick copious purulent capacitance
discharge and matting of the eyelashes, which among the ff. A. Hypovolemic
is the most likely causative agent? B. Septic
A. HSV C. Cardiogenic
B. Amebic keratitis D. Neurogenic
C. Gonorrhoea All of the ff. are true regarding antibiotic prophylaxis in
D. Syphilis surgery except?
Which of the ff. responses is immediately activated A. Select an agent for organisms commonly found at
following severe traumatic injury? surgical site
A. Acute proinflammatory response B. Initial Dose of antibiotic given within 30min prior to
B. Anti-inflammatory response incision
C. Suppression of adaptive immunity C. Antibiotic should be redosed depending on half life of
D. All of the above the agent
Which of the ff. events promote the greatest severity of D. Antibiotics should only be used up to 24-48 hours
nitrogen wasting? after surgery
A. Skeletal trauma Patient is about to undergo a cholecystectomy, this is
B. Severe sepsis classified under which type of surgical wound?
C. Major burns A. Class 1
D. Elective surgery B. Class 2
The intravascular volume or plasma composes how many C. Class 3
% of the body weight? D. Class 4
A. 5% E. Class 1D
B. 15% All of the ff. are considered manifestations of the bloody
C. 20% vicious cycle and is an indication for damage control
D. 40% surgery except?
E. 60% A. Coagulopathy
The most common cause of volume deficit in surgical B. Hypotension
patients is C. Hypothermia
A. Intraoperative blood loss D. Metabolic acidosis
B. Inadequate oral intake Which of the ff. accurately describes flail chest?
C. Loss of GI fluids A. Paradoxical abdominal inspiration
D. Increased urinary excretion of free water B. 2 or more contiguous ribs are fractured in at least 3
GI secretion with the highest potassium content? locations
A. Stomach C. 3 or more contiguous ribs are fractures in at least 2
B. Small intestine locations
C. Colon D. Paradoxical chest inspiration
D. Pancreas All of the following are part of the secondary survey in
GI secretion with the highest HCO3 content? trauma except?
A. Stomach A. Allergies
B. Small intestine B. Neurological deficits
C. Colon C. Pregnancy status
D. Pancreas D. Past illnesses
Which of the ff. electrolyte disturbances is associated with
nausea, vomiting and weakness?
A. Hyperkalemia
B. Hypermagnesemia END OF SURGERY – PHASE 3
C. Hypercalcemia
D. All of the above
Which of the ff. intravenous solutions most closely
approximates extracellular fluid?
A. Lactated Ringer’s
B. 0.9% NaCl
C. D5 0.45% NaCl
D. D5W
All of the ff. are basic components of damage control
resuscitation except?
A. Permissive hypotension
B. Minimizing crystalloid based resuscitation
C. Administration of predefined blood products
D. Appropriate use of vasopressors
Type of von Willebrand factor deficiency where there is a
qualitative defect of the von Willebrand factor?
A. Type 1
B. Type 2
C. Type 3
D. Type 4
The most common abnormality of hemostasis that results
in bleeding in the surgical patient?
A. Thrombocytopenia
B. Anticoagulant use
C. Coagulation factor deficiency
D. Functional platelet defect

TOPNOTCH MEDICAL BOARD PREP SURGERY PHASE 3 HANDOUT BY DR LOUBOMIR ANTONIO Page 12 of 15
For inquiries visit www.topnotchboardprep.com.ph or email us at topnotchmedicalboardprep@gmail.com
This handout is only valid for the September 2021 PLE batch. This will be rendered obsolete for the next batch since we update our handouts regularly.
TOPNOTCH MEDICAL BOARD PREP SURGERY PHASE 3 HANDOUT BY DR LOUBOMIR ANTONIO
For inquiries visit www.topnotchboardprep.com.ph or https://www.facebook.com/topnotchmedicalboardprep/
This handout is only valid for the September 2021 PLE batch. This will be rendered obsolete for the next batch since we update our handouts regularly.
Complications of • Infection
BUZZWORDS exploratory • Atelectasis
QUESTION ANSWER laparotomy • Ileus
• Gradient Pressure <30mmHg Causes of stress- • Glucagon
(diastolic P - compartment P) related hyperglycemia • Epinephrine
• Absolute compartment P > in post-op patients • Glucocorticoid
Fasciotomy is
30mmHg • Eye opening
indicated if Components of
• Ischemic periods >6 hours • Verbal response
• Combined arterial and venous Glasgow Coma Scale
• Motor response
injuries • Central venous pressure
• Remnants of amnion are not Invasive monitoring of
• When large amounts of fluid
reabsorbed patient’s fluid status
are administrated
• Abdominal organs in the Most common thyroid
Omphalocele features hernia cancer affecting • Papillary thyroid carcinoma
• Not covered by membranes younger patients
• Abdominal wall defect in Management for
midline middle ear and • Radical mastoidectomy
• Encounters less resistance mastoid disease with
Monofilament suture
when passing through tissues cholesteatoma
advantages vs
• Less infection-causing • Primary tumor
multifilament Structures excised in
organisms • Cervical nodes
• Pliability for ease of handling composite resection
Ideal suture • Part of the mandible
and knot security Prognostic indicator in
characteristics
• Uniform tensile strength medullary thyroid • Calcitonin doubling time
Closed by secondary • For heavily contaminated carcinoma patients
intention wounds • Extranodal extension
Coagulation factors • Factors VIII, IX, XI, XII Indications of post-op • Perineural invasion
tested by aPTT (intrinsic) radiation therapy • Lymphovascular invasion
• Retained surgical items • Positive cervical lymph nodes
• Wrong-site surgery Indications for • Suspicious for malignancy
Never events
• Death on the day of surgery of surgical removal of • Symptoms of pressure
a normal healthy patient nodular goiter • Substernal extension
• Deep breathing exercises Contributory factors to • Defective lower esophageal
Pulmonary
• Early mobilization the development of sphincter
rehabilitation
• Incentive spirometry GERD • Degree of hiatal herniation
Clean wounds • Hernia repair, breast biopsy • Pituitary apoplexy
• Requires short course of Acute painful visual • Acute onset of headache,
Post-operative UTI
antibiotics (3-5 days) loss without red eye nausea, visual field loss, and
Management of ocular paresis
transfusion-related • Stop transfusion • Glaucoma
acute lung injury Conditions that result
• Uveitis
in blindness
Manifestations of • RR > 30/min • Corneal ulcer
respiratory failure in • PaCO2 > 45 mmHg Anterior knee pain
post-op patient • PaO2 < 60 mmHg • Patellofemoral pain syndrome
involving the patella
Tests for pulmonary (runner’s knee)
and retinaculum
risk pre-operative • CXR, CBC, PFTs • Risk of trauma to the testicle
assessment Orchidopexy
located at the pubic tubercle
procedure
Candidates for • Patients on anticoagulants • Increased incidence of torsion
considerations in
bleeding risk pre- • Liver and kidney dysfunction • Psychological impact of an
cryptorchidism
operative assessment • Anemia empty scrotum
• Cardiac tamponade • Dehydroepiandrosterone
Virilizing adrenal
Life-threatening • Massive hemothorax • Androstenedione
tumors secretions
traumatic injuries • Hemoperitoneum • Testosterone and estrogen
• Unstable pelvic fractures Structure kyphosis
• Gibbus deformity
• Limit enteric content spillage affecting the lower
• Tuberculous spondylitis (Pott
Damage control • Prevent bloody vicious cycle thoracic and upper
disease)
surgery goals • Control bleeding and prevent lumbar regions
ischemia • Tear supplementation
Treatment for dry eye
• Hemoperitoneum • Environment coping strategies
syndrome
• Adjusting to eating patterns • Application of warm compress
Goals post-bariatric (keratoconjunctivitis
• Early identification of post- sicca) • Amelioration of eyelid
surgery
operative complications and abnormalities
preventive measures IV anesthetic drug
• Ketamine
• Hypothermia causing increased ICP
Causes of bleeding • Dilutional coagulopathy Contraindicated in • Prednisone ophthalmic drops
after massive blood • Fibrinolysis or Herpes simplex • Steroids could potentiate the
transfusion hypofibrinogenemia conjunctivitis infection
• Platelet dysfunction • Surgery if completely
Treatment of choice in
• Sciatic nerve in over 90% of resectable
Common nerve palsy early-stage tracheal
cases followed by femoral • Primary resection and
after hip arthroplasty neoplasm
nerve anastomosis
Common cause of Causes of non- • Primary tumor invasion of the
inguinal hernia in • Patent processus vaginalis pulmonary thoracic vagus nerve and right
infancy symptoms laryngeal nerve

TOPNOTCH MEDICAL BOARD PREP SURGERY PHASE 3 HANDOUT BY DR LOUBOMIR ANTONIO Page 13 of 15
For inquiries visit www.topnotchboardprep.com.ph or email us at topnotchmedicalboardprep@gmail.com
This handout is only valid for the September 2021 PLE batch. This will be rendered obsolete for the next batch since we update our handouts regularly.
TOPNOTCH MEDICAL BOARD PREP SURGERY PHASE 3 HANDOUT BY DR LOUBOMIR ANTONIO
For inquiries visit www.topnotchboardprep.com.ph or https://www.facebook.com/topnotchmedicalboardprep/
This handout is only valid for the September 2021 PLE batch. This will be rendered obsolete for the next batch since we update our handouts regularly.
Management of early • Surgical resection via video- Presentation of
• Hypophosphatemia
lung cancer (T1, T2, assisted lobectomy or calcium stone
• Increased calciuria
and T3 without N1) pneumonectomy formation in
• Hypercalcemia
Malignancy that affects • Pancoast tumor hyperparathyroidism
the pulmonary apex • Presents with pain and Electrolyte that needs • Hypokalemia
progressively affecting weakness of the arm and hand to be controlled pre- • Hyperaldosteronism leads to
the brachial nerve muscles op for adrenalectomy hypertension and low K levels
Most appropriate Radical nephrectomy • Ipsilateral regional
• Insertion of chest tube
treatment for for renal cell retroperitoneal lymph nodes
drainage of the pleural space
pneumothorax carcinoma includes • Ipsilateral adrenal gland
Commonly injured the ff structures • Gerota’s fascia
nerve in the surgical • Recurrent laryngeal nerve Cause of traumatic • Tear in the arteries
closure of PDA epidural hematoma • Middle meningeal artery
• Shortness of breath • Caused by ascending coliform
• Unilateral chest pain bacteria
Physical exam findings • Decreased breath sounds Characteristics of
• Urinary retention managed by
of pneumothorax ipsilateral side bacterial prostatitis
transurethral catheterization
• Hyperresonant on percussion • Empiric systemic antiobiotic
of the ipsilateral side Drugs used in the • Aminoglutethimide
• Increase metabolic demand management of • Ketoconazole
Characteristics of
• Cause direct mucosal and heat Cushing’s disease • Metyrapone
inhalation injury
injury to upper airways Acute red eye with no
Lung cancer least • Subconjunctival hemorrhage
loss of vision risk
associated with • Adenocarcinoma Profuse purulent
smoking • Gonococcal conjunctivitis
discharge from eye
• Stop at the tentorium cerebelli Best approach for
Features of subdural and falx cerebri vertical band
hematoma • Hyperdense crescent-shaped • Laparoscopic
gastroplasty in
lesion on CT scan bariatric surgery
• Very young and old patient Treatment for
Risk factors for
• Cerebrospinal fluid leak cavernous sinus • Antibiotics
subdural hematoma
• Anticoagulant therapy thrombosis
Vulnerability of • Lacks a serosal layer Manipulations that • Prostate biopsy
esophagus to • Allows the smooth muscle to will cause a false • Cystoscopy
perforation dilate with ease elevation of PSA • Urethral catherization
• Older age, chronic alcoholic • High serum lactate
Poor prognosticating
• Cirrhosis symptoms dehydrogenase
Clinical features of index for metastatic
• Mass in the right lobe of the • High serum calcium
hepatocellular renal cancer
carcinoma
liver on imaging • Low hemoglobin
• Elevated alpha fetoprotein • Tumor with adjacent
level laboratory Surgical objectives for
uninvolved stomach
resection of a gastric
Important lab test for • Specimen of blood for cross • Duodenum
carcinoma
pre-op management of matching should be sent • Regional lymph node
patient with acute whenever urgent surgery is • Washboard abdomen
abdomen anticipated Clinical features of an
• Absent bowl sounds
Etiology of • Esophageal varices acute abdomen
• Involuntary guarding
hematemesis that will • Endoscopy with variceal band Symptom of carcinoma
require emergency ligation should be carried out • Migratory thrombophlebitis
of body and tail
management as soon as possible • Trousseau’s syndrome
carcinoma of pancreas
Next work-up for Definitive
substernal chest pain • Endoscopic biliary
management for
after trial of PPI and • Cardiac work-up decompression
cholangitis
unremarkable Goals of management • Relieving gastric outlet
endoscopic findings for late-stage obstruction and biliary
Signs of acute • Bleeding pancreatic cancer obstruction
abdomen requiring • Ischemic bowel Primary histologic
urgent surgery • Perforated viscus type of • Adenocarcinoma (>95%)
Self-limited disease cholangiocarcinoma
• Acute mesenteric adenitis
most often confused Management for • Conservative with clinical and
• Pain is usually diffuse
with acute paralytic ileus pharmacological management
appendicitis in • Tenderness not as sharply
localized Bowel preparation • Antibiotics
children prior to surgical • Enema
Signs of abdominal • Cullen’s sign (periumbilical resection of bowel • Laxatives
wall hemorrhage ecchymosis) Risk factors for • Ulcerative colitis
associated with acute • Grey Turner sign (flank development of colon • Crohn’s colitis
pancreatitis ecchymosis) cancer • Inflammatory bowel diseases
• Electrocardiogram Diagnostic modalities • Lymphoscintigraphy
Should be monitored
• Jaundice could be a
in patients with for lymphedema • Duplex ultrasound
presentation of cardiac disease
cholestatic jaundice Most common cause of
due to chronic liver congestion
mechanical bowel
Indication of surgical • Palpable mass • Adhesion
obstruction for post-
incurability for • Signs of metastasis in hysterectomy patients
pancreatic cancer advanced stages Condition associated
• Extramammary perianal
with synchronous GIT
Paget’s disease
adenocarcinoma

TOPNOTCH MEDICAL BOARD PREP SURGERY PHASE 3 HANDOUT BY DR LOUBOMIR ANTONIO Page 14 of 15
For inquiries visit www.topnotchboardprep.com.ph or email us at topnotchmedicalboardprep@gmail.com
This handout is only valid for the September 2021 PLE batch. This will be rendered obsolete for the next batch since we update our handouts regularly.
TOPNOTCH MEDICAL BOARD PREP SURGERY PHASE 3 HANDOUT BY DR LOUBOMIR ANTONIO
For inquiries visit www.topnotchboardprep.com.ph or https://www.facebook.com/topnotchmedicalboardprep/
This handout is only valid for the September 2021 PLE batch. This will be rendered obsolete for the next batch since we update our handouts regularly.
Laxative that produces
hydrogen and
methane gases that • Mannitol
can explode with
electrocautery
• Expeditious surgery
Management for
• “The sun should never rise and
complete small bowel
set on a complete bowel
obstruction
obstruction”
Depth of chest
• 5.6 cm (2.0-2.4 in) deep
compressions in CPR
Number of rescue
• 2 breaths
breath per cycle
Number of chest
• 30 compressions
compression per cycle
Number of cycles
rescuers should
• 5 cycles in 2 minutes
perform for the CPR to
be effective
Percentage of women
with BRCA1 mutation
• 55-65% of women
developing breast
cancer before age 70
Inguinal hernia repair
procedure with the • Shouldice repair (2.2%)
least recurrence rate
Repair procedure for • Ureteroureterostomy
intraoperative (upper or middle ureter)
iatrogenic ureteral • Ureteroneocystostomy
injury (lower ureter)
• Long-standing, severe
secondary HPT which has
Cause of tertiary
turned autonomous once the
hyperparathyroidism
cause of secondary HPT has
been removed
Cause of secondary • Deficiency in vitamin D or
hyperparathyroidism uremia
• Adenoma of one or more
Cause of primary
parathyroid glands
hyperparathyroidism
• Hyperplasia
Watershed area of the
colon supplied by the • Splenic flexure
SMA and IMA
• Hemodynamic instability
• Obvious peritoneal signs
Indication for • Impaled foreign body
laparotomy in • Pneumoperitoneum
penetrating • Herniated abdominal organs
abdominal trauma • GSW with evidence of
intraperitoneal penetration
• Blood in orifices
• MTC
• Pheochromocytoma
• Multiple neuromas
Diagnostic features of
MEN2B • Long narrow face,
retrognathia, highly arched
palate
• Marfanoid habitus

TOPNOTCH MEDICAL BOARD PREP SURGERY PHASE 3 HANDOUT BY DR LOUBOMIR ANTONIO Page 15 of 15
For inquiries visit www.topnotchboardprep.com.ph or email us at topnotchmedicalboardprep@gmail.com
This handout is only valid for the September 2021 PLE batch. This will be rendered obsolete for the next batch since we update our handouts regularly.

You might also like